Sbi Clerk Pyq
Sbi Clerk Pyq
Sbi Clerk Pyq
Tg:- @NextGenBankers
Typeset By
DISHA DTP Team
Tg:- @NextGenBankers
Buying Books from Disha is always Rewarding
This time we are appreciating your writing
Creativity.
Write a review of the product you purchased on Amazon/
Flipkart
Take a screen shot / Photo of that review
Scan this QR Code
Fill Details and submit | That’s it … Hold tight n wait.
At the end of the month, you will get a surprise gift from Scan this QR code
Disha Publication
Write To Us At
feedback_disha@aiets.co.in
www.dishapublication.com
Contents
1. SBI Clerk (Junior Associates) Mains Solved Paper-2023 2023-1-32
Tg:- @NextGenBankers
SBI Clerk (Junior Associates) Mains Solved Paper-2023 2023- 1
Reasoning Ability & Computer Aptitude DIRECTIONS (Qs. 5-9): Read the information carefully and
1. Six friends Deepak, Sachin, Dinesh, Dharm, Anup, Firoj answer the questions given below:
are sitting around a circular table facing towards the centre. There are few Buses travelling to some points. There are two
Sachin is sitting at an alternate place with respect to junctions, 1 and 2 where 1 is west of 2. Point K is north, J is
Deepak. Dinesh is to the immediate right of Sachin. Dharm west and I is south of Junction 1. Point C is north, A is east and
can’t sit adjacent to Deepak and Anup. Anup is to the B is south of Junction 2.
immediate right of Deepak. Firoj is at the alternate place Note: To reach one point to another, one must pass through at
to the left of Anup. Who is sitting opposite of Deepak? least one junction. In an expression movement is from left to
(a) Dinesh (b) Sachin right. Also distance between each point is unknown.
(c) Dharm (d) Firoj T$S: Bus T is at a certain point and moves left from the junction
(e) Anup to reach point S.
2. In the following question assuming the given statements T@S: Bus S is at a certain point and moves right from the
to be true, find which of the conclusion among given junction to reach point T.
conclusions is/are definitely true and then give your T%S: Bus T is at point S and moves left from the junction to
answers accordingly. reach a certain point.
Statements: N ³ K < T; M > L £ H; I ³ J £ T; M = G £ I; T#S: Bus S is at point T and moves right from the junction to
Z=F£H reach a certain point.
Conclusions: T&S: Bus T is at point S and moves straight through all the
I. G > L II. F > L III. F = L
(a) Only I is True
Tg:- @NextGenBankers
(b) Only I and Either II or III are True
junctions to reach a certain point.
T+S: Bus S is at a certain point and moves straight through all
the junctions to reach point T.
(c) Only II and Either I or III are True
T!S: Bus T is at a point H and moves through the 1st junction
(d) Only III and Either II or I are True
(e) None is true then takes left turn at the 2nd junction to reach a certain point.
3. In the following question assuming the given statements T*S: Bus S is at a certain point and moves through the 1st
to be true, find which of the conclusion among given junction then takes left turn at the 2nd junction to reach point T.
conclusions is/are definitely true and then give your 5. In the given below expression, what is the final position
answers accordingly. of W with respect to its initial position?
Statements: J > I £ G; T ³ L ³ K; A = M ³ G; J = N > T; A+W$B, W&B
K>U£H (a) North-East (b) North-West
Conclusions: (c) South-East (d) West
I. M > I II. L < H III. I = M (e) South
(a) Either II or III is True 6. In the given below expression what is the initial position
(b) Either I or III is True of bus Q with respect to initial position of bus G?
(c) Either I or II is True Q%B+G
(d) All are True (a) West (b) South
(e) None is True (c) North (d) East
4. In the following question assuming the given statements (e) None
to be true, find which of the conclusion among given 7. In the given below expression, what is the final position
conclusions is/are definitely true and then give your of N with respect to initial position of P?
answers accordingly. N!A#P&C
Statements: K £ N < R; U = F £ T; H = I £ U; R = T; (a) North-West (b) North
X=B>H (c) South-West (d) South-East
Conclusions: (e) North-East
I. T > H II. R ³ U III. H = T 8. In the given below expression what is the initial position
(a) Only I is True of bus U with respect to final position of bus Y
(b) Both I and III are True U%I#Y%A
(c) Only II and Either I or III are True (a) West (b) South
(d) Only III and Either I or II is True (c) North (d) South-West
(e) Only II is True (e) Can’t be determined
2023- 2 SBI Clerk (Junior Associates) Mains Solved Paper-2023
9. 1 is added to the first digit, subtracted from the second Statement III: Z is the mother-in-law of L.
digit, added to the third digit, and so on in the number (a) All of the above
‘778446748284’. The final number is formed by (b) Both statements II and III
interchanging the digits of the resulting number, that is (c) Only statement I
the first and the second digits are interchanged, the third (d) None of the above
and the fourth digits are interchanged and so on. What is (e) Both statements I and III
the difference between the largest two digit number in the
number system and the sum of all the digits of the final DIRECTIONS (Qs. 15-18): Study the following information
number thus obtained? carefully to answer the given questions:
(a) 30 (b) 33 (c) 35 (d) 41 If ‘P + Q’ means ‘P is the father of Q’
(e) 36 If ‘P $ Q’ means ‘P is the mother of Q’
If ‘P % Q’ means ‘P is the brother of Q’
DIRECTIONS (Qs. 10-14): Read the following information If ‘P & Q’ means ‘P is the sister of Q’
carefully and answer the questions. If ‘P # Q’ means ‘P is the daughter of Q’
A family of nine members – Z, X, V, T, R, P, N, L and J, which 15. In the following expression: ‘A # O + N & M ? B’. What
consists of three generations, went out for a winter picnic to a should come in the place of the question mark (?), to
garden. All of them sat on a straight bench present in the garden, establish that ‘M is the daughter of B’?
with an equal amount of distance between them, but not (a) + (b) $ (c) % (d) #
necessarily in the same order. Some of them sat facing the south (e) None of these
whereas some of them sat facing the north. The number of people 16. Which of the following options is true if the expression
facing north is exactly half the number of people facing south. ‘B & A + W # H & N % G’ is definitely true?
Also, the number of males in the family exceeds the number of (a) W is the nephew of G.
females in the family by the smallest positive integer. (b) A is the brother in law of G.
T, who is the brother-in-law of J, sat at one of the ends of the (c) B is the uncle of N.
bench. Z, who is sat third from the right end of the row facing (d) N is the aunt of W.
north, is the mother of V. L, who is married to J, sits second to (e) None of the above
the right of C, who is seated at the left end of the row and is an 17. Which of the given expressions indicates that U is the uncle
unmarried male. J, who is the son of Z, sits at the exact middle of V?
Tg:- @NextGenBankers
of the bench and faces the south. R, who is the son of L, sits
second from the right end of the row and faces the same direction
(a) U & A + B # O & V
(b) U % A $ B # O + V
as his mother that is south. V, who is the wife of T, sits seventh (c) U # A + B % O $ V
to the right of her husband and faces south. The persons who (d) U % A $ B & O + V
are sat the extreme ends of the row, face the opposite directions. (e) None of these
X is the elder brother of J and V. Z is married to P, who sits to 18. How is J related to A in the expression ‘C + B $ A & K #
the immediate right of J. N sits second to the right of her J % T’?
grandfather P and third to the right of his father T. (a) Brother (b) Father
10. How is Z related to N? (c) Mother (d) Grandfather
(a) Mother (b) Aunt (e) None of these
(c) Cousin (d) Cannot be determined
(e) Grandmother DIRECTIONS (Qs. 19): A passage is given and then three
11. Who among the following sits third to the right of J? conclusions – I, II & III are given. You have to consider the
(a) T (b) Z (c) R (d) P information given in passage as true though it seems to be
(e) V different from the actual facts. Then give your answer regarding
12. In which position the grandchildren of P are seated with the given passage that which of the conclusion/s is/are followed
respect to him? by it properly.
(a) Second to the left and fourth to the right The newly opened bookstore “White Whale” has quickly gained
(b) Immediate neighbours popularity among the city’s book lovers. Its extensive collection
(c) Third to the left and third to the right of books from various genres and periods has attracted readers
(d) Second and fourth to the right of all ages. The bookstore’s cozy reading books and tea shop
(e) Fourth to the left and second to the right have added to its charm. The store owner, Aakash, who has
13. The two persons sitting at the extreme ends of the row are studied literature at a prestigious university, ensures that every
related as_____ book on the shelves is a gem.
(a) Cousins (b) Sisters 19. Conclusions:
(c) Brothers-in-law (d) Cannot be determined I. “White Whale” only sells new books.
(e) Brothers II. The bookstore’s success is due to its book collection
14. Which of the following statements is/are not Correct? and cozy atmosphere.
Statement I: No married couple is seated side by side. III. Akash’s expertise is in management rather than
Statement II: R and N are cousins. literature.
SBI Clerk (Junior Associates) Mains Solved Paper-2023 2023- 3
(a) Only I (b) Only II 23. The positive difference between the sum of individual
(c) Both I and II (d) Both I and III digits present in the code for the first word and for the
(e) Only III second word in the phrase ‘DOCTOR ENGINEER’ is?
20. In each question below is given a statement followed by (a) 9 (b) 11 (c) 13 (d) 1
two assumptions numbered I and II. An assumption is (e) 3
something supposed or taken for granted. You have to 24. If all the vowels in the word ‘DEPOSIT’ are shifted by
consider the statement and the following assumptions and two forward places according to the English alphabet series
decide which of the assumptions is implicit in the and all the consonants are shifted by four backward places
statement. according to the English alphabet series, the alphabets thus
Statement: Deepak the increasing reliance on AI-driven obtained are then placed in alphabetical order, starting from
decision-making tools in the financial sector, a recent study the left to the right. What is the difference between the
warns that automated financial advisors may inadvertently position in the alphabetical series of the second letter from
perpetuate biased financial practices, potentially the right end and the fourth letter from the left end of the
exacerbating existing disparities. These AI systems, trained newly formed arrangement?
on vast datasets, have shown tendencies to offer biased (a) 10 (b) 12 (c) 11 (d) 2
advice, leading to concerns about the reinforcement of (e) 9
discriminatory financial practices. DIRECTIONS (Qs. 25-29): Study the given information
Assumption: carefully and answer the following questions.
I. The integration of AI-driven financial advisors may
Nine people Z, Y, X, W, V, U, T, S and Q are living on the
contribute to reinforcing biased financial practices.
different floors of a hotel not necessarily in the same order such
II. Financial institutions are actively working to eliminate
that four floors remains vacant. Ground floor is numbered one,
biases in AI systems used for decision-making in the
first floor is numbered two and so on.
financial sector. Top and bottom floor of the hotel are Vacant. There are four
(a) Only I implicit floors between Z and T. Z lives above T. Two persons live
(b) Only II implicit between the floors on which S and W live. Z lives immediate
(c) Both I and II implicit below S. T is not living in odd number floor. T lives immediately
(d) Either I or II implicit above Vacant floor. Number of floors between the floors on
(e) Neither I nor II implicit
questions.
Tg:- @NextGenBankers
21. Read the following information carefully and answer the
which X and Q live is same as the number of floors between the
ones on which T and Y live. X lives in one of the floor above Q
and T. W lives immediately below vacant floor. No two vacant
In a certain code language, words and phrases are coded floor are immediate next to each other. At most three floor are
in accordance with the following set of rules- there in between two vacant floor. Difference between Floor in
1. All the consonants except N, M, and I are coded as which U and V lives is prime number with U lives above V.
the numbers of the even number series starting from 25. Who lives immediately two floor below one of the vacant
2. For example, Y is coded as 2, X is coded as 4, W is floor?
coded as 6 and so on. (a) U (b) V (c) T (d) Y
2. N, M, and R are coded as $, @ and % respectively. (e) W
3. The vowels are coded as the position of letter we get 26. How many persons live between Z and T?
by reversing the English alphabetical series The codes (a) Three (b) One (c) Four (d) Six
for vowels are- (e) Five
4. If the third or the fourth letter of a word is a vowel, 27. How many floor are there in between Y and W?
then that vowel is coded as ‘#’. (a) 2 (b) 3 (c) 4 (d) 1
5. If the number of letters in a particular word is even, (e) 5
then the code for that particular word starts with an 28. Which floors are vacant?
‘C’ and if the number of letters is odd, then the code (a) 6, 2 (b) 2, 4 (c) 5, 1 (d) 6, 3
for that word starts with a ‘A’. (e) 2, 5
The word ‘BUSINESS’ is coded as? 29. Four among the five are the same and thus forms a group.
1. C2624#&242422 Who among the following does not belong to that group?
2. Z2426#&222424 (a) S (b) X (c) T (d) V
3. Z2624#&222424 (e) Z
4. C2624#&222424 DIRECTIONS (Qs. 30-32): Study the following information
5. C2426#&222424 and answer the following questions.
22. The first part of the code for the word ‘INTERMEDIATE’
is C18&26#%@. What is the remaining part of the code P # Q means All P is Q.
for the given word? P @ Q means Some P is not Q.
(a) 22618262622 (b) 22186262622 P * Q means Only a few P is Q.
P $ Q means Some P is Q.
(c) 22618222626 (d) 22626182622
P ^ Q means No P is Q.
(e) 22186222626
% means possibility e.g. P $% Q means Some P can be Q.
2023- 4 SBI Clerk (Junior Associates) Mains Solved Paper-2023
30. Statements: Book @ Table # Mobile $ Pen * Copy Statement I: Recent law changes in India have made it
Conclusions: possible for mother’s name to be used as a legitimate
I. Table #% Pen identification identifier on a variety of papers, including
II. Book @ Mobile passports and PAN cards.
III. Copy * Pen Statement II: There are international human rights treaties
(a) Only I follows (b) Only II follows and conventions that India is a signatory to, which uphold
(c) Only III follows (d) Only I and II follows gender equality.
(e) Only I and III follows (a) Statement I is the cause and statement II is the effect.
31. Statements: H4 # G3 * S6^ Y4# Q7 (b) Statement II is the cause and statement I is the effect.
Conclusions: (c) Both statement I and II are independent causes.
I. G3 @ Y4 II. H4#% S6 (d) Both statements I and II are effects of independent
III. S6 #% Q7 causes.
(a) Only I follows (e) Both statements I and II are effects of some common
(b) Only II and III follows
cause.
(c) All follows
35. The university has decided to implement a new policy in
(d) Only I and II follows
the upcoming academic session that mandates all students
(e) Only III follows
32. Statements: Man * Woman ^ House @ Car # Bike to renew their biometric identity cards every six months
Conclusions: which requires physical presence of the students. This was
I. Car @ Woman II. Man #% Bike done due to limited use of resources by alumni. What effect
III. Bike @ Woman will this policy have?
(a) Only I follows Possible Effects:
(b) Only II and III follows I. The policy will have a positive effect on both
(c) None follows academic performance and engagement, as it will
(d) Only I and III follows motivate the students to attend classes regularly,
(e) Only II follows interact with their peers and instructors and take the
33. In the question, the statement is given followed by some benefit of the available resources.
inferences. You must consider the statement to be true even II. The policy will have a negative effect on both
Tg:- @NextGenBankers
if it seems to be at variance from commonly known facts
and then decide which inference can be drawn from the
given statement.
academic performance and engagement, as it will
create an unnecessary burden and hassle for the
students, especially those who live far away or have
The Net has become an indispensable tool for finding other commitments.
information, communicate with people around the world (a) Only I (b) Only II
and manage your finance. However, it also poses many (c) Either I or II (d) Neither I nor II
risks and challenges for young generation, such as (e) Both I and II
radicalisation, race hate materials, excessive violence, and 36. In the question given below, there is a statement followed
suicide. Some experts argue that parents should monitor by four courses of action. On the basis of the situation
and limit their children’s internet use, while others claim given you have to decide which of the suggested course(s)
that this would violate their privacy and autonomy. Christie of action logically follow(s) for pursuing. There is an
Brinkley, a famous actress and mother of three, has increase in the number of waterborne diseases in Area W
revealed that she uses an app to track her kids’ online of Country H because the people of that area are using tap
activity and restrict their screen time. water without any filtration.
Inferences:
Course of action:
I. Christie Brinkley is a responsible parent who cares
I. The supply of water in area W should be reduced.
about her children’s well-being and safety.
II. To save from waterborne diseases public awareness
II. Parents have the right to interfere with their children’s
internet use as they see fit. programme should be arranged so that people can be
III. The internet is a dangerous place for children and aware of using clean water.
they need constant supervision and guidance. (a) If only I follows
(a) Only I (b) Only II (b) If only II follows
(c) Only III (d) Only I and III (c) If either I or II follows
(e) Only II and III (d) If neither I nor II follows
34. In the question given below, there are two statements (I) (e) If both I & II follow
and (II). These statements may be either independent 37. Statement: To manage the heavy workload, the workers
causes or may be effects of independent causes or a of company X are demanding to increase the number of
common cause. One of these statements may be the effect workers. The workers have also threatened the company
of the other statement. Read both the statements and decide to go on strike if the demand is not fulfilled by the company.
which of the following answer choice correctly depicts Which of the following will strengthen or weaken the above
the relationship between these two statements. statement?
SBI Clerk (Junior Associates) Mains Solved Paper-2023 2023- 5
I. For the last few years, due to the heavy workload, II. Incidents like these contribute to a negative
the workers have been working extra hours and did perception of safety in public spaces for women, and
not take any leave. such occurrences tarnish the reputation of the city.
II. In the last six months, the workers of company X (a) Only I implicit
have repeatedly taken rest in between the work, thus (b) Only II implicit
the work is not completed. (c) Either I or II implicit
(a) I is weak and II is strong (d) Both I and II implicit
(b) Both are strong (e) Neither I nor II implicit
(c) Both are weak 40. Read the statement below followed by two conclusions
(d) I is strong and II is weak and decide which of the assumptions is implicit from the
(e) Either I or II is weak statement.
38. Read the given information and answer the following Statement: The college authorities have decided to give
questions: seven grace marks in the Mathematics paper to all the
Our global civilization is said to be enhanced by variety. second-year students as the performance of these students
Sadly, despite recent biological evidence that has in Mathematics was below their expectations.
categorically refuted such views, theories of racial Assumptions:
prejudice continue to be spread and put into reality. I. Majority of the students in second-year may still fail
Numerous people suffered unimaginable pain as a result in Mathematics even after giving them grace marks.
of slavery, the slave trade, imperialism, and colonialism. II. Majority of the students in second-year may now pass
Which of the given statements will weaken the argument? in Mathematics after giving grace marks.
(a) Millions of people from the weaker segments of (a) Only assumption I is implicit
society now have voice and empowerment thanks to (b) Only assumption II is implicit
the institutions of our democratic polity and the (c) Either assumption I or II is implicit
growth of literacy. (d) Neither assumption I or II is implicit
(b) In comparison to the many advantages one person (e) Both assumptions I and II are implicit
enjoys, people from various racial and ethnic origins 41. Six people from three generations of a family live in a
may enjoy privileges. house. How many males are in the family?
(c) A National Committee should be established, and they Statement I – K is the daughter of G, who is married to H.
Tg:- @NextGenBankers
should be encouraged by the general support of the
legal protections against discrimination in the
constitution and other laws.
J is the father of a son and a daughter. F, the sister of H, is
unmarried. H is the son-in-law of I.
Statement II – J is the father of H, who is married to G. I
(d) The internet and social media have made the globe a is the mother of G, who has a daughter. F is the aunt of K.
more interconnected place. However, some people J has two children, a son, and a daughter.
are using this technical advancement to foment racial (a) Only Statement I alone is sufficient to answer the
animosity. question
(e) The government has established an institutional and (b) Only Statement II alone is sufficient to answer the
administrative framework to address various types question
of discrimination within our democratic system (c) Both statements together are sufficient to answer the
39. In each question below is given a statement followed by question
two assumptions numbered I and II. An assumption is (d) Either statements are sufficient to answer the question
something supposed or taken for granted. You have to (e) Neither statements are sufficient to answer the
consider the statement and the following assumptions and question
decide which of the assumptions is implicit in the 42. The question below consists of two statements numbered
statement. I and II given below it. You have to decide whether the
Statement: A popular food blogger, known for her data provided in the statements are sufficient to answer
Instagram account ‘Tasty Bites Explorer,’ was recently the question. Read all the three statements and give answer:
harassed by a man in a busy marketplace in Mumbai. The Six boxes namely – N, M, L, K, J and I are kept one above
entire incident was captured on her Instagram Live and other, each box contains different Vegetables viz. Tomato,
quickly gained attention. In the video, the man tries to Potato, Carrot, Onion, Corn and Garlic. Box M is kept
strike up a conversation with the blogger, asking if she third from top at a gap of one box from one that contains
would like to join him for coffee. However, his seemingly Onion. Box K contains Corn, which is neither kept adjacent
harmless approach turns uncomfortable as he makes to box M nor adjacent to the one that contain Onion, is
inappropriate remarks about the blogger’s appearance.
kept just above one that contains Potato. How many boxes
Despite her attempts to ignore him, the man persists in
are kept below one contains Garlic?
following her and making lewd comments.
I. Box which contains Carrot and Onion are kept
Assumption:
together. Box I, which neither contains Carrot nor
I. The behaviour of the man not only violated the
contains Onion, is kept at a gap of two from box N.
blogger’s right to privacy and dignity but also
Box which contains Carrot is kept at a gap of three
highlighted the broader societal issue of gender-based
from box J.
harassment that needs to be urgently addressed.
2023- 6 SBI Clerk (Junior Associates) Mains Solved Paper-2023
II. I which contains Tomato is kept at a gap of two from 47. Which of the following combinations represents the first
box N, which neither contains Carrot nor kept two and last two elements in step IV of the given input?
adjacent to box K. Only two boxes are kept between (a) SAFE ,PLAY and EOT, DGO
box J and one which contains box Garlic. (b) PLAY, AEFS and EOT, DGO
(a) If the data in statement I alone is sufficient to answer (c) PLAY, AEFS and TOE, DOG
the question, while the data in statement II alone is (d) CLOO, SAFE and CRY, ELT
not sufficient to answer the question. (e) ALPY, AEFS and EOT, DGO
(b) If the data in statement II alone is sufficient to answer 48. Seven persons namely – Ravi, Sonu, Kavita, Sameen, Tanu,
the question, while the data in statement I alone is Manu and Shyam have different salaries, Salary of how
not sufficient to answer the question. many people are less than Sonu?
(c) If the data either in statement I alone or in statement I. Salary of Ravi is more than Kavita and Sonu but not
II alone is sufficient to answer the question. as much as Shyam. The salary of Kavita is more than
(d) If the data in both statement I and II together are not Tanu and Sameen but not as much as Ravi and Shyam.
sufficient to answer the question. Salary of Tanu is only more than two Person and
(e) If the data in both statement I and II together are salary of Manu is lowest.
necessary to answer the question. II. Salary of Sonu is more than that Shyam and Tanu but
not as much as Shyam and Kavita . The salary of
DIRECTIONS (Qs. 43-45): Study the following information Manu is more than Sameen and Manu but not as much
carefully and answer the questions given below. as Ravi and Kavita . Salary of Shyam is the highest,
A arrangement machine, when given a particular input, and Salary of Sameen is not lowest.
rearranges it following a particular rule. The following is an (a) If the data in Statement I alone is sufficient to answer
illustration of the input and the steps of arrangement. the question, while the data in Statement II alone is
Input: LOVE, HOPE, FREE, GIFT, BEST, CAT, MAT, not sufficient to answer the question. b) Ic) d) . e)
MAN, TOP (b) If the data in Statement II alone is sufficient to answer
Step I: HOPE, FREE, GIFT, BEST, ELOV, ACT, MAT, MAN, the question, while the data in Statement I alone is
TOP not sufficient to answer the question.
Step II: FREE, GIFT. BEST, EHOP, ELOV, ACT, MAT, MAN, (c) If the data either in Statement I alone or in Statement
TOP II alone is sufficient to answer the question.
TOP
Tg:- @NextGenBankers
Step III: GIFT, BEST, EEFR, EHOP, ELOV, ACT, AMT, AMN, (d) If the data in both the Statements I and II together are
not sufficient to answer the question
Step IV: BEST, FGIT, EEFR, EHOP, ELOV, ACT, AMT, AMN, (e) If the data in both the Statements I and II together are
OPT necessary to answer the question
Step V: BEST, FGIT, EEFR, EHOP, ELOV, ACT, AMT, AMN, 49. Study the following information carefully and answer the
OPT given questions.
Step V is the last step of the rearrangement. As per the rules Seven persons Z, Y, X, W, V, U, and T are living in an
followed in the above steps, find out the answer to each of the eight-storey building where the bottommost floor is
following questions for the Input given below. numbered as 1 and the floor just above it as 2 and so on.
Input: CALM, TRUE, COOL, SAFE, PLAY, CRY, LET, One floor is vacant. Only one person lives on one floor. In
TOE, DOG which floor does U live?
43. Which of the following is the third element from the left I. One of the odd numbered floors is vacant. X lives in
end in step V? an odd floor below W who lives in an even numbered
(a) ERTU (b) AEFS floor but not on the top floor. W is not immediate
(c) ACLM (d) CLOO above X. 3 people lives between W and T.
(e) ALPY II. Number of floors above V is one more than the
44. What is the position of “PLAY” from “ERTU” in step III? number of floors below V. The number of persons
(a) 3rd to the left (b) 2nd to the left live between Z and V is same as the number of persons
(c) Immediate left (d) 3rd to the right live between V and T. Z lives on the top floor. Y lives
(e) None of these immediate above T.
45. Which word comes exectly between “AEFS” and “ERTU” (a) I is alone sufficient
in step V of the given input? (b) II is alone sufficient
(a) DGO (b) CLOO (c) Both statements are not sufficient
(c) ELT (d) ACLM (d) Either I or II alone sufficient
(e) ALPY (e) Both statements are needed to answer the question.
46. In which step are the elements ‘PLAY CLOO ERTU’ 50. In the following question, five different words are given
found in the same order? in the options. Along with each word, positions of four
(a) Second (b) Fourth letters in that word are given and the positions of the letters
(c) Third (d) Fifth are counted from the left end of the word. Four lettered
(e) None of the above meaningful word can be formed from the letters
SBI Clerk (Junior Associates) Mains Solved Paper-2023 2023- 7
corresponding to the positions given in each option without 54. If the marked price of each Tricycle in shop R is Rs.
jumbling the letters. Which of the following options will ______ and the average number of Tricycle sold in shops
give a meaningful English word? R and U is 23 and the total number of Tricycles sold in
(a) Claustrophobia – 2, 7, 12, 14 shop U is ______ then which of the following conditions
(b) Photosynthesis – 2, 7, 8, 12 satisfy the same order.
(c) Incomprehensive – 3, 4, 10, 12 (i) 250 and 21 (ii) 250 and 30
(d) Notwithstanding – 4, 5, 11, 12 (iii) 250 and 24
(e) Parallelogram – 6, 7, 12, 13 (a) I only (b) III only
(c) II only (d) I and II only
Quantitative Aptitude (e) None of these
55. If the total number of Tricycles sold in shop P and R
DIRECTIONS (Qs. 51-55): The given table shows the number together is 55. Find which of the following contexts is
of Tricycle sold in different shops P, Q, R and S and Cost price true.
of each article, Markup percentage of each article and discount I) The difference between the total revenue received
amount of each article in the same shops. by the shop P and R is Rs. 665.
II) The marked price of each Tricycle in shop R is Rs.
Shops Sold Cost price of Markup % of Total 250.
Tricycles each Tricycle each Tricycle Discount III) The selling price of each Tricycle in shop P is Rs.
135.
Amount
(a) I only (b) II only
P (x + 5) 4x 40 210 (c) I and II only (d) II and III only
Q y (y + 5) 20 3y (e) I and III only
56. Given that the combined ages of two individuals, J and K
R x 4x 150 117x
is a2 years, and the difference of their ages is '72 – 12a'
S (y + 10) y 80 27y years (with K being younger), and also the ratio of J's age
Note: two years ago to J's age ten years from now is 4 :7. can
(i) Selling price of Tricycle in shop P is equal to selling you determine the average between J's age three years
from now and K's age seven years ago?
Tg:- @NextGenBankers
price of Tricycle in shop R.
(ii) Selling price of Tricycle in shop Q is equal to selling
price of Tricycle in shop S.
(a) a + 12
(c) a + 15
(b) a + 10
(d) a + 8
(iii) The average cost price of Tricycle in each shops P, (e) None of these
Q and R is 135. 57. In a container of capacity 500 ml a mixture of juice and
51. If the cost price of Tricycles in shops S to T are in the water is kept. Juice is 250 ml and water is 'a' ml, Now
ratio 4 : 5 and the number of Tricycles sold in shop T is 70 ml of mixture is taken out without replacement and
(y – 120), Markup percentage of Tricycles in shop T is the process is repeated one more time, the container is
(x + 5)%. If the total discount amount of Tricycles in shop now completely filled by adding water. If the ratio of milk
T is Rs. 4800. Find which of the following conditions are and water in the final mixture is 3 : 7. What is the quantity
true? of water added?
I. Selling price of the Tricycles in shop T is ` 265. (a) 250 ml (b) 300 ml
II. Total revenue generated by the shop T by selling all (c) 290 ml (d) 260 ml
the Tricycles in shop T is `12500 (e) 80 ml
III. Number of Tricycles sold in shop T is 70. 58. In the given question, two quantities are given, one as
(a) I only (b) II only Quantity I and another as Quantity II. You have to
(c) III only (d) I and II only determine relationship between two quantities and choose
(e) I and III only the appropriate option.
52. Find the difference between the Sum of the marked price Umesh bought two articles A at Rs. x and article B at
and sum of the selling price of each Tricycles sold in the ` x + 50. He sold article A at 20% profit and article B at
shop P, Q, R and S. 10% loss, and earned Rs. 45 as profit on the whole deal.
(a) ` 156. 3 (b) ` 152. 7 Quantity I : Profit earned by Umesh on selling Article
(c) ` 141.3 (d) ` 147. 3 A(in Rs.)
(e) ` 149. 3 Quantity II : Loss incurred (in `) when an article which
53. If the number of Tricycles sold in the shop Q 20% are costs Rs. 560 is sold at 25% loss.
defective and sold at Rs. 2 more than the usual discount. (a) If Quantity I ³ Quantity II
Find the total discount amount given by Shop Q by selling (b) If Quantity I > Quantity II
all the Tricycles (c) If Quantity I < Quantity II
(a) 580 (b) 550 (d) Quantity I = Quantity II or the relationship cannot
(c) 600 (d) 680 be established from the information that is given
(e) 590 (e) Quantity I £ Quantity II
2023- 8 SBI Clerk (Junior Associates) Mains Solved Paper-2023
59. In this question, two equations are given. Answer the BPOs Ratio between
questions based on given equations: Inbound Messages to
I. 2x2 – Mx + 10 = 0 Outbond Messages
a and b are two roots of the given equation and their sum
is a + b = 4.5 (a > b) P 11:05
II. 4y2 – Ny + 19 = 0 Q 4:x
c and d are two roots of the given equation such as one R 19:05
root is 40% of the largest root of first equation. (c > d)
S -
Which of the following statements is are correct?
I. Values of a + c = b + d T 5:06
II. c + d = 5. 75 U 3:05
III. a > b > c > d 62. If the number of males to females bounded the inbound
(a) Only I (b) Only III messages in K are in the ratio 10 : 9. And the total number
(c) Only II (d) All of these of males bounded the all the messages in R is 70. Find
(e) None of these the difference between the number of females bounded
60. Rohan invested some money in the bank at the rate of the inbounded messages and outbound messages in the
20% compounded annually and after 2 years received centre R.
interest as Rs. 8800. If the man invested double of the (a) 50 (b) 40
principal in another scheme at the rate of 10% (c) 30 (d) 35
compounded annually for 2 years then find the compound (e) 45
interest earned by him in the second scheme 63. Inbound and Outbound messages of S are in the ratio
(a) Rs. 8300 23 : 7. Then Inbound messages of S are (a + 3)% more
(b) Rs. 8600 than total messages made by Q. If the ratio of domestic
(c) Rs. 8200 inbound messages and International inbound messages
of R is (a + 4) : 3. Find International Inbound messages
(d) Rs. 8400
of R.
(e) Rs. 8700
(a) 15 (b) 24
61. A box contains 10 yellow, 8 green and some white balls.
Tg:- @NextGenBankers
Find the probability of getting three different coloured
balls when 3 balls are drawn from box at random if total
(c) 18
(e) None of these
(d) 20
expenditure of 'A' and 'B' is `5920. If the monthly saving 73. The profit obtained on article L is ` 36 and the discount
and expenditure of ‘B’ is the ration of 5:7 respectively offered on article L is 20%. Find the difference between
then find the average monthly income of ‘A’ and ‘B’ the selling price of article J and the cost price of article L.
together? (a) ` 110 (b) ` 118
(a) ` 52000 (b) ` 54000 (c) ` 135 (d) ` 120
(e) None of these
(c) ` 56000 (d) ` 58000
(e) None of these 74. Selling price of article J to the selling price of article N
68. Yuvi and Ekansh started a business with the investment are in the ratio 3 : 5. The Marked price of article N is `50
of ` 30000 and ` 36000 respectively, after 6 months Yuvi more than its selling price. If the cost price of the article
withdrawn 20% of his initial investment and Kartik started N is Rs. 10 less than the cost price of the article M. If J is
the business with the investment of ` X. At the end of the sold at 20% profit. Find sum of the cost price of article
year, the total profit of the business is ` 132000 and profit N. Marked price of article N and Cost price of article J.
share of Ekansh is ` 48000 and find the vale of X? (a) Rs. 750 (b) Rs. 850
(a) 72000 (b) 76000 (c) Rs. 900 (d) Rs. 650
(c) 80000 (d) 64000 (e) None of these
(e) None of these 75. If the profit obtained on article M is Rs. 25 less than the
69. In this question, two equations are given, answer the profit obtained on article K, then the discount offered on
questions based on given equations: article M is
I. x2 – 12x + 32 = 0 (a) 24.67% (b) 26.89%
II. x2 – 26x + 169 = 0 (c) 22.82% (d) 21.25%
If one of the roots of given equation is taken from both (e) None of these
equation, a new equation is formed which is: 76. The ratio of the cost price of two chairs is 5 : 9 one chair,
(a) x2 + 10x + 36 = 0 (b) x2 – 17x + 52 = 0 in which cost price is less, is sold at profit of 24% and the
2
(c) x – 17x – 52 = 0 (d) x2 + 21x – 104 = 0 other one is sold for Rs. 2196 more than that of the first
2
(e) x – 16x + 63 = 0 one, if the overall profit earned after selling both the chairs
at 6%. What is the difference of cost price of the two
DIRECTIONS (Qs. 70-71): What approximate value should chairs ?
come in place of the question mark(?) in the following (a) 3150 (b) 3600
question?
70.
Tg:- @NextGenBankers
? × 8.23 + 44. 89% of 80 .31 = 32.92 × 4.14
(c) 1800
(e) None of these
(d) 2700
(a) 8 (b) 12 77. The ratio between the radius of the circle and the length
(c) 16 (d) 20 of the rectangle is 7 : 22 and the breadth of the rectangle
(e) 14 is equal to the base of the triangle. The area and height of
71. 25.12 × 4.29 ¸ 5.19 + 16. 16 + ( 7 . 91)2 = ?2 the triangle is 2100 cm2 and 60 cm. The perimeter of the
(a) 10 (b) 8 rectangle is 360 cm, then, find the perimeter of the circle?
(c) 9 (d) 5 (a) 220 cm (b) 340 cm
(e) 12 (c) 450 cm (d) 550 cm
(e) None of these
DIRECTIONS (Qs. 72-75): Read the following information
carefully and answer the questions given below. DIRECTIONS (Qs. 78-79): What approximately value should
come in the place of the question mark (?) in the following
The marked price of an article K and M is 50% above its cost equation?
price and the selling price of article J is half of the marked
price of article K. The ratio of the marked price of article M 78. 18.17 × 4.23 + (24.97 ¸ 5.31) – 33. 24 = ?
and the selling price of article J is 7 : 3. The sum of the marked (a) 50 (b) 51
price of the article L and M is Rs. 810. The profit of article K is (c) 44 (d) 40
Rs. 194 less than the cost price of M. The discount offered on (e) 47
article K is 15%. 79. 70 . 39 – 20. 19 × 1.91 + 36. 41 × 1. 17 = ?
72. If the Cost price of the article M is increase by 25% and (a) 60 (b) 66
the discount percentage of the article M is 20% then find (c) 76 (d) 73
which of the following is true in the given following (e) 81
statements? 80. A boat travels is 1.85 km downstream is 18 hours, while
I. New cost price of the article M is Rs. 325 a person can run S km in 24 hours. The speed of the boat
II. Loss of Rs. 13 occurred in still water exceeds the running speed of the person by
III. Selling price is Rs. 332 4 km /h, and the speed of the current is 3 km/h. What is
(a) I and II only (b) I only the speed of the boat in still water?
(c) II only (d) III only (a) 5 (b) 12
(e) I and III only (c) 9 (d) 6
(e) 10
2023-10 SBI Clerk (Junior Associates) Mains Solved Paper-2023
81. A train takes t seconds to pass a pole and t + 12 seconds DIRECTIONS (Qs. 88-91): In the question, two quantities
to pass a platform. The combined length of the train and (I) and (II) are given. You have to solve both quantities to
the platform is 540 meters and the train's length is L establish the correct relation between Quantity (I) and Quantity
meters. If the train's length is reduced by 30 meters, it (II) and choose the correct option.
then takes 18 seconds to cross a pole. Determine the value
of t. 88. Quantity I: 50x + 32 × x 64 such that 2 £ x £ 4
(a) 23.5 (b) 19.7
(c) 16.8 (d) 20.5
(e) None of these Quantity II: 3 216y3 –2y4 + 50y such that 8 £ y £ 10
(a) Quantity (I) > Quantity (II)
DIRECTIONS (Qs. 82-83): The Series contains one wrong (b) Quantity (I) < Quantity (II)
and one missing number. (c) Quantity (I) ³ Quantity (II)
0.5, 6, 84, 840, 6720, 40320, a (d) Quantity (I) £ Quantity (II)
82. What is the value of a - 112809 (e) Quantity (I) = Quantity (II) or No relation can be
(a) 41000 (b) 43245 established
(c) 48471 (d) 52231 89. Quantity I: Pipe P can fill a tank in 3 hours. Pipe Q can
(e) 45325 fill the same in 4 hours while Pipe R can fill the same
83. Find the wrong number of the given series. tank in 5 hours. What is the time taken when all of them
(a) 84 (b) 6 works together?
(c) 6720 (d) 840 Quantity II: Two pipes P and Q can fill a tank in 8 hours.
(e) 40320 While Pipe Q and Pipe R can fill the same in 10 hours.
Pipe R and Pipe S fill the tank in 15 hours. Pipe P and
DIRECTIONS (Qs. 84-86): In this question, two equations Pipe S can fill the same tank in 30 hours. Find the time
are given. Answer the questions based on given equations:
taken by them to fill the tank together.
I. a, b, c, 707, 563, 627, 611 (a) Quantity I > Quantity II
II. x, x + 18, 433, 451, 469, y (b) Quantity I < Quantity II
84. Tg:- @NextGenBankers
Which of the following are true?
æ 1ö
(c) Quantity I = Quantity II or no relation can be
established
(i) y = a + b (ii) y = 446 + çè ÷ø C (d) Quantity I £ Quantity II
11
(e) Quantity I ³ Quantity II
1 90. Quantity I : Perimeter of triangle cut, of a square of side
(iii) x + b = ( y) 22 cm, four isosceles triangles are cut from its four corners.
2
(a) Only (ii) (b) Only (iii) Area of remaining part is 196 sq. cm. Take 2 =1.4
(c) Only (i) and (iii) (d) Only (ii) and (iii)
(e) All of these Quantity II : Percentage increase in area of rectangle.
85. If the sum of z + y = 576, then find the average of 'c' and 'z'. Increase in length and breadth are 20% and 15%
(a) 300 (b) 270 respectively.
(c) 220 (d) 200 (a) Quantity I ³ Quantity II
(e) 250 (b) Quantity I £ Quantity II
86. Had 'a' been divided by the second largest single digit (c) Quantity I = Quantity II or no relation can be
prime number, then resultant would be.... established
(a) 45 (b) 38 (d) Quantity I > Quantity II
(c) 55 (d) 50 (e) Quantity I < Quantity II
(e) 40 91. Average of P and Q is 12, while the sum of Q and R is 12.
87. 3x2 – 26x + n = 0 14 ³ (P, Q) ³ 8 and 10 ³ Q and R ³ 2; Given P, Q, R are
Roots of the given equation are l and m, where, l > m and integers.
m = 8/3. Quantity I : Value of PQ – 54
Which of the following/s is/are true? Quantity II : (P + R) × Q
I. HCF of n and l is equal to 50% more than m (a) Quantity I ³ Quantity II
II. Value of n is a perfect cube (b) Quantity I £ Quantity II
III. n is multiple of l. (c) Quantity I = Quantity II or no relation can be
(a) Only I and II (b) Only II established
(c) Only II (d) Only I and III (d) Quantity I > Quantity II
(e) All of these (e) Quantity I < Quantity II
SBI Clerk (Junior Associates) Mains Solved Paper-2023 2023-11
DIRECTIONS (Qs. 92-93): The following pie chart shows 97. Average weight of four friends 'J', 'K', 'L' and 'M' is a kg.
the percentage distribution of the number of mobiles sold in a When 'M' is replaced by 'N' then the average increases by
shop in five different days Monday, Tuesday, Wednesday, 14.25 kg. When 'L' is replaced by 'N', average increases
Thursday and Friday. Total number of mobiles sold = 400 by 6.75 kg. If the average weight of 'L', 'M' and 'N' is 74
kg then the weight of 'N' is how much percent more than
Monday that of 'L'?
Friday (a) 30% (b) 36%
a%
(c) 42% (d) 45%
6% Tuesday (e) None for these
23%
DIRECTIONS (Qs. 98): The following questions are
20%
accompanied by two or three statements . You have to determine
1.25a% which statements(s) is/are sufficient/necessary to answer the
Thursday Wednesday questions.
Note: Difference between number of mobiles sold on Monday 98. What is the present age of Father?
and Wednesday is 12. I. The difference between the ages of Father and Son
92. Number of mobiles sold on Tuesday and Monday is what is 45 years.
percentage of the number of mobiles sold on all the II. The present age of Son is 1/4th of the present age of
remaining days. the Father.
(a) 56.87% (b) 50.78% III. The sum of the ages of Father and Son is 75 years.
(c) 42.89% (d) 53.84% (a) Any two of them (b) Only I and III
(e) None of these (c) Only I and II (d) All I, II and III
93. If the cost of each mobiles on Friday is `P and cost of (e) None of these
each mobiles on Thursday is ` (P – 10) if the total cost 99. If a wire is cut into three pieces of unequal lengths, what
earned by the shop by selling mobiles on the both the day is the length of the shortest of these pieces of wire?
is ` 9200. If the cost of each mobiles on Monday's Statement I. The combined length of the longer two
` (P – 20). Find the total amount earned by the shop on pieces of wire is 35 meters.
(a) ` 1280
(c) ` 1440
Tg:- @NextGenBankers
selling mobiles on Monday.
(b) ` 1360
(d) ` 1560
Statement II. The combined length of the shorter two
pieces of wire is 30 meters.
(a) The data is statement I alone is sufficient to answer
(e) None of these the question, while the data in statement II alone is
94. Number of mobiles sold on Wednesday is 66.667% of not sufficient to answer the question.
the number of mobiles present in the shop on Wednesday (b) The data is statement II alone is sufficient to answer
initially and the number of mobiles sold on Thursday is the question, while the data in statement I alone is
33.33% of the number of mobiles present in the shop not sufficient to answer the question.
initially. Find the ratio between the number of unsold (c) The data in statement I alone or in statement II alone
mobiles on Wednesday and the number of unsold mobiles is sufficient to answer the question.
in the shop on Thursday (d) The data in both statements I and II are not sufficient
(a) 2 : 11 (b) 2 : 13 to answer the question.
(c) 3 : 16 (d) 3 : 17 (e) The data in both statements I and II together are
(e) None of these necessary to answer the question.
95. Average number of mobiles sold on Monday and Sunday 100. Two friends P and Q together can complete a piece of
is 8 less than the number of mobiles sold on Monday and
work in 20 days. In how many days can P alone complete
number of mobiles sold to unsold on Sunday are in the
the piece of work?
ratio 8 : 5. Find the total number of mobiles unsold on
Statement I. Q alone completes half of the work in 25 days.
Sunday.
(a) 28 (b) 25 Statement II. The efficiency of P is 50% more than that of Q.
(c) 35 (d) 20 (a) The data is statement I alone is sufficient to answer
(e) 44 the question, while the data in statement II alone is
96. Difference between the mobiles sold on Tuesday and not sufficient to answer the question.
Monday is twice the difference between the number of (b) The data is statement II alone is sufficient to answer
mobiles sold on Wednesday and Saturday. If the ratio of the question, while the data in statement I alone is
number of mobiles sold to unsold on Saturday is 19 : 15. not sufficient to answer the question.
Then find the number of mobiles unsold Saturday. (Note: (c) The data in statement I alone or in statement II alone
Number of mobiles sold on Wednesday is more than the is sufficient to answer the question.
number of mobiles sold on Saturday). (d) The data in both statements I and II and not sufficient
(a) 52 (b) 38 to answer the question.
(c) 30 (d) 44 (e) The data in both statements I and II together are
(e) None of these necessary to answer the question.
2023-12 SBI Clerk (Junior Associates) Mains Solved Paper-2023
life perspectives. However, a cautious approach towards (d) The Mediocre Influence of Motivational Books
accepting these books as the definitive path to success is (e) Youth Motivation: The Role and Limitations of
recommended. They are not magic solutions or silver bullets, Motivational Books
as they often portray themselvesto be. They should be seen as 115. How does an individual ultimately feel motivated to do
only one element of a broader journey towards self-growth and something?
development. The youth, while inspired by these motivational (a) Understanding that ‘one-size-fits-all’ path towards
reads, should understand that lasting motivation can only be success.
derived from within themselves. It is fuelled by their (b) Through introspection, life experiences and personal
impassioned desires, intrinsic values, and dreams. So, while values.
motivational literature has its place, the youth are encouraged (c) Reading motivational books, but these are only a
to primarily cultivate their own intrinsicmotivation that can temporary source.
withstand the test of time. (d) When an individual does not receive the much needed
108. What is the synonym for ‘sheer’ as used in the passage? guidance.
(a) Obscurity (b) Infinity (e) When motivational books drive the self-ignited
(c) Throw (d) Utter passion among individuals.
(e) Blur 116. According to the passage, why should young readers take
109. According to the passage, what is the complexity a cautious approach to motivational literature?
associated with the convenience of motivational books (a) The books cannot replace practical life experience.
aimed at youths? (b) The books lack credibility and authenticity.
(a) They offer too many success stories. (c) The books might discourage readers from pursuing
(b) They disrupt the reading habits of the youth. their own goals.
(c) They oversimplify the subjective nature of motivation. (d) The books cannot be the definitive path to success.
(d) They are too expensive for young people. (e) The books often lack quality and depth.
(e) They are often outdated.
110. Which of the following is a synonym for the phrase ‘retain’ DIRECTIONS (Qs. 117-123): Fill in the blanks with the
as given in the passage? most appropriate words to complete the passage.
(a) Maintain (b) Disperse In today’s fast-paced world, time management is (117) _____
(c) Relinquish (d) Neglect
(e) Dismiss Tg:- @NextGenBankers
111. Which of the following is mentioned in the passage as a
for success. Whether you are a student or a professional,
(118) _____ the most out of your day can be a challenge.
The first step is to (119) _____ a daily schedule. Start by
challenge in the effectiveness of youth motivational listing your (120) _____ tasks and prioritize them. Allocate
books? specific time slots for each task and stick to your schedule.
(a) The lack of personal testimonies Avoid (121) _____ distractions and stay (122) _____ on your
(b) The broad scope of the contents goals. Remember that (123) _____ planning and discipline
(c) Difficulty in sustaining the inspired motivation can help you make the most of your time.
(d) The language used is too complex for the youth 117. (a) crucial (b) irrelevant
(e) The books are not accessible enough
(c) ineffective (d) optional
112. What is an antonym for ‘essential’ as mentioned in the
(e) necessary
passage?
118. (a) maximizing (b) minimizing
(a) Invigorate (b) Sheer
(c) optimizing (d) ignoring
(c) Seldom (d) Intrinsic
(e) sulking
(e) Indictment
119. (a) ignore (b) create
113. Based on the passage, which of the following statements
is correct? (c) eliminate (d) cherish
(i) Motivational books potentially serve as a start (e) build
towards self-growth and development. 120. (a) favourite (b) essential
(ii) Motivational literature for youth counsels a one-size- (c) random (d) unnecessary
fits-all method to success. (e) boring
(iii) The motivational surge from these books is often 121. (a) enticing (b) eliminating
short-lived. (c) embracing (d) ignoring
(a) Only i (b) i and ii (e) unnecessary
(c) ii and iii (d) i and iii 122. (a) focused (b) distracted
(e) i, ii and iii (c) relaxed (d) anxious
114. Which of the following would be an appropriate title for (e) aimed
the passage? 123. (a) haphazard (b) strategic
(a) The Unseen Dangers of Motivational Literature (c) random (d) spontaneous
(b) Dissecting Motivational Literature for Youth (e) good
(c) The Ineffectiveness of Self-help Books
2023-14 SBI Clerk (Junior Associates) Mains Solved Paper-2023
DIRECTIONS (Qs. 124-127): In the following questions, 131. Activists in the country have been _________ since
there are some errors in every sentence. Find the part of the ages about its _________ society that prohibits women
sentence which has an error. That particular part is your from travelling, marrying or attending college without
answer. If there is no error, your answer is (d), i.e., No error. __________ from a male member of their family, also
known as the guardian according to the law.
124. She is a qualified doctor (a)/ but none of her daughters (a) denying, rigid, permit
(b)/ pursue the medical field. (c)/ No error (d) (b) fighting, strong, restrictions
(a) She is a qualified doctor (c) proposing, lenient, allowance
(b) but none of her daughters (d) curious, patriarchal, permitting
(c) pursue the medical field (e) protesting, patriarchal, permission
(d) No error 132. The new world driven by ___________ changes is
125. After knowing the truth, (a)/ he moved ahead (b)/ and seeing a __________ of startups who are questioning
made the right choice. (c) the status quo and ___________ industries.
(a) After knowing the truth (a) automatically, upsurge, disorganized
(b) he moved ahead (b) robotic, decline, Indian
(c) and made the right choice (c) technological, proliferation, disrupting
(d) No error (d) mechanical, enlargement, organized
126. Reading and writing (a)/ provides (b)/ nutrition to the (e) patriotism, storehouse, astounding
brain. (c) 133. The ______ of time does not remain the same. The new
(a) Reading and writing (b) provides lamp had _______ for reading. Therefore, he opened
(c) nutrition to the brain (d) No error his books for a ________ understanding of concepts.
127. His brother (a)/ was wet in the rain (b)/ while returning (a) lapse, more, better (b) span, less, clear
from his office. (c) (c) cycle, much, detailed (d) cycle, enough, thorough
(a) His brother (e) span, enough, better
(b) was wet in the rain 134. There cannot be _________ among Indian children
(c) while returning from his office because there are two _______ of children. They get
(d) No error two __________ kinds of education in the country.
DIRECTIONS (Qs. 128-130): In the below given questions, (a) peace, kinds, varied
Tg:- @NextGenBankers
a part of the sentence is bold. There are five alternatives
provided as (A), (B), (C) and (D) which may improve the
sentence. Choose the correct alternative. In case no
(b) equality, classes, different
(c) happiness, types, indirect
(d) peace, dimensions, major
replacement is required, choose (E) as your answer. (e) rights, kinds, limited
135. The government h as failed to provide n ecessary
128. The organization may be hiring for vacancies, but _________ for the __________ of children. Most of
cannot scold freely. them go to a ________ without blackboards.
(a) cannot scold at will (a) facilities, education, school
(b) cannot give umbrage (b) amenities, upliftment, school
(c) cannotjust scold anyway (c) schools, education, place
(d) cannot scold wilfully (d) requirements, well-being, school
(e) No improvement (e) reports, betterment, educational institute
129. Many of the conservationists proclaim to save the
environment against degradation. DIRECTIONS (Qs. 136-138): In the question given below, a
(a) to save the environment well set of sentences is given, which when properly sequenced, form
(b) that they save the environment a coherent paragraph. Arrange the sentences in the correct
(c) to prevent the environment sequence, and answer the questions.
(d) to save the environment P. Among the biggest implications of the coming drop in
(e) No improvement population is the end of plentiful labour.
130. Except for you and I, everyone went for the dinner to Q. China’s population is shrinking faster than expected.
their house. R. According to the Chinese Academy of Social Sciences,
(a) With the exception of you and I the long-anticipated population decline will start in 2027.
(b) Except for I and you S. Over the past 40 years, companies benefited from the
(c) Except for you and me seemingly endless scale of the Chinese workforce.
(d) Everyone except for me and you T. Now, that era is drawing to a close, and companies need
(e) No improvement to prepare for a shrinking talent pool.
DIRECTIONS (Qs. 131-135): In each of the following 136. Which of the following would be the SECOND statement
sentences, there are three blank spaces. There are five after rearrangement?
options and each option consists of three words which will (a) P (b) R
be filled in the blanks to make the sentence grammatically (c) Q (d) S
correct. (e) T
SBI Clerk (Junior Associates) Mains Solved Paper-2023 2023-15
137. Which of the following would be the FIFTH statement (a) Ladakh (b) Himachal pradesh
after rearrangement? (c) Punjab (d) Jammu and kashmir
(a) P (b) R (c) Q (d) S (e) None of the above
(e) T 147. In which state/UT, Pench Tiger Reserve (PTR) is located?
138. Which of the following would be the THIRD statement (a) Gujrat (b) Rajasthan
after rearrangement? (c) Uttar pradesh (d) Madhya pradesh
(a) P (b) R (c) Q (d) S (e) None of the above
(e) T 148. Under the PM-JANMAN, India has a Scheduled Tribe
(ST) population with ______ communities across _____
DIRECTIONS (Qs. 139-140): In the given questions, a states.
word in the sentence is printed in bold. Below the sentence, (a) 75 & 15 (b) 75 & 18
alternatives are provided for the bold part which may help (c) 53 & 68 (d) 63 & 98
improve the sentence. Choose the correct alternative out of (e) 43 & 48
the five given options. 149. In August 2020, COVID-19 stress committee headed by
139. North Korean leader Kim Jong-un met with Chinese (a) Shri Diwakar Gupta (b) Shri T N Manoharan
President Xi Jinpingand reaffirmed his commitment to (c) Shri Ashvin Parekh (d) K.V. Kamath
the peninsula’s denuclearization. (e) None of the above
(a) Emphasized (b) Redundant 150. Regional Rural Banks (RRBs) located in which of the
(c) Forced (d) Imposed following Union Territory (UTs)
(e) Slapped (a) Punjab, Jammu & Kashmir and Ladakh.
140. This special magnetic appeal or charm is reinforce to his (b) Delhi, Jammu & Kashmir and Ladakh.
power and presence in front of an audience. (c) Puducherry, Jammu & Kashmir and Ladakh.
(a) Ornate (b) Opine (d) Madhya pradesh, Jammu & Kashmir and Ladakh.
(c) Paramount (d) Peculiar (e) Jammu & Kashmir and Ladakh.
(e) None of these 151. How much % of Indian population come under 15 – 64 working
age, as per the United Nations Population Fund (UNFPA)
General/Finance Awareness (a) 65% (b) 68% (c) 70% (d) 80%
(e) 90%
141. DLabs at the Indian School of Business (ISB) launched 152. The manipulation of facial appearance through deep
Tg:- @NextGenBankers
“Build for Billions”, a startup accelerator program, in
collaboration with the Reserve Bank Innovation Hub
(RBIH) and which bank?
generative methods called
(a) Line art (b) Deepfake
(c) Fake (d) Duplicate
(a) State Bank of India (e) None of the above
(b) Punjab National Bank 153. What is the inspiration for G20 2023 logo?
(c) Axis Bank (a) India’s national flag (b) Russia’s national flag
(d) Union Bank of India (c) France’s national flag (d) China’s national flag
(e) Bank of Baroda (e) None of the above
142. What is the architectural style of the Ayodhya Ram 154. Who is the first unicorn in 2024?
Mandir? (a) Krutrim (b) BYJU’s
(a) Gothic (b) Neoclassical (c) Swiggy (d) OYO Rooms
(c) Nagara (d) Modernist (e) Dream11
(e) None of the above 155. Eway Bill should be generated when the value exceeds
143. Name the DRDO Multi-Barrel Rocket Launcher (MBRL) systems (a) ` 50,00 (b) ` 50,000
(a) bazooka (b) Pinaka (c) ` 50,0000 (d) ` 70,000
(c) Panzerfaust 3 (d) M-160 Rocket Launcher (e) ` 60,000
(e) None of the above 156. The RBI has given permit to resident individuals to make
144. Which states are in startup ranking 2022? remittances to IFSCs that was established in India under
(a) Gujarat. Karnataka. Kerala. Tamil Nadu. which scheme?
(b) Maharashtra. Odisha. Punjab. Rajasthan. Telangana (a) Liberalized remittance scheme (LRS)
(c) Andhra Pradesh. Assam. Madhya Pradesh. Uttar (b) Money Transfer Service Scheme(MTSS)
Pradesh. Uttarakhand (c) Rupee Drawing agreement(RDA)
(d) Bihar. Haryana (d) Restructuring scheme by RBI
(e) All of the above (e) One Time Settlement Scheme (OTSS)
145. Name the Argentina’s state-owned enterprise that has 157. Which company signed MoU with NPCI to expand UPI
signed an agreement with Khanij Bidesh India Limited globally?
(KABIL) to explore and develop 5 lithium brine blocks (a) Amazon (b) Flipkart
(a) CEMYIN (b) CAMYEN (c) Google Pay India (d) Bharat pay
(c) CEMYAN (d) CIMYEN (e) None of the above
(e) None of the above 158. Windfall tax levied in the form of
146. Hanle as India’s first dark sky reserve, is located in which (a) Special Additional Extra Duty
state/UT? (b) Special Additional Excise Duty
2023-16 SBI Clerk (Junior Associates) Mains Solved Paper-2023
(c) Super Additional Excise Duty 168. Which of the following statements is incorrect with respect
(d) Special Advantage Excise Duty to TReDS?
(e) None of the above (a) It stands for Trade Related Discounting System.
159. Tata Consultancy Services (TCS) is India’s most valuable (b) It is an initiative of the Reserve Bank of India.
brand with a brand value of (c) Its aim is to help cash-starved micro, small and
(a) $50 billion (b) $43 billion medium enterprises (MSMEs) promptly encash
(c) $60 billion (d) $70 billion receivables.
(e) $80 billion (d) It is an online bill discounting platform that enables
160. What is the meaning of ISO IEC 27001? discounting of invoices/bills of exchanges of MSME
(a) International Organization for Steel sellers against large corporate, including Government
(b) International Organization for Standardization departments and PSEs through an auction mechanism
(c) Internal Organization for Standardization at competitive market rates.
(d) International Outlook for Standardization (e) None of the above
(e) None of the above 169. The National Quantum Mission (NQM) was launched for
161. An account becomes inoperative after how many years? how many years?
(a) 3 years (b) 4 years (a) 4 years (b) 8 years
(c) 2 years (d) 5 years (c) 5 years (d) 6 years
(e) 1 years (e) 9 years
162. Which company ranked first among arrangers for Indian 170. State Bank of India (SBI) had raised Rs. ________
offshore loans in 2023? through its second Basel III compliant Additional Tier 1
(a) HDFC (b) HSBC Holdings plc bond issuance for the current financial year.
(c) SBI (d) AXIS (a) `4,000 crore (b) ` 5,000 crore
(e) None of the above (c) `6,000 crore (d) `7,000 crore
163. Which of the following is NOT a classical language of (e) `8,000 crore
India? 171. The transaction limit for UPI payments for Retail Direct
(a) Odia (b) Pali Scheme and IPO subscriptions is upto Rs
(a) 1 lakh (b) 2 lakh
(c) Kannada (d) Telugu
(c) 5 lakh (d) 4 lakh
(e) None of the above
(e) 6 lakh
164.
Tg:- @NextGenBankers
REC Limited issues inaugural Yen Denominated Green
Bonds aggregating to JPY 61.1 Billion for how many
years?
172. The Central Board of Directors of the RBI (Reserve Bank
of India) is appointed/nominated for a period of ______
years.
(a) 4-year (b) 2-year (a) five (b) six (c) three (d) four
(c) 1-year (d) 5-year (e) None of the above
(e) 3-year 173. How much fine issued for coaching centre after the 1 breach
165. Non-Banking Financial Companies (NBFCs) are the of Guidelines for Regulation of Coaching Center 2024?
Financial Intermediaries engaged primarily in the (a) Rs 35,000 (b) Rs 45,000
business of (c) Rs 25,000 (d) Rs 55,000
i. Accepting Deposits (e) Rs 65,000
ii. Lending loans and advances 174. The UDGAM portal to simplify the process for the public
iii. Leasing to locate their _______in various banks through a single
iv. Hire purchasing platform.
(a) i and ii (b) iii and iv (a) Fixed Deposits (b) Unclaimed Deposits
(c) i ad iii (d) i, ii, iii and iv (c) Current Deposits (d) Claim Deposits
(e) None of the above (e) None of the above
166. Based on the recommendations of which Committee was 175. Ayhika Mukherjee is associated with which sports?
the creation of a separate category for NBFCS operating (a) Football (b) Table Tennis
in the microfinance sector (NBFC-MFI) done? (c) Cricket (d) Hockey
(a) Narasimhan Committee (e) None of the above
(b) Malegam Committee 176. Which of the following are the Domestic Systemically
(c) Deepak Parekh Committee Important Insurers (D-SIIs)?
(d) PK Mohanty Committee (a) Life Insurance Corporation of India (LIC), General
(e) None of the above Insurance Corporation of India Limited (GIC Re)
167. Who is the regulatory authority for Merchant Banking (b) Life Insurance Corporation of India (LIC), General
in India? Insurance Corporation of India Limited (GIC Re),
(a) Securities and Exchange Board of India and New India Assurance
(b) Reserve Bank of India (c) Life Insurance Corporation of India (LIC)
(c) Union Ministry of Corporate Affairs (d) Life Insurance Corporation of India (LIC), General
(d) Union Ministry of Finance Insurance Corporation of India Limited (GIC Re),
(e) None of the above and HDFC
(e) None of the above
SBI Clerk (Junior Associates) Mains Solved Paper-2023 2023-17
177. Under the Member of Parliament Local Area (a) ICICI (b) Punjab National Bank
Development (MPLAD) scheme, Rs _________ will be (c) SBI (d) HDFC
available to each Member of Parliament (MP) at the start (e) None of the above
of the financial year. 185. When was the Pension Fund Regulatory and Development
(a) 1 crore (b) 2 crore Authority of India (PFRDA) established?
(c) 5 crore (d) 4 crore (a) 2000 (b) 2005 (c) 2003 (d) 2001
(e) 6 crore (e) None of the above
178. Which Bank has applied to the Monetary Authority of 186. How many times has NASAApollo reached the moon till now?
Singapore for a banking licence? (a) Two missions (b) Six missions
(a) SBI Bank (b) Axis Bank (c) Five missions (d) Four missions
(c) ICICI Bank (d) HDFC Bank (e) Ten missions
(e) Union Bank 187. The Constitution (Scheduled Castes) Order (Amendment)
179. How many banks were nationalized in India on 15th April 1980? Bill, 2023 was passed to include two synonyms for a
(a) 4 (b) 5 (c) 6 (d) 8 community in which state?
(e) 9 (a) Madhya Pradesh (b) Sikkim
180. Which team Netherlands defeated in World Cup 2023? (c) Chhattisgarh (d) West Bengal
(a) India (b) Pakistan (e) Bihar
(c) Bangladesh (d) South Africa 188. Which Union Ministry has set up a working group to
(e) None of the above decriminalise laws to further promote ease of doing
181. Which of the following is not a regulatory body? business?
(a) CCI (b) RBI (c) SIDBI (d) SEBI (a) Ministry of Finance
(e) None of the above (b) Ministry of MSME
182. What is the compensation amount per day of delay in (c) Ministry of Commerce and Industry
releasing documents as per RBI’s recent directions? (d) Ministry of Corporate Affairs
(a) ` 2,000 (b) ` 5,000 (c) `3,000 (d) ` 4,000 (e) None of the above
(e) `6,000 189. India and which bloc reached an agreement to review their
183. Presence of Indian banks increased in overseas through free trade pact for goods by 2025?
________ (a) SAARC (b) BIMSTEC
(a) subsidy
(c) annexation Tg:- @NextGenBankers
(e) None of the above
(b) Subsidiaries route
(d) appropriation
190.
(c) ASEAN
(e) None of the above
(d) G-20
I B (Q¢)
Anup Dinesh So, it is clear that the initial position of Q is in the south
Sachin to the initial position of G.
7. (c)
Clearly, Dinesh is sitting opposite of Deepak C (P ¢)
K
2. (b) After Combining
N ³ K £T £ J £ I £ G = M > L£ H £ F = Z
Conclusions: (P)
I. G > L ® True (G = M > L) J 1 2 A
II. F > L ® False (L £ H £ F ® L £ F) (N)
III. F = L ® False (L £ H £ F ® L £ F)
So, con clusion II and con clusion III form
complementary pairs. (N ¢) I (P ¢¢) B
3.
Conclusions:
Tg:- @NextGenBankers
Hence, only I and Either II or III are True.
(b) H ³ U < K £ L £ T < N = J > I £ G £ M = A So, we know that the final position of Bus 'N' from
initial position of Bus 'P' is in South-West.
I. M > I ® false (I £ G £ M ® I £ M) 8. (c)
II. L < H ® false (H ³ U < K £ L) C
K
III. I = M ® false (I £ G £ M ® I £ M)
So, con clusion I and con clusion III forms
complementary pair. (U¢) (Y¢)
Hence, Either I or III is true. J 1 2 A
4. (c) K £ N < R = T ³ F = U ³ I = H < B = X
Conclusions:
(I) T > H ® False (T ³ F = U ³ I = H ® T ³ H)
(II) R £ U ® True (R = T ³ F =n U) (U)I(Y) B(Y¢¢)
(III) H = T ® False (T ³ F = U ³ I = H ® T ³ H)
The distance between each point is unknown so we
So, coclusion I and III forms complementary pair.
can not exactly tell the initial position of Bus U from
Hence, only II and either I or III are true.
final Position of Bus Y. It can not be determined.
Sol. (5- 8):
9. (a)
(W¢¢¢) Digits 7 7 8 4 4 6 7 4 8 2 8 4
K C Subtract-1 +1 –1 +1 –1 +1 –1 +1 –1 +1 –1 +1 –1
Add 1
Resultant 8 6 9 3 5 5 8 3 9 1 9 3
(W) (W¢) Digit
J 1 2 A
Final 6 8 3 9 5 5 3 8 1 9 3 9
Number
The sum of all the digits of the final number is _____
I B (W¢) = 6 + 8 + 3 + 9 + 5 + 5 + 3 + 8 + 1 + 9 + 3 + 9 = 69
The largest two digit number is = 99
5. (a) So, the answer is North-East. Therefore, the difference between 99 and 69 is = 30
SBI Clerk (Junior Associates) Mains Solved Paper-2023 2023-19
(+) (–) B V
(–) (+) PZ U is the uncle of V
L J (+) Hence, the answer is U%A$B#O+V
X
18. (b) C
(+) (+) (–)
R T V
(–)
N
10. (e) N is the grand daughter of Z B J T
Hence, Z is the grandmother of N.
11. (e) V is seated third to the right of J
Hence, V sits third to the right of J. A K
12. (d) The grandchildren of P are R and N. N is seated
second to the right and R is seated fourth to the right Hence, J is the father of A.
of P. Hence, the grandchildren sit second and fourth 19. (b) I. "White Whale" only sells new books ® The
to the right of P. passage does not provide any information about
whether the bookstore only sells new books or not.
13. (c) The two persons sitting at the two ends of the row
Therefore, we cannot conclude that statement I is
are X and T. They are brothers-in-law.
correct.
Hence, X and T are brothers-in-law.
II. The bookstore’s success is due to its book collection
14. (d) Statement 1: No married couple is seated side by
and cozy atmosphere ® The passage mentions that
side. This is true. There are three married couples in the bookstore has gained popularity due to its
the family and none are seated side by side. "extensive collection of books" and "cozy reading
Tg:- @NextGenBankers
Statement II: R and N are cousins. This is true.
Statement III: Z is mother-in-law of L. This is true.
L is J's wife who is the son of Z.
books and tea shop". Therefore, statement II is
correct.
III. Aakash’s expertise is in management rather than
Hence, none of the statements are false. literature ® The passage states that Aakash, the store
Sol. (15-18): owner, “has studied literature at a prestigious
university”. The passage does not provide any
Symbol in diagrams Meaning information about Aakash’s management skills or
education. Therefore, we cannot conclude that the
Female Male statement III is correct.
Hence, the correct answer is Only II.
Married Single 20. (c) Statement 1: It suggests that the integration of AI-
driven financial advisors may contribute to rein-
15. (d) forcing biased financial practices, aligning the
O B warning in the study.
Statement II: It assumes that financial institutions are
actively working to eliminate biases in AI system used
for decision-making in the financial sector, which is
A N M not explicitly mentioned in the given information.
However, it is a common expectation that institutions
M is being the daughter of B when the symbol of '#' would work towards reducing biases. Therefore, the
between M and B can be palced here. correct answer is C, as both assumptions are implicit
Hence, the option (d) is Correct. and align with the concerns raised in the study about
AI - driven financial advisors.
16. (b) A H N G
B The correct answer is option (c). Both I and II
implicit.
21. (d) The codes for the 21 consonants and 5 vowels of the
English alphabetical series. They are as follows-
W 1. The consonants are coded as the numbers of the even
number series except N. M and I, which are coded
Therefore, only statement (b) is true. with special characters. Therefore, the codes for the
Hence, A is the brother-in-law of g. consonants are-
2023-20 SBI Clerk (Junior Associates) Mains Solved Paper-2023
LETTERS D E P O S I T
CONSONANT CODE CONSONANT CODE
CONSONA NTS C V C V C V C
B 2 N &
(C)/VOW ELS(V)
C 4 P 20
+2 if(V) –4if (C) –4 +2 –4 +2 –4 +2 –4
D 6 Q 22
RESULTA NT Z G L Q O K P
F 8 R % LETTER
G 10 S 24 REA RRA NGEM G K L O P Q Z
ENT
H 12 T 26
J 14 V 28 The second letter from right end is 'Q'. The fourth
letter from the left end is 'O'. Difference between
K 16 W 30 them is 17 – 15 = 2
L 18 X 32 Sol. (25-29):
Floor Person
M @ Y 34 13 VACANT
Z 36 12 S
11 Z
10 X
2. The vowels are coded as the position of letter we get 9 VACANT
by reversing the English alphabetical series. The 8 W
codes for vowels are- 7 Q
6 T
VOWEL MIRROR LETTER CODE 5 VACANT
4 U
A Z 26 3 Y
E V 22 2 V
1 VACANT
I R 18 25. (d) 'Y' lines two floor below vacant floor.
O
U
Tg:- @NextGenBankers
L
F
12
6
26. (a) Number of person between Z and T is three.
27. (c) There are four floor in between W and Y.
28. (c) So, floors S and 1 are vacant, Hence, option(c) is
the correct answer.
The given word is 'BUSINESS'. It is a eight letter 29. (e) All except 'z' lines in even number floor.
30. (a)
word. So the code for it should begin with 'c'. Also
Book
the fourth letter is a vowel. So it should be replaced
by '#' in the code.
Therefore, in accordance with the given set of rules, Mobile
the code for the given word is C2624#&222424 Table Pen Copy
Hence, the code for the word 'BUSINESS', is
'C2624#&222424'. Hence, Only I follows.
22. (a) In accordance with the given set of rules, the code 31. (c)
for the word 'INTERMEDIATE' is C18&26#%
H4 Y4
'22618262622'.
Hence, the second part of the code for the word G3
Q7
INTERMEDIATE is 226182622. S6
23. (b) 'DOCTOR' is coded as 'C61242612%. Hence, the correct answer is all follows.
The sum of the individual digits in the code is _____. 32. (e)
6 + 1 + 2 + 4 + 2 + 6 + 1 + 2 = 24 Man
'ENGINEER' is coded as 'C22&10#&2222%'. Woman
The sum of the individual digits in the code is _____
2 + 2 + 1 + 0 + 2 + 2 + 2 + 2 = 13
The positive difference between these two quantities House
is 24 – 13 = 11
24. (d) The vowels are shifted by two forward places and Car
the consonants are shifted by four backward places Bike
according to the English alphabetical series 8______.
Hence, the correct answer is Only II follows.
SBI Clerk (Junior Associates) Mains Solved Paper-2023 2023-21
33. (a) Inference 1:- II. People of different racial and ethnic origins may have
Christie Brinkley is decision to use an app to track privileges in comparison to the numerous advantages
her kids' Online activity and restrict their screen time one individual receives. Thus, this statement weakens
suggests that she is a responsible parent who cares the argument.
about her children's well-being and safety. Therefore, III. The establishment of a National Committee and the
the inference I is valid. widespread endorsement of the constitutional and
34. (b) According to statement I, India has made a other legal provisions that prohibit discrimination
progressive and landmark change in law that should be supported. This statement does not weaken
recognizes gender equality in India by making it the argument.
possible to use mother’s name in offical documents, IV. The world is now more linked thanks to the internet
which is the effect of the fact that India is bound by and social media. However, some people are stoking
international law to respect and protect such rights racial tension with this technological breakthrough
as per statement II. which does not weakens the argument.
Thus statement I is the effect and II is the V. Our democratic system’s institutional and
cause.Hence, the correct answer is Statement II is administrative structures have been set up by the
the cause and statement I is the effect. government to counter different forms of prejudice.
35. (e) Let’s analyze the possible effects of the new policy this will strengthen the argument.Thus, option II is
mandating students to renew their biometric identity the answer.
cards every six months with physical presence: 39. (d)
Effect I: The policy will have a positive effect on I. The incident showcases a clear violation of the food
both academic performance and engagement, as it blogger’s privacy and dignity, highlighting an
will motivate the students to attend classes regularly, instance of gender-based harassment. Such behavior
interact with their peers and instructors, and take not only infringes on individual rights but also points
advantage of the available resources. ’! to a larger societal issue that demands attention.
Effect II: The policy will have a negative effect on II. The statement implies that incidents like these
both academic performance and engagement, as it contribute to a negative perception of safety for
will create an unnecessary burden and hassle for the women in public spaces, and such occurrences
students, especially for those who live far away or tarnish the reputation of the city. This indicates a
36.
Tg:- @NextGenBankers
have other commitments. ’! Hence, the correct
answer is Both I and II.
(b) To determine which of the suggested courses of
broader impact on the safety perception and
reputation of the city due to incidents like the one
described. Therefore, assumption II is implicit.
action logically follow for addressing the situation Hence, both assumptions I and II are implicit in the
of an increase in waterborne diseases in Area W of given context.
Country H, let’s analyze each course of action: 40. (a) The given statement is-
Course of Action II: To save from waterborne The college authorities have decided to give seven
diseases, a public awareness program should be grace marks in the Mathematics paper to all the
arranged so that people can be aware of using clean second-year students as the performance of these
water. ’! This course of action logically follows for students in Mathematics was below their
addressing the situation. Waterborne diseases are expectations. This mainly implies that a majority of
likely occurring because people in Area W are using the students in second year have not passed their
tap water without filtration. Conducting a public Mathematics paper and giving them grace marks
awareness program would help educate the residents might help them clear their paper.
about the risks associated with using untreated water Considering this, we can infer that:
and promote the importance of using clean water to Assumption I: Majority of the students in second-
prevent waterborne diseases. Hence, the correct year may still fail in Mathematics even after giving
answer is If only II follows. them grace marks. This statement is implicit as those
37. (d) seven grace marks may still not be enough for many
38. (b) The passage talks about effects of racial students of second year to pass their examination as
discrimination faced by the weaker section of people they might have performed very poorly. They might
so that still the weaker section faces racial need more than seven marks to pass the examination.
discrimination, which was believed to be a thing of Hence, only assumption I is implicit.
the past. Thus, the racism faced should be put to an 41. (b)
end. I
J
I. Because of the institutions of our democratic politics
and the increase of literacy, millions of individuals
from the weaker parts of society now have a voice F H G
and empowerment which is positive note and it does
not weaker the statement.
K
Hence, statement alone is sufficient to answer the question.
2023-22 SBI Clerk (Junior Associates) Mains Solved Paper-2023
42. (b) 45. (b) In step V 'CLOO' comes exactly between "AEFS"
and "ERTU".
No. Box Vegetables 46. (c) In Step III "PLAY CLOO ERTU" are in same order.
6 N Onion 47. (b) From the above steps it is clear that PLAY, AEFS
5 and EOT, DGO represents the first two and last two
4 M Garlic elements in Step IV.
3 I Tomato
48. (e) From Statements I and II :
2 K Corn
1 J Potato Salary
(Decreasing Shyam Ravi Kavita Sonu Tanu Sameen Monu
This statement is sufficient to answer. How many Order
boxes are kept below one contains Garlic.
Thus, the salary of only three persons are less than
Hence, the data in statement II alone is sufficient to 'Sonu' Hence. Statement I and II together are
answer the question, while the data in statement I sufficient.
alone is not sufficient to answer the question.
Sol. (43-47): 49. (e)
For the four letter words:- The first word from the four letter
words shifted to the end (arranged from right to left) and the Floor Person
letters are arranged in alphabetical order. 8 Z
For the three letter words:- The letters in words are arranged 7 U
in alphabetical order from the beginning. 6 W
g 5
Input : CALM TRUE COOL SAFE PLA Y CRY LET TOE 4 V
DOG 3 X
Step I: TRUE COOL SAFE PLAY ACLM CRY LET TOE DOG 2 Y
Step II: COOL SAFE PLAY ERTU ACLM CRY ELT TOE DOG
Tg:- @NextGenBankers
Step III: SAFE PLAY CLOO ERTU ACLM CRY ELT EOT
DOG
1 T
Sol. (51-55):
Shops Sold Cost Markup Selling Total Discount
Tricycles price per Price Per Price per Discount per
Tricycle Tricycle Tricycle Tricycle
P 30 `100 `140 `133 `210 `7
Q y = 200 `205 `246 `243 `600 `3
R x = 25 `100 `250 `133 `2925 `117
S 210 `200 `360 `334.29 `5400 `25.71
Total 465 `605 `996 `843.29 `9135 `152.71
200 130
51. (a) Cost price each Tricycle in shop T = ´ 5 =`250 = 250 ´ =`325
4 100
Number of tricycles is shop T = 200 – 120 = 80
So, condition III is not true. 4800
Discount of each Tricycles in shop T = =`60
Markup % of Tricycles is shop T = 25 + 5 = 30% 80
\ Markup price of each Tricycles in shop T \ Selling price of eachTricycle in shop T
SBI Clerk (Junior Associates) Mains Solved Paper-2023 2023-23
BPOs Inbound
Tg:- @NextGenBankers
Outbound Total
So, b + 3 = 60 + 3 = 63
And 33.33% =
1
3
Messages Messages
4
P 55 25 80 So, Outbound messages by Q = 63 × = 84
3
Q 100
Inbound message by Q = 100 – 84 = 16
R 95 25 120 \ 4 : x = 16 : 84 = 4 : 21
S 115 35 150 So, x = 21
\ m2 – 21m + 108 = 0
T 100 120 220
Þ m2 – 12m – 9m + 108 = 0
U 60 100 160 Þ (m – 12) (m – 9) = 0
Þ m = 9, 12
62. (b) Males bounded the inbound message in R Hence, smaller root (a) = 9
95 67. (a) Let monthly income of A and B are 100a and 300a
= ´ 10 = 50 respectively.
19
Females bounded the inbound message in R x
= 98 – 50 = 45 A's expenditure = 100a ´ = ax
100
Males bounded the outbound messages in R
= 70 – 50 = 20 7
B's expenditure = 300a ´ = 175a
\ Females bounded the outbound messages in R 12
= 25 – 20 = 5 According to question,
Hence, required difference = 45 – 5 = 40. 12480 + ax = 100a
Þ (100 – x)a = 12480 – (i)
23
63. (a) Inbound messages by S = 150 ´ = 115 And, ax + 175a = 59020
30 Þ (175 + x)a = 59020 – (ii)
7 By dividing equation (ii) by (i) –
Outbound messages by S = 150 ´ = 35
30 175 + x 59020 227
Total messages made by Q = 100 = =
100 - x 12480 48
Þ 48x + 8400 = 22700 – 227x
SBI Clerk (Junior Associates) Mains Solved Paper-2023 2023-25
=
( 390 - 301)
´ 100 = 22.82%
Þ x = 13, 13 390
So, two equations can make – 76. (b) Let cost price of two chairs are 50x and 90x.
(i) x2 – (13 + 4)x + 13 × 4 = 0 Selling price of chair in which CP is less
Þ x2 – 17x + 52 = 0
124
(ii) x2 – (13 + 8)x + 13 × 8 = 0 = 50x ´ = 62x
Þ x2 – 21x + 104 = 0 100
Hence, equation in option (b) is formed. According to question,
45 106
70. (b) ?´ 8 + ´ 80 = 33 ´ 4 62x + 62x + 2196 = 140x ×
100 100
Þ 124x + 2196 – 148.4x
Þ ? × 8 = 132 – 36
Þ 24.4x = 2196
96 Þ x = 90
Þ ?= = 12 Hence , required difference = 90x – 50x = 40x
8
40 × 90 = `3600
25 ´ 4 2
71. (a) + 16 + ( 8) = ?2 1
5 77. (a) × base × 60 = 2100
Þ ?2 × 20 + 16 + 64 2
2100
Þ ? = 100 = 10 Þ Base = = 70
Sol. (72 – 75): 30
\ Breadth of rectangle = 70cm
Articles Cost - Selling - Marked Discount Profit \ 2(70 + l) = 360
price Price Price Þ 70 + l = 180 Þ l = 110
J – `180 – – – 110
\ Radius of circle = ´ 7 = 35cm
K `240 `306 `360 `54 `66 22
Hence, perimeter of the circle = 2pr
L – – `420 – –
22
M `260 – `390 – – = 2´ ´ 35 = 220 cm
7
2023-26 SBI Clerk (Junior Associates) Mains Solved Paper-2023
56 × 8 – 2 × 84 Sol. (92-96):
Þ 448 – 8192 = – 7744
So, also for y = 9, 10 value will be negative. Day Mobiles Sold
Hence, Quantity I > Quantity II. Monday 48
89. (b) Quantity I : Tuesday 92
P ¾®3 20 Wednesday 60
15 Thursday 80
Q ¾®4 60 (Total capacity) Friday 120
R ¾® 5 12 40
So, time taken by pipes P, Q and R together to fill (1.25 a – a) = 12
100
60 Þ a = 12%
tank = = 1.28 hrs. \ 1.25 a = 15%
47
Quantity II: So, b = 100 – (12 + 23 + 15 + 20) = 30%.
92. (d) Mobiles sold on Monday and Tuesday
P+Q® 8 15 = 48 + 92 = 140
Q + R ® 10 12 Mobiles sold on all the remaining days
8 120 (Total capacity) = 400 – 140 = 260
R + S ® 15 140
Hence, required percentage = × 100 = 53.84%.
4 260
P + S ® 30
So, 2 (P + Q + R + S) = 15 + 12 + 8 + 4 = 39 93. (c) According to question.
120 P + 80 (P – 10) = 9200
39 Þ 120 P + 80P – 800 = 9200
Þ (P + Q + R + S)'s efficiency =
2 Þ 200 P = 10,000
So, time taken by P, Q, R and S together to fill tank Þ P = 50
Hence, total amount earned on Monday
120 240
= = = 6.15 hrs = (50 – 20) × 48 = ` 1440.
39 39 94. (c) Mobiles present in the shop initially on Wednesday
2
Tg:- @NextGenBankers
Hence, Quantity I < Quantity II. = 60 ´
100
66.67
3
= 60 ´ = 90
2
90. (d) Quantity I:
Area of square = 22 × 22 = 484 cm2 Mobiles present in the shop initially on Thursday
Area of 4 isosceles triangles = 484 – 196 = 288 cm2| 100
= 80 × = 80 × 3 = 240
288 33.33
\ Area of 1 triangle = = 72 cm2 Hence, required ratio of unsold mobiles on
4
Wednesday and Thursday = (90 – 60) : (240 – 80)
1 = 30 : 160 = 3 : 16.
\ × side × side = 72
2 95. (d) Let mobiles sold on Sunday is x.
Þ (side)2 = 144
48 + x
Þ side = 12 cm \ = 48 – 8
2
\ Perimeter of triangles = 12 + 12 + 12 2 Þ 48 + x = 40 × 2 Þ x = 80 – 48 = 32
= 24 + 12 × 1.4 = 40.8 cm. Hence, total number of unsold mobiles on Sunday
Quantity II:
Percentage increase in area of rectangle 5
= 32 × = 20.
8
20 ´ 15
= 20 + 15 + = 35 + 3 = 38% 96. (c) Let the mobiles sold on Saturday is x.
100 \ (92 – 48) = 2 × (60 – x)
Hence, Quantity I > Quantity II.
91. (e) P + Q = 24 44
Þ 60 – x = = 22 Þ x = 60 – 22 = 38
Q + R = 12 2
As, 10 ³ Q and 14 ³ (P, Q) ³ 8 and R ³ 2 Hence, number of unsold mobiles on Saturday.
\ Q = 10 15
R= 2 = 38 × = 30
P = 14 19
Quantity I : PQ – 54 97. (b) (J + K + L + M)'s weight = 4a ...(i)
Þ 14 × 10 – 54 Þ 140 – 54 = 86 (J + K + L + N)'s weight = 4 × (a + 14.25)
Quantity II : (P + R) × Q = (14 + 2) × 10 = 4a +57 ...(ii)
= 16 × 10 = 160. From equ (i) and (ii)
Hence, Quantity I < Quantity II. 4a – M + N = 4a + 57
2023-28 SBI Clerk (Junior Associates) Mains Solved Paper-2023
ÞN – M = 57 ...(iii) So, P + Q ® 20 5
And, J + K + N + M = 4 (a + 6.75) = 4a + 27...(iv) 100
From equ (i) and (iv) Q ® 50
2
4a – L + N = 4a + 27 \ P alone complete the work in
Þ N – L = 27 . ..(v)
And, L + M +N = 74×3 = 222 100 100
= =
Þ N – 27 + N –57 + N =222 ( 5 - 2) 3 days.
Þ 3N = 222 + 84 = 306
Þ N = 102 So, statement I alone is sufficient.
\ L = 102–27 = 75 From II:
Let efficiency of Q is 1.
(102 - 75) 150
Hence, required percentage = ´ 100 \ P's efficiency = 1´ = 1.5
75 100
27 \ Total work = 20 × (1+ 1.5) = 50
= ´ 4 = 36%
3 50 100
98. (a) From II: \ P alone complete the work in = =
days
1.5 3
Let present age of father is 4x years. So, statement II alone is sufficient.
1 Hence, either statement I or statement II alone is
\ Present age of son = 4 x ´ = x years. sufficient to answer.
4 101. (a) In the given sentence, only ‘staggered’ will make it
From I and II: grammatically as well as contextually correct.
4x – x = 45 Therefore, Option A is the correct alternative among
Þ 3x = 45 the following as staggered perfectly fits in the blank
Þ x = 15 both grammatically and contextually. The sentences
\ Father's age = 4×15 = 60 years will be formed as:
From I and II: She staggered on trembling legs.
4x + x = 75 She staggered at the sensation, taking in the crumbled
Þ 5x = 75
Þ x = 15
Tg:- @NextGenBankers 102. (d)
world around her.
After making the replacements, the thus formed
\ Father's age = 4 × 15 = 60 years. sentence is “A partial reopening of the economy is
Let father's age is x and Son's age is y years. being proposed, but a lot will depend on the extent
x –y = 45 ...(i) of success in containing the spread of the COVID-
x + y = 75 ...(ii) 19 pandemic in particular areas.”
By solving (i) and (ii) 103. (c) Hire is the correct synonym for engage. Hire means
x = 60 and y = 15 to give somebody a job. While, engage also means
So, Father's age is = 60 years. to give someone a task.
Hence, any two of statements I, II and III will give 104. (e) All parts are correct.
the answer. 105. (d) In the given sentence, only ‘normalcy’ makes it
99. (d) Let three pieces of wire are a,b and c and a > b < c. grammatically as well as contextually correct. The
From I: remaining options do not fit in the context of the
a + b = 35 ...(i) sentence.
So, statement I alone is not sufficient, 106. (a) The most appropriate antonym for ‘voluntary’ is
From II: ‘mandatory’. Voluntary means an action carried out
b + c = 30 ...(ii) as per one’s will. Whereas, mandatory means
So, statement II alone is not sufficient. something that is made necessary to be done.
From I and II: 107. (d) ‘Oparate’ is incorrect here. It should be ‘operate’.
By solving equ (i) and (ii) Therefore, option D is the correct answer choice for
a–c=5 this question.
No other information is given. 108. (d) Utter is the most appropriate synonym for the word,
Hence, statements I and II both are not sufficient to sheer. The meaning of sheer is to emphasize the
answer. degree or amount of something. While, utter refers
100. (c) P + Q ® 20 to something complete or total.
From I : 109. (c) They oversimplify the subjective nature of
25 motivation. It is evident from these lines from the
Q's full work in = = 50 days passage: “Motivation, inherently a personal and
1 subjective experience, is often simplified in these
2 books. Theyoffer a sequence of simplified steps, a
‘one-size-fits-all’ path towards success.”
SBI Clerk (Junior Associates) Mains Solved Paper-2023 2023-29
110. (a) Maintain. The word, ‘retain’ means to hold on to or to new ways of thinking andprovide alternate life
keep something. The most appropriate synonym to perspectives. However, a cautious approach towards
retain is maintain which means to keep something in accepting these books as the definitivepath to success
good condition by checking or repairing is recommended. They are not magic solutions or
continuously. silver bullets, as they often portray themselvesto be.
111. (c) Difficulty in sustaining the inspired motivation. It is They should be seen as only one element of a broader
explained through the following lines: “…Though journey towards self-growth and development.
such guidance mighttemporarily inspire, it rarely Theyouth, while inspired by these motivational reads,
contributes to long-lasting motivation, rendering should understand that lasting motivation can only
them ineffective in the long run. Acentral issue lies be derivedfrom within themselves. It is fuelled by
in the external nature of motivation these books their impassioned desires, intrinsic values, and
attempt to foster. It’s widely accepted thatmotivation dreams.”
should be an intrinsic quality, a self-ignited passion 117. (a) 118. (c) 119. (b) 120. (a) 121. (a) 122. (a)
driving one forward, even in the face of adversity. 123. (b)
This motivation, which develops from introspection, 124. (c) The error is in the third part of the sentence. Since
life experiences, and personal values, cannot be the sentence finishes in past tense, the word
derivedfrom predetermined methods or uniform ‘pursue’ will become ‘pursued’.
strategies.” 125. (b) The error is in the second part of the sentence.
112. (d) Intrinsic. The word, ‘essential’ refers to completely The grammatically correct way of writing the
necessary. Whereas, ‘intrinsic’ is an antonym for sentence will be- “After knowing the truth, he went
essential, meaning belonging to the essential nature ahead and made the right choice.”
of a thing. Therefore, intrinsic is an antonym for 126. (b) The error lies in the second part of the sentence.
essential. Since two nouns are used, the verb will be singular.
113. (e) All three statements are correct as per the passage. The improved sentence will be- “Reading and
It is evident from the following lines of the passage: writing provide nutrition to the brain.”
“Motivation, inherently a personal and subjective 127. (b) The error lies in the second part of the sentence.
experience, is often simplified in these books. The grammatically correct way of writing the
Theyoffer a sequence of simplified steps, a ‘one-size- sentence is- “His brother got wet in the rain while
Tg:- @NextGenBankers
fits-all’ path towards success. Though such guidance
mighttemporarily inspire, it rarely contributes to
long-lasting motivation, rendering them ineffective
128.
returning from office.”
(a) cannot scold at will. Option (A) will replace
correctly as ‘cannot scold at will’ means the higher
in the long run. Acentral issue lies in the external authorities in the organization cannot scold as and
nature of motivation these books attempt to foster.” when they want to as they are not permitted to.
114. (e) Youth Motivation: The Role and Limitations of Options (C) and (D) nearly mean the same as
Motivational Books. The passage talks about option (A) and are pointing towards a direct means
motivational books, how it enhances development of punishing their employees.
as well as limits it. It is clearly evident from these 129. (b) that they save the environment. ‘That’ should be
lines: “Motivation, inherently a personal and used after proclaim in order to make the sentence
subjective experience, is often simplified in these grammatically correct and meaningful. The
books. Theyoffer a sequence of simplified steps, a sentence will now be read as- “Many of the
‘one-size-fits-all’ path towards success. Though such conservationists proclaim that they save the
guidance mighttemporarily inspire, it rarely environment against degradation.”
contributes to long-lasting motivation, rendering 130. (c) Except for you and me. When the word, ‘except’
them ineffective in the long run.” is used, ‘me’ is used with it instead of I, so the
115. (b) Through introspection, life experiences and personal most appropriate form of this phrase will be
values. It is given in the passage through these lines: ‘except for you and me’.
“It’s widely accepted thatmotivation should be an 131. (c) proposing, lenient, allowance. These three words
intrinsic quality, a self-ignited passion driving one will fit appropriately to make the sentence
forward, even in the face of adversity.This grammatically correct.
motivation, which develops from introspection, life 132. (c) technological, proliferation, disrupting. These
experiences, and personal values, cannot be words will be paced appropriately in the blanks
derivedfrom predetermined methods or uniform to attain a grammatically correct sentence.
strategies.” 133. (d) cycle, enough, thorough. These words will fit the
116. (d) The books cannot be the definitive path to success. blanks and complete the sentence to make it appear
This is evident from the following lines in the grammatically correct.
passage: “…These books undoubtedly play their 134. (b) equality, classes, different. The three words will complete
part. They provide practical tips, inspiration from the sentence and make it grammatically correct.
shared stories, and caneven stimulate one’s journey 135. (a) facilities, education, school. These three words
of self-growth and discovery. They expose readers will complete the sentence to make it sound
grammatically correct.
2023-30 SBI Clerk (Junior Associates) Mains Solved Paper-2023
136. (b) The correct rearrangement of sentences will be Q, India and will be one of the world›s highest-located
R, P, S, T. Therefore, sentence R will be the second sites for optical, infra-red, and gamma-ray telescopes.
statement after rearrangement. 147. (d) Pench Tiger Reserve or Pench National Park is one
137. (e) The correct rearrangement of sentences will be Q, of the premier tiger reserves of India and the first
R, P, S, T. Therefore, sentence T will be the fifth one to straddle across two states - Madhya Pradesh
statement after rearrangement. and Maharashtra. The reference to Pench is mostly
138. (a) The correct rearrangement of sentences will be Q, to the tiger reserve in Madhya Pradesh.
R, P, S, T. Therefore, sentence P will be the third 148. (b) Pradhan Mantri Janjati Adivasi Nyaya Maha Abhiyan
statement after rearrangement. (PM-JANMAN) According to the 2011 census, India
139. (a) emphasized. The word, ‘emphasized’ will be an has a Scheduled Tribe (ST) population of 10.45 crore,
appropriate alternative to replace ‘reaffirmed’ that with 75 communities across 18 states and the Union
will improve the sentence. The newly formed Territory of Andaman and Nicobar Islands identified
sentence will be- “North Korean leader Kim Jong- as Particularly Vulnerable Tribal Groups (PVTGs).
un met with Chin ese President Xi Jinping 149. (d) The Reserve Bank of India has set up the K.V.
and emphasized his commitment to the peninsula’s Kamath Committee for restructuring of loans
“denuclearization.” impacted by the Covid-19 pandemic. As per the
140. (c) paramount. The word, ‘paramount’ will be an Supreme Court’s order, the Kamath panel report was
appropriate alternative to replace ‘reinforce’ that released on September 7, 2020, by the central bank
will improve the sentence. The newly formed as a special window under the Prudential Framework
sentence will be- “This special magnetic appeal or on Resolution of Stressed Assets. The committee was
charm is paramount to his power and presence in headed by K.V. Kamath and its main objective was
front of an audience.” to recommend parameters for one-time restructuring
141. (d) DLabs at the Indian School of Business (ISB) has of corporate loans.
announced the initiation of ‘Build for Billions’, a 150. (c) RRBs are sponsored by Public Sector Banks (PSBs).
startup accelerator program centered around financial There are currently 43 RRBs supported by 12 public
inclusion for the informal economy. This innovative sector banks with 21,856 branches across 26 states
program has been launched in partnership with the and 3 Union Territories — Puducherry, Jammu &
Reserve Bank Innovation Hub (RBIH) and Union Kashmir and Ladakh.
Bank of India.
Tg:- @NextGenBankers
142. (c) The Ram Mandir, which is being supervised by the
Shri Ram Janmabhoomi Teerth Kshetra Trust, is
151. (b) India’s population offers a significant advantage in
terms of a large workforce, which can help drive
economic growth. India’s 68 % population are in the
designed in the Nagara style of architecture by 15 to 64 years age group, providing a significant
Chandrakant Sompura and his son Ashish Sompura. contribution to the working or able-to-work
The temple will have a length of 360 feet, a width of population.
235 feet, and a height of 161 feet. 152. (b) Deepfakes are synthetic media that have been
143. (b) Pinaka multi-barrel rocket launcher (MBRL) was digitally manipulated to replace one person’s likeness
developed by Armament Research and Development convincingly with that of another. It can also refer to
Establishment (ARDE), an important arm of Defence computer-generated images of human subjects that
Research and Development Organisation (DRDO). do not exist in real life.
144. (e) States’ Startup Ranking 2022 153. (a) The G20 Logo draws inspiration from the vibrant
• Best performer. Gujarat. Karnataka. Kerala. Tamil colours of India’s national flag – saffron, white and
Nadu. green, and blue. It juxtaposes planet Earth with the
• Top Performer. Maharashtra. Odisha. Punjab. lotus, India’s national flower that reflects growth
Rajasthan. Telangana. amid challenges. The Earth reflects India’s pro-planet
• Leader. Andhra Pradesh. Assam. Madhya Pradesh. approach to life, one in perfect harmony with nature.
Uttar Pradesh. Uttarakhand. Below the G20 logo is “Bharat”, written in the
• Aspiring leader. Bihar. Haryana. Devanagari script.
• Emerging Ecosystems. Chhattisgarh. Delhi. Jammu 154. (a) Krutrim announced that it has raised $50 million in
and Kashmir. a funding round led by investment firm Matrix
145. (b) Ministry of Mines, Government of India has achieved Partners India and other backers. The funding
a significant milestone with the signing of an equates to a valuation of $1 billion, making Krutrim
agreement between Khanij Bidesh India Limited the first company this year to attain unicorn status,
(KABIL) and the state-owned enterprise of according to Indian media.
Catamarca province of Argentina CATAMARCA 155. (b) As per the e-way bill rules, e-way bill is required to
MINERA Y ENERGÉTICA SOCIEDAD DEL be carried along with the goods at the time of
ESTADO (CAMYEN SE) at Catamarca, Argentina transportation, if the value is more than Rs. 50,000/
146. (a) The Dark Sky Reserve will be located at Hanle -. Under this circumstance, the consumer can get the
village in Eastern Ladakh as a part of Changthang e-way bill generated from the taxpayer or supplier
Wildlife Sanctuary. It will boost Astro-tourism in 156. (a) The RBI has decided to permit resident individuals
SBI Clerk (Junior Associates) Mains Solved Paper-2023 2023-31
to make remittances to IFSCs established in India 166. (b) The creation of a separate category for NBFCs
under the liberalized remittance scheme (LRS). This operating in the microfinance sector (NBFC-MFI)
scheme allows resident individuals to send a certain was done based on the recommendations of the
amount of money during a financial year to another Malegam Committee. The Malegam Committee was
country for investment and expenditure. An IFSC appointed by the Reserve Bank of India (RBI) in
caters to customers outside the jurisdiction of the 2010 to review the microfinance sector in India and
domestic economy. provide recommendations for its growth and
157. (c) Google Pay In dia sign s MoU with NPCI development. The committee recommended the
International to expand UPI to countries beyond creation of a separate category of NBFCs for
India. Google Pay is a popular mobile payments app microfinance to regulate and supervise them
in India. The free app allows you to send and receive effectively, and to ensure that they comply with the
money and also pay utility bills on the go. prescribed norms and standards. The NBFC-MFIs
158. (b) The government has hiked windfall tax on are required to obtain a certificate of registration from
domestically produced crude oil to Rs 4,600 per the RBI and comply with the regulations on interest
tonne from Rs 3,300 per tonne with effect from rates, lending practices, borrower protection, and
reporting requirements.
Friday. The tax is levied in the form of Special
Additional Excise Duty (SAED). 167. (b)
159. (b) Tata Consultancy Services (TCS) has retained the 168. (a) Trade Receivables e-Discounting System (TReDS)
top spot as India’s most valuable brand, with a value is the institutional mechanism for facilitating the
of $43 billion, according to Kantar’s BrandZ India financing of trade receivables of MSMEs from
corporate and other buyers, including Government
ranking. “TCS remains a strong brand as it continues
Departments and Public Sector Undertakings
to have a very strong corporate reputation.
(PSUs), through multiple financiers.
160. (b) ISO/IEC 27001 is the international standard for
169. (b) National Quantum Mission
information security. It sets out the specification for
an effective ISMS (information security management • It’ll be implemented by the Department of Science
& Technology (DST) under the Ministry of Science
system). ISO 27001’s best-practice approach helps
& Technology.
organisations manage their information security by
• The mission planned for 2023-2031 aims to seed,
addressing people, processes and technology.
161. (c)
Tg:- @NextGenBankers
A savings as well as current account should be treated
as inoperative / dormant if there are no transactions
nurture, and scale up scientific and industrial R&D
and create a vibrant & innovative ecosystem in
Quantum Technology (QT).
in the account for over a period of two years. The • With the launch of this mission, India will be the
accounts which have not been operated upon over a seventh country to have a dedicated quantum
period of two years should be segregated and mission after the US, Austria, Finland, France,
maintained in separate ledgers. Canada and China.
162. (b) HSBC Holdings Plc ranked first among arrangers Salient features of NQM:
for Indian offshore loans last year, toppling Japanese • It will target developing intermediate scale
lenders from the top spot they›d held since 2020. quantum computers with 50-100 physical qubits
Data compiled Bloomberg show the British bank was in 5 years and 50-1000 physical qubits in 8 years.
the bookrunner on roughly $4 billion of US-currency 170. (b) State Bank of India (SBI) said it had has raised
transactions in 2023. `5,000 crore through its second Basel III compliant
163. (b) Pali is not a classical language of India. It is an Additional Tier 1 bond issuance for the current
ancient language. Currently, there are 6 languages financial year at a coupon rate of 8.34%
that have been accorded this status: Sanskrit, Telugu, 171. (c) In December 2021, the transaction limit for UPI
Tamil, Odia, Malayalam, and Kannada. As per the payments for Retail Direct Scheme and for IPO
requirements set by the Government of India in 2004, subscriptions was increased to ` 5 lakh.
a language is accorded the status of “Classical 172. (d) The Central Board of Directors of the RBI (Reserve
Language of India” if it fulfils the following criteria: Bank of India) is appointed/nominated for a period of
It should have a recorded history of existing for over 4 years. The Reserve Bank of India was established on
a period of 1500-2000 years. Should be original and April 1, 1935, in accordance with the provisions of the
not borrowed from any other language Should have Reserve Bank of India Act, 1934. The Central Office
ancient text/literature which is considered heritage. (Headquarters) of the Reserve Bank was initially
164. (d) REC Limited, a Maharatna Central Public Sector established in Kolkata but was permanently moved to
Enterprise under the Ministry of Power, has Mumbai in 1937. Though originally privately owned,
successfully issued its inaugural Japanese Yen (JPY) since its nationalization in 1949, the Reserve Bank is
61.1 billion 5-year, 5.25-year and 10-year Green fully owned by the Government of India.
bonds, issued under its US$ 10 billion Global 173. (c) The first violation will cost the coaching center a
Medium Term Notes Programme. penalty for `25,000, while the second offense can
165. (d) cost a penalty up to Rs. 1 lakh. The subsequent
breaches may involve registration revocation.
2023-32 SBI Clerk (Junior Associates) Mains Solved Paper-2023
174. (b) Balances in savings / current accounts which are not foreign branches and subsidiaries, respectively,
operated for 10 years, or term deposits not claimed during 2022-23.
within 10 years from date of maturity are classified 184. (b) Indian Renewable Energy Development Agency Ltd.
as “Unclaimed Deposits”. These amounts are (IREDA) and Punjab National Bank (PNB) have
transferred by banks to “Depositor Education and signed a Memorandum of Understanding (MoU)
Awareness” (DEA) Fund maintained by the Reserve aimed at advancing renewable energy initiatives
Bank of India. across the nation.
175. (b) Ayhika Mukherjee (born 10 June 1997) is an Indian 185. (c) Pension Fund Regulatory and Development
table tennis player from Naihati, West Bengal. She Authority (PFRDA) is a pension regulator of India.
was part of the Indian team for the 2018 Asian Games It headquartered in New Delhi. It was established
and 2022 Asian Games. She, along with Sutirtha by the Government of India on 23rd August 2003
Mukherjee, won the bronze medal for India in Ami is to promote old age income security by
women›s doubles table tennis in the Asian Games. establishing, developing and regulating pension
176. (b) India’s Insurance Regulatory and Development funds. It administers and regulates National Pension
Authority (Irdai) has declared three state-run System (NPS) and also administers Atal Pension
insurers, Life Insurance Corporation of India (LIC), Yojana (APY).
General Insurance Corporation of India (GIC), and 186. (b) Between 1968 and 1972, crewed missions to the
New India Assurance, as Domestic Systemically Moon were conducted by the United States as part
Important Insurers (D-SIIs). of the Apollo program. Apollo 8 was the first crewed
177. (c) Under the scheme, each MP has the choice to suggest mission to enter orbit in December 1968, and it was
to the District Collector for works to the tune of Rs. followed by Apollo 10 in May 1969. Six missions
5 Crores per annum to be taken up in his/her landed humans on the Moon, beginning with Apollo
constituency. The Rajya Sabha Members of 11 in July 1969, during which Neil Armstrong
Parliament can recommend works in one or more became the first person to walk on the Moon. Apollo
districts in the State from where he/she has been 13 was intended to land; however, it was restricted
elected. to a flyby due to a malfunction aboard the spacecraft.
178. (d) India’s HDFC Bank seeks expansion into Singapore, All nine crewed missions returned safely to the Earth.
applying for a banking license with the Monetary 187. (c) In the Lok Sabha, the Constitution (Scheduled
Tg:- @NextGenBankers
Authority of Singapore (MAS). In a strategic move,
HDFC Bank Ltd, India›s largest private sector lender,
Castes) Order (Amendment) Bill, 2023 was
introduced to make changes to the Constitution
is actively pursuing its first branch in Singapore. (Scheduled Castes) Order, 1950.
179. (c) Six Indian banks were nationalized on 15th April The purpose of this amendment is to include two
1980. Nationalization is the transfer of ownership synonyms for the Mahar community in the State of
and management of an undertaking from private Chhattisgarh in its Scheduled Castes list.
hands to the states. Banks were nationalized in India 188. (c) The Ministry of Commerce and Industry has set up
through an ordinance passed in the year 1969. Indira a working group to decriminalise laws to further
Gandhi was the prime minister who nationalized promote ease of doing business.
banks in India. The group comprised representatives from industry
180. (c) The win against Bangladesh was the second victory associations, business chambers, legal professionals,
for the Netherlands in the 2023 World Cup after and officials of seven ministries. It would also have
inflicting a shock defeat to South Africa earlier in representatives of the National Housing Bank,
the group stages. NABARD and CPCB.
181. (c) RBI :- RBI is India’s central bank and regulatory 189. (c) India and the ASEAN countries reached an
body under the jurisdiction of ministry of finance. agreement to review their free trade pact for goods
EBI :- SEBI was first established in 1988 as non- and set a 2025 timeline for concluding the review to
statutory body for regulating the securities market. address the asymmetry in bilateral trade.
CCI :- CCI is the sole quasi-judicial and regulatory The decision came at a meeting of economic
body established under the competition Act, 2002. ministers from the two sides in Indonesia. The
Therefore RBI , CCI , SEBI are regulatory body but ASEAN-India Trade in Goods Agreement (AITIGA)
SIDBI is not a regulatory body. was signed in 2009.
182. (b) In case of delay in releasing documents, the bank/ 190. (a) The Government of India and the Government of New
NBFC has to pay a fee to the borrower – ¹ 5,000 for Zealand have signed a Memorandum of Understanding
each day of delay. 6. If there is a loss/damage to (MoU) to boost cooperation in civil aviation.
documents, the lender must assist you in getting the This will cover the scheduling of new routes, code
duplicate/certified copies by covering the extra share services, traffic rights and capacity entitlement.
charges. An Air Services Agreement was signed between New
183. (b) Indian banks increased their overseas presence Zealand and India in 2016. The New Zealand and
through the subsidiaries route and their employee India have reviewed the existing arrangements
strength rose by 0.5 per cent and 6.2 per cent for relating to air service between the two countries.
SBI Clerk (Junior Associates) Prelim Solved Paper-2023 2023-33
16. If C doesn't have the least number of colors, then who has (a) None of the statements are correct
the least number of colors? (b) Only Statements II and III are correct
(a) A (b) D (c) C (d) F (c) Only Statement II is correct
(e) E (d) Only Statements I and II are correct
17. How many persons have more colors than E? (e) All the Statements are correct
(a) Four (b) Five 25. How many boxes are kept between the box kept sixth
(c) Two (d) Cannot be determined from the top and the box kept 2 boxes above Box X?
(e) Three (a) 2 boxes (b) 5 boxes (c) 1 box (d) 4 boxes
(e) No boxes
DIRECTION (Qs. 18-22): Read the following information 26. Four of the five options given below follow a certain
carefully and answer the questions given below. logic and form a group. Identify the odd one out.
There are 7 persons – G, H, I, J, K, L and M – attended a seminar (a) Box M (b) Box N (c) Box O (d) Box X
on different days among Monday, Tuesday, Wednesday, (e) Box Z
Thursday, Friday, Saturday and Sunday, Starting the week from 27. If all the boxes are rearranged in alphabetical order from
Monday but not necessarily in the same order. (Assume that the buttom to the top, which box will be placed third from
they attended in the same month of the year.) the top?
Three persons attended between M and I. Neither M nor I is (a) Box O (b) Box N
either the first or the last person to attend the seminar. Number (c) Box P (d) Box X
of persons who attended before G is the same as the number of (e) Box Z
persons who attended after J. G attended after I but immediately DIRECTION (Qs. 28-32): Study the following information
before K. H did not attend the seminar after L. carefully to answer the given questions.
18. Which of the statements is true?
(a) L attended on Monday. Ten people are sitting in two parallel rows of five each such that
(b) Only two persons attended before G. each person of one row is facing the other person of the other
(c) I attended before H. row. G, H, I, J and K sit in Row 2 facing North, while M, N, O, P
(d) J attended after Friday. and Q sit in the Row 1 facing South. Q sits in the middle of the
(e) All the above row and is immediate left of the person who sits opposite J. I
19. Who attended the seminar on Thursday? sits at an end of the row and is sitting opposite to N. O sits to
(a) I
(e) L
Tg:- @NextGenBankers
(b) J (c) K (d) M the immediate left of Q. J sits exactly is between G and H. P. sits
at an end of the row. H sits to the right of J.
28. Who sits in the middle of Row 2?
20. On which day does H attended?
(a) Sunday (b) Monday (a) I (b) G (c) J (d) H
(c) Tuesday (d) Wednesday (e) K
(e) Cannot be determined 29. How many people sit between the one who sits beside G
21. In a certain way I is related to K, and G is related to J. and the one who sits at the right end of Row 2?
Following the same – A is related to M? (a) 1 person(b) 2 people (c) 3 people (d) 4 people
(a) I (b) K (c) J (d) G (e) 5 people
(e) No one 30. Who sits opposite to the one who sits second to the left
22. How many persons attended between G and L? of the one sits beside P?
(a) Three (b) None (c) Two (d) One (a) H (b) G (c) I (d) K
(e) More than three (e) N
DIRECTION (Qs. 23-27): Study the following information 31. Who sits at the end of the row?
carefully to answer the given questions. (a) P and M(b) M and N (c) G and K (d) G and N
(e) I and M
Eight boxes – M, N, O, P, W, X, Y and Z are kept in a room in
32. Who sits diagonally opposite to I?
such a way that each box is stacked on bottom to top of one
(a) M (b) N
another. Box Z is kept below Box O but not immediately. 3 boxes
are kept between Box N and Box Y, which is kept below Box N. (c) O (d) P
Box D is kept 3 places above Box X, which is kept immediately (e) Q
above Box P. Box Z is kept above Box N. Box P is not kept below DIRECTION (Qs. 33-35): In the question below are given
Box M. Box W is kept second from the top of the stack. two statements followed by two conclusons numbered I and II.
23. Which box is kept 5 boxes below the box kept third from You have to take the given statements to be true even if they
the top? seem to be at variance with commonly known facts. Read all
(a) Box N (b) Box O (c) Box W (d) Box Y the conclusions and then decide which of the given conclusions
(e) Box Z logically follows from the given statements disregarding
24. Which of the following statements is /a re correct? commonly known facts.
I. Four boxes are kept between Box N and Box O
II. Box P is kept immediately below Box X 33. Statements :
III. Box Z is kept three boxes above Box M Only a few P is Q.
No Q is R.
SBI Clerk (Junior Associates) Prelim Solved Paper-2023 2023-35
36. Find the wrong term in the following series. Amount invested by Amount invested by
150, 141, 131, 120, 110, 95 Scheme Dinesh (in ` ) Deepak (in ` )
(a) 141 (b) 131 (c) 110 (d) 120 ITC 25,000 40,000
(e) 95 HDFC BANK 20,000 30,000
37. Find the wrong term in the following series.
ICICI BANK 15,000 10,000
50, 72, 98, 128, 164, 200
(a) 50 (b) 72 (c) 164 (d) 200 45. Find the ratio of the total investment of Deepak in ICICI
(e) 98 BANK and ITC together with the total investment of Dinesh
38. Find the wrong term in the following series. in HDFC BANK and ICICI BANK together.
81 118 151 180 197 206 (a) 10 : 7 (b) 11 : 8 (c) 11 : 9 (d) 13 : 19
(e) 11 : 19
(a) 81 (b) 118 (c) 151 (d) 180
(e) 197 46. If the investment of Gagon in HDFC BANK is 2500 more
than the investment of Dinesh in ICICI BANK and the
39. Find the wrong term in the following series.
investment of Gagan in ICICI BANK is 3500 less than the
10 10 20 25 240 1200 investment of Deepak in ITC, then find the average
(a) 10 (b) 20 (c) 25 (d) 240 investment of Gagan in HDFC BANK and ICICI BANK.
(e) 1200 (a) `29000 (b) `28000
40. Find the wrong term in the given series. (c) `26000 (d) `27000
5 8 17 25 37 48 (e) `24000
(a) 17 (b) 5 (c) 37 (d) 8 47. The investment of Dinesh in ICICI BANK and HDFC BANK
(e) 25 together is what percentage more or less than the
41. The ratio of the perimeter to the length of a rectangle is investment of Deepak in HDFC BANK and ITC together?
18:5. If the perimeter of the rectangle is 72 cm, then find the (a) 50% (b) 100% (c) 150% (d) 200%
ratio of breadth of the rectangle to the perimeter of the (e) 250%
rectangle. 48. If the investment of Pawan in ICICI BANK is 50% of the
(a) 1 : 6 (b) 2 : 9 (c) 3 : 5 (d) 4 : 7 investment of Dinesh in HDFC BANK and ITC, then find
(e) 2 : 5 the total investment of Pawan in ICICI BANK.
2023-36 SBI Clerk (Junior Associates) Prelim Solved Paper-2023
(a) `18,500 (b) `14,500 59. What will come in the place of the question mark ‘?’ in the
(c) `13,500 (d) `15,500 following question?
(e) 22,500
49. Find the average of the total investment of Dinesh in HDFC 45 × 18 + 135 × –49 + 89 × 43 = ? + 169
BANK, Deepak in HDFC BANK and Dinesh in ICICI BANK (a) 10159 (b) 11519 (c) 12259 (d) 11219
together. (e) 11239
(a) ` 21666 (b) ` 19566 60. What will come in the place of the question mark ‘?’ in the
(c) ` 17566 (d) ` 23566 following question?
(e) `13566
50. What value will come in the place of the question mark ‘?’ 324 ´ 4 - 40 + 72 = 53 + 65 – ?
in the following question? (a) –125 (b) 109 (c) –117 (d) –115
500 × 18 ¸ 6 + 3 + 2 of 5 of 3 (e) 119
(a) 1516 (b) 1512 (c) 1514 (d) 1510 61. What will come in place of question mark ‘?’ in the following
(e) 1533 question?
51. What value will come in the place of the question mark ‘?’ (952 + 348) ¸ 260 + 142 = ?
in the following question? (a) 284 (b) 272 (c) 292 (d) 282
28 × 7 + 5 × 4 – 56 × 2 + 169 ¸ 13 = ? ¸ 24 (e) 201
(a) 2970 (b) 2897 (c) 2766 (d) 2808 62. What should come in the place of ‘?’ in the following
(e) 2976 question?
52. What value will come in the place of the question mark ‘?’
289 ´ 5 + 25 ´ 20% of 750 – 486 ¸ 3 = ?
in the following question?
(a) 2088 (b) 3415 (c) 3375 (d) 2812
20% of 2000 – 25% of 1000 = 2% of 500 + (?)
(e) 3673
(a) 130 (b) 140 (c) 150 (d) 149
63. What should come in the place of the question mark ‘?’ in
(e) 160
the following question?
53. What value will come in the place of the question mark ‘?’
in the following question? 4 2 2 3
(a) 56
Tg:- @NextGenBankers
760 ¸ 19 + 8 × 5 + 7 – 2 + ? = 156
(b) 60 (c) 34 (d) 71
17
3
¸ 3 + 6 ´ - 10.5% of 110 = ?
(a) 3.15
3 3 2
(b) 3.45 (c) 3.65 (d) 4.25
(e) 65 (e) 4.75
54. What value will come in the place of the question mark ‘?’ 64. What value will come in the place of ‘?’ in the following
in the following question? question?
25 × 400 – 20 × 200 + 100 ¸ 2 = ? × 30 ¸ 3 24% of 840 + 64% of 940 = 32% of ?
(a) 505 (b) 400 (c) 605 (d) 500 (a) 3210 (b) 3010 (c) 2310 (d) 2510
(e) 305 (e) 2810
55. What value will come in the place of the question mark ‘?’ 65. Ratio of milk and water in a mixture of 54 litres is 5 : 4. 18
in the following question? litres of the mixture is taken out from the mixture and then
5 litres of water is added to it. Find the final difference
90 × 2 + 6 + ? + 18 ¸ 3 × 9 = 300
between milk and water.
(a) 96 (b) 100 (c) 120 (d) 110 (a) 2 (b) 4 (c) 3 (d) 1
(e) 60
(e) None of these
56. What will come in the place of the question mark ‘?’ in the
66. A sum is lent at simple interest of 8% p.a. If at the end of
following question?
1105 – 15 × [4 × (140 – 95)] ¸ 5 + 100 = ? three years, the amount received is ` 3,720, then find the
sum lent.
(a) 618 (b) 625 (c) 623 (d) 612
(e) 665 (a) ` 4000 (b) ` 3000
57. What value will come in the place of the question mark ‘?’ (c) ` 5000 (d) ` 5600
in the following question? (e) ` 4400
50% of 180 – 30% of 150 = ? + (500 – 55 × 4) ¸ 8 67. A dealer purchased a washing machine. He allows a
(a) 10 (b) 13 (c) 15 (d) –12 discount of 20% on its marked price and gains ` 3000. If
(e) –13 he allows a discount of 30% on its marked price and gains
58. What will come in the place of the question mark ‘?’ in the ` 2000. Find the marked price of the machine.
following question? (a) ` 7,000 (b) ` 9,000
(842 + 598 – 111) ¸ 3 = 693 – 73 + ? (c) ` 12000 (d) ` 10,000
(a) –165 (b) –196 (c) –177 (d) –162 (e) ` 15,000
(e) None of these
SBI Clerk (Junior Associates) Prelim Solved Paper-2023 2023-37
68. A and B can complete a work in 18 days. If A can complete such as jewelry made of bronze, which is an alloy of copper and
the work in 24 days, then in how many days B alone can tin. The eruption of Mount Vesuvius and its tragic consequences
complete the work? have provided everyone with a wealth of data about the effects
(a) 24 (b) 40 (c) 48 (d) 72 that volcanoes can have on the surrounding area. Today,
(e) None of these volcanologists can locate and predict eruptions, saving lives
69. The ratio of present age of A and B is 3 : 1 and the ratio of and preventing the destruction of other cities and cultures.
present age of A and C is 2 : 5. The difference between the 71. Poisonous ___________ gases were spread in the
present age of A and C is 4 years. 10 years hence, find the atmosphere.
sum of there ages. (a) Hydrogen (b) Nitrogen
(a) 70 (b) 64 (c) 60 (d) 56 (c) Sulfuric (d) Acidic
(e) 68 (e) Non-toxic
70. A train moving at a speed of 45 km/hr crosses a standing 72. Find the synonym of the word, ‘solidify’ from the above
man in 20 seconds. Find the time taken by it to cross a passage.
platform of 200 m. (a) Molten (b) Saturate
(a) 10 s (b) 36 s (c) 12 s (d) 16 s (c) Coagulate (d) Ignite
(e) 20 s (e) Excavate
73. __________ with acrylic paints, scientists examine the
English Language
skeletons, thus concluding the type of diet consumed by
the residents of a particular area.
(a) By strengthening the brittle bones
DIRECTIONS (Qs. 71-78): Read the following passage and
(b) When brittle bones are strengthened
answer the questions given below.
(c) By the combination of brittle bones
Mount Vesuvius, a volcano located between the ancient Italian (d) When one strengthens brittle bones
cities of Pompeii and Herculaneum, has received much attention (e) The strength of brittle bones
because of its frequent and destructive eruptions. The most 74. What is used by archaeologists to clean the skeletons
famous of these eruptions occurred in A.D. 79. covered with volcanic ash when they have to be studied?
The volcano had been inactive for centuries. There was little (a) Fresh water (b) Sodium bicarbonate
warning of the coming eruption, although one account (c) Distilled water (d) Nitrogen gas
Tg:- @NextGenBankers
unearthed by archaeologists says that a hard rain and a strong
wind had disturbed the celestial calm during the preceding night.
(e) Sulfur
75. Bronze is an alloy of ________ and ___________.
Early the next morning, the volcano poured a huge river of (a) Tin and potash (b) Aluminium and copper
molten rock down upon Herculaneum, completely burying the (c) Copper and tin (d) Potash and copper
city and filling the harbor with coagulated lava. (e) Aluminium and sodium
Meanwhile, on the other side of the mountain, cinders, stone 76. Who studies the effect of tidal waves on the world’s
and ash rained down on Pompeii. Sparks from the burning ash climate?
(a) Geologists (b) Archaeologists
ignited the combustible rooftops quickly. Large portions of the
(c) Meteorologists (d) Volcanologists
city were destroyed in the conflagration. Fire, however, was
(e) Scientists
not the only cause of destruction. Poisonous sulfuric gases
77. ___________ can locate and predict volcanic eruptions
saturated the air. These heavy gases were not buoyant in the that can prevent cities from destruction.
atmosphere and therefore sank toward the earth and suffocated (a) Historians (b) Volcanologists
people. (c) Scientists (d) Volcano engineers
Over the years, excavations of Pompeii and Herculaneum have (e) Meteorologists
revealed a great deal about the behavior of the volcano. By 78. Find the antonym for the word, ‘weak’ from the passage.
analyzing data, much as a zoologist dissects an animal (a) Disruptive (b) Yielded
specimen, scientists have concluded that the eruption changed (c) Examined (d) Strengthen
large portions of the area’s geography. For instance, it turned (e) Destruction
the Sarno River from its course and raised the level of the beach
DIRECTIONS (Qs. 79-85): In the following questions, two
along the Bay of Naples. Meteorologists studying these events
words are to be interchanged to be placed in the other sentence
have also concluded that Vesuvius caused a huge tidal wave to make it grammatically correct. Choose the appropriate
that affected the world’s climate. word to swap to improve the sentence.
In addition to making these investigations, archaeologists have
been able to study the skeletons of victims by using distilled 79. Parents improve (A) their children by overindulgence (B)
water to wash away the volcanic ash. By strengthening the and granting their wishes. Let us try and spoil (C) the
brittle bones with acrylic paint, scientists have been able to situation by discussing (D) positive measures.
examine the skeletons and draw conclusions about the diet and (a) A-C (b) B-D
habits of the residents. Finally, the excavations at both Pompeii (c) A-D (d) C-B
and Herculaneum have yielded many examples of classical art, (e) B-A
2023-38 SBI Clerk (Junior Associates) Prelim Solved Paper-2023
80. The contents (A) of the letter surprised (B) them. But he 90. They took blankets ______ keep themselves warm.
was alarmed (C) upon seeing (D) his son. (a) To (b) For
(a) A-D (b) B-D (c) In (d) Were
(c) B-C (d) D-C (e) Have
(e) C-A 91. The tourist who visited Kashmir was __________ by its
81. Trust is the basic quality (A) for all relationships (B). beauty.
His tenet (C) of being punctual in office is quite (a) Mesmerized (b) Enchanted
commendable (D). (c) Awe-struck (d) Shocked
(a) A-C (b) B-C (e) Elated
(c) D-C (d) B-D
DIRECTIONS (Qs. 92-97): The sentences given below have
(e) D-A an error in one of its five parts. Find out which part of the
82. Growing urbanization (A) and literacy damaged (B) the sentence has an error. The alphabet of that part is your answer.
plight of women in India. The car got changed (C) due to
the accident on the street (D). 92. (A) The scissors / (B) is / (C) kept / (D) on top of / (E) the
(a) D-A (b) B-C shelf.
(c) A-C (d) B-A (a) A (b) B
(e) B-D (c) C (d) D
83. Not all young (A) children are serious (B). They were (e) E
quite naughty (C) at the Board meeting (D). 93. (A) She enrolled / (B) herself in a /(C) three-years degree
(a) A-D (b) B-D / (D) course in / (E) Fashion Designing.
(c) B-C (d) C-A (a) A (b) B
(e) D-C (c) C (d) D
84. My friends (A) told me that there was no action (B) out. (e) E
They took no way (C) against the caught (D) culprit. 94. (A) Three requirements / (B) necessary for / (C) plant
(a) B-C (b) D-A growth is / (D) soil, temperature / (E) and moisture content.
(c) B-D (d) C-A (a) A (b) B
(c) C (d) D
85.
(e) A-B
Tg:- @NextGenBankers
The Bhagvad Gita is a poem (A) of 700 paragraphs (B).
The verses (C) of that story were well connected (D). 95.
(e) E
(A) He purchased / (B) some new / (C) and innovative /
(a) A-B (b) B-C (D) equipments / (E) for his company.
(c) A-C (d) C-D (a) A (b) B
(e) C-B (c) C (d) D
(e) E
DIRECTIONS (Qs. 86-91): In the following questions, there 96. (A) The man / (B) laid down / (C) besides me / (D) on the
is a blank in each sentence which has to be filled with an bench / (E) in the park.
appropriate word. Five options have been given. Choose the (a) A (b) B
correct one to fill the blank and complete the sentence.
(c) C (d) D
86. The snake _______ the man on his toes. (e) E
(a) Had bitten (b) Bit 97. (A) My brother / (B) was / (C) first / (D) to get / (E) a
(c) Bite (d) Was bitten degree.
(e) Was bit (a) A (b) B
87. During the recent hailstorm, one-third of our city _____ (c) C (d) D
devastated. (e) E
(a) Were (b) Are
DIRECTIONS (Qs. 98-100): In the following questions, options
(c) Was (d) Is
A and G are the first and the last sentences of a paragraph.
(e) Will be Rearrange the five sentences in a logical sequence and answer
88. Proper grammar, punctuation and spellings ______ the questions that follow.
essential in a sentence.
(a) Were (b) Have A Branded disposable diapers are available at many
(c) Is (d) Are supermarkets and drug stores.
(e) Can be i. If one supermarket sets a higher price for a diaper,
89. The roads ________ the state of Punjab are largely raised customers may buy that brand elsewhere.
and smooth. ii. By contrast, the demand for private-label products
(a) Of (b) Throughout may be less price-sensitive since it is available only
(c) In (d) From at a corresponding supermarket chain.
(e) Are
SBI Clerk (Junior Associates) Prelim Solved Paper-2023 2023-39
iii. So the demand for branded diapers at any particular (iv) For instance, only SavOn Drugs stores sell SavOn
store may be quite price sensitive. Drugs diapers.
iv. For instance, only SavOn Drugs stores sell SavOn Then stores should set a higher incremental margin
Drugs diapers. percentage for private label diapers.
v. These diapers are known to be water absorbent to a 98. What will be the third sentence after rearrangement?
high extent. (a) iii (b) i
G. Then stores should set a higher incremental margin (c) ii (d) iv
percentage for private label diapers. (e) v
The correct rearrangement is as follows: 99. What will be the second sentence after rearrangement?
Branded disposable diapers are available at many (a) iii (b) v
supermarkets and drug stores. (c) i (d) ii
(v) These diapers are known to be water absorbent to a (e) iv
high extent. 100. What will be the fifth sentence after rearrangement?
(iii) So the demand for branded diapers at any particular (a) i (b) iii
store may be quite price sensitive. (c) ii (d) iv
(i) If one supermarket sets a higher price for a diaper, (e) v
customers may buy that brand elsewhere.
(ii) By contrast, the demand for private-label products
may be less price-sensitive since it is available only
at a corresponding supermarket chain.
ANSWER KEY
1 (b) 11 (c) 21 (b) 31 (d) 41 (b) 51 (d) 61 (e) 71 (c) 81 (a) 91 (a)
2 (c) 12 (d) 22 (a) 32 (d) 42 (c) 52 (b) 62 (e) 72 (c) 82 (b) 92 (b)
3 (d) 13 (c) 23 (d) 33 (a) 43 (c) 53 (d) 63 (b) 73 (a) 83 (c) 93 (c)
4
5
6
(b)
(d)
(b)
14
15
16
Tg:- @NextGenBankers
(e)
(b)
(e)
24
25
26
(c)
(a)
(b)
34
35
36
(a)
(e)
(c)
44
45
46
(c)
(a)
(d)
54
55
56
(c)
(e)
(e)
64
65
66
(d)
(d)
(b)
74
75
76
(c)
(c)
(c)
84
85
86
(a)
(b)
(b)
94
95
96
(c)
(d)
(c)
7 (b) 17 (d) 27 (d) 37 (c) 47 (b) 57 (a) 67 (d) 77 (b) 87 (c) 97 (c)
8 (c) 18 (b) 28 (d) 38 (d) 48 (d) 58 (c) 68 (d) 78 (d) 88 (d) 98 (a)
9 (e) 19 (c) 29 (b) 39 (c) 49 (a) 59 (e) 69 (b) 79 (a) 89 (b) 99 (b)
10 (b) 20 (b) 30 (d) 40 (e) 50 (e) 60 (b) 70 (b) 80 (c) 90 (a) 100 (c)
2023-40 SBI Clerk (Junior Associates) Prelim Solved Paper-2023
M
(+)
NTg:- @NextGenBankers
(–) Monday
Tuesday
H
I
Wednesday G
Hence, L is married to a girl names O, then L is the
Thursday K
uncle of N.
Friday J
7. (b) Hence, the relationship between J and M is father
Saturday M
and son.
8. (c) Hence, N is niece of L is the correct statement. Sunday L
Sol. (9 - 13): 18. (b) Hence only two persons before 'G' is true.
On decoding the words 19. (c) Hence, K attended the seminar on Thursday.
20. (b) Hence, H attended on Monday.
Words Code
21. (b) I attended one day before K. Similarly, G attended
city pr
one day before J.
bus hr
Here, K attended one day before M
on ql/oz
22. (a) Hence, three persons attended between G and L
school oz/ql Sol. (23 - 27)
college nr
S . No Boxes
road yk 8 O
circle fg 7 W
through we 6 Z
navigation sn/qh 5 N
gone qh/sn 4 X
9. (e) The code for 'road' in the given code of language is 3 P
'yk'. 2 M
10. (b) The possible code of 'city college earth' in the given 1 Y
code language is 'pr nr cr'.
SBI Clerk (Junior Associates) Prelim Solved Paper-2023 2023-41
4 P
+37 +33 +27 +19 +9
3 O
2 N –4 –6 –8 –10
1 M
Tg:- @NextGenBankers
Hence, Box X is kept third from the top after the
39. (c)
10 10 20
60
25 240 1200
rearrangement.
Sol. (28 - 32) : ×1 ×2 ×3 ×4 ×5
P M Q O N 40. (e) (2)2 + 1 = 5
(3)2 – 1 = 8
(4)2 + 1 = 17
(5)2 – 1 = 24
G J H K I
28. (d) 'H' sits in the middle of Row 2. (6)2 + 1 = 37
(7)2 – 1 = 48
29. (b) 'J' sits beside 'G'
41. (b) The perimeter of the rectangle is = 72 cm
'I' sits at the right end of Row 2.
Hence, 2 people sit between J and I 72
The length of the rectangle is = ´ 5 = 20 cm
30. (d) 'M' sits beside 'P'. 'O' sits second to the left of 'M'. 18
Hence, 'K' sits opposite to 'O'. 72 - 20 ´ 2
The breadth of the rectangle is =
31. (d) 'G' and 'N' sits at the end of the row. 2
32. (d) 'P' sits diagonally opposite to 'I'. 32
= = 16 cm
Q 2
P R The ratio of breadth to the perimeter of the rectangle
33. (a) = 16 : 72 = 2 : 9
42. (c) The ratio of the amount invested by kamal and Surya
Hence, only conclusion I follows. (40,000 × 12) : (1,00,000 × 6)
Bat Ball 4,80,000 : 6,00,000
4 : 5
34. (a) The amount of profit share of Surya
Book
27000
Hence, only conclusion I follows. = ´ 5 = 15,000
9
2023-42 SBI Clerk (Junior Associates) Prelim Solved Paper-2023
4 2 2 3 5 25
63. (b) 17 ¸ 3 + 6 ´ - 10.5% of 110 = ? 70. (b) The speed of train = 45 ´ = m/sec
3 3 3 2 18 2
55 3 20 3 25
= ´ + ´ - 11.55 = ? The length of train = ´ 20 = 250 m
3 11 3 2 2
? = 5 + 10 – 11.55 The time taken by train to cross a platform of 200 m
? = 3.45 (250 + 200)2 450
64. (d) 24% of 840 + 64% of 940 = 32% of ? Þ Þ ´ 2 Þ 36 sec
25 25
201.6 + 601.6 = 0.32 × ?
71. (c) Sulfuric gases. This has been mentioned in third
303.2 paragraph, “Poisonous sulfuric gases saturated the
?=
0.32 air.”
? = 2510 72. (c) Coagulate. The word is in the last line of the second
paragraph, “completely burying the city and filling
54 the harbor with coagulated lava.”
65. (d) Quantity of milk in a mixture = ´ 5 = 30 litres
9 73. (a) By strengthening the brittle bones. It is the second
54 sentence of the last paragraph of the passage.
Quantity of water in a mixture = ´ 4 = 24 litres 74. (c) Distilled water. It is mentioned in the first sentence
9
of the last paragraph, “In addition to making these
After 18 litres of the mixture is taken out: investigations, archaeologists have been able to
18 study the skeletons of victims by using distilled
Quantity of milk = 30 - ´ 5 = 20 litres water to wash away the volcanic ash.”
9
75. (c) Copper and tin. It is mentioned in the lines of last
18 paragraph, “…the excavations at both Pompeii and
Quantity of water = 24 - ´ 4 = 16 litres
9 Herculaneum have yielded many examples of
After 5 litres of the water is added classical art, such as jewelry made of bronze, which
Tg:- @NextGenBankers
Quantity of water = 16 + 5 = 21 litres
The final difference between milk and water 76. (c)
is an alloy of copper and tin.”
Meteorologists. It is mentioned in the fourth
paragraph, last sentence, “Meteorologists studying
= 21 – 20 = 1 litre
these events have also concluded that Vesuvius
3720 ´ 100 3720 caused a huge tidal wave that affected the world’s
66. (b) The sum lent = = ´ 100 = 3000
100 + 8 ´ 3 124 climate.”
67. (d) According to question 77. (b) Volcanologists. It is mentioned in the last line of the
Let the marked price = `x passage, “Volcanologists can locate and predict
eruptions, saving lives and preventing the
80 70 destruction of other cities and cultures.”
x´ - 3000 = x ´ - 2000
100 100 78. (d) Strengthen. The antonym for the word, ‘weak’ from
the passage is ‘strengthen’. It is given in second
10x
= 1000 sentence of the second paragraph, “By strengthening
100 the brittle bones with acrylic paint, scientists have
x = ` 10,000 been able to examine the skeletons and draw
The marked price of the machine is = `10,000 conclusions about the diet and habits of the
68. (d) A and B can complete a work in = 18 days 4 residents.”
A can complete a work in = 24 days 72 79. (a) This is an example of word swapping in which words
3
in options A and C will be interchanged. The newly
72 formed sentence will be- “Parents spoil their children
B alone can complete the work in = 4 - 3 = 72 days. by overindulgence and granting their wishes. Let us
try and improve the situation by discussing positive
69. (b) The ratio of present age of A, B and C = 6 : 2 : 5
measures.”
The difference between the present age of A and C is
80. (c) These are examples of word swap wherein a word in
4 years.
one sentence fits the second sentence in a more
The age of A = 6 × 4 = 24 years appropriate way. In this question, option (B) i.e.,
The age of C = 5 × 4 = 20 years ‘surprised’ will be replaced by option (C), i.e.,
The sum of their ages = 24 + 20 + 10 + 10 = 64 years ‘alarmed’ and vice-versa. The newly formed
sentences will be- “The contents of the letter alarmed
them. He was surprised upon seeing his son.”
2023-44 SBI Clerk (Junior Associates) Prelim Solved Paper-2023
81. (a) In this question, two words will be swapped to form The grammatically correct sentence will be: ‘The
grammatically correct sentences. Here, option (A) roads throughout the state of Punjab are largely
will be replaced by option (C). The new sentences raised and smooth.’
thus formed will be- “Trust is the basic tenet for all 90. (a) To. The correct preposition to be used is ‘to’.
relationships. His quality of being punctual in office 91. (a) Mesmerized. The word means, to hold somebody’s
is quite commendable.” attention completely.
82. (b) In this question, the options, (B) and (C), i.e., 92. (b) This is an example of spotting errors in a sentence.
‘damaged’ and ‘changed’ will be swapped to form Since some nouns are always used in plural form, so
correct sentences. The newly formed sentences will they always follow a plural verb. Therefore, ‘is’ will
be- “Growing urbanization and literacy changed the be replaced by ‘are’ since scissors is always used in
plight of women in India. The car got damaged due a plural form. So, option (b) has an error.
to the accident on the street.” 93. (c) This is an example of error spotting in sentences.
83. (c) In this question, the words, ‘serious’ (option B) and When nouns indicate measure or length, they remain
‘naughty’ (option C) will be swapped with each other unchanged in form so long as they are followed by
as they will be appropriate in the other sentence. another noun or pronoun. In this sentence, since
The new sentences formed will be- “Not all young the noun ‘degree’ follows the measure or time period,
children are naughty. They were quite serious at the i.e., ‘three years’, hence it will be written as ‘three-
Board meeting.” year degree course’. So, option (c) is the incorrect
84. (a) This is an example of word swapping wherein option part.
(B) will be interchanged with option (C). The two 94. (c) plant growth is. This is the error spotted in the
words that will be interchanged to form appropriately sentence where when there are more than one items,
correct sentences will be ‘action’ and ‘way’. The new ‘are’ will be used. Therefore, option (c) of the given
sentences will be formed as- “My friends told me sentence has an error.
that there was no way out. They took no action against 95. (d) equipments. Since it is an uncountable noun, its plural
the caught culprit.” will remain ‘equipment’. Therefore, the error in option
85. (b) This is an example of word swap in which options (d) is the correct answer.
Tg:- @NextGenBankers
(B) and (C) will be interchanged. In this question,
‘paragraphs’ and ‘verses’ will be swapped with each
other to make meaningfully correct sentences. The
96. (c) besides me. In option (c), beside should be used
instead of ‘besides’ since besides means in addition
to and beside means next to or at the side of. The
new sentences will be- “The Bhagvad Gita is a poem grammatically correct sentence would be read as –
of 700 verses. The paragraphs of that story were well “The man laid down beside me on the bench at the
connected.” park.”
86. (b) Bit. Since the sentence is in present continuous 97. (c) first. ‘The’ should be added before ‘first’ because
tense, ‘bit’ (form of bite) will be used to complete the the sentence is expressing a quality and at times,
sentence. common nouns are used as abstract nouns to express
87. (c) Was. Since the sentence is in past tense, therefore qualities. Therefore, option (C) is the correct answer.
‘was’ will fit in the blank and complete the sentence. 98. (a) iii. It follows from a logical and coherent sequencing
88. (d) Are. Since ‘are’ is the plural form of ‘is’, the of the sentences to form a paragraph.
grammatically correct sentence will be formed as: 99. (b) v. Through a logical sequencing of the sentences,
‘Proper grammar, punctuation and spellings are option (b) will be the correct answer.
essential in a sentence.’ 100. (c) ii. From the above sequence of sentences, the fifth
89. (b) Throughout. Though ‘in’ may be used here, but sentence will be option (c).
‘throughout’ will fit the context more appropriately.
SBI Clerk (Junior Associates) Mains Solved Paper-2022 2022- 1
Reasoning Ability & Computer Aptitude and remaining friends sit at the middle of two adjacent sides of
the table. All the friends face away from the center of the table.
DIRECTIONS (Qs. 1-5): Study the following information S sits third to the right of P, who sits at the middle of one of the
carefully and answer the questions given below: sides of the table. R sits second to the right of S. V sits immediate
right of T. At least one immediate neighbour of T sits on the side
There are three horizontally parallel rows i.e. row 1, row 2 and
row 3. Row 1 is north of row 2, which is in north of row 3. Four of the table. W sits immediate right of U. Friend, who sits second
persons sit on row 1 facing south, four persons sit in row 3 to the right of Q, does not sit at any corner. Both the neighbours
facing north and eight persons sit in row 2 such that first three of W sit at the middle of sides of the table.
persons (from the left end of the row, considering all are facing 6. Who sits immediate right of Q.
north) face north while rest of the five persons face south. (a) S (b) R (c) P (d) W
Persons, who sit in row 1 and row 3 face the persons, who sit on (e) None of these
row 2. The distance between each of the adjacent persons in all 7. Four from the following are similar in a certain way and
the rows is same. The persons at the rightmost seat of row 2 is forms a group. Find out the one who does not belong to
opposite to the rightmost seat of row 3 (considering all are facing that group.
the north direction). And, the leftmost seat of row 2 is opposite (a) R (b) Q (c) U (d) W
to leftmost seat of row 1 (considering all are facing the north (e) S
direction). X faces the person, who sits 3rd to the right of U. Q 8. How many friends sit between Q and T, when counted
sits 2nd to the left of U and adjacent to W. W faces the person, from the right of Q?
who sits 2nd to the right of K. Neither X nor W sits at any of the (a) One (b) Two
Tg:- @NextGenBankers
extreme ends. K faces the person, who sits 2nd to the right of V.
Only one person sits between V and Z who faces R. L sits
(c) Three
(e) None of these
(d) Four
immediate right of M, who doesn’t face K. O and T sit adjacent 9. Which of the following pair sit at the middle of one of the
to each other. S faces Y, who doesn’t sits 2nd to the right of N. sides of the table?
O doesn’t sit immediate right of U. N doesn’t sit in row 3. P (a) T and P (b) P and R
doesn’t face Q. (c) U and P (d) P and S
1. Who among the following faces the person, who sits
(e) None of these
immediate left of M?
10. Who sits opposite to V?
(a) V (b) N (c) U (d) W
(e) None of these (a) R (b) Q (c) S (d) No one
2. Who among the following doesn’t sit in row 2? (e) None of these
I. Q II. V III. P IV. N DIRECTIONS (Qs. 11- 15): Answer the questions based on
(a) I and II (b) II and III the information given below.
(c) I and IV (d) III & IV
(e) II, III and IV Six employees joined the Disha organizaiton in 2019. All are
3. How many persons sit between Y and the person, who sits joined on different dates of either the same or the different month.
immediate right of Q? Each of them already has some working experience. One of them
(a) One (b) Three (c) Four (d) Two joins on 18th July. Less than two employees joined before T,
(e) More than four who has an experience in an even number. The number of
4. Who among the following pairs of persons doesn’t form a employees joined before T is the same as after the one whose
group? experience is 13 years. Q, who does not have an experience in
(a) S, X (b) P, M (c) Y, V (d) Z, L prime number, joined before the one whose experience is 16
(e) T, Q years but after the one whose experience is 5 and 9 years. The
5. Who sits opposite to S. one whose experience is 5 years joins before the one whose
(a) Z (b) V (c) Y (d) O experience is 9 years, who joins on 24th May. S has an experience
(e) None of these
twice that of T’ s experience. The number of employees joining
DIRECTIONS (Qs. 6-10): Answer the questions based on the between the one whose experience is 5 years and Q is the same
information given below. as between the one whose experience is 9 years and S, who
Eight friends P, Q, R, S, T, U, V and W sit around a regular joins in the organization on 27th September. A joins after R but
hexagonal table, such that six of them sit at different corners, before U. The one who joins on 26th July joined before S but
after Q. The one whose experience is 10 years joined after the
2022- 2 SBI Clerk (Junior Associates) Mains Solved Paper-2022
one who joined on 12th January, which is not the joining date of 19. Which of the following combination is true?
T. Q does not join on 15th January. (a) A-Gaya (b) C-Noida
Based on their working experience a foreign company BUD (c) B-Churu (d) F-Dehradun
gives the project with certain conditions: (e) D-Churu
20. Who among the following students is from Gaya?
1. The project is given to those employees who have a
(a) F (b) C (c) B (d) G
minimum experience of 3 years after joining the Disha
(e) None of these
Organization (Calculate their experience on the following
21. How many pairs of letters are there in the word
date: 31st August 2022).
“HEADPHONE’’ which have as many letters between them
2. The project is given to the employees who joins the
organization at first according to the month. (both forward and backward direction) in the word as in
3. If two employees join in the same month then give priority alphabetical series?
to the one who joins the organization first according to date. (a) None (b) One (c) Two (d) Three
11. Which among the following is the correct order of the (e) Four
employees who get the project? DIRECTIONS (Qs. 22-24): In each of the questions below are
(a) RTPQU (b) QUPRS given some statements followed by some Conclusions. You have
(c) URTQP (d) UTRQS to take the given statements to be true even, if they seem to be at
(e) None of these variance from Commonly known facts. Read all the conclusions
12. If the project is given according to only the date of joining and then decide which of the given conclusions logically follows
of the employees, then how many employees remain from the given statements disregarding commonly known facts.
unchanged in their positions?
(a) 1 (b) 3 (c) 2 (d) None (a) If only conclusion I follows.
(e) 4 (b) If only conclusion II follows.
(c) If either conclusion I or II follows.
13. Who among the following employee has a maximum work
experience before joining this organization? (d) If neither conclusion I nor II follows.
(e) If both conclusions I and II follow.
(a) P (b) Q (c) R (d) T
22. Statements:
(e) None of these
Only a few Yellow is Orange.
14. How many employees joined after U?
No orange is Green.
(a) 1
(e) 5
(b) 2
Tg:- @NextGenBankers
(c) 3
25. Statement: F & D, D * X, X $ Z, B * X 33. The question given below consists of two statements
Conclusions: numbered I and II given below it. You have to decide
I. F * B II. X # F III. B*F whether the data provided in the statements are sufficient
(a) None is true (b) Only II is true to answer the question. Read all the statements and give
(c) Only I and II are true (d) Only II and III are true answer.
(e) All are true How many persons are sitting in the row (assuming all are
26. Statement: D * H $ X @ F & M facing north)?
Conclusions: Statement I: S sits 5th from the right end of the row. R
I. M # X II. F # D III. H@M sits 11th from the left end of the row.
(a) None follows (b) Only I is true Statement II: T sits 6th from the left end. R sits 3rd to the
(c) Only III is true (d) Either I or II is true left of the one who is 7th from the right end.
(e) All are true (a) Data in statement I alone is sufficient to answer the
question.
DIRECTIONS (Qs. 27- 29): Study the following information (b) Data in either statement I or statement II alone is
carefully to answer the given questions. sufficient to answer the question
A @ B means A is mother-in-law of B. (c) Data in statement II alone is sufficient to answer the
A % B means A is child of B. question.
A * B means A is parent of B. (d) Data in both statement I and statement II together is
A # B means A is sibling of B. sufficient to answer the question.
A ! B means A is daughter in law of B. (e) Data in statement I and II together is not sufficient to
A & B means A is married to B. answer the question.
27. P#Q!R*S&T, then how Q’s husband is related to T? 34. The question given below consists of three statements
(a) Sister (b) Brother numbered I, II and III given below it. You have to decide
(c) Brother in law (d) Sister in law whether the data provided in the statements are sufficient
(e) None of these to answer the question. Read all the statements and give
answer.
28. A % B * C & D ! E, then how C is related to E?
Seven persons P, Q, R, S, T, U and V live on different
(a) Daughter in law (b) Son in law
floors of a seven storey building, where the bottommost
(c) Daughter (d) Son
(e) None of these Tg:- @NextGenBankers
29. J @ K & L ! M * N, then how L is related to N?
floor is I and the floor above it is 2 and so on. Who live on
the 2nd floor?
Statement I: Three persons live between S and R, who
(a) Brother in law (b) Sister in law lives just below U. Two persons live between U and Q.
(c) Sister (d) Brother Statement II: P lives above U. T doesn’t live below S. R
(e) None of these lives above Q.
DIRECTIONS (Qs. 30-32): Read the following information Statement III: T doesn’t live just below R. At least 2 persons
carefully and answer the given questions. live between Q and P.
(a) Data in all statement I, statement II and statement III
A # B-B is in the south direction of A at distance of 8m. together are sufficient to answer the question.
A * B-B is in the north direction of A at distance of 6m. (b) Data in either statement I alone or both statement I
A & B-B is in the east direction of A at distance of 12m. and statement II together is sufficient to answer the
A @ B-B is in the west direction of A at distance of 10m. question.
A * @ B-B is in the northwest direction of A. (c) Data in either statement II or statement III alone is
A # & B-B is in the southwest direction of A. sufficient to answer the question.
P # @ R, R * Q, Q & S * T, P @ Q (d) Data in either statement II or statement I alone is
30. P is in which direction with respect to S and what is distance sufficient to answer the question.
between point P and point S? (e) Data in both statement I and II together is sufficient to
(a) West and 2m (b) East and 9m answer the question.
(c) North and 11m (d) South and 13m
(e) None of these DIRECTIONS (Qs. 35-39) : Answer the questions based on
31. What is the distance between T and Q? the information given below.
(a) 195m (b) 85m In a certain language,
‘India and Nepal are friends’ is coded as ‘#1E,%6T, @ 1B,
(c) 180 m (d) 176 m
@12M, #1F’ ‘People with country should grow’ is coded as
(e) None of these ‘$8I, !4E, !5F, %3Z, $7X’ ‘Hard work becomes core for success’
32. T is in which direction with respect to P and what is distance ‘%2T, #6S, & 8L, %19T, $3F’
between point P and point T? 35. How is the word ‘Monkey’ coded in the given language?
(a) South-west, 40m (b) North-west 40m (a) @13M (b) !13Z
(c) South-east 40m (d) North-east, (c) $12M (d) %11Z
40m
(e) %11M
(e) None of these
2022- 4 SBI Clerk (Junior Associates) Mains Solved Paper-2022
36. How is the word ‘Cabbage’ coded in the given language? 43. Which of the following element is fourth to the right of
(a) %3D (b) &5F (c) *5D (d) %3F the third element from the left end in second last step of
(e) &4G the given input?
37. How is the word ‘Packer’ coded in the given language? (a) 22 (b) 09 (c) 24 (d) 26
(a) @18Q (b) %15R (c) !16S (d) %17P (e) None of these
(e) !18S 44. How many even numbers are there in the last step?
38. How is the word ‘Radarmen’ coded in the given language? (a) 1 (b) 4 (c) 5 (d) 3
(a) &14O (b) &18S (c) !14M (d) !18Q (e) None of these
(e) @14S 45. How many elements are to the right of the ‘88’ in step 3?
39. How is the sentence ‘This can direct’ coded in the given (a) 5 (b) 6 (c) 3 (d) 4
language? (e) None of these
(a) #12T &4V !19X (b) #11A &5T %18Y
(c) #13Z @3U $20Z (d) #3D $20U !4E DIRECTION (Q. 46) : In the question below is given a
(e) #3O !4U $19T statement followed by two assumptions numbered I and II. An
assumption is something supposed or taken for granted. You
DIRECTION (Q. 40) : In this question, two rows are given have to consider the statement and the following assumption
and to find out the resultant of a particular row you need to and decide which of the assumption is implicit in the statement.
follow the following steps:
Give answer:
Step 1: If an even number is followed by an odd (prime) number
(a) If only assumption I is implicit
then the resultant will be the addition of both the numbers.
Step 2: If an odd number is followed by a perfect square then (b) If only assumption II is implicit
the results will be the subtraction of the square number from the (c) If either I or II is implicit
odd number. (d) If neither I nor II is implicit
Step 3: If an odd number is followed by another odd number (e) If both I and II are implicit
then the resultant will be the addition of both the numbers. 46. Statement: Government should deploy army to rehabilitate
Step 4: If an even number is followed by an odd (non-prime) the people displaced due to earthquake.
number then the resultant will be the subtraction of the odd Assumptions:
number from the even number. I. Army can be used for purposes other than war also.
Step 5: If an odd number is followed by an even number then
Tg:- @NextGenBankers
the resultant comes by multiplying the numbers.
40. Find the sum of two rows.
II. Only army can rehabilitate the displaced victims of
earthquake.
4 5 2 DIRECTION (Q. 47) : In the questions below is given a statement
13 9 3 followed by two courses of action numbered I and II. On the
(a) 18 (b) 25 (c) 11 (d) 14 basis of the information given in the statement, you have to assume
(e) None of the above everything in the statement to be true, and then decide which of
the suggested courses of action logically follow(s) for pursuing.
DIRECTIONS (Qs. 41- 45): A number and word arrangement
machine when given an input line of number and words Give answer:
rearranges them following a particular rule in each step. The (a) If only I follows.
following is an illustration of an input rearrangement. (b) If only II follows.
(c) If either I or II follows.
Input: name narrow nest nostalgic nature nagative night nephew
(d) If neither I nor II follows.
Step I: am aorr es agilost artu aegitvghi eehp
(e) If both I and II follow.
Step II: 14 19 24 21 22 23 16 21
47. Statement: A number of school children in the local
Step III: 56 95 96 105 88 115 64 105
schools have fallen Ill after, the consumption of their
Step IV: K25 N81 D81 F1 P64 G1 J36 F1
subsidized tiffin provided by the school authority.
Step V: 18 23 24 7 26 8 19 7
Courses of action:
Step VI: 81 25 36 49 64 64 100 49
I. The tiffin facility of all schools should be discontinued
Step VI is the final output of the above arrangement
Input: Hard Height Handsome Hour Husband Hypertext Honest Hindi with immediate effect.
II. The government should implement a system to certify
41. How many steps are required to obtain the final output of the quality of tiffin provided by the school.
the given input.
48. Statement: A slump in home sales has pushed builders’
(a) Four (b) Five
inventory to “unsustainable levels” in the National Capital
(c) Six (d) Seven
Region of Delhi, creating conditions for a real price
(e) None of these
correction that developers have so far avoided.
42. Which among the following is the final step of the given input? Which of the following statements is inferred on the basis
(a) 25 64 1 81 16 36 25 9 of the given statement?
(b) 25 81 81 1 64 16 36 16 (a) Demand for houses is quite low in most of the big cities.
(c) 64 4 81 16 25 64 25 49 (b) The high inventory level will hamper the ability of
(d) 36 25 49 09 64 25 49 9 launching new projects by the builders.
(e) None of these
SBI Clerk (Junior Associates) Mains Solved Paper-2022 2022- 5
(c) Builders in the market who have shown a good track travels for next 3 hrs 2 min and he covered 60 km more
record of delivering on time and on their promises, than his earlier covered distance. Find the approx. value
have been able to garner sales even in this slow market. of A?
(d) Unless this unsold stock gets absorbed, it will be difficult (a) 36 (b) 40 (c) 30 (d) 45
for consumer confidence to come back in this market. (e) None of these
(e) High prices have pushed most of the on-sale 52. A tank contains 80 litre mixture of Milk and water in which
apartments beyond the reach of average home buyers. ratio was 4 : 1 respectively. 20 litre mixture was taken out
49. World’s forests are ‘in emergency room’. Study shows that and 24 litre milk was replaced by it. Now 28 litre new
the world lost 12 million hectares of tropical tree cover mixture was removed to get final mixture. Find the final
last year - the equivalent of 30 football pitches a minute - quantity of water in final mixture?
researchers said on Thursday, warning the planet’s health (a) 10 (b) 6 (c) 12 (d) 8
was at stake as we depend on forests for our survival from (e) 5
the air we breathe to the wood we use and so on. 53. P started a business with Rs x, after 8 months Q joined
Which of the following may be the reason of the warning him with Rs (x + 3000). After another one month R also
given in the statement? joined them with some investment. Now profit share of Q
(I) Forests and trees make vital contributions to both and R become equal after a year. Profit share of P is twice
people and the planet, bolstering livelihoods, of profit share of Q. Find the investment made by R?
providing clean air and water, conserving biodiversity (a) Rs. 8000 (b) Rs. 10000
and responding to climate change. (c) Rs. 12000 (d) Rs. 15000
(II) How to increase agricultural production and improve (e) None of these
food security without reducing forest area is one of 54. 10 boys can finish a work in 30 days and same work can
the great challenges of our times. be finished by 15 girls in same days. Now 20 boys start
(III)There is quantitative evidence to show that forests are work and leave the work after 3 days. 30 girls finish
being managed more sustainably. remaining work. If total time to finish the work was 21
(a) Only I is implicit days. Find the no. of days in which no work was done?
(b) Only III is implicit (a) 4 days (b) 5 days
(c) Only III and II are implicit (c) 6 days (d) 8 days
(d) None is implicit (e) None of these
Tg:- @NextGenBankers
(e) Only I, II and III are implicit
DIRECTION (Q. 50) : In making decisions about important
55. Breadth (b) of a rectangle is 65% less than length (l) of
same rectangle. Radius of circle is r mm and area of circle
is 616 sq m. relation between r and b is as follow:
question, it is desirable to be able to distinguish between ‘strong’ r2 = 16 1/3 (b + 5). Find the area of rectangle.
arguments and ‘weak’ arguments. ‘Strong’ arguments are those (a) 140 sq m (b) 560 sq m
which are important and directly related to the question. ‘Weak’ (c) 420 sq m (d) 280 sq m
arguments are those which are of minor importance and also (e) None of these
may not be directly related to the question or may be related to 56. Yuvi invest in Scheme X certain sum Rs. (a + 960) for
a trivial aspect of the question. Each question below is followed 6 years at 15% rate of interest, he received Rs. (3a – 420)
by two arguments numbered as I and II. You have to decide amount after 6 years. If he invest Rs. b in scheme X for same
which of the arguments a strong argument is and which a weak time than he received Rs. (2a – 280). Find the value of b?
argumetn is. (a) Rs 3000 (b) Rs 2000
Give answer: (c) Rs 2600 (d) Rs 1600
(a) If only Argument I is strong (e) None of these
(b) If only Argument II is strong 57. Dinesh invest total sum of Rs 20000 in two schemes P and
(c) If either Argument I or II is strong Q. He invest in Scheme P and Scheme Q for 3 years and 2
(d) If neither Argument I nor II is strong years respectively. Rate of interest in both schemes on SI
(e) If both Argument I and II are strong is 20% and 35% respectively. Interest received from
50. Statement: Should only reputed NGO’s be authorized to scheme Q is Rs 1600 less than same received from scheme
distribute the commodities to the public under the P. Find the 50% of total amount received from scheme P
programme of Public Distribution System (PDS)? and Q?
Arguments: (a) Rs 14400 (b) Rs 17600
I. Yes, the move will be helpful to implement the (c) Rs 15600 (d) Rs 16400
programme more effectively and will keep a tab on (e) None of these
various problems like black marketing of the 58. Women in city A is 150% of Men in same city. Men in city
commodities supplied under PDS. B is thrice of Men in city A. Women in city B is 500 more
II. Yes, NGO’s have helped government on many occasions. than women in city A. Total population of city B is 180%
more than that of city A. Find the number of Men in city A?
Quantitative Aptitude (a) 300 (b) 400
(c) 200 (d) 500
51. Ekansh cover a certain distance in 3 hrs 55 min with A (e) None of these
kmph after that he increased his speed by 30 kmph and he
2022- 6 SBI Clerk (Junior Associates) Mains Solved Paper-2022
DIRECTIONS (Qs. 59-60) : Following questions contain two 64. Which of following equation will give you same roots?
statements as statement I and statement II. You have to determine (a) 20x2 – 13x + 2 = 0 (b) 10x2 – 9x + 1 = 0
which statement/s is/are necessary to answer the question and (c) 15x2 – 23x + 6 (d) 15x2 – 22x + 8 = 0
give answer as, (e) 15x2 – 13x + 2 = 0
(a) The data in statement I alone is sufficient to answer the DIRECTIONS (Qs. 65-66) : Study the given quadratic
question, while the data in statement II alone is not equations and give answer of the questions.
sufficient to answer the question. P = 2x2 – 11x + 12 = 0
(b) The data in statement II alone is sufficient to answer the Q = 5x2 – 12x + 4 = 0
question, while the data in statement I alone is not sufficient R = 4x2 – 25x + 6 = 0
to answer the question. 65. Sum of smallest root of equation P and smallest root of
(c) The data either in statement I alone or in statement II alone equation R?
is sufficient to answer the question.
(d) The data given in both statements I and II together are not 3 1 1 2
(a) 1 (b) 1 (c) 1 (d) 1
sufficient to answer the question. 4 4 2 3
(e) The data given in both statements I and I together are (e) None of these
necessary to answer the question. 66. Which of the following statements are TRUE?
59. Rs. 15000 was distributed between A, B, C and D. Find Statement 1: Product of roots of equation P is equal to
the share of B? largest root of equation R.
Statement 1: A and B together received 25% more than Statement 2: Difference between smallest root of equation
share of C. B received Rs. 1000 less than share of A. Q and smallest root of equation R is 3/10
Statement 2: Ratio of share of D and C is 3 : 2. Statement 3: Ratio of the product of roots of equation P
60. Find the sum of the perimeter of the rectangle and the and equation R is 3 : 1.
perimeter of the square. (a) Only 1 and 2 (b) Only 1
Statement I: The difference between the length of the (c) Only 3 (d) Only 1 and 3
rectangle and the side of the square is 20 m. (e) Only 2
Statement II: The ratio of the length of the rectangle to
the breadth of the rectangle is 4:3 and the radius of the DIRECTIONS (Qs. 67-72) : Study the following pie chart
circle is 20% less than the sum of the length and the breadth carefully and answer the questions.
of the rectangle. Tg:- @NextGenBankers Pie chart given below shows the percentage distribution of
number of visitors (Male + Female) visited a circus on five
DIRECTIONS (Qs. 61-62) : Following questions have two different days in a week.
quantities as Quantity I and Quantity II. You have to determine Note: y is 1/4th of the total pie.
the relationship between them and give an answer as, Total number of person visited the circus on five different days
(a) Quantity I > Quantity II is 500.
(b) Quantity I > Quantity II
(c) Quantity II > Quantity I
(d) Quantity II > Quantity I y%
(x + 20)%
(e) Quantity I = Quantity II (or) Relation cannot be established
61. x, y and z are three positive integers.
x > y and z = 2x (x + 5)%
sum of reciprocal of x and y is three times of difference 2x%
69. On Saturday visitors came circus are 60% more than that 76. If the total number of students in college O is 3 times the
of Friday. Find the ratio of people came to circus on number of students in course P in college J and the number
Thursday to the same on Saturday? of students in course Q in college O is 3/4th of the total
(a) 5 : 4 (b) 4 : 5 (c) 8 : 3 (d) 3 : 8 students in that college. Then find the average of total
(e) None of these number of students in course P in college O and the number
70. Which of the following are incorrect from given statements? of students in course Q in college J.
1. y = 2.5 x 2. y – x = 20 3. y + 2x = 55 (a) 378 (b) 412 (c) 356 (d) 424
(a) Only 2 (b) Only 1 (e) None of these
(c) Only 2 and 3 (d) Only 1 and 2 77. If the number of students in course Q in college L is 78
(e) Only 1 and 3 more than four times the difference between total students
71. On Friday, if 2/5th of total person visited the circus were in college J and M, then find the sum of total number of
females and out of the total females visited on Friday students in college K and total number of students in
1/5th visited for the first time. The total first time visitors college L.
on Friday was 10% of the total people visited on all five (a) 1460 (b) 1514 (c) 1626 (d) 1392
days. Then find the number of males who visited the circus (e) None of these
not for the first time on Friday.
(a) 30 (b) 40 (c) 35 (d) 45 DIRECTIONS (Qs. 78-82) : Study the line graph carefully
(e) 50 and answer the questions.
72. On Saturday if the number of males visited the circus was Line graph shows data of 5 different gym members in 2015 and
double the difference of the number of persons visited the circus 2020.
on Monday and Wednesday and on Saturday females visited
140
the circus was 3/4th of the total person visited on Saturday.
Then find the total person visited the circus on Saturday. 120
120
Tg:- @NextGenBankers
There are 4 colleges in a city for example J, K, L, and M. Here
two type of certificate courses is being opted-courses are (P, Q).
In the table given total numbers of students and either percentage
60
40
48
36 40
32
or actual value in numbers of the students who has taken that
20
particular certificate course. 0 20
12
J 968 50% – 78. In 2015 male members of gym Q is less than by male
K – 44% 504 members of gym R. Female member of gym R is 4 times
of female members of gym Q. Sum of female members in
L – 40 8 –
both gyms are more than 10. Find the possible number of
M 840 65% – male members in gym R.
73. In college N total students are equal to the sum of 3 times 1. 12 2. 24 3. 16 4. 20
of students of course P in college J and 1/3 of course Q in 5. 28
college M. Find the ratio of total students of college N (a) Only 1 and 3 (b) Only 2, 3 and 4
and total students of college K? (c) Only 2, 4 and 5 (d) Only 1, 3 and 4
(a) 41 : 36 (b) 31 : 18 (e) All are possible value.
(c) 15 : 23 (d) 31 : 27 79. Average members of gym S, T and U is 57 in 2020. If male
(e) None of these are twice of female members in gym U in given year. Ratio
74. Ratio of the students in course P to course Q in college L of male to female members in gym S in the same year is 5 :
is 2 : 3. Total students in college L are approx what percent 3. Find the male members of gym U and S together is what
more than that of College K? percent more than female members in the same gym.
(a) 22% (b) 13% (c) 17% (d) 24% (a) 94% (b) 82%
(e) None of these (c) 89% (d) 78%
75. In college L, If the percentage range of students in course (e) None of these
P lies within 15 < P < 30 and P is a even multiple of 6 then 80. In gym U total member in 2020 is equal to difference
find the number of students in course Q in college L. between total members in both years of gym Q. If difference
(a) 1100 (b) 1076 between male and female of members in gym U is 4. Then
(c) 1292 (d) 1154 find the number of male in gym U
(e) None of these (a) 16 (b) 12 (c) 18 (d) 20
(e) Cannot be determined
2022- 8 SBI Clerk (Junior Associates) Mains Solved Paper-2022
81. Ticket price for male and female is ` 30 and ` 20 90. 124, ?, 100, 256, 904, 4078
respectively in both year for gym R. Ratio of male and (a) 74 (b) 78 (c) 64 (d) 56
female is 3 : 5 of total male and female of 2015 and 2020 (e) 84
for gym R then find revenue which was generated by selling 91. The average cost price of all products in an mobile shop is
tickets of female in gym R in 2015 and 2020. Rs__. If the cost of a mobile and a smart watch is increased
(a) ` 720 (b) ` 650 (c) ` 580 (d) ` 600 by Rs.550 and Rs.290 respectively and the average
(e) None of these becomes Rs.70 more, then the total number of products in
82. Total number of female in 2020 gym P is equal to total an mobile shop is___.
number of persons in 2015 in gym Q. If total number of A. Rs. 750,12 B. Rs. 1200,8
female in gym P in both years is 100 then find the male of C. Rs. 510,7
gym P in both years? (a) Only A (b) Only A and B
(a) 30, 20 (b) 40, 50 (c) 40, 60 (d) 20, 80 (c) Only C (d) Only A and C
(e) 60, 30 (e) All A, B and C
DIRECTIONS (Qs. 83-87) : Study the data carefully and 92. The quantity of juice in vessel P is 20% more than that of
answer the question given below: vessel Q and the ratio of the quantity of juice to water in
vessels P and Q is 9:2 and 5:2 respectively. The quantity
There are three companies which hired employees in three
different years. Total number of employees hired in company A of juice in vessel R is equal to the average of the quantity
in 2016 is 116. Total number of employees hired by company B of juice in vessels P and Q. If the difference between the
in three years are 212 and in 2018 are 50. Total number of total quantity of vessel P and Q is 4 liters, then find the
employees hired by company C are 255. Ratio of employees ratio of the average quantity of juice in vessel P and vessel
hired in 2016 and 2017 by company C is 11:20. Number of Q together to the quantity of water in vessel Q.
employees hired by company A in 2018 is 3/4 of hired by itself (a) 9:7 (b) 8:5 (c) 7:6 (d) 11:4
in 2017. Employees hired in 2017 by company C is 125% of (e) None of these
employees hired by company A. Ratio of employees hired by 93. Kiyansh sold two times A and B. Sum of marked price of
company B in 2018 and company C in 2018 is 1:2 respectively. A and B is Rs 10000. Sum of cost price of A and B is
The average of employees hired by company C in 2018, 2017 Rs 7200. Sum of selling price of A and B is Rs 8260.
and employees hired by company B in 2017 is 100. Kiyansh gave discount of 20% and earns a profit of Rs.
(a) 60
Tg:- @NextGenBankers
83. Total number of employees hired by company A is how many
more than total number employees hired by company B?
(b) 44 (c) 56 (d) 72
640 on item A. If markup percentage on item B is 30%.
Then find the selling price of item B?
(a) Rs 4360 (b) Rs 4420
(e) None of these (c) Rs 4580 (d) Rs 4640
84. Find the ratio between the employees hired by company (e) None of these
C in 2016 and company B in 2017. 94. Difference between curved surface area of cone and
(a) 21:80 (b) 27:53 (c) 11:20 (d) 13:50 cylinder is 64p and height of cylinder and cone are equal.
(e) 7:15 Slant height of cone is __% more than its radius. If radius
85. Total number of employees hired by company A in 2018 of cylinder is __% more than radius of cone, Then the
is how many percentage more than in the same year volume of cylinder will be 6912p.
employees hired by company B? (a) 30% and 20% (b) 15% and 40%
(a) 25% (b) 30% (c) 20% (d) 10% (c) 20% and 60% (d) 25% and 50%
1 (e) None of these
(e) 12 %
2 95. Three flexible pipes that can serve as inlet as well as outlet
86. Find the average of employees in all three years in companyA? pipes have been attached to a cistern. The rates of filling
(a) 85 (b) 76 (c) 82 (d) 79 or emptying the cistern by these three pipes are in the ratio
(e) None of these 1:2:4. The largest pipe can alone fill the empty cistern in 7
87. Find the ratio between the employees hired by company hours. Initially the cistern is full and the three pipes were
B and C in 2016 and employees hired by company A and used as outlet pipes for A hours. After the cistern is emplied,
B in 2017? later the smallest pipe alone filled the entire cistern in 28
(a) 11:17 (b) 12:19 (c) 13:21 (d) 13:20 hours. Find the value of A.
(e) 21:13 (a) 3 Hours (b) 4 Hours
DIRECTIONS (Qs. 88-90) : Find out the missing number in (c) 4.5 Hours (d) 5 Hours
the following number series. (e) None of these
88. 256, 304, ?, 436, 520, 616 DIRECTIONS (Qs. 96-100) : What approximate value should
(a) 340 (b) 388 (c) 364 (d) 336 come in place of question mark(?).
(e) 316
96. 83.33% of 119.98 + 28.15% of 225.12 = ? + 16.12% of
89. 512, 567, 502, ?, 492, 607
99.89
(a) 587 (b) 576 (c) 565 (d) 548
(a) 147 (b) 153 (c) 159 (d) 141
(e) 592
(e) None of these
SBI Clerk (Junior Associates) Mains Solved Paper-2022 2022- 9
97. (2856.06 ¸ 101.891) × 14.91 = ? + 219.891 (c) The government has made the decision to eliminate
(a) 160 (b) 188 (c) 172 (d) 195 any friction between the various economic sectors.
(e) None of these (d) The government has ensured that all economic
98. 89.904% of 319.879 + (682.07 ¸ Ö960) × 15.891 = ? × sectors receive the necessary attention for expansion
7.891 and development.
(a) 76 (b) 90 (c) 80 (d) 84 (e) None of the above
(e) None of these 102. Which of the following is true about the government’s
99. 69.56% of 479.912 + 85.714% of 489.921 + 42.857% of attitude toward the country’s startup economy?
273.12 = ? (a) The economy of the country has been given a lifeline
(a) 873 (b) 843 (c) 753 (d) 927 but there are a lot of questions to ponder over that.
(e) None of these (b) Despite the fact that the government has done nothing
100. (1273.12 ¸ Ö360) × 14.899 + (6360.087 ¸ Ö2810) ¸ Ö65 to assist startups, the nation’s economy is primarily
= ? × 11.991 concerned about them.
(a) 71 (b) 85 (c) 88 (d) 94 (c) The nation continues to rely on large industrial
(e) None of these corporations to maximize their tax payments..
English Language
(d) The government has understood the importance of
the startups in the economy and is positive about the
development of this economy.
DIRECTIONS Qs. (101- 105): Read the following passage (e) None of the above
carefully and answer the questions that follow. 103. Which among the following is /are correct regarding the
Start-ups are heaving a sigh of relief as the Finance Minister TV channel that has been announced by the government
announced measures to do away with the long pending in the Budget?
contentious issue of Angel Tax, which many of them had to I. This new channel will be run by the government with
cough up for raising Angel funding under Section 56 of the the experts from the government organizations.
Income Tax Act. II. This new channel will be financed by the public
Also, special administrative arrangements will be made by the sector banks with a fresh round of capital from the
Central Board of Direct Taxes for pending assessments of start- government only.
III. This new channel will attempt at saving the taxes of
Tg:- @NextGenBankers
ups and redressal of their grievances. Start-ups have been
assured that no inquiry or verification in such cases can be
carried out by the Assessing Officer without obtaining approval
the startup founders by the experts.
(a) Both II and III (b) Both I and III
of his supervisory officer. This provision will do away with (c) Both I and II (d) Only III
much of the angst among start-ups who were subjected to (e) None of I, II and III
aggressive questioning by the Income Tax department. 104. What is the opinion of the author regarding the TV channel
A TV channel exclusively for start-ups under the Doordarshan that has been proposed to be set up by the government?
boutique will be designed and executed by start-ups themselves (a) The author is excited about the new TV channel since
and will also serve as a platform for promoting them, discussing it will bring a lot of interest in the sector.
issues affecting their growth, match making with VCs and for (b) The author has nothing to say regarding the new TV
funding and tax planning, came as a pleasant surprise to many. channel that has been proposed in the Union Budget.
Although it remains to be seen how it will be executed on the (c) The author is cautious in his reaction because he
ground. wants to see if the government is able to make it
Th e Budget sounded the bugle for fosterin g rural work actually.
entrepreneurship for those depending on agriculture and (d) The author is disappointed with this decision for the
traditional industries by announcing the setting up of 80 fact that the government should have done something
Livelihood Business Incubators and 20 Technology Business more worthwhile for the startups in India.
Incubators this fiscal with the aim to develop 75,000 skilled (e) None of the above
entrepreneurs in agro-rural industry sectors. “The Government 105. Which among the following is/are correct regarding the Angel
has tried to broad base and foster entrepreneurship beyond Tax levied by the government on the startups in India?
metros to Tier 2 and 3 cities which is a good sign. And doing I. Angel Tax is levied on companies that raise capital
away with Angel Tax is welcome” said Bhaskar Majumdar, from the angel investors only and not from others
Managing Partner, Unicorn India Ventures. sources.
101. Which of the following statements can be said regarding II. Angel Tax is not defined in the Income Tax
the government’s sectoral thrust on start-ups in the nation? legislation and the government had to implement it
(a) The government has made sure that the primary separately.
sector of the country gets the fair share of attention III. Angel Tax is not going to be levied on the companies
from the start-up industry. that go out of business from this year onwards.
(b) According to the government, the start-up industry (a) Both I and II (b) Only I
should first prove to be profitable in the country (c) Both II and III (d) Both I and III
before expanding to other areas. (e) All I, II and III
2022-10 SBI Clerk (Junior Associates) Mains Solved Paper-2022
DIRECTIONS Qs. (106- 112): Read the following passage on us that television executives love to fill comedy shows with
carefully and answer the questions that follow. canned laughter.
Experiments by lots of behavioural scientists have found that
We did an interesting experiment in Mumbai some time back. the use of canned laughter causes an audience to laugh longer
We got 98 households across a few housing societies in Bandra and more often when humorous material is presented. People
and Khar to provide us with their electricity bills before the rate the material as funnier. In addition, evidence indicates that
bills reached each member’s house. We then calculated the canned laughter is most effective for poor jokes.
average bill amount in that particular society. In another experiment conducted by behavioural scientists Noah
Let’s say the average was Rs 1,022. For all above-average users, Goldstein, Robert Cialdini and Vladas Griskevicius (‘A Room
we put a stamp stating that the average in that society is Rs with a Viewpoint: Using Social Norms to Motivate
1,022. Next to their above average amount, we put Environmental Conservation in Hotels’, goo.gl/OJT1pb),
a frownie indicating that they could do better. different kinds of signs were placed in hotel rooms. One of the
The average number set the social norm and got the above signs asked guests to help save the environment by reusing their
average users to act like their neighbours and reduce their towels.
electricity consumption by 1.33 per cent. 1.33 per cent sounds The second one informed them that the majority of guests at
small, but it can power 17,465 villages for one whole year. We the hotel recycled their towels to help save the environment.
called the experiment People Power because it gives people The second sign had a success rate of 26 per cent more than the
the power to make a difference at no cost. first sign.
Human behaviour is contagious. Our actions are often guided A third sign informed guests that majority of people who had
by how people around us are behaving. The information previously stayed in their particular room recycled their towels
provided by the stamp let the above-average users know how to help save the environment. The third sign had a success rate
much their neighbours were consuming. of 33 per cent more than the first sign.
That set the social norm and got them to reduce their power Now only if hotels could apply the same principle to reducing
consumption. We do as others do. If people see other people theft of towels, shampoos, bed sheets, stationary and, yes,
littering, they litter too. If people see other people throwing appliances too.
waste in dustbins, they use dustbins too. If people see other (Source:‘ Economic Times’ dated December 29th, 2016 and
people cheating, they cheat too. If people see other people being Jagran Josh)
honest, they behave honestly too. 106. Why do you think that human behaviour is contagious,
Tg:- @NextGenBankers
Behavioural science studies show that people dress in the same
styles as their friends, pick dishes preferred by other diners,
choose restaurants that are more crowded, are more likely to
according to the author?
(a) We are wired to interact with others and form bonds.
(b) We imitate the behaviours and habits of others.
get fat if people around them become fat, are more likely to (c) Our behaviour is influenced by how those around us
quit smoking if their friends quit, pay taxes if others are paying, behave.
vote if their spouse votes, and so on. A five-star review on (d) Other than those given in the options.
Amazon leads to approximately 20 more books sold than one- (e) All of the above
star reviews. 107. How this behavioral science principle helped a popular
This behavioural science principle of ‘social proof ‘ made a American infomercial?
popular American infomercial for a home shopping channel (a) Their mental image was improved.
change the all-too familiar call-to-action line at the end of the (b) Their sales were skyrocketing.
infomercial, “Operators are waiting, please call now” to “If (c) The perception of their customers changed.
operators are busy, please call again”. This simple change led (d) It was hard to resist their offer.
to its sales skyrocketing. (e) Other than those given in the options.
On the face of it, the change seems foolhardy. After all, the 108. Why, according to the author, the television executives
message indicates that one may have to waste their time fill comedy shows with canned laughter?
redialling till they reach a sales representative. Yet it worked (a) They don’t have anything else to show.
so brilliantly. (b) People like to laugh at others.
(c) People laugh more on poor jokes.
Consider the kind of mental image that’s likely to get generated
(d) Both (b) and (c).
when you hear, ‘Operators are waiting, please call now’ —
(e) Other than those given in the options.
scores of bored phone representatives while they wait by their
109. Why, according to the passage, the experiment was called
silent telephones — an image indicative of low demand and poor
as “People Power”?
sales.
(a) Because it gives people the power to socialize and
Consider how your perception of the popularity of the product mingle with others.
would change when you hear, ‘If operators are busy, please (b) Because it gives people the power to make a
call again’ — operators going from phone call to phone call difference at no cost.
without a break, right? That made people thinks: ‘If the phone (c) Because it gives people the power to copy others,
lines are busy, then other people like me who are also watching without feeling ashamed.
this infomercial must be calling too. (d) Because it gives people the power to listen to their
Most people think they are different. But in reality most of us souls.
behave the way others do. So powerful is the effect of others (e) Other than those given in the options.
SBI Clerk (Junior Associates) Mains Solved Paper-2022 2022-11
110. Which among the following is similar in meaning to the (a) DACB (b) DBCA
word ‘skyrocketing’ used in the passage? (c) CBDA (d) BCAD
(a) Descending (b) Tumble (e) No rearrangement required
(c) Escalating (d) Smashing
(e) Other than those given in the options DIRECTIONS Qs. (118-121): In this question, a sentence has
111. Which among the following is similar in meaning to the been divided into four parts marked as I, II, III and IV. You
word ‘contagious’ as used in the passage? need to find which part/parts does not/do not have an error in
(a) Transmissible (b) Immoral terms of its grammatical or contextual usage. If the sentence is
(c) Non violent (d) Precious absolutely correct, mark (E) as your answer.
(e) Other than those given in above options 118. I. The supreme court of India disapproved
112. Which among the following is opposite in meaning to the II. The practice of constituting commissions of inquiry
word ‘foolhardy’ as used in the passage? III. At the cost of taxpayers and let them continue for
(a) Bold (b) Wise years together
(c) audacious (d) Foolish IV. Without serving any useful purpose.
(e) Other than those given in the options (a) Only I and III (b) Only II and IV
(c) Only IV (d) Only III
DIRECTIONS Qs. (113-117): In the questions given below, a (e) No error
sentence has been broken down into four fragments labeled 119. I. Between 2010 and 2017, several countries
(A), (B), (C) and (D) and arranged, not necessarily in the II. Make rapid progress
correct order. You have to find the correct order of arrangement III. In reducing HIV incidence and
from the options given below. In case, the sentence is correct IV. Getting antiretroviral therapy of patients.
in its original form, please select (E) as your answer. (a) Only I and IV (b) Only I and III
113. 2016, but little money has come in as retailers want (A) / (c) I, III and IV (d) Only I
(e) No error
like soaps and shampoos for customers (B) / permission
120. I. A group of banks, including public sector, private
to stock a few non-food items (C) / 100% FDI was allowed
sector and foreign banks,
in the food retail business in (D)
II. have recently signed an inter-creditor agreement
(a) CBAD (b) BACD III. to push for the speedy resolution of
(c) BADC
Tg:- @NextGenBankers
(e) No rearrangement required
(d) DACB IV. Non-performing loans on their balance sheets.
(a) Only II and III (b) Only II and IV
114. of plastic articles also show little (A) / remain on paper (c) I, III and IV (d) I, II and III
while the producers (B) / the Solid Waste Management (e) No error
Rules mostly (C) / concern about their negative 121. I. We continue to be hidebound with tradition
environmental impact (D) II. And waste precious time and money
(a) CABD (b) CBAD III. In rituals which may have been relevant in earlier
(c) DBCA (d) DABC times
IV. But which have no relevance to modern living.
(e) No rearrangement required (a) Only II (b) Only II and IV
115. A federation of 130 farmer bodies has decided (A) / dairy (c) I, III and IV (d) Only III and IV
produce to major cities and hold a (B) / to stop supplies (e) No error
of vegetables and (C) / dharna on 30 national highways
to protest against rising prices (D) DIRECTIONS Qs. (122-125): In each of the questions below,
(a) ACBD (b) BCDA four words are in bold. These four words may or may not be in the
(c) CABD (d) CBDA correct position. The sentence is followed by options containing
the correct combination of words. These options must replace each
(e) No rearrangement required
other to make the sentence grammatically and contextually correct.
116. the dignity of the government lies in its moral (A)/ capacity Find suitable combinations of words that replace each other. If
to perceive and confront the lived truth, (B)/ such as the the sentence is correct, please select option (e)
widespread despair, frustration, and (C)/ irreparable loss
of dignity leading to farmers’ suicides (D)/ 122. Delhi has benefited (1) from stringent testing measures,
priority offered (2) of cases and their contacts and the
(a) BCAD (b) CDBA
medical (3) support that has been containment (4) to
(c) ADCB (d) DCAB patients at home isolation.
(e) No rearrangement required (a) 2-3 (b) 1-4 (c) 2-4 (d) 1-3
117. the Emergency, the real story of the buying and selling of (e) None of these
the media began with (A)/ of media organisations began
123. India’s clinical testing (1) has focused on early detection
to be chipped away at until it disappeared (B)/ the advent (2) through widespread protocol (3)surveillance, prompt
of liberalisation, when the wall dividing the business from triaging (4) and clinical management of cases.
the editorial side (C)/ leaving aside the compromises the
(a) 2-3 (b) 1-4 (c) 2-4 (d) 1-3
majority of media houses made in the 1970s during (D)/
(e) None of these
2022-12 SBI Clerk (Junior Associates) Mains Solved Paper-2022
124. The agency’s application (1) seeking his custody was on A. Providing benefits for women and children is a societal
the grounds (2) that it had to confront (3) him with data responsibility which can be funded in a large country
recovered from seized (4) electronic articles. through a combination of general taxation and
(a) 1-2 (b) 3-4 (c) 2-3 (d) 1-4 contributory payments.
(e) None of these B. This should further lead to closer scrutiny of the
125. I got disillusioned (1) by CPI ideology but the recent difficulties faced by unorganised workers who fall beyond
motivated (2) being undertaken in Swabhiman Anchal the scope of any worthwhile labour welfare measures.
development (3) me to join the mainstream (4) C. The enhancement of paid maternity leave for women in
(a) 1-2 (b) 3-4 (c) 2-3 (d) 1-4 the organised sector to 26 weeks from 12 is a progressive step.
(e) None of these D. The reported move to restrict even this meagre benefit to
the first child for budgetary reasons is retrograde and must
DIRECTIONS Qs. (126-130): In each of the following be given up.
questions, a sentence is given with a phrase highlighted. These E. Positive though it is, the amended law is expected to cover
phrases may or may not be correct. Following the sentences only 1.8 million women, a small subset of women in the
are four phrases, from which one phrase will replace the workforce.
incorrect phrase. The number of that correct phrase will be F. For many poor millions in the unorganised sector, the only
your answer. If the phrase is correct then option (e) i.e. “No support available is a small conditional cash benefit of
improvement” will be your answer. Rs. 6,000 during pregnancy and lactation offered under
126. People scramble to stocked essentials ahead of the 10- the Maternity Benefit Programme.
day lockdown commencing at the midnight of July 13. G. It is wholly welcome that such a benefit is being introduced
(a) Scrambled to stock essentials with an amendment to the Maternity Benefit Act, 1961.
(b) Scrambles to stock essential 131. Which of the following would be the SECOND sentence
(c) Scrambled to stocked essentials after rearrangement?
(d) Scrambled to stocks essentials (a) F (b) A (c) E (d) B
(e) No improvement (e) G
127. Economic policies formulated by the IAS officers are 132. Which of the following would be the FOURTH sentence
generally above the power of comprehension off most of after rearrangement?
the ministers.
Tg:- @NextGenBankers
(a) the powers of comprehension off
(b) the powerful of comprehension of 133.
(a) D
(e) B
(b) A (c) E (d) G
138. The terrorists wanted to ________ communal tensions (a) Two (b) Three
and _______ Sri Lanka’s fragile ethno-religious matrix. (c) Four (d) Five
(a) stoke, exploit (b) fuel, enhance (e) Six
(c) regret, helps (d) stop, provide 150. Holcim Limited is a multinational compan y that
(e) improves, outsource manufactures building materials belongs to which country?
139. In her, the _______ Gujarat riots have forged an iron spirit (a) England (b) Switzerland
that will ________ other embattled women to fight for (c) Germany (d) France
justice. (e) Italy
(a) emotional, change (b) ancient, intend 151. PCA Framework does not includes
(c) horrific, inspire (d) terrified, motive (a) Customer Service (b) Leverage
(e) violent, assure (c) Capital (d) Asset Quality
140. China _______ its arrival in the private sector space race (e) CRAR
last week, as OneSpace became the first Chinese company 152. Banking Ombudsman is a quasi judicial authority created
to launch a small rocket—its first step towards sending in 2006, and the authority was created pursuant to a
_______ small satellites in space. decision made by the Government of India to enable
(a) Portrayed, smooth (b) Colored, some resolution of complaints of
(c) Marked, numerous (d) Dealt, more (a) customers (b) Banking Staff
(e) None of the above (c) Farmer (d) Labour
(e) Politician
General/Finance Awareness 153. Who is the Richest self made women according to Hurun
Global Rich List 2022?
141. The Nobel Prize in Chemistry 2022 was awarded to (a) Falguni Nayar (b) Kiran Mazumdar
(a) Carolyn R. Bertozzi (b) Morten Meldal (c) Roshni Nadar (d) Wu Yajun
(c) K. Barry Sharpless (d) All of The Above (e) All of the above
(e) None of the above 154. Kumbhalgarh Fort is related to which State?
142. As announced in the Monetary Policy Statement 2022-23 (a) Madhya Pradesh (b) Uttarakhand
dated December 07, 2022, the Bank Rate is revised (c) Maharashtra (d) Rajasthan
upwards by (e) Andhra Pradesh
(a) 6.2% (b) 6.3%
(e) 6.6% Tg:- @NextGenBankers
(c) 6.4% (d) 6.5%
162. ..........is a cyber crime that uses the phone to steal personal (a) 10 (b) 15 (c) 17 (d) 19
confidential information from victims. (e) 21
(a) Phishing (b) Vishing 174. RBI appointed R S Gandhi as Non Executive Chairman of
(c) Online Fraud (d) Skimming the Yes Bank for a period of
(e) Sim Swap (a) 2 Years (b) 3 Years
163. Who is the trustee of PM Cares Fund? (c) 4 Years (d) 5 Years
(a) Sudha Murthy (b) Arundhoti Roy (e) 6 Years
(c) Geeta Gopinath (d) Gautam Adani 175. RBI has recently issued guidelines for Digital Lending.
(e) Mukesh Ambani Asset Performance Report includes
164. Which of the following External debt is contracted? (a) Cost of funds, (b) Credit cost
(a) Currency & Deposits (c) Operating cost (d) Processing fee
(b) Trade Deficit & advances (e) All of the above
(c) Trade Credit & advances 176. Which organisation threshold B2B e Invoice (annual) ?
(d) Long Term Borrowing (a) Central Board of Direct Taxes
(e) Debt Securities (b) Central Board of Indirect Taxes
165. Eliud Kipchoge, kenyan marathon runner won Olympic (c) Reserve Bank of India
2020. Previously, he won the Gold Medal in which (d) Niti Ayog
olympics? (e) Ministry of Agriculture
(a) 2018 (b) 2016 (c) 2014 (d) 2012 177. Which Indian topped the IIFL List?
(e) 2013 (a) Mukesh Ambani (b) Indira Nooyi
166. Who is called “glasnost Russian”? (c) Gautam Adani (d) Radhakishan Damani
(a) Vladmir Putin (e) Kiran Mazumdar
(b) Boris Nikolayevich Yeltsin 178. What is the maximum limit of Credit Guarantee Scheme
(c) Mikhail Gorbachev for Startups for Non Banking Financial Companies ?
(d) All of The Above (a) Rs 10 Crore (b) Rs 8 Crore
(e) None of the Above (c) Rs 6 Crore (d) Rs 4 Crore
167. Windfall tax applied to which sector 179. Liability Index is related to
(a) Road (b) Land (a) Marginal Rate (b) Discount Rate
(c) Energy
(e) Water
Tg:- @NextGenBankers
(d) Health (c) Profit Rate
(e) Floating Rate
(d) Fixed Rate
168. Financial assets to BSNL and MTNL by govt in form of 180. India has signed MOU with which organization to raise
the government has listed for sale real estate assets of state- Fund for Pradhan Mantri Matsya Sampada Yojana?
run telecom firms MTNL and BSNL at a reserve price of (a) IMF
(a) Rs. 970 Crore (b) 1000 Crore (b) World Bank
(c) 1270 Crore (d) 1470 Crore (c) Asian Infrastructure Investment Bank
(e) 1480 Crore (d) National Development Bank
169. In which Year, Indian Population expected to cross China? (e) Asian Development Bank
(a) 2040 (b) 2050 181. Education 4.0 India report is the result of a collaboration
(c) 2060 (d) 2080 between YuWaah (Generation Unlimited India) and
(e) 2090 (a) UNESCO (b) UNICEF
170. Which ministry of India has awarded Meghalaya as best (c) World Bank (d) a & b
start up ecosystem? (e) None of the above
(a) Ministry of Commerce 182. How many Banks are listed in National Stock Exchange
(b) Ministry of Power Limited?
(c) Ministry of Home Affairs (a) 10 (b) 12 (c) 15 (d) 20
(d) Ministry of Micro, small, medium Enterprises (e) 21
(e) Ministry of Earth Science 183. Under which Government Scheme ,an assured monthly
171. As per the States, which state has topped the chart in the pension of Rs. 3000/- given to beneficiary after the age
category of best-performing states in the Swach Survekshan of 60 years?
Awards 2022? (a) Atal Pension Scheme
(a) Madhya Pradesh (b) Uttar Pradesh (b) Pradhan Mantri Vaya Vandana Scheme
(c) Meghalaya (d) Uttarakhand (c) Pradhan Mantri Kisan Maandhan Yojana
(e) Karnataka (d) Varistha Pension Scheme
172. Who is the Sponsor of NARCL? (e) Indira Gandhi National Old Age Pension
(a) Bank of Baroda (b) Canara Bank 184. Banking Ombudsman applies to which FIs
(c) Bank of India (d) Union Bank of India (a) NBFC (b) SEBI
(e) Allahabad Bank (c) MSME (d) Credit Agency
173. How many cities surveyed under Inflation Expectations (e) Credit Information Companies
Survey of Households?
SBI Clerk (Junior Associates) Mains Solved Paper-2022 2022-15
185. On September 2022, RBI lifts curbs on which Financial (a) 1 Only (b) 2 only
Institution for loan recovery through outsourcing? (c) 1 & 2 (d) None of The Above
(a) India Post Payment Bank (e) 1 & 3
(b) Mahindra & Mahindra Financial Services 188. Which of the following statement about National Assets
(c) Shriram Transport Finance Company Limited Construction Limited is/are correct?
(d) Bajaj Finance Limited. 1) The National Asset Reconstruction Company Ltd.
(e) Muthoot Finance Ltd (NARCL), set up to take over large bad loans of more
186. Under Pradhan Mantri Garib Kalyan Yojana (PMGKY), than ¹ 500 crore from banks.
the limit for collateral-free loans for women’s SHGs was 2) The SARFAESI Act, 2002 provides the legal basis for
doubled from Rs.10 lakh to the setting up of ARCs in India.
(a) 20 Lakh (b) 30 Lakh 3) Setting up of NARCL, the proposed bad bank for
(c) 40 Lakh (d) 50 Lakh taking over stressed assets of lenders, was announced
(e) 60 Lakh in the Budget for 2022-23.
187 Which of the following statement about Global Innovation (a) 1 Only (b) 2 Only
index is/are correct? (c) 1 & 2 (d) 1 & 3
1) The Global Innovation Index is an annual ranking of (e) None of the above
countries by their capacity for, and success in, 189. How many companies are in upper layer list of NBFC?
innovation, published by the World Intellectual (a) 12 (b) 14 (c) 16 (d) 18
Property Organization. (e) 20
2) It was started in 2007 by INSEAD and World Business, 190. The EXIM Letter of Credit policy can reduce a bank’s
a British magazine. risks on confirmations and negotiations of irrevocable
3) It reveals the most innovative economies in the world, letters of credit issued by overseas financial institutions
ranking the innovation performance of around 130 for the financing of exports of which Country?
economies. (a) UK (b) USA
(c) Switzerland (d) Poland
(e) Australia
1 (c) 21
Tg:- @NextGenBankers
(c) 41 (c) 61 (e) 81
ANSWER KEY
(d) 101 (a) 121 (e) 141 (d) 161 (a) 181 (d)
2 (d) 22 (a) 42 (b) 62 (c) 82 (c) 102 (d) 122 (c) 142 (d) 162 (b) 182 (c)
3 (c) 23 (c) 43 (c) 63 (a) 83 (b) 103 (d) 123 (d) 143 (b) 163 (a) 183 (c)
4 (d) 24 (d) 44 (b) 64 (e) 84 (c) 104 (c) 124 (e) 144 (a) 164 (d) 184 (a)
5 (c) 25 (b) 45 (c) 65 (a) 85 (c) 105 (b) 125 (c) 145 (a) 165 (b) 185 (b)
6 (b) 26 (a) 46 (a) 66 (b) 86 (a) 106 (e) 126 (a) 146 (b) 166 (c) 186 a)
7 (c) 27 (c) 47 (b) 67 (a) 87 (d) 107 (b) 127 (d) 147 (d) 167 (c) 187 (c)
8 (d) 28 (d) 48 (e) 68 (c) 88 (c) 108 (d) 128 (e) 148 (b) 168 (a) 188 (c)
9 (c) 29 (b) 49 (a) 69 (d) 89 (a) 109 (b) 129 (b) 149 (c) 169 (b) 189 (c)
10 (a) 30 (a) 50 (a) 70 (c) 90 (c) 110 (c) 130 (b) 150 (b) 170 (a) 190 (b)
11 (a) 31 (c) 51 (c) 71 (c) 91 (a) 111 (a) 131 (d) 151 (a) 171 (a)
12 (e) 32 (d) 52 (d) 72 (d) 92 (d) 112 (b) 132 (c) 152 (a) 172 (b)
13 (e) 33 (d) 53 (c) 73 (b) 93 (b) 113 (d) 133 (a) 153 (d) 173 (d)
14 (a) 34 (a) 54 (c) 74 (b) 94 (e) 114 (b) 134 (e) 154 (d) 174 (b)
15 (d) 35 (b) 55 (a) 75 (c) 95 (b) 115 (a) 135 (b) 155 (b) 175 (e)
16 (d) 36 (d) 56 (b) 76 (d) 96 (a) 116 (e) 136 (c) 156 (c) 176 (b)
17 (c) 37 (c) 57 (d) 77 (b) 97 (e) 117 (a) 137 (d) 157 (c) 177 (c)
18 (a) 38 (a) 58 (c) 78 (b) 98 (c) 118 (b) 138 (a) 158 (a) 178 (a)
19 (e) 39 (e) 59 (e) 79 (c) 99 (a) 119 (b) 139 (c) 159 (d) 179 (b)
20 (b) 40 (b) 60 (d) 80 (e) 100 (b) 120 (c) 140 (c) 160 (a) 180 (b)
2022-16 SBI Clerk (Junior Associates) Mains Solved Paper-2022
V Tg:- @NextGenBankers
W Bag
P
T
6. (b) 7. (c) 8. (d) 9. (c) 10. (a)
Sol. (11-15) :
Tag
Employees Date of J oining W ork ing Experience
R 12th Jan 5 Years
T 15th Jan 8 Years Hence, neither conclusion I nor II follows.
P 24th Jan 9 Years Sol. (25-26) :
Q 18th Ju ly 10 Years
# * @ & $
U 26th Ju ly 13 Years
< > = ³ £
S 27th Sept 16 Years
25. (b) F ³ D > X £ Z, B > X
11. (a) 12. (e) 13. (e) 14. (a) 15. (d)
I. F > B (false)
Sol. (16-20) :
II. X < F (true)
Person Colour City III. B > F (false)
A Grey Dehradun 26. (a) D > H £ X = F ³ M
E W hite Kolkata I. M < X (false) II. F < D (false) III. H = M (false)
D Blue Churu 27. (c) R
F Black Gangtok
G Red Patna
P Q(–) (+) S T
B Pink Noida
Hence, Q’s husband is brother-in-law of T.
C Bronze Gaya
16. (d) 17. (c) 18. (a) 19. (e) 20. (b)
SBI Clerk (Junior Associates) Mains Solved Paper-2022 2022-17
b´9 A+B 5
Þ = 2 × 2040 – 280 – b (by putting a’s value) Þ =
10 C 4
9b D 3 2 6
Þ + b = 3800 And, = ´ =
10 C 2 2 4
\ (A + B) : C : D = 5 : 4 : 6
19b
Þ = 3800
10 15000
(A + B)’s share = ´ 5 = 5000
15
3800 ´ 10
Þb= = ` 2000 A – B = 1000
19
Hence, both statements together are necessary to
57. (d) Let invested in scheme P is ` x.
answer the question.
According to question,
60. (d) From I:
x ´ 20 ´ 3 (20000 - x) ´ 35 ´ 2 l – a = 20 m
- = 1600
100 100 So, statement I alone is not sufficient to answer.
From II:
3x 20000 ´ 7 7x
Þ - + = 1600 l = 4x, b = 3x (let)
5 10 10
80 28x
13x r = 7x ´ =
Þ = 14000 + 1600 = 15600 100 5
10
So, statement II also alone is not sufficient to answer.
Þ x = 1200 × 10 = `12000
61. (e) z = 2x
\ Amount received from scheme P
12000 ´ 60 1 1 æ1 1ö
And, + = 3ç - ÷
= 12000 + = `19200 x y èy xø
100
Tg:- @NextGenBankers
And, amount recieved from scheme Q
Þ
x+y
=
3(x - y)
8000 ´ 70 xy xy
= 8000 + = ` 13600
100 Þ x + y = 3x – 3y
Hence, 50% of amount recieved from scheme P and Þ 2x = 4y
50 Þ x = 2y
Q = (19200 + 13600) ´ = ` 16,400 Þ x : z = 1 : 2 and x : y = 2 : 1
100
58. (c) Let men in city A is 2x. So, x : y : z = 2 : 1 : 4
QI: 0.5 × 1 × 4 = 2
150
Women in city A = 2x ´ = 3x 4 ´1
100 QII: ´ (2)2 = 2
Men in city B = 3 × 2x = 6x 2´4
Women in city B = 3x + 500 Hence, QI = QII
According to question, 62. (c) Number (A) = 10x + y
280 y = 2x
(6x + 3x + 500) = (2x + 3x) According to question,
100
(10x + 2x) + x × 2x = 54
28
Þ 9x + 500 = ´ 5x Þ 12x + 2x2 = 54
10
Þ x2 + 6x – 27 = 0
9x + 500 = 14 x
(x + 9) (x – 3) = 0
Þ 5x = 500 Þ x = 100
Þ x = 3, –9 (Neglected)
Hence, number of men in city A = 2 × 100 = 200
\ A = 10 × 3 + 2 × 3 = 36
59. (e) From I & II:
QI = 3 × 36 = 108
125 QII = 2 × 36 + 48 = 120
A+B= C´
100 Hence, QI < QII
2022-20 SBI Clerk (Junior Associates) Mains Solved Paper-2022
50 1 35
N = 3 ´ 968 ´ + ´ 840 ´ M F M F
100 3 100 x 4x (Let)
= 1452 + 98 I. 17 3 24 12 (Ö)
= 1550 II. 16 4 20 16 (Ö)
504 III. 15 5 16 20 (Ö)
Total students in college K = ´ 100 = 900 IV. 14 6 12 24 (×)
56
Tg:- @NextGenBankers
Hence, required ratio = 1550 : 900 = 31 : 18
408
´ 5 = 1020
Hence, only (2), (3) and (4) are possible numbers.
79. (c) Total members in gym S, T and U in 2020 = 57 × 3 = 171
74. (b) Total students in college L =
2 \ Members in gym U in 2020 = 171 – 32 + 61 = 78
504 78
Total students in college K = ´ 100 = 900 Males in gym U = ´ 2 = 52
56 3
Hence, required percentage Females in gym U = 78 – 52 = 26
(1020 - 900) 120 32
= ´ 100 = Males in gym S = ´ 5 = 20
900 9 8
= 13% (approx) Females in gym S = 32 – 20 = 12
75. (c) P ® 6%, 12%, 18%, 24%, 30% Total male members of gym S and U = 52 + 20 = 72
P = 24% Total female members of gym S and U = 26 + 12 = 38
Hence, the number of students in course Q in college Hence, required percentage
408 (72 - 38) 34
L= ´ 76 = 1292 = ´ 100 = ´ 100
24 38 38
= 89.47% » 89%
50
76. (d) Total students in college O = 3 ´ 968 ´ = 1452 80. (e) Total members in gym U in 2020 = (48 – 20) = 28
100
So, M + F = 28
1 and, (M – F) = 4
\ Students in course P in college O = 1452 ´ = 363
4 \ M = 16, 12 and F = 12, 16
So, we cannot detemined the exact male members in
50
And, students in course Q in college J = 968 ´ gym U.
100
81. (d) Total members in gym R in 2015 and 2020
= 484 = 36 + 12 = 48
2022-22 SBI Clerk (Junior Associates) Mains Solved Paper-2022
48 This is satisfied
\ Females in gym R in 2015 and 2020 = ´ 5 = 30 From B,
8
Hence, required revenue = 30 ´ 20 = ` 600 1200 ´ x + 550 + 290
82. (c) Females in gym P in 2020 = Total members in gym Q = 1200 + 70
x
in 2015
Þ 70x = 840 Þ x = 12
= 20 This is not satisfied.
\ Males in gym P in 2020 = 80 – 20 = 60 From C,
Females in gym P in 2015 = 100 – 20 = 80
510 x + 840
\ Males in gym P in 2015 = 120 – 80 = 40 = 510 + 70
x
Sol. (83-87) :
Þ 70x = 840 Þ x = 12
Year Company A Company B Company C This is also not satisfied.
2016 116 62 55 Hence, only option (a) is correct.
2017 80 100 100 92. (d) Let total quantity of vessel P = 11x
2018 60 50 100 Quantity of juice in vessel P = 9x
Total 256 212 255 Quantity of water in vessel P = 2x
101. (a) Refer to, “The Budget sounded the bugle for fostering
Weight (h) = l2 - r 2 rural entrepreneurship for those depending on
= (5r) 2 - (4r) 2 = agriculture and traditional industries by announcing
9r 2
the setting up of 80 Livelihood Business Incubators
Þ h = 3r and 20 Technology Business Incubators this fiscal
According to question, with the aim to develop 75,000 skilled entrepreneurs
Þ pr1l - 2pr2 h = 64 p in agro-rural industry sector.”
It is evident that the government wants start-up
Þ p ´ 20r 2 - p ´ 36r 2 = 64 p founders to expand beyond metropolitan areas in
order to ensure that the agriculture sector receives
Þ 5pr 2 - 9pr 2 = 16 p the attention it deserves. It will emphasize that the
Þ 4r2 = 16 country’s rural economy will also grow. Option (a) is
Þ r =2 our choice here because it explains this fact, and the
\ r2 = 6 ´ 2 = 12 other options can be discarded because they don’t
h=3´2=6 follow the information in the passage.
So, volume of cylinder = pr2h This makes Option (a) the correct choice among the
= p ´ 12 ´ 12 ´ 6 given options.
= 864p 102. (d) It has been described in the passage that the
Hence, none of the option is correct. government has announced a number of steps in order
95. (b) Ratio = 1 : 2 : 4 to boost the startup economy of the country. The
Capacity = 7 ´ 4x = 28x government has announced tax exemptions and also
Empty rate of all three pipes = x+ 2x + 4x = 7x more networking in order to help the startups develop
more and more. Among the given options, we can
28 x
\ A= = 4 hours easily pick up Option (d) whereas Option (a) is correct
7x in the first part but for the second part, we can say
84 28 16 that no question has been put forward by the
´ 120 + ´ 225 = ? + ´ 100
96. (a)
100 100
Þ 100 + 63 = ? + 16
Tg:- @NextGenBankers
100
government for the startup sector of the country. Other
options can be eliminated since they do not at all
follow from the passage.
Þ ? = 147 This makes Option (d) the correct choice among the
2856 given options.
97. (e) ´ 15 = ? +220 103. (d) Statement I is not correct for the fact that the new
102
channel has been proposed to se set up and run
Þ 28 ´ 15 = ? + 220
by the startup companies themselves so that they
Þ ? = 420 - 220 = 200
can utilize the platform properly. Refer to, “A TV
90 æ 682 ö channel exclusively ......surprise to many.”( third
98. (c) ´ 320 + ç ÷ ´ 16 = ?´ 8 paragraph)
100 è 961 ø
Statement II is also not correct because there is no
Þ 288 + 352 = ? ´ 8
such reference in the passage.
640 Statement III is correct because from the above quoted
Þ ?= = 80 lines it can be understood that the new channel will
8
try to match the startup founders with the VCs so that
70 6 3 they can raise capital and also plans the taxes so that
99. (a) ´ 480 + ´ 490 + ´ 273 = ?
100 7 7 there is tax saving by these small companies.
Þ 336 + 420 + 117 = ? Hence Option (d) is the correct choice among the
Þ ? = 873 given options.
104. (c) Among the available choices, Option (c) explains
1273 æ 6360 ö the author’s reaction, while the others can be
100. (b) ´ 15 + ç ÷ ¸ 64 = ? ´ 12
361 è 2809 ø skipped because they don’t follow the passage.
hence Option (c) is the correct choice among the given
1273 æ 6360 ö options.
Þ ´ 15 + ç ÷ ¸ 8 = ? ´ 12
19 è 53 ø 105. (b) According to Statement I, start-ups that receive
Þ 1005 + 15 = ? ´ 12 funding from angel investors are subject to a tax
known as “Angel Tax.” Therefore, it is correct that
1020
Þ ?= = 85 this tax will only apply to businesses that raise capital
12 from this source.
2022-24 SBI Clerk (Junior Associates) Mains Solved Paper-2022
Statement II is incorrect because it cannot be said 111. (a) Among the given options, ‘transmissible is the right
that the country’s Income Tax Act does not contain choice since the given word implies that the human
such a provision. behavior is contagious in nature. Their consumption
Statement III is incorrect because, it applies to all and their habits are influenced by their natural
businesses, not just those that have chosen to close surroundings.
their doors. 112. (b) Among the given options, ‘wise’ is the right choice.
Because of this, Option (b) is the best option out of Bold and audacious
all the ones offered. Are the synonyms of foolhardy. We need antonym.
106. (e) The passage claims that because most of our actions Since the behavioral change adopted by a home
are influenced by the behaviour of those around us, shopping channel led to skyrocketing sales but it was
human behaviour is contagious in nature. Our brains rather an uncalculated move.
are designed to foster relationships with others. As a 113. (d) The fragment most suitable as the first one is D.
result, we often copy the actions, preferences, and Pairs:
other behaviours of others. As a result, of the D and A A and C C and B
available possibilities, option (e) is the best one.
The correct sequence is D-A-C-B. Hence, option (d)
107. (b) According to the passage, the behavioural science
is correct.
principle of social influence made a popular
114. (b) The ideal first fragment is C clearly.
American infomercial for a home shopping channel
Pairs
change by making customers believe that the
operators are waiting for them at the other end of C and B B and A A and D
the line. This simple change led to its sales The correct sequence is C-B-A-D. Hence, option (b)
skyrocketing. So, this makes (b) the right choice is correct.
among the given options. 115. (a) The most suitable first fragment is A here.
108. (d) According to the passage, the use of canned laughter Pairs:
causes an audience to laugh longer and more often A and C C and B B and D
when a humorous material is presented. Also, the
The correct sequence is A-C-B-D. Hence, option (a)
109. (b)
Tg:- @NextGenBankers
canned laughter is most effective for poor jokes as
well. This makes option (d) an apt choice.
According to the passage, the experiment was called
is correct.
116. (e) The original sequence is correct and there is no need
to rearrange it.
as “People Power” because it gives people the power
117. (a) The first fragment is clearly D as no other fragment
to make a difference at no cost. So, this makes (b) an
provides a sensible starting point.
apt choice among the given options.
Pairs:
110. (c) Among the given options, ‘Escalating’ is the right
choice since the given word implies that the D and A A and C C and B
behavioral change adopted by a home shopping The correct sequence is D-A-C-B.
channel led to its rising sales.
118. (b) Here, the preposition ‘of’ must follow the verb
Fragment I The supreme court of India disapproved
‘disapprove’ to make it correct.
121. (e) All fragments are correct, no errors. Option (e) is cannot be placed elsewhere, and placing the B
hence the correct answer. sentence elsewhere makes the sentence context
122. (c) Offered, Containment: Should replace each other in absurd. Also, the demonstrative pronoun “This” at
order to make the sentence grammatically and the beginning of sentence B goes well with the noun
contextually correct. Hence, option (c) is the right “step” at the end of sentence C. So proposition B
answer choice. follows.
123. (d) Testing, Protocol: Should replace each other in order Keywords that connect B to C:
to make the sentence grammatically and contextually ‘this’ (B) – ‘step’ (C)
correct. Hence, option (d) is the right answer choice G follows the sequence as it welcomes and describes
124. (e) The sentence is correct. Hence, option (e) is the right the idea discussed in the first sentence.
answer choice. Keywords/phrases that connect G to C:
125. (c) Motivated, Development: Should replace each other ‘such a benefit’ (G) – ‘enhancement of paid maternity
in order to make the sentence grammatically and leave for women’ (C)
contextually correct. Hence, option (c) is the right E clearly seems to follow the sequence as it brings a
answer choice contrast to the point being discussed.
126. (a) Keywords/phrases that connect E to G:
127. (d) The original sentence is erroneous. ‘the amended law’ (E) – ‘an amendment’ (G)
Tg:- @NextGenBankers
Reason: The word ‘comprehension’ must be
followed the preposition ‘of’ instead of ‘off’ in this
F immediately follows as it supports the argument
made in sentence E.
context. Keywords/phrases that connect F to E:
The sentence after replacement becomes: ‘For many poor millions in the unorganised sector’
Economic policies formulated by the IAS officers are (F) – ‘a small subset of women in the workforce’ (E)
generally above the power of comprehension of most D must follow the sequence made so far as it criticizes
of the ministers. the current benefit scheme for women in the
128. (e) The original sentence is absolutely correct and hence unorganised sector whle quoting government’s apathy
the bold part needs no replacement. towards the women in unorganised sector.
129. (b) The original sentence is erroneous. Keywords/phrases that connect D to F:
Reason: The phrase ‘cave off’ is incorrect. ‘this meagre benefit’ – ‘a small conditional cash
The correct phrasal verb is ‘cave in’ which means benefit of Rs. 6,000’
‘capitulate or submit under pressure’. Now, the only sentence that is left is sentence A and it
The sentence after replacement becomes: will obviously take the last position in the sequence.
Non-performing assets have led to a seizure of new Hence, the correct sequence of sentences will be C-
lending and the caving in of credit culture. B-G-E-F-D-A.
130. (b) 131. (d) The second sentence is clearly B.
Sol (131-135) : 132. (c) The fourth sentence is clearly F.
In this article, the authors appear to applaud the Option (c) is hence the correct answer.
government’s move to provide benefits to women 133. (a) The first sentence is clearly C
working in the organized sector, while at the same 134. (e) The last but one (second from the last) sentence is
time criticizing the government’s neglect of other clearly D.
women in the unorganized sector. Sentence C clearly 135. (b) The third sentence is clearly G.
sets the tone of the passage and should be the first 136. (c) In option (c), ‘aspirations’ absolutely fits the context
statement. as well grammar. Aspiration is different from ‘need’
Finding the second movement of a sequence is a rather in a way that “need” refers to the necessities of an
difficult task. At first glance, the G sentence appears individual whereas ‘aspiration” is more about
to come next, but if you choose it, the B sentence luxurious desires of an individual.
2022-26 SBI Clerk (Junior Associates) Mains Solved Paper-2022
137. (d) Both the words of option (d) suit the context as well 147. (d) Sberbank is a state-owned banking and financial
as fulfill the grammar rules. services company of Russia headquartered in Moscow.
138. (a) It is very obvious that terrorist would do something As of 2014 it was the largest bank in Russia
negative. Thus both the words should be negative in and Eastern Europe, and the third largest in Europe,
nature. ranked 60th in the world and first in central and Eastern
Option (a) – Both the words absolutely fit the blanks. Europe in The Banker’s Top 1000 World Banks
139. (c) A strong spirit always motivates others, thus the ranking. In the world ranking of public companies
second blank must have words like motivate, inspire, Forbes ”Global 2000" Sberbank takes 51st place.
etc. 148. (b) First time in history, India has crossed USD 400
Hence option (c) is correct Billion billion in merchandise exports. India has, for
140. (c) Dealt and coloured do not make sense in the first blank the first time, met the government’s annual export
and are incorrect. Thus, options B and D can be target since 2014. The country crossed the crucial
eliminated. threshold of $400-billion annual merchandise export
Out of options (a) and (c), (a) is the only option where target.
both words fit both the blanks respectively 149. (c) India’s central bank, The Reserve Bank of India (RBI)
Hence, option (c) is correct. in a discussion paper has said the non-banking financial
141. (d) The Nobel Prize in Chemistry 2022 was awarded to services will be regulated in a four layered structure.
Carolyn R. Bertozzi, Morten Meldal, K. Barry There will be four tiers of NBFCs – Base Layer
Sharpless. The Nobel Prize in Chemistry 2022 was (NBFC-BL), Middle Layer (NBFC-ML),Top layer.
awarded to Carolyn R. Bertozzi, Morten Meldal, and 150 (b) The Holcim Group, legally known as Holcim Limited,
K. Barry Sharpless “for the development of click is a Swiss multinational company that manufactures
chemistry and bio orthogonal chemistry,” which building materials. It has a presence in around 70
involve simple, quick chemical reactions that can occur countries, and employs around 72,000 employees.
within living organisms without disrupting normal 151 (a) Capital, Asset Quality and Capital-To-Risk Weighted
biological functions. Assets Ratio(CRAR), NPA ratio, Tier I Leverage Ratio,
142. (d) As announced in the Monetary Policy Statement 2022- will be the key areas for monitoring in the revised
Tg:- @NextGenBankers
23 dated December 07, 2022, the Bank Rate is revised
upwards by 35 basis points from 6.15 per cent to 6.50
per cent with immediate effect.
framework.
152. (a) Banking Ombudsman is a quasi judicial authority
created in 2006, and the authority was created pursuant
143. (b) “The existing FTP 2015-20 which is valid up to to a decision made by the Government of India to
September 30, 2022 is extended up to March 31, enable resolution of complaints of customers.
2023,” . FTP provides guidelines for enhancing exports 153. (d) Richest self made women according to Hurun Global
to push economic growth and create jobs. It was first Rich List 2022 is Wu Yajun. Wu Yajun is a Chinese
extended on March 31, 2020 for one year due to the billionaire businesswoman who was at one time the
coronavirus outbreak and the lockdown. world’s richest self-made woman. She is the co-
144 (a) India was ranked 40th position out of 132 in the Global founder, chairwoman, and former CEO of Longfor
Innovation Index (GII) 2022 rankings released Properties.
by World Intellectual Property Organization (WIPO). 154. (d) Kumbhalgarh is also known as the Great Wall of
Switzerland is the most innovative economy in the India is a Mewar fortress on the westerly range of
world in 2022 - for the 12th year in a row - followed by Aravalli Hills, just about 48 km from Rajsamand
the United States, Sweden, the United Kingdom and city in the Rajsamand district of the Rajasthan state in
the Netherlands. western India. It is located about 84 km from Udaipur.
145. (a) Bomb Cyclone is related to USA. Bomb cyclone is It is a World Heritage Site included in Hill Forts of
actually used by meteorologists to indicate a mid- Rajasthan. It was built during the 15th century by Rana
latitude cyclone that intensifies rapidly. It is a massive Kumbha.
winter storm hammering the coast, bringing strong 155 (b) The cheetah was declared extinct from India in 1952.
winds, flooding, ice and snow. It is a combination of The Cheetahs that would be released are from Namibia
rapidly declining pressure and extreme cold. and have been brought under an MoU signed earlier
146. (b) China is the largest Producer of Cement. China this year. The introduction of Cheetah in India is being
produces the most cement globally by a large margin, done under Project Cheetah, the world’s first inter-
at an estimated 2.5 billion metric tons in 2021. China’s continental large wild carnivore translocation project.
cement production share equates to over half of the 156. (c) The SDR( Special Drawing Right) is an international
world’s cement. India was the world’s second-largest reserve asset, created by the IMF in 1969 to
cement producer, with production amounting to a supplement its member countries’ official reserves.
distant 330 million metric tons in 2021.
SBI Clerk (Junior Associates) Mains Solved Paper-2022 2022-27
157. (c) Indian plans to expand its regional satellite navigation commitment of the Gorbachev administration to
system NavIC (Navigation in Indian Constellation), allowing Soviet citizens to discuss publicly the
to increase its use in the civilian sector and ships, problems of their system and potential solutions.
aircraft travelling far from the country’s borders. The 167. (c) India imposed the windfall profit tax, from July, joining
constellations’ first satellite (IRNSS-1A) was launched a nations that tax super normal profits of energy
on 1st July 2013 and the eighth satellite IRNSS-1I was companies. The government has maintained that the
launched in April 2018. With the seventh launch of levy was introduced in view of the windfall gains made
the constellation’s satellite (IRNSS-1G), IRNSS by the domestic crude producers and refiners due to
was renamed NavIC by India’s Prime Minister in high global crude and product prices.
2016. 168. (a) The government has listed for sale real estate assets
158. (a) The Global Liveability Index has risen sharply in 2022. of state-run telecom firms MTNL and BSNL at a
Scores of Education, healthcare, and environment have reserve price of Rs 970 Crore.
improved, following the ease of restrictions post 169. (b) The 2019World Population Data Sheet indicates that
Covid-19. Austrian Capital, Vienna, topped the Global the world population reached 7.7 billion in 2019. By
Liveability Report, 2022, with a 99.1 rating. It replaced 2050, India will surpass China as the most populous
Auckland, which topped in 2021. country in the world, with an estimated 1.67 billion
159. (d) EU KLEMS is an industry level, growth and people.
productivity research project. EU KLEMS stands for 170. (a) Meghalaya won the best performer award in the 3rd
EU level analysis of capital (K), labour (L), energy edition of ranking of states on support to the start-up
(E), materials (M) and service (S) inputs. ecosystem which was released by Piyush Goyal,
160. (a) The 12 nationalised banks include Punjab National Minister of Commerce and Industry in a felicitation
Bank (PNB), Bank of Baroda (BoB), Bank of India ceremony in New Delhi .
(BoI), Central Bank of India, Canara Bank, Union 171. (a) As per the States, Madhya Pradesh has topped the
Bank of India, Indian Overseas Bank (IOB), Punjab, chart in the category of best-performing states in the
and Sind Bank, Indian Bank, UCO Bank, Bank of Swach Survekshan Awards 2022.
Maharashtra, and State Bank of India (SBI). 172 (b) NARCL( National Asset Reconstruction Limited.) is
Tg:- @NextGenBankers
161. (a) The Indian Super League, officially known as the Hero
Indian Super League for sponsorship reasons, is an
a government entity, has been incorporated on 7th July
2021 with majority stake held by Public Sector Banks
Indian professional league for men’s association and balance by Private Banks with Canara Bank being
football clubs. the Sponsor Bank.
162. (b) Vishing is a cyber crime that uses the phone to steal 173. (d) The survey aims at capturing subjective assessments
personal confidential information from victims. Often on price movements and inflation, based on their
referred to as voice phishing, cyber criminals use individual consumption baskets, across 19 cities, viz.,
savvy social engineering tactics to convince victims Ahmedabad, Bengaluru, Bhopal, Bhubaneswar,
to act, giving up private information and access to bank Chandigarh, Chennai, Delhi, Guwahati, Hyderabad,
accounts. Jaipur, Jammu, Kolkata, Lucknow, Mumbai, Nagpur,
163 (a) The Board of Trustees of the PM CARES Fund has Patna, Raipur, Ranchi.
also nominated following to the Advisory Board of 174. (b) The Reserve Bank has approved the appointment of
the Trust are Shri Rajiv Mehrishi, Smt. Sudha Murthy its former deputy governor R Gandhi as non-executive
and Shri Anand Shah. part time chairman of Yes Bank for three years.
164. (d) India’s external debt, at US$ 620.7 billion as at end- 175. (e) RBI, APR include cost of funds, credit cost and
March 2022, grew by 8.2 per cent over US$ 573.7 operating cost, processing fee, verification charges,
billon as at end-March 2021. External debt of the maintenance charges, and exclude penalty charges, late
country continues to be dominated by the long-term payment charges.
borrowings, but long term borrowings reduces in 2022- 176. (b) The Indian Central Board of Indirect Taxes and
23 as stated by Government of India. Customs (CBIC) has refuted reports of a cut to the
165. (b) Eliud Kipchoge, kenyan marathon runner won annual sales threshold for clearing e-invoices to ¹ 5
Olympic 2020. Previously, he won the Gold Medal in per annum for 1 January 2023. It had last been
2016 olympics Eliud Kipchoge EGH is a Kenyan long- decreased to ¹ 10 annual sales.
distance runner who competes in the marathon and 177. (c) Hurun India finds 1,103 individuals with INR 1,000
formerly specialized at the 5000 metre distance. crore wealth, up by 96, an increase of 62% over the
166. (c) Glasnost was taken to mean increased openness and last five years With INR 10,94,400 crore, Gautam
transparency in government institutions and activities Adani (60) overtakes Mukesh Ambani (65), to bag the
in the Soviet Union (USSR). Glasnost reflected a richest Indian title
2022-28 SBI Clerk (Junior Associates) Mains Solved Paper-2022
178. (a) The maximum amount of debt (fund based or non- 184. (a) In order to improve complaint redressal in finance
fund based facilities) eligible for guarantee cover under companies, the Reserve Bank of India (RBI) will
the scheme is Rs. 10 crore per borrower, irrespective introduce the Internal Ombudsman Scheme (IOS)
of the amount of debt facilities extended to the for NBFCs having a higher customer interface.
borrower by MI(s). 185. (b) RBI lifts curbs on M&M Finance for loan recovery
179. (b) The index represents the single discount rate that through outsourcing. On September 22, 2022, the
would produce the same present value as calculated Reserve Bank of India had directed Mahindra &
by discounting a standardized set of liabilities using Mahindra Financial Services to immediately cease
the PDC. carrying out any recovery or repossession activity
180. (b) India has signed MOU with world Bank to raise Fund through outsourcing arrangements.
for Pradhan Mantri Matsya Sampada Yojana. Pradhan 186. a) Under Pradhan Mantri Garib Kalyan Yojana
Mantri Matsya Sampada Yojana is a scheme to bring (PMGKY), the limit for collateral-free loans for
about Blue Revolution through sustainable and women’s SHGs was doubled from Rs.10 lakh to Rs.
responsible development of fisheries sector in 20 lakh.
India” with highest ever investment of Rs. 20050 187. (c) The Global Innovation Index is an annual ranking of
crores in fisheries sector comprising of Central share countries by their capacity for, and success in,
of Rs. 9407 crore, State share of Rs 4880 crore and innovation, published by the World Intellectual
Beneficiaries contribution of Rs. 5763 crore. PMMSY Property Organization. It was started in 2007 by
will be implemented over a period of 5 years from FY INSEAD and World Business, a British magazine. It
2020-21 to FY 2024-25 in all States/Union Territories. reveals the most innovative economies in the world,
181. (d) The Education 4.0 India report is the result of a ranking the innovation performance of around 132
collaboration between the World Economic economies.
Forum, the United Nations Children’s Education Fund 188. (c) The National Asset Reconstruction Company Ltd.
(UNICEF) and YuWaah (Generation Unlimited (NARCL), set up to take over large bad loans of more
India). It tracks the progress and findings of the than¹ 500 crore from banks. The SARFAESI Act,
Education 4.0 India initiative, which focuses on how 2002 provides the legal basis for the setting up of ARCs
Tg:- @NextGenBankers
Fourth Industrial Revolution technologies can enhance
learning and reduce inequalities in access to education
in India. Setting up of NARCL, the proposed bad bank
for taking over stressed assets of lenders, was
among children in India. announced in the Budget for 2021-22.
182. (c) NSE Clearing has empanelled 15 clearing 189. (c) 16 companies are in the NBFC-UL list are: LIC
banks namely Axis Bank Ltd., Bank of India Ltd., Housing Finance, Bajaj Finance, Mahindra &
Canara Bank Ltd., Citibank N.A., The Hongkong & Mahindra Financial Services, Shriram Transport, Tata
Shanghai Banking Corporation Ltd., ICICI Bank Ltd., Sons, L&T Finance, Indiabulls Housing Finance,
HDFC Bank Ltd., IDBI Bank Ltd., IndusInd Bank Ltd., Piramal Capital & Housing Finance, Cholamandalam
JPMorgan Chase Bank, Kotak Mahindra Bank Ltd., Investment and Finance Co., Shanghvi Finance Pvt.
Standard Chartered Bank, Union Bank of India, State Ltd, Muthoot Finance, PNB Housing Finance, Tata
Bank of India and Yes Bank. Capital Financial Services, Aditya Birla Finance, HDB
183. (c) PM Kisan Maandhan Yojana that provides an assured Financial Services and Bajaj Housing Finance.
monthly pension of Rs. 3000/- to all the small and 190. (b) The EXIM Letter of Credit policy can reduce a bank’s
marginal farmers (who own cultivable land up to 2 risks on confirmations and negotiations of irrevocable
hectares) after the age of 60 years. This scheme was letters of credit issued by overseas financial institutions
introduced with an aim to secure the lives of small for the financing of U.S. exports.
and marginal farmers in India.
SBI Clerk (Junior Associates) Prelim Solved Paper-2022 2022-29
two more than the number of persons below V. Q is living above Juhi went 30 kilometers to the coast from my house, then
R, which is not living below S. turned left and walked 10 kilometers. She then turned west
13. How many persons are living between R and P? and walked 25 kilometers and finally turning left covered
(a) None (b) Two 10 kilometers. How far was he from his house?
(c) More than three (d) One (a) 20 kilometers east (b) 40 kilometers
(e) Three (c) 30 kilometers (d) 5 kilometers east
14. Who is living three places below U? (e) 10 kilometers east
(a) V (b) Q (c) S (d) T
(e) R DIRECTIONS (Qs. 23-27): Study the following information
15. Number of persons living below Q is same as the number carefully to answer the given questions.
of persons living above _______. Eight boxes A, B, C, D, W, X, Y and Z are placed in a four-story
(a) P (b) R (c) S (d) T almirah such that ground rack is numbered as 1, the rack
(e) None of these immediately above rack 1 is numbered as 2 and so on. Each of
16. Which of the following is false? the rack has two portions in it as portion P and portion Q.
A. T is not living at the bottom. Portion Q is to the east of portion P. Portion P of rack 2 is
B. V is living immediately above R. immediately above the portion P of rack 1 and immediately below
C. More than four persons are living between Q and S. the portion P of rack 3 and so on. In the same way, portion Q of
(a) Only B and C (b) Only A and C rack 2 is immediately above the portion Q of rack 1 and
(c) Only C (d) Only A immediately below the portion Q of rack 3 and so on.
(e) Only A and B D is placed immediately above the portion in which A is placed
17. If all the persons are arranged to place in the alphabetical in same portion but not in portion Q. There are two racks between
order from top to bottom, then how many persons remain the rack in which A and Z placed. There is only one rack between
unchanged? the rack on which Z and B placed. Y who is placed on an even
(a) One (b) Two (c) Three (d) None numbered rack places above the rack in which C placed. C is
(e) More than three placed below the rack in which X is placed. C and X is placed in
18. How many such pairs of letters are there in the word the same portion but not with Z.
“FREQUENT” which has as many letters between them in 23. In which portion and on which rack does W placed?
the word (in both forward and backward directions, as (a) Portion Q, rack 3 (b) Portion P, rack 4
(a) One Tg:- @NextGenBankers
they have between them in the English alphabetical series?
(b) Two
(e) More than three
(c) Three (d) None
(c) Portion P, rack 3
(e) Portion P, rack 2
(d) Portion Q, rack 1
29. If we remove all the symbols from the above series then 39. I. 12x2 – 46x + 40 = 0 II. 6y2 – 30y + 24 = 0
which among the following alphabet is the 9th from the (a) x>y (b) x < y
right end? (c) x³y (d) x £ y
(a) S (b) E
(e) x = y or relation between x and y can be established.
(c) B (d) None of these
40. I. a2 + 30a + 81 = 0 II. b2 + 26b + 48 = 0
(e) R
30. Which among the following elements is 5th to the left of (a) a<b
7th from the right end of the series? (b) No relation in a and b or a = b
(a) None of these (b) U (c) a>b
(c) B (d) & (d) a£b
(e) @ (e) a³b
31. How many such vowels are in the series given each of 41. I. x2 + 7x – 60 = 0 II. y2 – 11y + 30 = 0
which is immediately followed by symbol and immediately (a) x£y (b) x > y
preceded by a letter? (c) y³x (d) x < y
(a) Two (b) One (c) Three (d) Four (e) No relation in x and y or x = y
(e) Five 42. I. 2x2 + 26x + 84 = 0 II. y2 + 21y + 104 = 0
32. If all the symbols are replaced with a digit 5 then how (a) x<y (b) x > y
many such 5’s are there in the new series each of which is (c) y£x (d) x ³ y
followed by a consonant? (e) x = y or the relation between x and y can’t be
(a) Five (b) Seven (c) Four (d) Eight established.
(e) Two
43. What approximate value should come in the place of
DIRECTIONS (Qs. 33-35): Read the following information question mark (?) in the following question?
carefully and answer the questions that follow: [(14.98 + 8.95) ÷ 0.99] × 3.98 = ?4 + 10.98
There are six friends in a class Z, Y, X, W, V and U who score (a) 11 (b) 8 (c) 2 (d) 3
different marks in an exam. All are sitting in a row according to (e) 5
their marks. Y scores the second highest mark. Marks of U is 44. What approximate value should come in place of the
less than W. Z scores fewer marks than X and V. U is the third question mark (?) in the following question?
(221.22 + 72.90) ÷ 5.99 = 3.99% of 199.99 + 19.99 + ?
Tg:- @NextGenBankers
highest score in the group and has scored 46 marks. W is the
highest scorer in the group.
33. Who score second highest in the group?
(a) 31
(e) 51
(b) 41 (c) 23 (d) 11
(a) X (b) Y (c) V (d) U 45. What approximate value should come in place of the
(e) Z question mark (?) in the following question?
34. Who score second lowest marks in the group? 219.89 + 2.993 + 199.96 = 2.99 × ? + 209.98
(a) V (b) Y (c) X (d) W (a) 90 (b) 100 (c) 120 (d) 79
(e) Can’t be determined. (e) 110
35. If sum of U and Y mark is 106, then what is possible marks of W. 46. What will come in the place of the question mark ‘?’ in the
(a) 50 (b) 62 (c) 48 (d) 47 following question?
(e) 46
Numerical Ability 26% of 300 + 30 × 17 – 961 × 4 = ?
(a) 472 (b) 473 (c) 471 (d) 464
36. If the numerator of a certain fraction is increased by 40%, (e) None of these
while the denominator is increased by 100%, then the 47. What will come in the place of the question mark ‘?’ in the
ratio of the numerator and denominator becomes 42 : 75, following question?
what will be the actual fraction? 40% of 70 = 25 × 4 – ? + 6 × 4
(a) 5/7 (b) 4/5 (c) 7/10 (d) 8/11 (a) 47 (b) 37 (c) 96 (d) 10
(e) 5/8 (e) None of these
37. A certain work was completed by A in 7 days, B in 14
48. What will come in the place of the question mark ‘?’ in the
days, C in 21 day. If A and C did work for 3 days and then
left then B will finishes rest work in how many days? following question?
(a) 4.5 (b) 4 (c) 5.5 (d) 3 131 × 3 + 63 = 70 ÷ 7 + ?% of 21 + 5
(e) 3.5 (a) 1400 (b) 2000 (c) 2100 (d) 2150
(e) None of these
DIRECTIONS (Qs. 38-42): In the given questions, two 49. What will come in the place of the question mark ‘?’ in the
equations numbered I and II are given. You have to solve both following question?
the equations and mark the appropriate answer.
17 × 16 + 60% of 250 = 25% of 40 + ? × 13
38. I. 5a2 + 26a + 33 = 0 II. b2 + 18b + 65 = 0
(a) 17 (b) 11 (c) 19 (d) 16
(a) a > b (b) a < b (c) a ³ b (d) a £ b
(e) None of these
(e) a = b or relation between a and b can not established.
2022-32 SBI Clerk (Junior Associates) Prelim Solved Paper-2022
50. What will come in the place of the question mark ‘?’ in the 58. Find the total number of the Bat and Gloves sold by T &
following question? T company in 2020?
22 × 23 – 40% of 800 = 36 × ? (a) 700 (b) 695 (c) 705 (d) 585
(a) 33 (b) 34 (c) 38 (d) 31 (e) 735
(e) None of these 59. If each company has targeted to increase their sale of gloves
51. What will come in the place of the question mark ‘?’ in the by at least 25% in 2020, but it was found that I, J and K have
following question? scale up their target only by 20%, while M has scale up by
? % of 400 × (3/4) + 300 = 66.66% of 900 30%. The sale of Gloves by M in 2020 is approximately
(a) 50 (b) 101 (c) 260 (d) 180 what percent to the sale of I, J and K in 2020?
(e) 70 (a) 83% (b) 85% (c) 87% (d) 77%
52. What will come in the place of the question mark ‘?’ in the (e) 80%
following question? 60. If the sale for Bat increased by 50% in 2020, and sale for
25% of 84 ÷ 20% of 140 = 2 + 11% of 500 + ? Ball is increased by 40% in 2020 for company M, then
(a) – 8 (b) – 6 (c) – 4 (d) 8 what will be the new ratio of sale of Bat and Ball in 2020?
(e) 4 (a) 5 : 17 (b) 15 : 7 (c) 17 : 11 (d) 7 : 15
53. What will come in the place of the question mark ‘?’ in the (e) 8 : 13
following question? 61. If in 2020, the total sales of bat for all the shops are
increased by 120%, what will be the number of bats sold
121 + 81 ÷ 9 = 144 ÷ 36 + (?)2 by company M in 2019 and 2020, if it was considered that
(a) 8 (b) 6 (c) 4 (d) 3
sales of other company are same as 2019 except M and L.
(e) None of these
M and L sell an equal number of bat in 2020?
54. What will come in the place of the question mark ‘?’ in the
(a) 1045 (b) 1095 (c) 1535 (d) 1120
following question?
(e) 995
33 × 25 – 30% of 200 = 25% of 1600 – ? × 59 62. What will be the total ratio of the Bat and the Gloves sold
(a) 7 (b) 8 (c) 9 (d) 5 by all of the company in 2019?
(e) None of these (a) 65 : 31 (b) 64 : 31 (c) 67 : 31 (d) 68 : 31
55. What approximate value will come in the place of the (e) 69 : 31
question mark ‘?’ in the following question? 63. A boat having a speed of 12 kmph covering a distance of
68. A sum of money gives an interest of 2/5 of itself in T DIRECTIONS (Qs. 76-80): In each question below, four words
years, when it is given in the rate of 20%. Find amount printed in bold type are given. These are numbered (a), (b),
after (T + 4) years. (c) and (d). One these words printed in bold might either be
(a) 2.2 P (b) 2.25 P (c) 2.35 P (d) 2.5 P wrongly spelt or inappropriate in the context of the sentence.
(e) 2.6 P Find out the word that is inappropriate or wrongly spelt, if
69. Monthly income of Ram was ` 18,000, if the spent 25% on any. The number of the word is your answer. If the words printed
entertainment, 12% on his travelling and save x% in bank in bold are correctly spelt and appropriate in the context of
account. After the calculation, he found that he was left the sentence then mark (e), i.e. ‘All Correct’, as your answer.
with ` 2160. Find the sum of amount which he saves in his
bank. 76. Ramesh had an aversion (a) / to alcohol and would (b) / avoid
(a) ` 8840 (b) ` 9180 (c) ` 10520 (d) ` 12140 going to partyes (c) / with his friends. (d) / All correct (e)
(e) ` 14720 (a) aversion (b) would
70. A train having a certain length of x m covers the two (c) partyes (d) friends
different platforms having lengths of 280 m and 360 m in (e) All correct
36 sec and 40 sec, respectively. Find speed of train (in 77. Ramesh spiled (a) / juice all over Raj’s new (b) / clothes and
kmph). did not even care (c) / to apologise. (d) / All correct (e)
(a) 80 kmph (b) 78 kmph (c) 75 kmph (d) 72 kmph (a) spiled (b) new (c) care
(e) 70 kmph (d) apologise (e) All correct
78. Daisy loved (a) / children and so she would distrebute (b)
English Language / sweets in an orphanage (c) / on her birthday. (d) / All
correct (e)
DIRECTIONS (Qs. 71-75): Rearrange the following six sentences (a) loved (b) distrebute
(A), (B), (C), (D), (E) and (F) in a proper sequence to form a (c) orphanage (d) birthday
meaningful paragraph. Then answer the questions given below. (e) All correct
79. All the competitors (a) / completed (b) / the race, (c) /
(A) Detritus that the worm deems unworthy may become a
with just one exeption. (d) / All correct (e)
meal for other tiny beach denizens, such as crabs, shrimp
(a) competitors (b) completed
and clams.
(c) race (d) exeption
(B) This is where the worm’s “fat innkeeper” nickname comes
(e) All Correct
from.
Tg:- @NextGenBankers
(C) From the front end of its burrow, the worm coughs up a
net of mucus to catch tiny seaside plankton, bacteria and
80. Lucy was working (a) / overtime because (b) / she had to
prepare (c) / for an important meating. (d) / All correct (e)
(a) working (b) because (c) meating (d) prepare
other detritus that happen to pass by.
(e) All correct
(D) It does this by spraying a jet of water out of its butt.
(E) When the worm sucks this net back into its mouth, it DIRECTIONS (Qs. 81-88): Read the following passage
holds onto choice morsels and tosses the rest away carefully and answer the questions given below. Some words
through the back end of its burrow. are printed in bold in order to help you locate them while
(F) In fact, it’s common for the worm’s burrow to host various answering some of the questions.
opportunistic animals looking for a free bed and meal.
The catastrophic monsoon floods in Kerala and parts of
71. Which of the following should be the SIXTH statement
Karnataka have revived the debate on whether political
after rearrangement?
expediency trumped science. Seven years ago, the Western
(a) A (b) B (c) C (d) F
Ghats Ecology Expert Panel issued recommendations for the
(e) E
preservation of the fragile western peninsular region. Madhav
72. Which of the following should be the FIFTH statement
Gadgil, who chaired the Union Environment Ministry’s WGEEP,
after rearrangement?
has said the recent havoc in Kerala is a consequence of short-
(a) D (b) B (c) C (d) F
sighted policymaking, and warned that Goa may also be in the
(e) E
line of nature’s fury. The State governments that are mainly
73. Which of the following should be the FOURTH statement
responsible for the Western Ghats — Kerala, Karnataka, Tamil
after rearrangement?
Nadu, Goa and Maharashtra — must go back to the drawing
(a) A (b) B (c) C (d) F
table with the reports of both the Gadgil Committee and the
(e) D
Kasturirangan Committee, which was set up to examine the
74. Which of the following should be the THIRD statement
WGEEP report. The task before them is to initiate correctives to
after rearrangement?
environmental policy decisions. This is not going to be easy,
(a) A (b) B (c) C (d) D
given the need to balance human development pressures with
(e) F
stronger protection of the Western Ghats ecology. The issue of
75. Which of the following should be the SECOND statement
allowing extractive industries such as quarrying and mining to
after rearrangement?
operate is arguably the most contentious. A way out could be
(a) A (b) B (c) C (d) F
to create the regulatory framework that was proposed by the
(e) E
Gadgil panel, in the form of an apex Western Ghats Ecology
Authority and the State-level units, under the Environment
2022-34 SBI Clerk (Junior Associates) Prelim Solved Paper-2022
(Protection) Act, and to adopt the zoning system that it (a) The reports of the expert panel show no proper
proposed. This can keep incompatible activities out of the framework for controlling the Ecologically Sensitive
Ecologically Sensitive Zones (ESZs). Zone in the Western Ghat Area.
At issue in the Western Ghats — spread over 1,29,037 sq km (b) The Western Ghat region doesn’t have any
according to the WGEEP estimate and 1,64,280 sq km as per the problems, but the main issue is that governments
Kasturirangan panel — is the calculation of what constitutes don’t have enough money.
the sensitive core and what activities can be carried out there. (c) Because they will use up all of the groundwater in
The entire system is globally acknowledged as a biodiversity the area, the reservoirs in the area’s immediate vicinity
hotspot. But population estimates for the sensitive zones vary will spell disaster.
greatly, based on interpretations of the ESZs. In Kerala, for (d) Since policymakers don’t care about the environment
instance, one expert assessment says 39 lakh households are until it affects their voter base, the area will flood..
in the ESZs outlined by the WGEEP, but the figure drops sharply (e) The balance between development and preservation
to four lakh households for a smaller area of zones identified by should be there in order to develop the area properly.
the Kasturirangan panel. The goal has to be sustainable 84. Which among the following should be the objective of all
development for the Ghats as a whole. The role of big concerned regarding the development of the Western
hydroelectric dams, built during an era of rising power demand Ghats Area?
and deficits, must now be considered afresh and proposals for (a) The development plan should be well supported by
new ones dropped. Other low-impact forms of green energy led money and also manpower by all the states.
by solar power are available. A moratorium on quarrying and (b) The Western Ghats Area should be preserved properly
mining in the identified sensitive zones, in Kerala and also other so that there is sustainable development of the area.
States, is necessary to assess their environmental impact. (c) Since the development plan will help gain an
Kerala’s Finance Minister, Thomas Isaac, has acknowledged advantage throughout the process, it must be
the need to review decisions affecting the environment, in the correctly drafted from the beginning.
wake of the floods. Public consultation on the expert reports (d) The states should take the development of the
that includes people’s representatives will find greater Western Ghats region seriously so that the area is
resonance now, and help chart a sustainable path ahead. actually preserved.
81. Which among the following has been attributed by the experts (e) Because the states are not in charge of developing
as a reason of the recent floods in Kerala and Karnataka? the Western Ghats Area, this area should not be taken
Tg:- @NextGenBankers
(a) The political decision making strategy has always
taken the upper hand as compared to the real interests
of the environment.
85.
for granted.
Which among the following should be the course of
action of the government in order to ensure that the
(b) Rainwater always spills in these two states since Western Ghats Area is preserved properly?
there is an improper drainage infrastructure in place. I. There should be utilization of various clean sources
(c) The states are unable to handle any issue, no matter of energy such as the solar power in the area
how minor it may be, because they have no idea how II. There should not be any restriction in mining
to handle any type of natural calamity. activities as well as quarrying activities in the area
(d) The states should be entrusted with the responsibility III. There should not be new construction of
of protection of environment in the areas within their hydroelectric dams in the area from now onwards
jurisdiction. (a) Both I and III (b) Only II
(e) None of the above (c) Both I and II (d) Only I
82. Which of the following should the flood-affected states (e) All I, II and III
do to stop this from happening again, according to the 86. Which among the following is true regarding the
passage? regulatory framework recommended by the expert panel
(a) The states need to invest more money in avoiding regarding the conservation of the Western Ghats Area?
natural disasters there. (a) There should be one central authority along with
(b) In order for the federal government to anticipate the state-wise authorities so that the coordination is
possibility of any natural disaster, the states should proper for sustainable development of the area.
establish appropriate warning mechanisms. (b) There must be a central regulator that does not authorize
(c) The states should plan properly so that they can states to avoid any kind of dispute between them.
implement the recommendations of the expert panels (c) There should be a decentralized regulatory authority
regarding the preservation of the Western Ghats at the state level, accompanied by the heads of
Area. ministries.
(d) The states should not take any action at this time; (d) There should be only one regional authority to avoid
instead, they should concentrate solely on the idea confusion about the responsibilities of different
of putting everything into disaster management authorities.
operations. (e) None of the above
(e) None of the above 87. Which among the following is the view of the Finance
83. Which among the following is the main issue pointed out Minister of Kerala regarding the efforts towards
in the passage in the implementation of the expert panel development of the Western Ghats Area?
reports in various states?
SBI Clerk (Junior Associates) Prelim Solved Paper-2022 2022-35
(a) It should not be carried out since any development 95. Our company is yet starting (a) / offering this facility to
activity in region would definitely put a lot of people (b) / our customers as we are (c) / awaiting approval from
in problem. the Board. (d) / No error (e)
(b) It should not be stopped because the central
DIRECTIONS (Qs. 96-100): In the following passage, some of
government has already sanctioned it and the funds
the words have been left out, each of which is indicated by a
have also been released. letter. Find the suitable word from the options given against
(c) Kerala should get special assistance from the central each letter and fill up the blanks with appropriate words to
government in the present Finance Commission in make the paragraph meaningful.
view of the ongoing flood situation.
(d) The state government should be the sole deciding This year, the world’s largest democracy, India, and the biggest
authority regarding the amount of compensation to country by ___ (96) ___, Russia, are celebrating the 70th
anniversary of establishment of diplomatic relations between
be given.
them. Russia continues to be among India’s major politico-
(e) The state should engage public in the process of diplomatic and defence partner nations. While India has ___ (97)
consultation r egarding the blueprint of the ___ separate strategic partnership pacts with more than two dozen
development of Western Ghats Area countries, the Indian and Russian governments in December 2010
88. Which of the following sentences has a similar meaning ___(98)___ their bilateral ‘Strategic Partnership’ to what they
to the word “resonance” used in this sentence? termed a “Special and Privileged Strategic Partnership.”
(a) Delightful (b) Blissful The New Delhi-based ___(99)___ ‘Foundation for National
(c) Consonance (d) Credence Security Research’, which did a comparative assessment of India’s
(e) None of the above strategic partnerships — meaning, ‘political-diplomatic, defence
and economic cooperation’ (during the 10-year period prior to
DIRECTIONS (Qs. 89-95): Read the sentence to find out November 2011), had said, “Russia emerges as the most important
whether there is any grammatical or idiomatic error in it. The strategic partner of India (followed by the U.S., France, the U.K.,
error, if any, will be in one part of the sentence. Mark the part Germany and Japan in that order).” The November 2011 report
with the error as your answer. If there is no error, mark (e). had found that Russia had provided strong political and
(Ignore errors of punctuation, if any) diplomatic support to India and helped enormously in building
89. If I will have an even faster (a)/ computer than I have India’s defence capability. However, it warned that the “economic
content of the (India-Russia) partnership is extremely weak,” and
90.
Tg:- @NextGenBankers
today, (b)/ I could come up with really (c)/ interesting
questions to ask it.(d)/ No error (e)
One of my colleagues (a)/ who wanted to help me with(b)/
recommended that “urgent and ___(100)___ steps need to be
taken to improve economic relations if this (India-Russia)
partnership is to be sustained and made durable.”
my projects have been transferred (c)/to some other 96. (a) Population (b) Density
department (d)/ No error(e) (c) Area (d) Democracy
91. The type of qualities you acquire (a)/ depend upon your (e) Economy
company(b)/ and so you associate yourselves with (c)/ 97. (a) Inked (b) Considered
simple and good natured people.(d)/ No error(e) (c) Contemplated (d) Refuted
92. Everybody among the traders (a)/ were enjoying dance (e) Revoked
and music (b)/ when the manager of the (c)/event was 98. (a) Called (b) Elevated
shot dead.(d)/ No error(e) (c) Refreshed (d) Nullified
93. There are many insurance (a) / disputes nowadays (e) Revived
because of (b) / most people do not fully (c) / understand 99. (a) NGO (b) Startup
the terms and conditions of their policies (d) / No error (e) (c) Personnel (d) Think tank
94. The stubborn child (a)/ started crying and laid on (b)/ the (e) Avenue.
ground after his (c) / mother refused to get him a chocolate. 100. (a) Various (b) Precarious
(c) Vigorous (d) Minuscule
(d) / No error(e)
(e) Exhilarating
ANSWER KEY
1 (c) 11 (c) 21 (b) 31 (c) 41 (c) 51 (b) 61 (c) 71 (b) 81 (d) 91 (b)
2 (d) 12 (e) 22 (d) 32 (b) 42 (b) 52 (a) 62 (b) 72 (d) 82 (c) 92 (b)
3 (c) 13 (b) 23 (c) 33 (b) 43 (d) 53 (c) 63 (e) 73 (a) 83 (e) 93 (b)
4 (a) 14 (c) 24 (a) 34 (e) 44 (c) 54 (d) 64 (c) 74 (d) 84 (b) 94 (b)
5 (a) 15 (d) 25 (b) 35 (b) 45 (d) 55 (c) 65 (c) 75 (e) 85 (a) 95 (a)
6 (b) 16 (b) 26 (c) 36 (b) 46 (d) 56 (d) 66 (b) 76 (c) 86 (a) 96 (c)
7 (d) 17 (d) 27 (d) 37 (d) 47 (c) 57 (e) 67 (a) 77 (a) 87 (e) 97 (a)
8 (b) 18 (b) 28 (d) 38 (a) 48 (c) 58 (d) 68 (a) 78 (b) 88 (c) 98 (b)
9 (b) 19 (c) 29 (c) 39 (e) 49 (d) 59 (a) 69 (b) 79 (d) 89 (a) 99 (d)
10 (d) 20 (e) 30 (c) 40 (b) 50 (d) 60 (b) 70 (d) 80 (c) 90 (c) 100 (c)
2022-36 SBI Clerk (Junior Associates) Prelim Solved Paper-2022
+
B
Two i.e. R-U, Q N.
19. (c) Statement:
+ – V
J D G ||
A£E<Q
w Brother in law
in la Conclusion:
Son I. V = E (False) II.V < E (False)
+ – + – Either I or II is true.
L C Z F
20. (e) Statement:
“ =” ® husband wife relation
“—” ® Brother sister relation
“ | ” ® Children relation F>D
O+ ® Boy / male
O– ® Girl/female C<B
10. (d) J is father of Z. Conclusion:
11. (c) J is son in law of B. I. Z > D (True) II. D > C (False)
12. (e) B belongs to first generation. Only I is true.
SBI Clerk (Junior Associates) Prelim Solved Paper-2022 2022-37
26. (c) D
Tg:- @NextGenBankers
different portion. Þ y2 – 4y – y + 4 = 0 Þ y (y – 4) – 1 (y – 4) = 0
Þ (y – 1) (y – 4) = 0 Þ y = 1, 4
27. (d) D is placed below the rack in which W is placed. No relation between x and y can be established.
Sol. (28-32) : 40. (b) (I) a2 + 30a + 81 = 0
OU* J B^ $ K #C * CV BS RE &M P@ QU !G Þ a2 + 27a + 3a + 81 = 0 Þ a (a + 27) + 3 (a + 27) = 0
28. (d) “U * J”, “E & M”, “U ! G” Þ a = – 27, – 3
29. (c) 30. (c) 31. (c) (II) b2 + 26b + 48 = 0
32. (b) O U 5 J B 5 5 K 5 C 5 C V B S R E 5 M P 5 Q U 5 G Þ b2 + 24b + 2b + 48 = 0 Þ b (b + 24) + 2 (b + 24) = 0
Seven Þ (b + 2) (b + 24) = 0 Þ b = – 24, – 2
Sol. (33-35) : No relation can be made.
Highest — W 41. (c) (I) x2 + 7x – 60 = 0
— Y Þ x2 + 12x – 5x – 60 = 0 Þ x (x + 12) – 5 (x + 12) = 0
46 — U Þ (x – 5) (x + 12) = 0 Þ x = 5, – 12
— either X or V (II) y2 – 11y + 30 = 0
— either X or V Þ y2 – 6y – 5y + 30 = 0Þ (y – 6) (y – 5) = 0
Lowest — Z Þ y = 6, 5 Þ y ³ x
33. (b) Y score 2nd highest. 42. (b) (I) 2x2 + 26x + 84 = 0
34. (e) Can’t be determined. Dividing by “2” by b.t.s.
35. (b) u + y = 106 Þ y = 106 – 46 = 60 x2 + 13x + 42 = 0
w = 62 according to option. Þ x2 + 7x + 6x + 42 = 0Þ x (x + 7) + 6 (x + 7) = 0
36. (b) Let the numerator be 100x. Þ (x + 6) (x + 7) = 0 Þ x = – 6, – 7
Then the denominator be 100y. (II) y2 + 21y + 104 = 4
According to question, Þ y2 + 13y + 8y + 104 = 0 Þ y (y + 13) + 8 (y + 13) = 0
æ 40 ö Þ (y + 8) (y + 13) = 0 Þ y = – 8, – 13 Þ x > y.
çè100 x + 100 x + ÷ 43. (d) [(14.98 + 8.95) ¸ 0.99] × 3.9 = ?4 + 10.93
100 ø 49 140 x 14
= Þ =
æ 100 ö 50 200 y 25 Þ [23 × 4] = ?4 + 11 Þ 81 = ?4 Þ ? = 4 81 = 3
çè 100 y + 100 y + ÷ 44. (c) (221.22 + 72.90) ¸ 5.99 = 3.99% of 199.99 + 19.99 + ?
100 ø
3
x 4 Þ 49 = ´ 200 + 20 + ? Þ ? = 23
Þ = 100
y 5
2022-38 SBI Clerk (Junior Associates) Prelim Solved Paper-2022
45. (d) 219.89 + 2.99 + 199.96 = 2.99 × ? + 209.98 130
237 Number of gloves made by M = 250 ´ = 325
Þ 220 + 27 + 200 = 3 ? + 210 Þ = ? Þ 79 = ? 100
3 325
26 Required percentage = ´ 100 = 83.33% ~ 83%
46. (d) ´ 300 + 510 - (31 ´ 4) = 588 – 124 = 464 390
100 150
40 60. (b) Sale of Bat by M in 2020 = 250 ´ = 375
47. (c) ´ 70 = 25 ´ 4 - ?+ 6 ´ 4 100
100 140
Þ 28 = 100 – ? + 24 Þ ? = 96 Sale of Ball by M in 2020 = 125 ´ = 175
100
? Required ratio = 375 : 175 = 15 : 7
48. (c) 131 × 3 + 63 = 70 ÷ 7 + ´ 21 + 5
100 61. (c) Total sale of bat in 2019
441´100 = 250 + 325 + 375 + 400 + 250 = 1600
Þ = ? Þ ? = 2100
21 220
Total sale of bat in 2020 = ´ 1600 = 3520
60 25 100
49. (d) 17 ´ 4 + ´ 250 = ´ 40 + 13?
100 100 As the sales of other are same except M and L.
208 So, number of bats sold in 2020 from I, J, K = 950
Þ 68 + 150 = 10 + 13 ? Þ = ? Þ ? = 16 Number of bat sold by M and I in 2020
13
= 3520 – 950 = 2570
40 Number of bat sold by M and I in 2020
50. (d) 22 ´ 23 - ´ 800 = 36 ´ ?
100
2570
186 = = 1285.
Þ 506 – 320 = 6 × ? Þ ? = = 31 Þ ? = 31 2
6 Total number of bats sold by M in 2019 and 2020
? 3 = 250 + 1285 = 1535
51. (b) ´ 400 ´ + 300 = 67 ´ 9
100 4 62. (b) Required ratio = 1600 : 775 = 64 : 31
Þ 3 × ? = 603 – 300 Þ ? = 101 63. (e) Let the speed of stream = x km/hr
25 20 11 Speed of boat in upstream = 12 – x
52.
53.
(a)
100
´ 84 +
100 Tg:- @NextGenBankers
´ 140 = 2 +
100
(c) 121 + 81 ¸ 9 = 144 ¸ 36 + (?) 2
´ 500 + ? Þ ? = – 8 Speed of boat in downstream = 12 + x
According to the question,
36
Þ 11 + 9 = 4 + ?2 Þ 16 = ? Þ ? = 4 12 - x = 2 3(12 + x) 1
54. (d) 33 × 5 – 60 = 25 × 16 – ? × 59 30 1 Þ 5(12 - x) = 1
295 12 + x
Þ 105 = 400 – 59 ? Þ ? = =5 Þ 36 + 3x = 60 – 5x Þ 24 = 8x Þ 3 km/hr = x
59
? 2 29 1
55. (c) ´ 550 ´ + 625 = ´ 630 - 10 64. (c) Area of triangle = × Base × Height = 84
100 5 100 2
Þ 2 ? + 25 = 2.9 × 63 – 10 Þ 2 ? + 25 = 189 – 10 1
Þ × 12 × Base = 84 Þ Base = 14
Þ ? = 179 - 25 Þ ? = 77
2
According to the question,
2
56. (d) 132 + 171 ¸ 19 × 2 = ? + 15 Perimeter of rectangle = 2 (L + B) = 42
Þ 169 + 18 = ? + 15 Þ 169 + 3 = ? Þ ? = 172 L + B = 21
25 20 Þ L + 14 = 21
57. (e) ´ 28 - ´ 30 + 4 = 5 ´ ? Þ ? = 1 {Q Base of triangle = Breadth of rectangle}
100 100 Þ L=7
120 65. (c) Let the ratio of income = x
58. (d) Sale of bat for T&T company = 400 ´ = 480
100 and the ratio of expenditure = y
70 According to the question,
Sale of gloves by T&T companry = 150 ´ = 105 14x – 10y = 2000 } × 5
100 10x – 6y = 2000 } × 7
Total number of gloves and bats sold by T&T in 2020
= 480 + 105 = 585. 70x – 50y = 10000
59. (a) Number of gloves made by I in 2020 = 90 70x – 42y = 14000
Number of gloves made by J in 2020 = 120 – + –
Number of gloves made by K in 2020 _______________
8y = 4000
120 Þ y = 500
= 150 ´ = 180
100 Income of Ist friend,
Total number of gloves made by I, J, K together = 390 14x – 5000 = 2000 Þ 14x = 7000
SBI Clerk (Junior Associates) Prelim Solved Paper-2022 2022-39
Income of 2nd friend, 70. (d) Time taken by train to cross Ist platform:
10x – 3000 = 2000 Þ 10x = 5000 x be the long of train,
Sum of monthly income of both = 7000 + 5000 = 12000 280 + x
66. (b) Total weight of class = 15 × 40 = 600 kg 36 = {Q ST is the speed of train}
According to the question, ST
Boys + Girls = 400 280 + x
Þ ST = ...(1)
é 40B ù 36
B + êB - = 40
ë 100 úû Time taken by same train to cross IInd platform:
Þ 8B = 40 × 3 Þ B = 25, G = 15 360 + x
Total weight of Girls = 15 × 12 = 180 kg 40 =
ST
Total weight of Boys = 600 – 180 = 420 kg
360 + x
420 Þ ST = ...(2)
Avg. weight of Boys = = 16.8kg 40
25
From (1) and (2),
Required difference = 16.8 – 12 = 4.8 kg
67. (a) Total age of Sunil and Tanisq = 44 280 + x 360 + x
=
Age of all three friends = 72 36 40
Age of Depank = 72 – 44 = 28 Þ 2800 + 10x = 3240 + 9x
According to the question, Þ x = 440 m
Sahil 's age 5 æ 280 + 440 18 ö
Sahil : Depank = 5 : 7 Þ = Þ ST = ç ´ ÷ km/hr
Depank 's age 7 è 36 5 ø
5 ´ 28 = 72 km/hr.
Þ Sahil’s age = = 20 Sol. (71-75):
4
The sentences flow naturally from one another. The sentences
Present age of Tanisq = 44 – 20 = 24 years.
detail a specific worm's behaviour. A closer look reveals that
68. (a) According to the question,
the first sentence of the paragraph is phrase C. Sentence C
P´R´T
I=
100 Tg:- @NextGenBankers
Let the principal be ` P.
therefore acts as the introduction. The remaining sentences
come after sentence C.
Hence, option B is the correct answer.
Pairing
2P P ´ 20 ´ T
=
5 100 the worm sucks this net
C and E: the worm coughs up a net
ÞT=2 back
Now, interest after (T + 4) years, It does this by spraying a
tosses the rest away through
P ´ 20 ´ T 6P E and D: jet of water out of its
I= = the back end of its burrow.
butt.
100 5
6P a meal for other tiny
Amount after 6 years = P + = 2.2P It does this by spraying a jet
beach denizens, such as
5 D and A:
of water out of its butt
69. (b) Amount spent by Ram on entertainment crabs, shrimp and clams.
25 a meal for other tiny beach various opportunistic
= 18000 ´ = ` 4500
100 A and F: denizens, such as crabs, animals looking for a
Amount spent by Ram on travelling shrimp and clams. free bed and meal.
12 In fact, it's common for the
= 18000 ´ = ` 2160
100 worm’s burrow to host This is where the worm's
Amount he saves in bank account F and B: various opportunistic "fat innkeeper"
animals looking for a free nickname comes from.
x
= ´ 18000 = 180 x bed and meal.
100
According to the question, 71. (b)
4500 + 2160 + 2160 + 180 x = 18000 72. (d) The correct sequence of the sentences: CEDAFB
Þ x = 51% 73. (a) The correct sequence of the sentences: CEDAFB
74. (d) The correct sequence of the sentences: CEDAFB
51 75. (e) The correct sequence of the sentences: CEDAFB
Amount he saves in his bank account = ´ 18000
100 76. (c) The correct spelling is ‘parties’.
= ` 9180 77. (a) The correct spelling is ‘spilled’.
2022-40 SBI Clerk (Junior Associates) Prelim Solved Paper-2022
78. (b) The correct spelling is ‘distribute’. 88. (c) Hence, Option (c) is the right choice among the given
79. (d) The correct spelling is ‘exception’. options.
80. (c) 89. (a) The error is in part A of the sentence.
81. (d) From the text given, it is clear that experts believe Reason:
that policy decisions that ignore the environmental When an “if” clause and a principal clause (which is
aspects of the region are the main cause of the in future tense) are present in a single sentence, the
country’s floods. ‘if’ clause should always be in the present or past
tense.
It makes Option (d) the right choice among the given
Correct Sentence:
options. If I had an even faster computer than I have today, I
82. (c) It implies from the given lines that the states should could come up with really interesting questions to
ponder over the steps to be taken in order to preserve ask it.
the ecology of the Western Ghats Area and they Therefore, option A is the correct answer choice to
should think about implementation of the expert panel this question.
report on this issue. 90. (c) Reason: one of/none of/ either of/ neither of/ + plural
This makes Option (c) the right choice among the noun/pronoun+ singular verb
91. (b) The type of (singular so) depend upon DEPENDS
given options.
upon
83. (e) Statement A is incorrect since the expert panel has 92. (b) Everybody is subject. It takes singular subject
recommended formation of committee and authority 93. (b) “disputes now a days because of “by “disputes now
to oversee the development in the Western Ghats a days because”. It is superfluous to use preposition
Area. Statements B, C and D are not in sync with the of as subordinate clause follows.
given context though they may sound logical 94. (b) Lie Lay Lain ( to relax in or on something)
otherwise. Only Option (e) implies the same as has Lay Laid (to spread Something Somewhere)
been depicted in the passage. Incorrect verb (v2) is used.
This makes Option (e) the right choice among the 95. (a) Replace our company is yet starting by we are yet to start.
Yet is occasionally used in affirmative sentences,
given options.
giving the sentences a similar meaning as the use of
84. (b)
85. Tg:- @NextGenBankers
(a) To protect the Western Ghats region, it is clear that
solar energy in the region should be encouraged,
still. Note that this is more formal and not
common."Yet to " is followed by 1st form of verb.
I am yet to complete this project.
along with a moratorium on mining and quarrying I am still working on this project.
activities in the region. Apart from that, the 96. (c) To answer this question correctly one needs to be
construction of new hydroelectric dams in this area good at general awareness as per which Russia is
should be restricted. Therefore statements I and III world’s biggest country by ‘area’.
are true. Option (c) is hence the correct answer.
97. (a) As the words “considered” and “contemplated” are
Hence, Option (a) is the right choice among the given
synonyms, neither of them could be picked for the
options. blank. Options (b) and (c) hence get eliminated.
86. (a) It is assumed that there must be a central authority Moreover, the words “refuted” and “revoked” are
to regulate local development with the assistance of negative in meaning and thus would not go well with
state level authorities so that the whole process is the context. Options (d) and (e) get eliminated as
well coordinated. Statement A implies this, but the well.
rest of the options clearly do not imply this kind of Clearly, the word “inked”, the present form of which
regulation in this area. refers to ‘sign or enter a pact or contract’ is the word
that’d be fitting the gap appropriately.
This makes Option (a) the right choice among the
Option (a) is hence the correct answer.
given options. 98. (b) The word “elevated” clearly fits the gap both
87. (e) From the given lines, it is clear that the Kerala Finance grammatically and contextually.
Minister believes that the public should be involved Option (b) is hence the correct answer.
in the process of building a blueprint for the Western 99. (d) Of the options given, only the noun phrase “think
Ghats region of the country. It will then be possible tank,” referring to “a group of experts who provide
to outline a development roadmap for the region. advice and ideas on a particular political or economic
Statements E only implies the same with the other issue,” is the most appropriate option for choosing
options being irrelevant in the given context. Gap. Option (d) is hence the correct answer.
100. (c) Clearly, the word “vigorous” which means ‘strong
This makes Option (e) the right choice among the
and forceful’ would be the most appropriate word for
given options. the blank in the given context of the passage.
Option (c) is hence the correct answer.
SBI Clerk (Junior Associates) Mains Solved Paper-2021 2021-1
Reasoning Ability & Computer Aptitude 6. How many numbers are there between the one which is 3 rd
from the right end and 38 in step 3?
DIRECTIONS (Qs. 1-5): Study the following information (a) Four (b) One
carefully to answer the given questions: (c) Three (d) None
(e) More than four
Ten friends - Q, R, S, T, U, V, W, X, Y and Z are sitting around a 7. Which of the following number is 7th from the left end in
circular table, but not necessarily in the same order. All friends step 5?
are facing inside. Adjacent name as in alphabetical series are (a) 46 (b) 79 (c) 56 (d) 49
not sitting nearby to each other in the circle. (e) None of these
There are two persons sitting between the Q and Z when count 8. What is the position of 88 from the right end in 2nd last
from to the left of Z. More than three persons are sitting between step?
the R and Z when count both left and right of Z. Q sits just to the (a) Fourth (b) Fifth
left of U who sits third to the left of X. More than four persons (c) Sixth (d) Third
sit between T and Y when counted to the left of Y. Y sits near to (e) None of these
R. W does not sit near to Y. S sits just right of V. 9. How many numbers are there between 72 and the one
1. How many persons are sitting between X and R when which 4th to left of 76 in step 5?
counted to the right of X ?
(a) None (b) Three
(a) Two (b) Three
(c) Two (d) More than three
(c) Four (d) Five
(e) None of these
(e) Six
2.
(a) U
Tg:- @NextGenBankers
Who is sitting fourth to the left of V ?
(b) T (c) S (d) R
10. Which of the following number is 5th to left of 49 in step 4?
(a) 60 (b) 38
(e) None of these
(c) 72 (d) 40
(e) Q
3. Who is sitting just to the near of U and T ? DIRECTIONS (Qs. 11- 14): In the questions below are given
(a) W (b) R (c) Z (d) X four statements followed by two or three conclusions numbered
(e) Q I, II or III. You have to take the given statements to be true even
4. How many persons are sitting between W and T when if they seem to be at variance with commonly known facts.
counted to the right of W ? Read all the conclusions and then decide which of the given
(a) Four (b) Five
conclusions logically follows from the given statements
(c) Six (d) Seven
disregarding commonly known facts.
(e) Eight
5. Who sits third to the right of S ? 11. Statements :
(a) Q (b) U (c) W (d) T Some Small are Shirt.
(e) R All Shirt are Cloth.
No Cloth is Rectangle.
DIRECTIONS (Qs. 6-10): Study the following information
All Jeans are Rectangle.
carefully and answer the questions given below. A number
Conclusions :
arrangement machine, when given a particular input,
I. Some Small are Cloth.
rearranges it following a particular rule. The following is the
illustration of the input and the steps of arrangement. II. Some Small are not Rectangle.
(a) None follows (b) Either I or II follows
Input : 62 97 38 74 55 12 86 45 68 22 (c) Only I (d) Only II
Step I : 13 62 97 38 74 55 86 45 68 23 (e) All follow
Step II : 39 13 62 97 74 55 86 68 23 46 12. Statements:
Step III : 56 39 13 97 74 86 68 23 46 63 Some Redmi are Lava.
Step IV : 69 56 39 13 97 86 23 46 63 75 Some Lava are Iron.
Step V : 87 69 56 39 13 23 46 63 75 98 Some Iron are Samsung.
And Step -V is the last step of the rearrangement as the desired Some Samsung are Nokia.
arrangement is obtained. As per rules followed in the above Conclusions:
steps, find out in each of the questions the appropriate step for I. Some Lava are Redmi.
the given input. II. Some Samsung are Lava.
Input : 88 59 28 94 37 75 15 64 71 48
2021-2 SBI Clerk (Junior Associates) Mains Solved Paper-2021
(iii) Have post qualification work experience of at least (a) Only I follows. (b) Only II follows.
three years in the admin division of an organization. (c) Both I and II follow. (d) Neither I nor II follows.
(iv) Have secured at least 55% marks in the selection (e) Either I or II follows.
examination.
(v) Have a post Graduate degree/diploma in Admin- DIRECTIONS (Qs. 27-28): In each question below is given a
Management with at least 60% marks. statement followed by two conclusions numbered I and II.
Study the following information carefully and find which Youhave to assume everything in the statement to be true, then
of the following condition shows candidate is selected? consider the two conclusions together and decide which of
(a) Candidate is daughter of a renowned freedom fighter them logically follows beyond a reasonable doubt from the
from another state. information given in the statement. Give answers accordingly.
(b) Candidate has a post Graduate degree in Finance with 27. Statement: Population increase coupled with depleting
60% marks. resources is going to be the scenario of many developing
(c) Candidate has completed his graduation with 80% countries in the days to come.
marks. Conclusions:
(d) Candidate does not own a house in Noida. I. The population of developing countries will not
(e) Candidate has secured 56% marks in Cap Gemini’s continue to increase in future.
interview. II. It will be very difficult for the governments of
DIRECTIONS : In the following question, a statement has been developing countries to provide its people decent
given which is followed by three courses of action numbered I, quality of life.
II, III. A course of action is a step or administrative decision to (a) Only conclusion I follows.
be taken for improvement, follow up, or further action in regard (b) Only conclusion II follows.
to the problem, policy etc. On the basis of the information (c) Either I or II follows.
provided. You have to assume everything in the statement to be (d) Neither I nor II follows.
true, then decide which of the given /suggested courses of action (e) Both I and II follow.
logically follows for pursuing. 28. Statement: It is unhealthy to keep plants in bedrooms,
especially at nights. Due to the excess carbon dioxide
25. Statement: Every year thousands of able students, both produced during those hours, it can lead to suffocation.
in rural as well as in urban areas cannot get admission in
their schools.
Tg:- @NextGenBankers
colleges despite passing the last certificate examination of
Conclusion
I. We should not grow plants at home.
II. Plants need to be placed in outer spaces like windows
Courses of Action: or balconies.
I. More colleges should be established in rural as well (a) Only I follows. (b) Only II follows.
as in urban areas. (c) Both I and II follow. (d) Neither I nor II follows.
II. The number of schools in rural as well as in urban (e) Either I or II follows.
areas should be decreased.
III. A good number of schools should conduct vocational DIRECTIONS (Qs. 29- 30): One statement is given followed
courses. So that the students could start preparing to get by two arguments, I and II. You have to consider the statement
employed after completing the school education. to be true, even if it seems to be at variance from commonly
(a) Only I follows (b) Only II and III follow known facts. You are to decide which of the given arguments
(c) All follow (d) Only I and III follow be definitely drawn from the given statement.
(e) None of the above
29. Statement: The usage of mobile phones by students
26. Direction: In the question below are given a statement
should be permitted in all schools.
followed by two courses of action numbered I and II. A
Arguments:
course of action is a step or administrative decision to be
I. No, it might become an unproductive gadget under
taken for improvement, follow-up or further action in regard
those age group students.
to the problem, policy, etc. On the basis of the information
given in the statement to be true, then decide which of the II. Yes, students can travel safely and communicate to
suggested courses of action logically follow(s) for parents whenever necessary.
pursuing. (a) Argument I alone is true.
Statement: Air pollution has emerged as a major (b) Argument II alone is true.
environmental challenge in Indian cities and across the (c) Both arguments I and II are true.
states. Emissions from multiple sources are responsible (d) Either argument I or II is true.
for the condition which can bring down the life expectancy (e) Neither argument I nor II is true.
by 5 years on average. 30. Statement: Should government ban Cigarette industries?
Course of action: Arguments:
I. Government should impose a limit on the manufacturing I. Yes, as it is very injurious for any person consuming
of vehicles across the country. them and causes cancer.
II. Awareness programs should be conducted and people II. No, forbidding them would mean that those people
should be encouraged to pool vehicles. who work in cigarette industries would lose their jobs.
2021-4 SBI Clerk (Junior Associates) Mains Solved Paper-2021
(a) Only argument I is strong DIRECTIONS (Qs. 37-40) : Read the instruction carefully and
(b) Only argument II is strong answer the questions given below.
(c) Neither I nor II is strong
(d) Both I and II are strong R, S, T, U, V, W, X, Y, and Z are the nine members of a family.
(e) Either I or II is strong Either both the parents of a person are alive or neither of them is
alive. T is the daughter of Y who is the daughter-in-law of W. S
DIRECTIONS: A series is given with five word in which few is elder than Y. X is the daughter of V who is the brother of Z. R
operations are performed. is elder than W who is the mother of Z. T is the elder sister of X.
FLAW DARK FLOW PACT JOKE R is the paternal grandfather of U who is the only son of S. Z is
elder than S but younger than V. W has no daughters. Z got
31. How many letters are there in alphabetical series, between
married after X was born. T is younger than Y
3rd letter from the right end of 2nd word from left end and 1st
37. How many are older than T’s father?
letter from the left end of the word which is 2nd from the
(a) No one (b) One
right end?
(c) Two (d) Three
(a) 15 (b) 12 (c) 13 (d) 18 (e) More than three
(e) 14 38. How is V related to U?
DIRECTIONS (Qs. 32-36): Study the following information (a) Father (b) Grandfather
carefully and answer the given below questions. (c) Paternal Uncle (d) Sister-in-law
(e) Brother-in-law
Eight people S, T, U, V, W, X, Y and Z go for a trip either in 2009 39. How is S related to W?
or 2010 in the month of January, April, July, October. Only two (a) Mother (b) Mother-in-law
persons go on the trip in a month but they go in different years. (c) Daughter-in-law (d) Son-in-law
They go for trips to eight different countries namely Russia, (e) Grandson
Canada, Maldives, Malaysia, Australia, USA, Switzerland and 40. Who among the following is 4th oldest?
UAE but not necessarily in the same order. T goes UAE in 2010 (a) R (b) W (c) Z (d) V
in the month having even no of days. Z and W go in the same (e) Y
month. Z neither go in October nor go to Switzerland and USA.
Not more than two people go on the trip before S, who goes on DIRECTIONS: In the questions below consists of a question
Tg:- @NextGenBankers
Russia trip. S does not go in the month having an even number
of days. One person go between W and X. X does not go for the
and three statements numbered I, II and III. You have to decide
whether the data provided in the statements are sufficient to
answer the question. Read all the three statements and give
trip in the first month of any year and He goes to the Maldives.
One person go between Y and U. Y does not go just before the answer accordingly.
one who goes to Canada. Three person goes for the trip between Six members of the family X, Y, Z, P, Q and R are standing in a row
the one who goes for Malaysia and the one who goes for and all are facing the north direction. Who is standing between
Switzerland. Neither Y nor W goes to Switzerland. The one who Z and Q?
goes to Canada goes after the USA. 41. Statement I : X is standing second from the left end. Z is
32. Who among the following goes to Australia for trip? standing third to the left of Y who is near to Q.
(a) Z (b) Y (c) X (d) W Statement II : Y is not the immediate neighbour of R who
(e) V does not sits any extreme end of the row.
33. Y goes for trip in which of the following Month? Statement III : Q is not the neighbour of X.
(a) January 2010 (b) January 2009 (a) If the data in statement I alone or in statement II alone
(c) April 2009 (d) April 2010 or in statement III alone is sufficient to answer the
(e) October 2009 question.
34. Number of persons go for trip before U is same as the (b) If the data in statement I and II are sufficient to answer
number of person go for trip after ______? the question, while the data in statement III are not
(a) The one who go to Maldives sufficient to answer the question.
(b) Y (c) If the data in statement I and III are sufficient to answer
(c) The one who go to Canada. the question, while the data in statement II is not
(d) X sufficient to answer the question.
(e) None of the given option is true (d) If the data in statement II and III are sufficient to answer
35. Who among the following go for trip in July? the question, while the data in statement I is not
(a) S (b) W sufficient to answer the question.
(c) X (d) Both S and W (e) If the data in all the statement I, II, and III are necessary
(e) Both S and X to answer the question.
36. Which of the following combination is correct?
DIRECTIONS (Qs. 42- 43): In the following questions, a given
(a) Z - Malaysia (b) X - Switzerland
question is followed by information in two statements.You have
(c) S - January (d) W - October
to find out the data in which statement (s) is are sufficient to
(e) Y - USA
answer the question and mark your answer accordingly.
SBI Clerk (Junior Associates) Mains Solved Paper-2021 2021-5
42. On which day of the second week of the month Rahul’s 47. Who lives just below the one who likes Blue color ?
exam is scheduled? (a) E (b) B (c) D (d) C
I. The exam is scheduled on the sixth day of the week, (e) A
the first day of the week is Sunday. 48. What will be the correct combination from the given
II. The exam is scheduled after Thursday but before last option?
day of the week i.e. Saturday. (a) F- Flat 2 - Floor 1
(a) II alone is sufficient while I alone is not sufficient. (b) Yellow - Flat 2 - Floor 2
(b) I alone is sufficient while II alone is not sufficient. (c) B - Flat 1 - Floor 3
(c) Either I or II alone is sufficient to answer the question. (d) Green - Flat 1 - Floor 1
(d) I and II together are sufficient to answer the question. (e) White - Flat 1 - Floor 3
(e) Neither I nor II is sufficient to answer the question. 49. Who lives just to the east of D ?
43. There are 4 people sitting around a circular table W, X, Y (a) F (b) A (c) E (d) C
and Z at equal distant points around the circle, facing the (e) B
centre of the circular table. 50. Who likes red color ?
Who sits to the second position at the right of W? (a) D (b) A (c) F (d) C
I. X sits to the second position to the left of W. (e) E
II. Y sits to the second position to the right of Z.
(a) II alone is sufficient while I alone is not sufficient. Quantitative Aptitude
(b) I alone is sufficient while II alone is not sufficient.
(c) Either I or II alone is sufficient to answer the question. DIRECTIONS (Qs. 51-55) : Read the following information
(d) I and II together are sufficient to answer the question.
carefully and answer the questions that follow:
(e) Neither I nor II is sufficient to answer the question.
Five members of a family viz. Mukesh his wife Taniya, their two
DIRECTIONS (Qs. 44- 45) : There are Six dices numbered 1, 2, children Nitu and Vikash and their daughter-in-law Arsha spent
3, 4, 5, 6 are placed one above another (not in the same different amount from their respective salaries in a month. Total
pattern).Two dices are placed between 1 and 3 is placed expenditure of the family per month is ` 3,30,000 which is 75% of
immediately below (5) One dice is placed between 6 and 4, 2 is the total income of the family per month. Taniya saving is l/3rd
placed one of the above place of 4 and one of the below place of the savings of her daughter-in-law who earns ` 60,000 a month
Tg:- @NextGenBankers
of 5 but not immediate above or below. 6 is not placed at the
bottom. Odd numbered dice is placed at the top and the Even
numbered dice is placed at the bottom.
and spent 60% of her salary. Salary earned by Nitu 'is the highest
in the family i.e. ` 1,20,000 per month and her expenditure is
twice the expenditure of Arsha. Taniya saves 10% of her income
44. How many dices were placed between 3 and 2? and her husband saves 20% of his income. Mukesh earns
(a) 1 (b) 2 (c) 3 (d) 4 ` 90,000 per month.
(e) None 51. Income of which of the following two members of the family
45. Which of the following dice is placed just above 4? is same?
(a) 5 (b) 6 (c) 1 (d) 3 (a) Taniya and Vikash (b) Mukesh and Taniya
(e) 2 (c) Nitu and Arsha (d) Mukesh and Vikash
(e) None of these
DIRECTIONS (Qs. 46 -50) : Study the following information 52. What is the average salary of the family per month?
carefully and answer the question given below: (a) ` 72,000 (b) ` 64,000
In a building of three floors and each floor having two flats as (c) ` 88,000 (d) ` 90,000
Flat - 1 and Flat - 2 There are six people live in these flats named (e) None of these
as A, B, C, D, E and F. They also like different color Red, Blue, 53. What is the ratio of the salary earned to salary spent by
Green, Yellow, Black and White but not necessary in the same Arsha in a month?
order. The bottom floor is numbered 1 and the top floor is (a) 5 : 2 (b) 5 : 3
numbered 3 Flat -1 is in the west direction of Flat - 2. (c) 2 : 5 (d) 3 : 5
E likes green color lives on the odd number floor and flat which (e) None of these
is also odd in number. There is one floor between A and E but A 54. What is the difference between the expenditure of Taniya
not lives in the same flat of E. There is one floor between the one and Nitu in a month?
who like red color and the one who like black color. No person (a) ` 4000 (b) ` 6000
lives between the one who like white color and C who lives on (c) ` 8000 (d) ` 10,000
even number floor. D neither like red nor black color. B is not like (e) None of these
red color and lives one of the floor below the one who likes red 55. Salary earned by Arsha is what % of salary earned by
color but not just below. A is not like the red color. Person who Taniya?
likes white color and yellow color did not live in same flat. (a) 65% (b) 45% (c) 50% (d) 75%
46. Person who likes Yellow color lives on which flat and floor? (e) None of diese
(a) Flat -1 Floor - 1 (b) Flat - 2 Floor 1 56. The distance between the two trains is 300 km and moving
(c) Flat 1 Floor 2 (d) Flat 2 Floor 2 in the ooposite direction on the same track. The speed of
(e) Flat 1 Floor 3
2021-6 SBI Clerk (Junior Associates) Mains Solved Paper-2021
one train is 72 krn/hr and the speed of another train is 48 DIRECTIONS (Qs. 64-65): Find out the wrong number .
km/hr. A bird starts flying at a speed off 68 km/hr at the
location of the faster train. When it reaches the slower 64. 12, 12,16, 34, 80, 182, 362
train, it turns around and flies in the opposite direction at (a) 182 (b) 34 (c) 16 (d) 80
the same speed. When it reaches the faster train again it (e) 12
turns around and so on. When the train collide, how far 65. 74, 125, 76, 122, 79,117
has the bird flown (in km)? (a) 76 (b) 79 (c) 117 (d) 122
(a) 180 km (b) 150 km (e) 74
(c) 170 km (d) 160 km DIRECTIONS (Qs. 66-70): The following question are
(e) None of these
accompanied by two or three statements (I) and (II) or (III).
57. Income of P is ` x and income of Q is ` y. Expenditure on
You have to determine which statement(s) is/are sufficient/
food by both is 10% of their income. Expenditure on
necessary to answer the following question.
entertainment by P is twice the expenditure on food. Saving
of P is ` 35,000. Expenditure on entertainment by Q is 66. What is the amount received by Vinita?
` 12,000. Saving of Q is 75 % of his income. Find their I. Ekansh and Vinita started a business with an
income? investment in the ratio of 5 : 4. After 4 months Ayush
(a) ` 72,000 and ` 54,000 (b) ` 80,000 and ` 60,000 joined their business with an amount 20% more than
(c) ` 48,000 and ` 50,000 (d) ` 80,000 and ` 50,000 that of Ekansh, after 2 more months Vinita took out
(e) None of these 25% of her investment. The total profit obtained after
58. Chemical mixtures A consist of chemicals P and Q in ratio the end of year is ` 25000.
of 1 : 3, Chemical B consist of chemicals P which chemical II. Ekansh invested ` 7000 more than that of Vinita.
B is present. Both the chemicals A and B are mixed in a Ayush invested 9000 more than Ekansh and the
large container. What is % of concentration is present in difference between the profit earned by them is `
final mixture as P? 2000.
(a) 35% (b) 50% (c) 40% (d) 25% (a) If the data in statement I alone is sufficient to answer
(e) None of the above the question, while the data in statement II alone are
59. Yuvi sells two types of milk. In the first type of milk, there not sufficient to answer the question.
is 40% milk is present and the rest is from water, whereas, (b) If the data in statement II alone is sufficient to answer
Tg:- @NextGenBankers
in the second types only 40% water is present. He takes
30 litres and 20 litres of milk from both types respectively.
the question, while the data in statement I alone are
not sufficient to answer the question.
What is the percentage of milk present in the new mixture? (c) If the data either in statement I alone or in statement
(a) 64% (b) 36% (c) 50% (d) 48% II alone is sufficient to answer the question.
(e) None of these (d) If the data even in both the statements I and II together
60. J, K and L invest together in business wherein they receive are not sufficient to answer the question.
profit ` 63,000. J reinvests his share of profit in a scheme (e) If the data in both statements I and II together are
giving 10% rate of interest after two years and K reinvests needed to answer the question.
his share of profit which gives him 20% rate of interest at 67. Rahul and Yogesh are traveling towards each other with
end of two years. If interest received by J and K after 2 certain speeds from point A and point B respectively. At
years is ` 3969 and ` 8316 then find interest earned by L if what time will both meet each other at point R if both start
he reinvests his share in same scheme as J did for 2 years. traveling at 3:00 pm.
(a) ` 5475 (b) ` 7690 I. Rahul takes 4 hours to reach point Q from point R
(c) ` 6658 (d) ` 4768 while Yogesh takes 9 hours to reach point P from
(e) ` 5292 point R.
DIRECTIONS (Qs. 61-63) : What approximate value should II. Distance between point P and point Q is 1200 km,
come in the place of question mark (?) in the following and the ratio of the speed of Rahul and Yogesh is 2 : 3.
question? III. The difference between the speed of Rahul and
Yogesh is 40 km/hr.
61. 12.973 × 3.99 ÷ 52.08 + 107.99 + 3 1727.98 × 6.93 = ? (a) Only I is sufficient to answer
(a) 240 (b) 228 (c) 232 (d) 244 (b) Only II and III are sufficient to answer
(e) None of these (c) Either I or II and III together are sufficient to answer
62. (345.97 + 129.88 – 45.03) + (34.87 – 6.96 × 2.99) = ? – (d) All are necessary to answer
(e) None of them is sufficient to answer
1521
68. There are four member A, B, C and D partners in the
(a) 485 (b) 501 (c) 595 (d) 364
business. What is the profit share of B?
(e) None of these
I. A and B started the business with an investment of
63. (24.99% of 250 × 3.95) + (21.05% of 300 – 49.99) = ? –
` Q and ` 3Q respectively and after 6 months C and D
19.99% of 40
joined them with an investment of ` (a + 5000) and
(a) 260 (b) 271 (c) 336 (d) 192
(e) None of these Rs. 4a respectively.
SBI Clerk (Junior Associates) Mains Solved Paper-2021 2021-7
II. At the end of the year profit share of C is ` 16000. Q is 240% of the quantity of water in vessel P. Total profit earned
III. At the end of one year, the profit ratio of C and D is by milkman when he sold the mixture from vessel S at the cost of
1 : 3. pure milk is (300/7)% and total loss occurred to milkman when
(a) Only statement II and III are sufficient. he sold the mixture from vessel T at ` 25.2 per liter is 25%.
(b) Any two of them are sufficient Total quantity of pure milk in vessel R is one third of the total
(c) Only statement I and II are sufficient. quantity of pure milk in vessel B and vessel T together.
(d) All of them are sufficient. 71. Diary owner sold 3 liters mixture from vessel R and added
(e) None of the statement is sufficient. the same amount of water, next time he sold 5 liters from
69. Ram and Shyam invested ` 3000 together. Ram invested the same vessel and added the same amount of water to it.
his amount at compound interest for ‘x/10’ years at x% Finally, he removes 2 liters of mixture and adds 2 liters of
rate interest and Shyam invested half of his amount at water. What is the ratio of milk to water in vessel R after
compound interest and remaining half at simple interest at these operations?
the same rate of interest for the same time period. [It is (a) 8 : 7 (b) 5 : 7 (c) 7 : 8 (d) 7 : 5
given that ‘x/10’ is a natural number] (e) None of these
What is the value of ‘x’? 72. If the diary owner uses the same cane to measure milk,
Statement I : Ratio of investment of Ram and Shyam is 1 : once from vessel P and once from vessel S, then what is
2 respectively and total sum received by them together the amount of pure milk purchased by the customer out of
after a particular time at x% rate of interest is ` 4280. 3 liters mixture?
Statement II : Simple interest received by Shyam is ` 40 (a) 1.4 L (b) 2.3 L (c) 2.1 L (d) 1.8 L
less than compound interest received by him. (e) None of these
Statement III : Sum of simple interest and compound 73. If dairy owner mixes the mixture of vessel Q with mixture of
interest received by Shyam after a certain time at x% rate vessel T and sells this mixture at cost price of pure milk,
of interest is ` 2500. then what is the profit percent of dairy owner?
(a) Only statement I alone is sufficient.
3 2 1
(b) Only Statement II alone is sufficient. (a) 25% (b) 42 % (c) 18 % (d) 33 %
(c) Only Statement III alone is sufficient. 4 3 3
(d) Either statement I and II together or statement II and (e) None of tliese
70.
Tg:- @NextGenBankers
III together are sufficient.
(e) Statement I, II and III together are sufficient
Find the ratio of alcohol and water in the new mixture if the
DIRECTIONS (Qs. 74 -78): Read the following data carefully
and answer the questions given below.
content of the two containers is mixed. The following pie chart shows the member of TCS in degree in
I. Two containers contain alcohol and water in the ratio 2021. Table shows the ratio of male and female in every
of 2 : 3 and 4 : 5 respectively. Both the mixtures are department.
mixed and formed a new mixture. Total member of TCS in 2021 = 720
II. Containers contain 10 L and 12 L alcohol respectively.
(a) The data in statement I alone is sufficient to answer TCS
the question, while the data in statement II alone is 5°
H.R.
not sufficient to answer the question. 50°
(b) The data in statement II alone is sufficient to answer Software
the question, while the data in statement I alone is 20°
not sufficient to answer the question. 200° Tech.
(c) Either statement I or statement II alone is sufficient
Marketing
to answer the question. 60°
(d) The data in both the statements I and II are not Product
sufficient to answer the questions. 25°
(e) The data in both the statement I and II together are QA
necessary to answer.
DIRECTIONS (Qs. 71-73) : Read the data given in paragraph
Department of TCS Male : Female
carefully and answer the question. The paragraph given below
shows the information about a Dairy owner who has five vessels H. R. 2:03
P, Q, R, S and T out of which volume of P, Q and R is 30 liters Software 4:01
each and volume of vessels of S and T is 50 liters each. Tech. 3:07
Cost of pure milk is ` 40 per liter and the cost of water is 0. In Marketing 3:02
each vessel, dairy owner has a mixture of milk and water.
When the dairy owner sold the mixture from vessel P at ` 50 per Product 1:01
liter he earned a total profit of 50%. Quantity of water in vessel QA 3:07
2021-8 SBI Clerk (Junior Associates) Mains Solved Paper-2021
74. How many members should be added in H.R. department (c) x > y (d) x < y
to become the same number of product department? (e) x = y or relationship between x and y cannot be
(a) 20 (b) 25 (c) 30 (d) 15 established.
(e) None of these 84. A mobile shop owner marks up the cost price of an mobile
75. What is the percentage of the number of male in software phone by 50% and offers a discount of 20%. He asks the
department to the number of female in tech, department? customer to pay GST of 18% on the selling price. The
(a) 964.29% (b) 914.29% customer refuses to pay the tax due to which the shop
(c) 928.32% (d) 934.29% owner himself pays the GST. Find his profit or loss
(e) None of these percentage.
76. The number of QA members will be increased by 80% and
3 3
the number of marketing members will be decreased by (a) 2 % Profit (b) 1 % Loss
50% in 2022 but the ratio of male and female remains same 5 5
as 2021. Find the ratio of the number of males in QA 3
department and the number of females in marketing (c) 2 % Loss (d) No profit no loss
5
department in 2022.
(a) 3 : 5 (b) 9 : 4 3
(e) 1 % Profit
(c) 5 : 3 (d) 4 : 9 5
(e) None of these 85. Jay wants to cross a river and reach a point opposite to
77. Find the number of total males in all departments in TCS. where he is now on other side of river. He needs to travel
(a) 665 (b) 566 (c) 457 (d) 461 north but flow of river is in west direction. Jay’s boat speed
(e) None of these is 5 km/hr and stream speed is 4 km/hr. River is 15 km wide.
78. If every male earns ` 5000 pr day in tech, department, then Find the time to reach the point where he intends to reach
find the total income of males in tech, department for one if he walks at the speed of 2 km/hr after reaching shore on
month.(in lakh) the other side(in hour)
(a) 12.5 (b) 30 (c) 22.5 (d) 27.5 (a) 9 (b) 6
(e) None of these (c) 3 (d) 12
DIRECTIONS (Qs. 79-83): In the given question, two equations (e) None of these
Tg:- @NextGenBankers
numbered I and II are given. Solve both the equations and
mark the appropriate answer.
86. The sum of the ages of 6 members of a family 4 years ago
was 112 years. Today, when the daughter has been married
off and replaced by a daughter-in-law, the sum of their
79. I. 3x2 – (2 + 15 11 )x + 10 11 = 0 ages is 98. Assuming that there has been no other change
II. 9y2 – (5 + 18 11 )y + 10 11 = 0 in the family structure and all the people are alive, what is
the difference in the age of the daughter and the daughter-
(a) x > y (b) x < y in-law?
(c) x > y (d) x < y (a) 30 years (b) 34 years
(e) x = y or relation between x and y cannot be (c) 27 years (d) 38 years
established.
(e) None of these
80. I. 9x2 – 45x + 50 = 0
87. Dev invested ` 50000 in gold forthe whole year. After
II. 12y2 – 35y + 25 = 0
5 months of Dev his sister joined him and at invested
(a) x > y (b) x < y
` 60000. Next year Dev invested `10000 more and his sister
(c) x > y (d) x < y
withdrew ` 5000 and at the end of two years profit earned
(e) x = y or relationship between x and y cannot be
by Dev is ` 42000. Find the total profit earned if they
established.
81. I. x2 + 43x + 406 = 0 distributed half of the total profit equally and rest in the
II. y2 + 46y + 513 = 0 capital ratio.
(a) x > y (b) x < y (a) ` 90,000 (b) ` 64,000
(c) y > x (d) y < x (c) ` 80,000 (d) ` 75,000
(e) x = y or relationship between x and y cannot be (e) None of these
established. 88. In a company, bottles are manufactured, out of which some
82. I. 4x2 – 20x + 16 = 0 are defected. Bottles are packed in a box. Three bottles are
II. 4y2 – 80y + 336 = 0 picked from box by inspector, if two of the bottles out of
(a) x > y (b) x < y three are perfect, box passes otherwise it fails. Find the
(c) x > y (d) x < y probability of box getting passed.
(e) x = y or relationship between x and y cannot be 1 1 1 2
determined. (a) (b) (c) (d)
4 2 3 3
83. I. 7x2 – 92x + 96 = 0
II. 9y2 – 128y + 143 = 0 (e) None of these
(a) x > y (b) x < y 89. Mohan is having two varieties of sugar. First variety of
sugar costing `25 per kg while second variety of sugar is
SBI Clerk (Junior Associates) Mains Solved Paper-2021 2021-9
` 32 per kg. Find how much quantity of 1st variety should DIRECTIONS (Qs. 96-100): Study the chart carefully and the
he added to 30 kg of 2nd variety such that the cost of the questions that follow.
mixture of both varieties be worth ` 27 per kg?
(a) 75 kg (b) 72 kg The pie charts show the distribution of the total number of
(c) 76 kg (d) 70 kg books sold by book store P,Q,R and S the distribution of Maths
(e) None of these book sold by book store P, Q, R and S respectively. Also, Total
90. A piece of work is completed by 3 boys and 4 girls in 7 books = Maths books + Physics books.
days. The same piece of work is completed by 5 boys and Number of Physics books sold by P = 110
3 girls in 6 days. Find the time taken by 1 boy and 2 girls to Number of Physics books sold by Q = 30
complete the 4 times of the same work? Total Books
(a) 58 days (b) 69 days
(c) 60 days (d) 66 days
(e) None of these (a+5)%
DIRECTIONS (Qs. 91- 95): Read the following table carefully 2a%
and answer the given questions :
a%
The following table shows No. of hours per day, No. of Days in
Total, No. of Days on leave, Income of three persons J, K and L. (a+20)%
100. Total books sold by P and Q together is how much If a list of eminent institutions in the country is at all needed, the
percentage more than the physics books sold by R? absence of the Jawaharlal Nehru University (JNU) from the first
(a) 48% (b) 72% (c) 50% (d) 60% list of IoEs is striking. Its faculty has brought many of the world's
(e) None of these leading ideas to Indian students and in at least area came close
to building a new school of thought, however controversial. It
English Language is not as if similar efforts in the social sciences have not occurred
elsewhere in India but JNU has perhaps sustained its reputation
DIRECTIONS (Qs. 101-107): Read the passage and answer as a university for longer. It already had schools of Computer
the questions that follow: Science and the Life Sciences over four decades ago when these
were fledging disciplines giving it a certain breadth early on.
The government has announced a list of 'Institutes of Eminence'
Even as we may wonder at the exclusion of JNU from the list of
(IoE) among India's institutions of higher education. This was
IoEs released by the government one might wonder at how the
awaited for the simple reason that finding a place on it would
private institutions that are on it made the cut. While BITS Pilani
save an educational institution from the clutches of a dreaded
may have made a significant contribution to the country at a
regulator. Regulators are meant to ensure that we have a socially
time when it desperately needed engineers, but is yet not what
desirable outcome, but in the case of higher education in India
may be considered a university, the presence of the two others
the opposite seems to have been the case. The University Grants
on the list leave one nonplussed. One of them, we are told, has
Commission (UGC) has over half a century micro-managed this
been conferred the status on grounds of its promise, a dubious
space to an unimaginable level of silliness. The result has been
position to take as this institute has little to show except for the
publicly-funded universities that are cavernous wastes,
financial heft that will surely undergird it. The other is known
shattering the aspirations of our youth and producing low-level
largely for its association with the practice of charging capitation
'knowledge'. Evidence of the role of India's higher-education
fees for education.
regulator may be seen in the feature that the few instances when
101. As per your understanding of the passage studied above,
this is not the case the institutions have enjoyed privilege that
what can be some reasons for lack of quality in higher
leaves them protected from its depredations.
education?
The latest offering is in the form of a proposed Higher Education I. State universities recruited a lot of faculty members
Commission of India (HECI). The intention is to leave the HECI on contract basis who have little incentive to perform.
to focus on quality while leaving funding of public institutions
Tg:- @NextGenBankers
to the Ministry of Human Resource Development (MHRD). Even
as we observe the progress of the HECI and wonder if it is going
II. Public universities are insulated from political pressure.
III. The amount spent on research is very less as compared
to foreign Institutions.
to be any more than old wine in a new bottle, we have already (a) Only I (b) Only II
have an inkling of what could go wrong. This springs from the (c) Only I and III (d) Only II and III
government's announcement of a list of IoEs. The government (e) All of the above
has chosen three public and three private institutions for this 102. Which of the following is/are true as per the passage?
status. The public institutions are the Indian Institute of Science, I. Among countries with a comparable research output,
Bengaluru, and the Indian Institutes of Technology at Delhi and India with 0.8% R&D spending trails Russia, Brazil,
Mumbai. The private ones are the Birla Institute of Technology South Korea and even Singapore, according to Unesco
and Science Pilani, the JIO Institute and the Manipal Academy data.
of Higher Education. This list suffers from a serious lack of II. HECI would focus on funding while quality would be
credibility. Where in it are the universities of India? We regulated by the Ministry of Human Resource
understand that the government's aim is to rectify the low Development.
presence of Indian institutions in the global rankings of III. The Institution of Eminence (IoE) status has been
universities. given to six institutes, three each from the public and
While the early European universities may have started as private sectors.
academies of the arts they were soon to have medicine and (a) Only I (b) Only III
astronomy as areas that they pursued with vigour. Somewhere (c) Only I and II (d) Only II and III
along the line we seem to have lost this breadth and come to (e) None of the above
revel in a landscape dominated by engineering schools. These 103. Which of the following best describes the tone of the
engineering schools, notably the IITs, have done us proud but author in paragraph 1?
cannot be equated with the great universities of the world for (a) Euphoric (b) Castigating
the simple reason that they are focussed on a narrow domain. (c) Deploring (d) Lamenting
Also, if the idea behind IoEs is that they will be left alone and (e) None of the above
given enhanced financial support, it must be acknowledged that 104. What could be a/some result/s of the function of funding
until very recently the IITs have not been meddled with neither of public institutions being left to the Ministry of Human
have they been starved of resources. The IISc is of course Resource Development instead of HECI?
broader than the IITs but does not embrace the social sciences I. The government may use its discretion to reward
and the humanities, the presence of which would be considered institutions according to its ideological predilections.
necessary for an university.
SBI Clerk (Junior Associates) Mains Solved Paper-2021 2021-11
II. The Institutions may be forced to comply with even over 2012, and confirming an upward trend over the past years.
some dubious rules setup by the government. In 2008, our global plastic consumption worldwide has been
III. The government can be made accountable for attaining estimated at 260 million tons, and, according to a 2012 report by
excellence in education. Global Industry Analysts, plastic consumption is to reach 297.5
(a) Only II (b) Only I and II million tons by the end of 2015.
(c) Only II and III (d) Only I and III Plastic is versatile, lightweight, flexible, moisture resistant, strong,
(e) All of the above and relatively inexpensive. Those are the attractive qualities
105. Which of the following may be inferred from paragraph 3? that lead us, around the world, to such a voracious appetite and
I. Universities should embody knowledge across a wide over-consumption of plastic goods. However, durable and very
range of disciplines. slow to degrade, plastic materials that are used in the production
II. There is an emphasis on a depth of knowledge across of so many products all, ultimately, become waste with staying
a broad horizon in Indian Universities today. power. Our tremendous attraction to plastic, coupled with an
III. In India, a lot of focus is given to Institutions which undeniable behavioural propensity of increasingly over-
are focused on only few areas. consuming, discarding, littering and thus polluting, has become
(a) Only I (b) Only III a combination of lethal nature.
(c) Only II and III (d) Only I and III A simple walk on any beach, anywhere, and the plastic waste
(e) All of the above spectacle is present. All over the world, the statistics are ever
106. Which of the following may strengthen the argument for growing, staggeringly. Tons of plastic debris (which by definition
putting the Jawaharlal Nehru University in the list of IoEs? are waste that can vary in size from large containers, fishing
I. If there is an Indian institution that engages as an nets to microscopic plastic pellets or even particles) is discarded
equal in the global commons it is JNU. every year, everywhere, polluting lands, rivers, coasts, beaches,
II. Research from JNU has adapted and contested ideas and oceans.
floating in the global pool of knowledge ranging from Published in the journal Science in February 2015, a study
history to economics. conducted by a scientific working group at UC Santa Barbara's
III. If the criterion of engagement with the global field of National Center for Ecological Analysis and Synthesis (NCEAS),
quantified the input of plastic waste from land into the ocean.
ideas is accepted JNU would count as one among
The results: every year, 8 million metric tons of plastic end up in
India's eminent educational institutions.
our oceans. It's equivalent to five grocery bags filled with plastic
(a) Only I
(c) Only I and II
(e) All of the above
Tg:- @NextGenBankers
(b) Only III
(d) Only II and III
for every foot of coastline in the world. In 2025, the annual input
is estimated to be about twice greater, or 10 bags full of plastic
per foot of coastline. So the cumulative input for 2025 would be
107. What best describes the central idea of the passage?
nearly 20 times the 8 million metric tons estimate - 100 bags of
(a) The government's approach to higher education
plastic per foot of coastline in the world!
reflects a short-sightedness due to the social sciences 108. As per the passage, which of the following statements are
and the humanities being ignored. true?
(b) The IoE list is a good start and can be modified going 1. Most of the plastic present in the ocean today has
forward by adding more institutions that focus on originated from the land.
some disciplines. 2. In recent years, the production of plastics has
(c) Eminence is not usually understood in terms of money declined.
but certain exceptions can be made. 3. Plastic pollution is present on almost every beach.
(d) Two previous governments have in the past decade (a) Only 1 (b) Only 2
tried to revamp the regulatory environment for higher (c) Only 1 and 3 (d) All 1, 2 and 3
education. (e) None of 1, 2 and 3
(e) None of the above 109. Which of the following is not the quality of plastic?
DIRECTIONS (Qs. 108-115): Read the passage carefully and (a) Cheap (b) Moisture Resistant
answer the questions given beside. (c) Brittle (d) Lightweight
(e) Flexible.
The world population is living, working, vacationing, 110. Which of the following statements are false according to
increasingly conglomerating along the coasts, and standing on the passage?
the front row of the greatest, most unprecedented, plastic waste 1. There is a 4 percent increase in the production of
tide ever faced. plastics in 2013, over 2012.
Washed out on our coasts in obvious and clearly visible form, 2. The production of plastics in 2012 is approximately
the plastic pollution spectacle blatantly unveiling on our beaches 299 million tons.
is only the prelude of the greater story that unfolded further 3. According to the Global Industry Experts, plastic
away in the world's oceans, yet mostly originating from where consumption is to reach 297.5 million tons by the end
we stand: the land. of 2015.
For more than 50 years, global production and consumption of (a) Only 1 (b) Only 2
plastics have continued to rise. An estimated 299 million tons of (c) Only 2 and 3 (d) Only 1 and 2
plastics were produced in 2013, representing a 4 percent increase (e) Only 3
2021-12 SBI Clerk (Junior Associates) Mains Solved Paper-2021
111. Which organisation has quantified the input of plastic 118. Public expenditure that produces public assets has a higher
waste from land into the ocean? As mentioned in the and more continuous growth multiplier set to the public
passage. consumption expenditure multiplier because it maintains
(a) UC Santa Barbara's National Center for Ecological demand and also reduces costs.
Analysis and Synthesis (a) building, laidback, comparison
(b) Global Industry Analysts (b) creates, persistent, compared
(c) Kano State Environmental Planning and Protection (c) furnishing, broken, correlation
Agency (d) Consume, eternal, distinguished
(d) Foundation for Environmental Education (e) No replacement required
(e) None of the above given options 119. While it is acknowledged that Electric Vehicles contribute
112. According to the author, What can be the reason behind to improving urban air quality, the government policy
the plastic pollution. should take into account the life-cycle impact of the
1. Plastic is cheaply available and that lead to the product and be agnostic to the underlying technology.
overconsumption. (a) Ascertained, indifferent, overtures
2. Plastic does not degrade easily thus it ended up being (b) Debated, enchanted, built
a pollutant. (c) Developed, believer, patented
3. There is no policy to curb plastic pollution. (d) Supported, atheist, seeming
(a) Only 1 (b) Only 2 (e) No replacement required
(c) Only 3 (d) Only 1 and 2 120. The blasts in emerging markets is hardly surprising as the
(e) All 1, 2 and 3 tightening of monetary policy in the U.S. Federal Reserve has
113. Which of the following word is the synonym of the given conventional caused the turning of the global credit cycle.
word? (a) complacency, for, typically
Propensity (b) chaos, of, newly
(a) Tendency (b) Dislike (c) turbulence, by, traditionally
(c) Inadequate (d) Antipathy (d) roil, with, always
(e) None of the above. (e) No improvement required
114. Which of the following word is the synonym of the given
word DIRECTIONS (Qs. 121-125): Answer the following questions
Staggeringly
(a) Predictable
Tg:- @NextGenBankers
(b) Startling
after rearranging thefollowing sentences into a coherent
paragraph andidentify the sentence that doesn't fit into the
(c) Ordinary (d) Unremarkable context ofthe paragraph.
(e) Unexceptional (A) What is apparent from this paper is that the NHA sees the
115. Which of the following word is the antonym of the given nascent state of digitization of our healthcare system as
word? an opportunity to learn from the experiences of other
Prelude countries that are much further down this road.
(a) Preface (b) Prologue (B) The National Digital Health Mission recently released a
(c) Conclusion (d) Preamble Strategy Overview document that laid out, in broad strokes,
(e) None of the above the National Health Authority's plan to build India into a
DIRECTIONS (Qs. 116-120): A sentence is given below with Digital Health Nation.
three words highlighted in bold. Select the option that gives (C) At the outset, it is good to see that the NHA has decided
the correct set of words. In case the sentence is correct, select to adapt and extend India's novel data portability
'No replacement required'. architecture to facilitate transfers across the digital health
ecosystem.
116. While stressing the role of investment in restored the (D) While this allows us to avoid the mistakes that other
economy the Economic Survey also states that the twin countries have made, the sheer scale of what the document
balance sheet issue has to be undertake on an urgent basis. attempts to deliver seems too vast for what is possible for
(a) Highlighting, slowing, tackled us to achieve.
(b) emphasizing, stimulating, addressed (E) Care delivery systems looking to make meaningful impact
(c) shouting, excited, focused through data face a number of challenges, the most critical
(d) Assuaging, invigorating, overlooked one is collecting data from various sources
(e) No replacement required (F) However, the technological scaffolding on which this
117. The concept of financial inclusion, special in developing portability will be implemented is contained in the Data
markets, has always met with challenges relating to Empowerment and Protection Architecture, a framework
accessibility and affordability. that is already being used for the sharing of financial
(a) Inspired, commonly, connect
information through the central bank's account aggregator
(b) Thought, peculiar, belonging
framework.
(c) Idea, particularly, pertaining
(G) The regulatory basis for India's unique brand of data portability
(d) Notion, heed, relevant
is set out in the draft Personal Data Protection Bill.
(e) No replacement required
SBI Clerk (Junior Associates) Mains Solved Paper-2021 2021-13
121. Considering statement (B) as the first sentence of the 126. Mr. Tharoor's urban manners charm friends and enemies alike.
rearranged paragraph, which of the following statement (a) urban manners charming friends
should be last sentence after the rearrangement? (b) urbane manners charming friends
(excluding the incoherent sentence) (c) urban mannerisms charm friends
(a) A (b) E (c) B (d) F (d) urbane manners charm friends
(e) D (e) No correction required
122. Considering statement (B) as the first sentence of the 127. Religious bigots look away on anyone who does not
rearranged paragraph, which of the following statement conform to their beliefs.
should be FOURTH sentence after the rearrangement? (a) bigots look away on
(excluding the incoherent sentence) (b) bigots look down on
(a) A (b) D (c) B (d) G (c) bigots look behind on
(e) C (d) bigots look in front of
123. Considering statement (B) as the first sentence of the (e) No correction required
rearranged paragraph, then which one among the following 128. Having leisure till outdoor activities such as taking a stroll
will be the PENULTIMATE sentence? down the park has become a rarity in this fast-paced life.
(A) What is apparent from this paper is that the NHA sees the (a) Having leisures till outdoor
nascent state of digitization of our healthcare system as (b) Having laziness for outdoor
an opportunity to learn from the experiences of other (c) Having leisure in outdoor
countries that are much further down this road. (d) Having leisure for outdoor
(B) However, the technological scaffolding on which this (e) No correction required
portability will be implemented is contained in the Data 129. The Punjab National Bank fraud demonstrates the extent
Empowerment and Protection Architecture, a framework of operational and risk management failures in PSBs.
that is already being used for the sharing of financial (a) demonstrateing the extent of
information through the central bank's account aggregator (b) demonstrates the extent in
framework. (c) demonstrates the extent with
(C) The regulatory basis for India's unique brand of data (d) demonstrates the extention of
portability is set out in the draft Personal Data Protection (e) No correction required
Bill.
(D) Either (a) or (c)
(E) None of the above
Tg:- @NextGenBankers DIRECTIONS (Qs. 130-133): Replace the words if needed
andchoose option 5 if no replacement is required.
124. Considering statement (B) as the first sentence of the 130. Under the affluence of the anticipated cyclone, light to
rearranged paragraph, then which among the following moderate rain is likely to commend over Kerala at a few
fails to become the part of the coherent paragraph? places on Sunday.
(a) A (b) B (c) D (d) F I. Affluence - influence
(e) E II. Anticipated - antecedent
125. Choose the most logical order of sentences among the given III. Commend - Commence
choices to construct a coherent paragraph. (a) Only (II) (b) Both (I) and (II)
(A) The Reserve Bank of India did not extend the moratorium (c) Both (I) and (III) (d) Only (III)
in its monetary policy review on Thursday. (e) None of these
(B) It allowed banks to restructure the loans of borrowers who 131. While Karthik is interested in pondering engineering, he is
are in financial difficulty and are unable to repay them. also infused about the pure sciences which shows that he
(C) This means that the lenders won't report the borrower as a has varied interests.
defaulter to credit bureaus if the borrower follows the new I. Pondering - pursuing
payment structure. II. Infused - enthused
(D) In his speech, RBI governor said, "The disruptions caused III. Varied - dissimilar
by COVID-19 have led to heightened financial stress for (a) Both (I) and (II) (b) Both (I) and (III)
borrowers across the board. (c) (I), (II) and (III) (d) Only (I)
(E) Once restructured, such loans would be considered as (e) None of these
standard. 132. With nearly 10 million people construed to die annually
(a) ABCDE (b) ACBDE from resistant infections by 2050, health-care costs and
(c) ABECD (d) ACDBE the cost of food production will spike, while income
(e) ABDCE inequality will lengthen.
I. Construed - Analysed
DIRECTIONS (Qs. 126-129): In each question, a part of the II. Spike - snap
sentence is made bold. Below are given alternatives to the III. Lengthen - widen
bold part at (A), (B), (C) and (D) which may improve the (a) Only (III) (b) Only (I)
sentence. Choose the correct alternative. In case no (c) Both (I) and (III) (d) Both (I) and (II)
replacement is needed, mark (E) as your answer. (e) None of these
2021-14 SBI Clerk (Junior Associates) Mains Solved Paper-2021
133. The positive April GST numbers have come as a surprise 138. An experienced set of economic policymakers is leading
to many experts, given the lacklustre economic activity the country back into the pre-1991 era; they may have a
witnessed across many sectors in recent months, which long reign in which to ruin the economy, The economy
should normally have impacted tax collections adversely. has collapsed under their stewardsship; they have no idea
I. Surprise - Panache how to revive it But Ahluwalia is not one to give up; he
II. Lacklustre - sheen has written an epilogue of advice on how to rescue it.
III. Adversely - evasively (a) Experienced (b) Reign
(a) Both (II) and (III) (b) Both (I) and (III) (c) Stewardsship (d) Epilogue
(c) Both (I) and (II) (d) (I) , (II) and (III) (e) All are correct
(e) None of these 139. Over the months, Indian Davy personnel train the eight
DIRECTIONS (Qs. 134-137) : In the question below, a sentence submariners and other Bangla volunteers to become
is given with two blanks, that indicate that someparts are underwater saboteours. By mid-August, these trainees
missing. Identify the correct pair of words that fit in the sentence infilterate East Pakistan from Meghalaya,Assam, Tripura
to make it grammaticallyand contextually correct. and West Bengal to launch attacks against Pakistani
shipping and dock facilities.
134. Being near running water and good shade, the explorers (a) Personnel (b) Volunteers
_________ it was a good _________ for setting up camp. (c) Saboteours (d) Infilterate
1. Aimed, place (e) Both (c) and (d)
2. Decided, locale 140. From August till end-November, when hostilities ramp up
3. Concluded, location into proper war, the naval comandos of the Mukti Bahini
(a) Only 3 (b) Only 1 and 2 inflict serious damage to Pakistani shipping, impeding
(c) Only 1 and 3 (d) Only 2 and 3 everything from international shipping and water-based
(e) All 1, 2 and 3 supply routes to troop movement across the vast riverine
135. The Indian government has ______ a broader strategy terrain.
that is specifically _______ on increasing foreign (a) Hostilities (b) Comandos
investment. (c) Riverine (d) Both A and B
1. launched, focused (e) Both B and C
General/Financial Awareness
Tg:- @NextGenBankers
2. formulated, centred
3. established, targeted
(a) Only 3 (b) Only 1 and 2
141. What is the name of the proposed budget airline funded
(c) Only 1 and 3 (d) Only 2 and 3
by Rakesh Jhunjhunwala?
(e) All 1, 2 and 3
(a) Ektara Airline (b) Akanksha Airline
136. Technological changes have ______ new job positions
and also led to an increase in the _____ for new skills. (c) Akasa Airline (d) Velvet Airlines
1. produced, requirement (e) None of the above
2. catered, descent 142. Pan India 'SUPACE' project is monitored and funded by
3. created, demand which ministry that was launched by the chief justice of
(a) Only 3 (b) Only 1 and 2 India, S A Bobde?
(c) Only 1 and 3 (d) Only 2 and 3 (a) Ministry of Law and Justice
(e) All 1, 2 and 3 (b) Ministry of Science and Technology
137. India's _______ as an education hub is the result of both (c) Ministry of Electronics and Information Technology
economic changes and ______ efforts. (d) Ministry of Social Justice and Empowerment
1. Advent, significant (e) None of the above
2. Rise, remarkable 143. According to the All India Survey on Higher Education
3. Emergence, adequate (AISHE) 2019-20 report, what is Gross Enrolment Ratio
(a) Only 3 (b) Only 1 and 2 (GER) in higher education in India?
(c) Only 1 and 3 (d) Only 2 and 3 (a) 22.1 per cent (b) 25.7 per cent
(e) All 1, 2 and 3 (c) 26.5 per cent (d) 27.1 per cent
(e) 28.3 per cent
DIRECTIONS (Qs. 138-140): In each question below, four 144. Who stepped down as Managing Director (MD) of HCL
words printed in bold type are given. These are numbered (A), Technologies Ltd?
(B), (C) and (D). One of these words printed in bold might be (a) Roshni Nadar (b) C Vijayakumar
either wrongly spelt or inappropriate in the context of the (c) Shiv Nadar (d) Vineet Nayar
sentence. Find out the word that is inappropriate or wrongly (e) Anant Gupta
spelt, if any. The number of the word is your answer. If the 145. What does 'P' stand for in the TOP scheme?
words printed in bold are correctly spelt and appropriate in (a) Palm Oil (b) Preservative
the context of the sentence then mark (E), i.e. 'All Correct', as (c) Post (d) Potato
your answer. (e) Promotion
SBI Clerk (Junior Associates) Mains Solved Paper-2021 2021-15
146. CREDAI is an apex body of which industry/ sector? (a) 2014 (b) 2015 (c) 2013 (d) 2012
(a) Tourism (b) Real estate (e) 2016
(c) Information Technology(d) Hospitality 158. Who is the current Chief Economic Advisor to the
(e) None of the above Government of India?
147. Which bank has signed an agreement with Ola Electric for (a) Arvind Subramanian (b) Surjeet Bhalla
the largest long-term debt financing in the Indian EV (c) K Subramanian (d) Amitabh Ghosh
industry? (e) None of the above/More than one of the above
(a) Bank of India (b) Bank of Baroda 159. The Reserve Bank of India (RBI) has decided to levy
(c) Punjab National Bank (d) Bank of Maharashtra monetary charges of what amount on ATMs that run out
(e) Canara Bank of cash, starting 1 October, 2021?
148. N Rangaswamy from All India NR Congress took oath as (a) Rs 2000 (b) Rs 5000
chief minister of Puducherry for the _______. (c) Rs 10000 (d) Rs 15000
(a) first time (b) second time (e) Rs 20000
(c) third time (d) fourth time 160. Which state will continue to sponsor Indian men's and
(e) None of the above women's national hockey teams for another 10 years, after
149. The government's stake in ITC Limited which is worth the current sponsorship ends in 2023?
about Rs 20,250cr currently _____ (a) Andhra Pradesh (b) Haryana
(a) 6.43% (b) 7.93% (c) Punjab (d) Odisha
(c) 8.33% (d) 10 .43% (e) Karnataka
(e) 15.53% 161. World Red Cross And Red Crescent Day observed annually
150. Which state has received a maximum Rural Infrastructure on____.
(a) 8th May (b) 9th May
Development Fund (RIDF) in the last three years?
(c) 7th May (d) 6 th May
(a) Haryana (b) Tamil Nadu
(e) 5th May
(c) Odisha (d) Assam
162. What is the revised overdraft limit under Pradhan Mantri
(e) Rajasthan
Jan Dhan Yojana?
151. World's highest motorable road was inaugurated in Umling
(a) Rs 5,000 (b) Rs 20,000
La in Eastern Ladakh at a height of ______, which (c) Rs 10,000 (d) Rs 2,000
connects Leh to the Pangong Lake.
(a) 19,300 feet
(c) 18,600 feet
Tg:- @NextGenBankers
(b) 20,200 feet
(d) 17,700 feet
163.
(e) Rs 1,000
Who has been selected for the Nobel Peace Prize, 2020?
(a) European Union (b) World Food Programme
(e) 16.700 feet (c) Robert B Wilson (d) Paul R. Milgrom
152. What is the full form of DEAF? (e) None of the above/More than one of the above
(a) Depositor Education Awareness Fund 164. Which Bank has launched a digital and contactless banking
(b) Defence Education Awareness Fund platform, especially for retail merchants named as 'Merchant
(c) Development Education Awareness Fund Stack'?
(d) Developers Education Awareness Fund (a) HDFC Bank (b) Yes Bank
(e) None of the above (c) ICICI Bank (d) IndusInd Bank
153. Which one is not a Fast-Moving Consumer Goods (e) Axis Bank
(FMCGs) company? 165. What is the rank of India in global hunger index 2020?
(a) Hindustan Unilever Ltd (b) Zomato (a) 107th (b) 110th (c) 95th (d) 94th
(c) ITC Ltd (d) Nestle India Ltd (e) None of the above/more than one of the above
(e) Dabur India Ltd 166. With which organisation, Intel has collaborated to launch
154. What does 'P' stand for in TOP Scheme launched by the 'AI For All' initiative?
Ministry of Youth Affairs and Sports? (a) AICTE (b) NITI Aayog
(a) Podium (b) Para (c) CBSE (d) UGC
(c) Performance (d) Pension (e) None of these
(e) None of the above 167. Which two Indian organisations have won the UNDP
155. Indian Ace Badminton player PV Sindhu won two Olympic Equator Prize 2021?
medals in individual events, who also achieved the same (a) Paramount Limited and Doshion Limited
feat? (b) Aadhimalai Pazhangudiyinar Producer Company
(a) Abhinav Bindra (b) Sushil Kumar Limited and Snehakunja Trust
(c) MC Mary Kom (d) Deepika Kumari (c) ASM Company and Enviro Engineer Pvt Ltd.
(e) Geeta Phogat (d) Excel Enviro Services and GreenTek Indika
156. The Hemis National Park is located in which state/UTs of India? (e) None of these
(a) Ladakh (b) Haryana 168. Which country will host the inaugural International Hockey
(c) Jammu and Kashmir (d) Madhya Pradesh Federation (FIH) Hockey5s World Cup?
(e) None of these (a) Indonesia (b) South Korea
157. In which year Kailash Satyarthi was awarded the Nobel (c) Oman (d) UAE
Peace Prize? (e) Japan
2021-16 SBI Clerk (Junior Associates) Mains Solved Paper-2021
169. Which fertilizer manufacturing company has introduced 180. Which institution released the "Tracking Universal
the world's first Nano Urea Liquid for farmers? Coverage - 2021 Global Monitoring Report"?
(a) Brahmaputra Valley Fertilizer Corporation Ltd (a) WTO (b) WHO (c) FAO (d) WEF
(b) Chambal Fertilisers and Chemicals Ltd. (e) None of the above
(c) Hindalco Industries Ltd. 181 "bob World Wave", launched by Bank of Baroda (BoB) is
(d) Greenstar Fertilizers Ltd a …………..
(e) Indian Farmers Fertiliser Cooperative Ltd (a) Global Debit Card (b) Wearable Device
170. What is the motto of the 2020 Tokyo Olympics? (c) AI Chatbot (d) Investment Platform
(a) Come Share (e) None of the above
(b) "Faster, Higher, Stronger - Together". 182. Which country is set to host the first tribal nations' summit
(c) Inspire a Generation since 2016?
(d) Connected by Emotion (a) India (b) USA (c) Japan (d) Russia
(e) None of the above/More than one of the above (e) None of the above
171. Along with Jio Platform which company have figured in Time 183. Which state has approved 'Kaiser-i-Hind' as the State
Magazine's first-ever list of 100 Most Influential Companies? butterfly?
(a) Byju's (b) Swiggy (a) Assam (b) Sikkim
(c) Big Basket (d) ReNew Power (c) Arunachal Pradesh (d) Odisha
(e) One97 (e) None of the above
172. What is India's rank in the 'Climate Change Performance 184. India signed a MoU with which country to exchange
Index (CCPI) 2021'? information in the field of public sector audit?
(a) 6th (b) 7th (c) 8th (d) 9th (a) Sri Lanka (b) Switzerland
(e) 10th (c) Maldives (d) Cayman Islands
173. India was elected to the Economic and Social Council (e) None of the above
(ECOSOC), one of the 6 main organs of the United Nations, 185. Which global bloc has entered into an agreement cross-
for the- ______term. border data use and digital trade?
(a) 2021-22 (b) 2022-25 (a) G-20 (b) G-7 (c) ASEAN (d) BRICS
(c) 2025-28 (d) 2022-24 (e) ASEM
(e) 2022-26 186. Which organization releases the All-India Consumer Price
Tg:- @NextGenBankers
174. SBI has launched NETC- SBI FASTag. What does N stands
for in NETC?
Index?
(a) NITI Aayog
(a) Network (b) National (b) National Statistical Office
(c) Numbered (d) Nominal (c) Centre for Agriculture costs and prices
(e) None of the above (d) Ministry of Finance
175. Pandit Jasraj was associated with which field? (e) None of the above
(a) Literature (b) Vocal Music 187. A working group of IRDAI has proposed to reintroduce
(c) Kathak (d) Painting ILIP. What is its expansion?
(e) None of the above (a) Interest- linked Insurance Policy
176. Which country has agreed to cooperate with the European (b) Inflation- linked Insurance Policy
Union on Energy Security? (c) Index- linked Insurance Policy
(a) USA (b) Australia (d) Investment- linked Insurance Policy
(c) India (d) Japan (e) None of the above
(e) China 188. Which Union Ministry announces the Index of Industrial
177. Where is Indravati Tiger Reserve located? Production (IIP)?
(a) Karnataka (b) Chhattisgarh (a) Ministry of Statistics and Programme Implementation
(c) Bihar (d) West Bengal (b) Ministry of Finance
(e) Madhya Pradesh (c) Ministry of Corporate Affairs
178. What is the name of the Indian Navy's indigenous stealth (d) Ministry of Heavy Industries
destroyer, whose maiden sea trials was conducted (e) None of the above
recently? 189. What per cent of India's exports is contributed by MSMEs,
(a) Mormugao (b) Kalinga approximately?
(c) Ashoka (d) Kalvari (a) 30 (b) 40 (c) 50 (d) 60
(e) Vikrant (e) 70
179. Which organisation released the 'State of the world's land 190. Shavkat Mirziyoyev has been re-elected as the President
and water resources for food and agriculture (SOLAW of which country?
2021) report? (a) Afghanistan (b) Uzbekistan
(a) IMF (b) FAO (c) World Bank (d) WEF (c) Syria (d) Maldives
(e) None of the above (e) Mauritius
SBI Clerk (Junior Associates) Mains Solved Paper-2021 2021-17
ANSWER KEY
1 (b) 21 (a) 41 (b) 61 (c) 81 (e) 101 (c) 121 (d) 141 (c) 161 (a) 181 (b)
2 (a) 22 (d) 42 (c) 62 (a) 82 (b) 102 (b) 122 (e) 142 (a) 162 (c) 182 (b)
3 (e) 23 (b) 43 (c) 63 (b) 83 (e) 103 (c) 123 (c) 143 (d) 163 (b) 183 (c)
4 (c) 24 (c) 44 (e) 64 (d) 84 (b) 104 (b) 124 (e) 144 (c) 164 (c) 184 (c)
5 (d) 25 (d) 45 (c) 65 (b) 85 (a) 105 (d) 125 (c) 145 (d) 165 (d) 185 (b)
6 (e) 26 (b) 46 (c) 66 (a) 86 (d) 106 (e) 126 (d) 146 (b) 166 (c) 186 (b)
7 (d) 27 (b) 47 (b) 67 (c) 87 (c) 107 (a) 127 (b) 147 (b) 167 (b) 187 (c)
8 (c) 28 (b) 48 (d) 68 (d) 88 (b) 108 (c) 128 (d) 148 (d) 168 (c) 188 (a)
9 (c) 29 (a) 49 (d) 69 (a) 89 (a) 109 (c) 129 (e) 149 (b) 169 (e) 189 (b)
10 (b) 30 (d) 50 (c) 70 (a) 90 (d) 110 (c) 130 (d) 150 (c) 170 (b) 190 (b)
11 (e) 31 (e) 51 (d) 71 (c) 91 (a) 111 (a) 131 (a) 151 (a) 171 (a)
12 (b) 32 (a) 52 (c) 72 (b) 92 (b) 112 (d) 132 (a) 152 (a) 172 (e)
13 (b) 33 (c) 53 (b) 73 (d) 93 (c) 113 (a) 133 (e) 153 (b) 173 (d)
14 (c) 34 (c) 54 (e) 74 (c) 94 (d) 114 (b) 134 (d) 154 (a) 174 (b)
15 (a) 35 (e) 55 (d) 75 (b) 95 (d) 115 (c) 135 (b) 155 (b) 175 (b)
16 (d) 36 (e) 56 (c) 76 (b) 96 (e) 116 (b) 136 (c) 156 (a) 176 (a)
17 (e) 37 (c) 57 (d) 77 (d) 97 (a) 117 (c) 137 (d) 157 (a) 177 (b)
18 (a) 38 (c) 58 (b) 78 (c) 98 (b) 118 (b) 138 (c) 158 (c) 178 (a)
19 (d) 39 (c) 59 (d) 79 (e) 99 (d) 119 (e) 139 (e) 159 (c) 179 (b)
20 (c) 40 (c) Tg:- @NextGenBankers
60 (e) 80 (c) 100 (e) 120 (c) 140 (b) 160 (d) 180 (b)
Tg:- @NextGenBankers
16. (d) Only II conclusion is true.
As, per the given Statements, we have; 27. (b)
vehicles.
Only conclusion II follows.
- It is not evident from the fact that the population of
N W Q; U < P = G; G Q > K; W > C; D U
developing countries will not continue to increase in
(i) Q @ D (False) (ii) K & P (True)
future it may or not increase.
(iii) C @ Q (True)
- With increasing population and deputing resources,
17. (e) None of the above conclusion is true. it will be definitely difficult for the government of
As, per the given statements, we have; developing countries to provide its people decent
N W Q; U < P = G; G Q > K; W > C; D U quality of life.
(i) D N (False) (ii) P @ N (False) 28. (b) Only II follows.
(iii) C $ K (False) - Keeping in mind, the unhealthy impact of plants at
18. (a) Only conclusion I isTrue. night, we should grow in outer spaces like windows
As, per the given statements, we have; or balconies, It can be grown at home, but outside the
N W Q; U < P = G; G Q > K; W > C; D U room.
(i) P W (True) (ii) D @ G (False) 29. (a) Argument I alone is True.
(iii) K # U (False) The usage of mobile phones by students in schools
will divert the minds of students and will become an
Sol. (19 - 23)
unproductive gadget under those age group students.
4m N The use of mobile phones is not directly related with
O
state travel and communication to parents. It can be
15 m done without mobile phones also.
Y 4m K 30. (d) Both I and II are strong.
18 m Clearly, cigaette should be banned. Because, it is
4m injurious to health, which causes cancer.
Also, Banning Cigarettes would surely render jobless
S 7m P the large number of workers involved in
manufacturing it.
18 m
W 31. (e) There are 14 letters between A and P.
3rd letter from right end of 2nd word from left = A
U Ist letter from left end of 2nd word from right = P
SBI Clerk (Junior Associates) Mains Solved Paper-2021 2021-19
Sol. (32 -36) 44. (e) None dice is placed between 3 and 2.
45. (c) 1 is placed just above 4.
Months 2009 2010 Sol. (46- 50) :
55. (d) Required percentage Total mixture after mixing both = 30 + 20 = 50 Litres
Arsha 's Salary 60000 And, milk in new mixture = 12 + 12 = 24 Litres
= 100 100 = 75%
Taniya 's Salary 80000 24
Hence, required percentage = 100 = 48%
56. (c) Time taken tocollide trains 50
60. (e) Let J and K's profit share is x and y respectively.
300 300 For J,
= = 2.5 hrs
72 48 120 2
Bird's speed = 68 km/hr 10 x121
CI = x 1 x x
Hence, total distance covered by bird 100 100
= 68 × 2.5 = 170 km
21x
10 x 10 3x 3969 =
57. (d) Total expenditure of P = x 2 100
100 100 10
P's saving = ` 35,000 3969
x= 100 = ` 18,900
21
3x 7x
x– = 35000 = 35000 For K,
10 10
2
20 36 y
35000 10 CI = y 1 y y
x= = ` 50,000 (P's income) 100 25
7
y 10 y 11 y
Total expenditure of Q = 12000 12000 8316 =
100 10 25
75 3y 8316 25
Q's saving = y y= = ` 18,900
100 4 11
L's share in profit = 63,000 – (18900 + 18900)
y 3y = ` 25,200
y– 12000
y–
10
y
Tg:- @NextGenBankers
3y
12000
4
0
Hence, interest earned by L = 25200 1
10
100
2
10 4
121 21
20 y 2 y 15 y = 25200 1 = 25200 × = ` 5292
= 12000 100 100
20
3y = 20 × 12000 61. (c) 133 × 4 ÷ 52 + 108 ÷ 3 1728 × 7 = ?
20 12000 13 13 13 4 108
y= = ` 80,000 (Q's income) ?= 7
3 52 12
58. (b) Let capacity fo vessel in which chemical B is present ? = 169 + 9 × 7 = 169 + 63 = 232
is 50.
62. (a) (346 + 130 – 45) + (35 ÷ 7 × 3) = ? – 1521
P Q
A (1 : 3 = 4) × 5 431 + 15 = ? – 39
B (3 : 2 = 5) × 10 ? = 446 + 39 = 485
63. (b) (25% of 250 × 4) + (21% of 300 – 50) = ? – 20% of 40
25 21 20
2 250 4 300 – 50 = ? – 40
A 5 : 15 = 20 [ Capacity of vessel A is of 100 100 100
5
250 + 63 – 50 = ? – 8
capacity of vessel B] ? = 263 + 8 = 271
B 30 : 20 = 50 64. (d) Pattern of the series is,
In large container 35 : 35 = 70 12 + (13 – 12) = 12
35 12 + (23 – 22) = 16
Hence, Chemical P in final mixture = 100 = 50%
70 16 + (33 – 32) = 34
34 + (43 – 42) = 82 80
40 65. (b) Pattern of the series is,
59. (d) Milk in 30 litres mixture = 30 = 12 Litres
100 74 + 51 = 125
125 – 49 = 76
60
Milk in 20 litres mixture = 20 = 12 Litres 76 + 46 = 122
100 122 – 42 = 80 79
SBI Clerk (Junior Associates) Mains Solved Paper-2021 2021-21
66. (a) For statement I: Let the investment of Ekansh and According to question,
Vinita is 5x and 4x respectively.
1
Ratio of profit = Ekansh : Vinita : Ayush Profit ratio of C and D =
3
75 120
= 5x × 12 : 4x × 6 + 4x × × 6 : 5x 8 a 5000 1
100 100 =
= 60x : 24x + 18x : 48x 4a 3
= 60x : 42x : 48x 3a + 15000 = 4a
Ratio of profit = 10 : 7 : 8 a = 15000
7 Profit ratio = A : B : C : D
Vinita's profit share = 25000 × = Rs. 7000 = 30000 : 90000 : 20000 : 60000
25
=3:9:2:6
So, Statement I alone is sufficient.
For statement II: Let Vinita invested in business is x. 9
Ekansh's investment = x + 7000 So, B's profit share = 16000 × = ` 72,000
2
Ayushi's investment = x + 16000 Hence, all the statements are sufficient.
One data is missing, so we can't find ratio of
69. (a) Let Shyam's investment is P.
investment or profit.
67. (c) From Statement I: Let speed of Rahul and Yogesh is x
x km/hr and y km/hr respectively and time taken by Time = year
10
both to meet each other is t. So, Ram's investment is (3000 – P).
Distance between P and R = 9 × y = 9y km
Rate = x%
And, distance between Q and R = 4 × x = 4x km
For Ram,
9y
t= [time taken by Yogesh between Q and R] x /10
x x
CI = 3000 P 1 1
100
4x
And, t = [time taken by Yogesh between Q and R]
4 For Shyam,
t×t=
9y
×
Tg:- @NextGenBankers
4x
= 36 P
1
x
x /10
1
x 4 CI = 2 100
t = 36 = 6 hrs
Hence, they will meet at 3:00 + 6 = 9:00 Pm each other. P x
x
So, statement I is sufficient. SI = 2 10 x2 P
From statement II: Let speed of Rahul and Yogesh is 100 2000
2x and 3x km/hr respectively
From statement I:
1200
So, time taken to meet each other = 2
5x Shyam's investment = 3000 × = ` 2000
3
Statement II alone is not sufficient.
From statement III: Difference between speed of 1
Rahul and Yogesh = 40 km/hr Ram's investment = 3000 × = ` 1000
3
Can't find anything from statement III.
According to statement I,
Now, from statement II and III together,
4280 – 3000
3x – 2x = 40
x = 40 x x
So, speed of Rahul = 2 × 40 = 80 km/hr x 10 x 10
= 1000 1 1 1000 1 1 x2
Speed of Yogesh = 3 × 40 = 120 km/hr 100 100
1200
Time taken to meet each other = = 6 hrs
80 120 x
Hence, they will meet at 9:00 PM each other. x 10
1280 = 2000 1 1 x2
Therefore , either I or II and III together are sufficient. 100
68. (d) From statement I, II and III,
A: B : C : D From above equation, x's value can be determined as
Ratio of investment there is only single variable in the equation.
= a × 12 : 3a × 12 : (a + 5000) × 6 : 4a × 6 Hence, statement I is sufficient.
= 2a : 6a : (a + 5000) : 4a
2021-22 SBI Clerk (Junior Associates) Mains Solved Paper-2021
20
Members in product department = 720 × = 40
Total Ratio 360
Vessel Milk Water Hence, members to be added in H.R. department
Quantity (Milk:Water)
= 40 – 10 = 30
P 30 L 25 L 5L 05:01
200 4
Q 30 L 18 L 12 L 03:02 75. (b) Male in software department = 720 × = 320
360 5
R 30 L 20 L 10 L 02:01
S 50 L 35L 15 L 07:03 25 7
Female in tech department = 720× = 35
T 50 L 42 L 8L 21:04 360 10
320
71. (c) Quantity of pure milk after replacement in the final Hence, required percentage = 100 = 914.29%
mixture 35
76. (b) The number of QA members in 2022
3 5 2
= 20 1 1 1 = 14 Litres 50 180
30 30 30 = 720 = 180
360 100
Quantity of water in the final mixture
= 30 – 14 = 16 Litres The number of marketing members in 2022
Hence, required ratio of Milk = 7 : 8 60 50
= 720 = 60
5 360 100
72. (b) Milk in vessel P =
6 3
Male in QA department = 180 = 54
7 10
Milk in vessel S =
10 2
Mixed mixture from vessel P and S in the ratio = 1 : And, Female in marketing department = 60 × = 24
5
1 Hence, required ratio = 54 : 24 = 9 : 4
Let part of pure milk in the mixture purchased by
SBI Clerk (Junior Associates) Mains Solved Paper-2021 2021-23
5 2 5 5
77. (d) Male in HR department = 720 × =4 y= ,
360 5 3 4
200 4 10 5 5 5
Male in software department = 720 × = 320 3 3 3 4
360 5
Hence, x > y
25 3 81. (e) I. x2 + 43x + 406 = 0
Male in tech department = 720 × = 15
360 10 x2 + 29x + 14x + 406 = 0
60 3
(x + 29) (x + 14) = 0
Male in marketing department = 720 × = 72 x = –14, –29
360 5
II. y2 + 46y + 513 = 0
20 1 y2 + 27y + 19y + 513 = 0
Male in product department = 720 × = 20 (y + 27) (y + 19) = 0
360 2
y = –19, –27 = 0
50 3 As, –14 > –19 > –27 > –29
Male in QA department = 720 × = 30
360 10 Hence, relationship between x and y can't be
Hence, the number of males in all department established.
= 4 + 320 + 15 + 72 + 20 + 30 = 461 82. (b) 1. 4x2 – 20x + 16 = 0
78. (c) Total income of males in tech department for one 4x2 – 16x – 4x + 16 = 0 4x(x – 4) –4(x – 4) = 0
month (4x – 4) (x – 4) = 0 x = 4, 1
= 15 × 5000 × 30 = 2250000 = 22.5 Lakh II. 4y2 – 80y + 336 = 0 4y2 – 24y – 56y +336 = 0
(4y – 56) (y – 6) = 0 y = 6, 14
79. (e) 1 . 3x2 – (2 + 15 11 )x + 10 11 = 0 As 14 > 6 > 4 > 1
3x2 – 2x – 15 11 x + 10 11 = 0 Hence, y > x.
83. (e) 1. 7x2 – 92x + 96 = 0
x(3x – 2) – 5 11 (3x – 2) = 0 7x2 – 84x – 8x + 96 = 0
Tg:- @NextGenBankers
(3x – 2) (x – 5 11 ) = 0
2
7x(x – 12) –8(x – 12) = 0
8
x= , (7x – 8) (x – 12) = 0 x= , 12
3 5 11 7
II. 9y2 – 128y + 143 = 0
II. 9y2 – (5 + 18 11 )y + 10 11 = 0 9y2 – 117y – 11y +143 = 0
9y2 – 5y – 18 11 )y + 10 11 = 0 9y(y – 13) –11(y – 13) = 0
(y – 13) (9y – 11) = 0
y(9y –5) – 2 11 (9y – 5) = 0
11
(9y –5) (y – 2 11 ) = 0 y= , 13
9
5 11 8
y= ,
9 2 11 As 13 > 12 >
9
>
7
2 5 Hence, relationship between x and y can't be
5 11 2 11 established.
3 9
84. (b) Let cost price of mobile phone is 100x.
Hence, relation between x and y cannot be
established. 150
M.P. = 100 x = 150x
80. (c) 1. 9x2 – 45x + 50 = 0 100
9x2 – 15x – 30x + 50 = 0 80
3x(3x – 5) – 10(3x – 5) = 0 S.P. = 150 x = 120x
100
(3x – 5) (3x – 10) = 0
18 108 x
5 10 18% GST on S.P. = 120 x
x= , 100 5
3 3
II. 12y2 – 35y + 25 = 0 108 x
So, total cost to shop owner of mobile = 100x +
12y2 – 15y – 20y + 25 = 0 5
3y(4y – 5) –5(4y – 5) = 0
608
(4y – 5)(4y – 5) = 0 = x
5
2021-24 SBI Clerk (Junior Associates) Mains Solved Paper-2021
Tg:- @NextGenBankers
for the lack of quality.
Hence, option (c) is correct.
Statement (d) is incorrect as this has not been
mentioned anywhere.
Statement (a) is correct as this neatly states the gist
102. (b) I has not been mentioned anywhere and is incorrect.
'The intention is to leave the HECI to focus on quality of the passage- that even with a new regulator, the
while leaving funding of public institutions to the government's approach remains short sighted and this
Ministry of Human Resource Development (MHRD).' can be seen in its choices for IoEs.
II is incorrect. Thus, option A is the best fit.
'This springs from the government's announcement 108. (c) Passage Para-2 : makes it clear that most of the plastic
of a list of IoEs. The government has chosen three that ended up being in the ocean, originates from the
public and three private institutions for this status. land. This makes the statement 1 true.
The public institutions are the Indian Institute of Refer to: Para-3 For more than 50 years, over 2012, and
Science, Bengaluru, and the Indian Institutes of confirming an upward trend over the past years.
Technology at Delhi and Mumbai. The private ones This makes the statement 2 invalid.
are the Birla Institute of Technology and Science Refer to: Para-5: A simple walk on growing,
Pilani, the JIO Institute and the Manipal Academy of staggeringly.
Higher Education.' Option (c) is the correct answer.
III is correct. 109. (c) All the qualities other than Brittle have been
Hence, option (b) is correct. mentioned in the passage.
103. (c) Euphoric: full of energy, excitement, and cheerfulness. Thus the option C is the correct answer.
Castigating: reprimand/rebuke (someone) severely. 110. (c) Refer to: Para-3: For more than 50 years, over 2012,
Deploring: feel or express strong condemnation of and confirming an upward trend over the past years.
(something). Thus the statement 1 is true.
Lamenting: a passionate expression of grief or sorrow. The production of plastics was 299 million tons in
The author is criticizing the regulation of the education 2013 and not in 2012. This makes the statement 2 false.
sector and elaborates on this in the entire paragraph. The report is published by the "Global Industry
Option (d) can also be eliminated as the author is not Analyst" and not by the 'Global Industry Experts'.
sad or expressing grief but is angry. This makes the statement 3 invalid.
Out of options (b) and (c), (c) is a better choice as the Hence, The answer will be option C i.e. Both statement
author is not verbally scolding anyone (castigating) 2 and 3 are false.
but is expressing disapproval and criticizing the heavy 111. (a) Option (a) is correct.
2021-26 SBI Clerk (Junior Associates) Mains Solved Paper-2021
112. (d) Plastic is comparatively cheaper. This quality of plastic disruptions caused by COVID-19 have led to heightened
has lead to over-consumption of plastic goods. Thus financial stress for borrowers across the board.
the statement 1 is true. 126. (d) 127. (b)
Plastic does not degrade easily, Ultimately it ended up 128. (d) The original sentence is erroneous.
being a pollutant. This makes the statement 2 valid. Reason: The word 'leisure' must be followed the
There has been no mention in the passage about the preposition 'for' instead of 'till' in this context. The
the statement 3. This makes the statement 3 invalid. expression "leisure for" means 'free time for'.
Evidently, option D is the correct answer. Hence 'for' should be used in place of 'till' to make the
113. (a) Refer to: Propensity means an inclination or natural sentence grammatically and contextually correct.
tendency to behave in a particular way which is the Option (d) is hence the correct answer.
same as Tendency and that makes them synonyms. 129. (e) The original sentence is absolutely correct and hence
Antipathy means a strong feeling of dislike while the bold part needs no replacement.
Inadequate means not appropriate. Option (e) is hence the correct answer.
Clearly, option A is the correct answer. 130. (d)
114. (b) Staggeringly: to an astonishing or shocking degree. 131. (a) Both (I) and (II)
Startling: very surprising, astonishing, or remarkable. Pondering means to think about something deeply
Predictable: able to be predicted. and carefully. Looking at the context of the sentence,
Ordinary: with no special or distinctive features; it is best to use pursuing which means to follow some
normal. measures in order to accomplish a goal or task. Here,
Unremarkable: not particularly interesting or Karthik is interested in pursuing engineering.
surprising. Infused - to fill someone or something with quality or
Unexceptional: not out of the ordinary; usual. emotion
115. (c) Preface, Prologue and Preamble all have the same Enthused - excited about something
meaning while Conclusion means the end or the finish So, the word enthused is a more appropriate word and
of an event. Thus, Conclusion is the antonym of can be used to replace infused to make the sentence
Prelude. sound more logical. Also, varied suits the context well.
So, option C is the correct answer. It is used to indicate that Karthik has different (variety
Tg:- @NextGenBankers
116. (b) Option (b): All the words fit in well.
117. (c) Option (c): All the words fit in well.
Hence, option C is correct. 132. (a)
of) interests. Use of dissimilar brings out an
unintended connotation.
118. (b) Option (b): All the words fit in well. 133. (e) None of these
119. (e) The statement is correct in its original form and no We have to read the sentence carefully to understand
word needs to be replaced. the context being expressed. We find that the sentence
Hence, option E is correct. tells us that the tax collections must have been affected
120. (c) The original statement is incorrect as the none of the unfavourably because of the unimpressive economic
words make sense and render the statement activity. However, the April GST numbers are
meaningless. surprisingly positive. This makes perfect sense. Now,
Option (a): Complacency is opposite of what is needed looking at the highlighted words, there is no need to
here. For is also incorrect. replace any of them.
Option (b): Newly is incorrect here. 134. (d) Only 2 and 3
Option (d): With does not make sense here. The sentence talks about the decision taken by the
Option (c): All the words are a perfect fit in the explorers to set up camp at a place which was near
statement. running water and good shade. First blank - we need
Correct: The turbulence in emerging markets is hardly a verb to link the subject EXPLORERS with the object
surprising as the tightening of monetary policy by pronoun IT. DECIDED and CONCLUDED will fit here.
the U.S. Federal Reserve has traditionally caused the Second blank - we need a noun to be modified by the
turning of the global credit cycle. adjective GOOD. PLACE, LOCALE and LOCATION
Hence, option C is correct. will all fit here.
121. (d) 122. (e) 123. (c) 124. (e) 125. (c) 135. (b) Only 1 and 2
The Reserve Bank of India did not extend the moratorium The sentence seeks to convey that the Indian
in its monetary policy review on Thursday. It allowed banks government has enacted a strategy to attract foreign
to restructure the loans of borrowers who are in financial investment. For the first blank, we need a verb in the
difficulty and are unable to repay them.Once restructured, past participle as the present perfect tense construction
such loans would be considered as standard. This means HAS + VERB always takes the past participle form of
that the lenders won't report the borrower as a defaulter to the verb. LAUNCHED and FORMULATED fit
credit bureaus if the borrower follows the new payment contextually. For the second blank, we need a verb to
structure.In his speech, RBI governor said, "The be modified by the adverb SPECIFICALLY.
SBI Clerk (Junior Associates) Mains Solved Paper-2021 2021-27
FOCUSED and CENTRED fit contextually. (3) is of depositors' interest and for any other related
incorrect. ESTABLISHED does not fit contextually. purposes deemed necessary by the RBI
TARGETED does not fit grammatically. 153. (b) Fast-moving consumer goods, also known as
136. (c) 137. (d) consumer packaged goods, are products that are sold
138. (c) From the given words, spelling of 'stewardsship' is quickly and at a relatively low cost. Zomato is an Indian
incorrect instead it should be 'stewardship' which multinational restaurant aggregator and food delivery
means higher rank officer. Hence, the correct answer company founded by Deepinder Goyal and Pankaj
choice would be option (c) Chaddah in 2008.
139. (e) From the given words, spelling of 'saboteours sand 154. (a) 155. (b)
infilterate' is incorrect, instead it should be 'saboteurs' 156. (a) Hemis National Park located in Ladakh is the largest
which means a person who intentionally causes the National Park in India. Hemis National Park is India's
destruction of property, and infiltrate means to secretly protected area inside the Palearctic realm, outside the
penetrate. Hence, the correct answer choice would be Changthang Wildlife Sanctuary northeast of Hemis,
option (e) and the proposed Tso Lhamo Cold Deser t
140. (b) From the given words, spelling of 'comandos' is Conservation Area in North Sikkim.
incorrect instead it should be 'commandos' which 157. (a) Kailash Satyarthi is an Indian 'Children's rights' activist.
means specially trained fighting force. Hence, the • He is a Nobel Peace Prize recipient (2014).
correct answer choice would be option (b) • He is the founder of Bachpan Bachao Andolan, Global
141. (c) 142. (a) 143. (d) March Against Child Labour, Global Campaign for
144. (c) Shiv Nadar steps down as HCL Tech MD, Vijayakumar Education, Kailash Satyarthi Children's Foundation,
to take over. Shiv Nadar, the founder of HCL and Rugmark now known as Good Weave
Technologies Ltd, and its chief strategy officer, has International.
tendered his resignation as managing director, as well • He has been a member of a UNESCO body established
as a director. with the goal of providing "Education for All".
145. (d) Ministry of Food Processing Industries (MoFPI) has 158. (c)
recently extended the Operation Greens Scheme from 159. (c) Launching the 'Scheme of Penalty for non-
Tomato, Onion and Potato (TOP) to all fruits & replenishment of ATMs', the central bank said that a
146. (b)
Tg:- @NextGenBankers
vegetables (TOTAL) for a period of six months on
pilot basis as part of Aatmanirbhar Bharat Abhiyan.
The Confederation of Real Estate Developers'
penalty of Rs. 10,000 per ATM will be levied in the
event of a cash-out situation for more than 10 hours
in a month.
Association of India, is the apex body for private real 160. (d) Odisha govt will continue to sponsor Indian men's
estate developers in India, representing over 20,000 and women's national hockey teams for another 10
developers through 21 states and 220 city chapters years, after the current sponsorship ends in 2023.
across the country. 161. (a) World Red Cross Red and Crescent Day is celebrated
147. (b) Ola Electric and Bank of Baroda have signed the on 8 May each year.
largest long-term debt financing agreement in Indian • The theme of World Red Cross Day 2021 is 'Together
EV industry. we are unstoppable'.
148. (d) • It is celebrated to commemorate the birth anniversary
149. (b) The government's 7.93 per cent stake in the Kolkata- of the founder of the Red Cross and International
based conglomerate is worth Rs 20,250 crore as it owns Committee of the Red Cross (ICRC) Henry Dunant.
the stake in ITC through the Special Undertaking of 162. (c) To make the Pradhan Mantri Jan Dhan Yojana (PMJDY)
the Unit Trust of India (SUUTI). scheme more attractive, the finance ministry decided
150. (c) Gujarat, Tamil Nadu, Andhra Pradesh and Maharashtra to double the overdraft facility from Rs 5,000 to Rs
are among the top 10 States that have received a 10,000.
maximum Rural Infrastructure Development Fund 163. (b) The Norwegian Nobel Committee has decided to award
(RIDF) in the last three years. the Nobel Peace Prize for 2020 to the World Food
151. (a) The world's highest motorable road was inaugurated Programme (WFP) for its efforts to combat hunger.
in eastern Ladakh. • It has been awarded to WEP for its contribution to
• It is situated at the height of 19,300 feet at Umling La bettering conditions for peace in conflict-affected areas
Pass in eastern Ladakh. and for acting as a driving force in efforts to prevent
• The BRO constructed a 52-km long tarmac road the use of hunger as a weapon of war and conflict.
through this high mountain pass. 164. (c) ICICI Bank has launched a digital and contactless
• The road at Umlingla Pass now connects the important banking platform for merchants.
towns in the Chumar sector of Eastern Ladakh. • The service, christened 'merchant stack', is targeted at
152. (a) The Depositor Education and Awareness Fund the over 2 crore retail merchants in the country to
Scheme (DEAF Scheme) was established by the render banking services digitally.
Reserve Bank of India (RBI) in 2014 for the promotion
2021-28 SBI Clerk (Junior Associates) Mains Solved Paper-2021
165. (d) 178. (a) The Indian Navy recently had maiden sea trials of its
166. (c) Chipmaker Intel has collaborated with the Central second indigenous stealth destroyer of the P15B class
Board of Secondary Education (CBSE) to launch the named Mormugao, which is built by the Mazagon Dock
'AI For All' initiative. About 'AI For All' initiative: Based Shipbuilders Ltd (MDSL). The ship has been named
on Intel's 'AI For Citizens' programme, 'AI For All' is a after the port town in Goa. It has been proposed to
4-hour, self-paced learning programme. commission the stealth destroyer in mid-2022 and
167. (b) Two Indian organisations bag the prestigious UNDP would add significantly to the Indian Navy's combat
Equator Prize 2021, for their exceptional achievement capabilities.
in showcasing local, innovative, nature based 179. (b) The state of the world's land and water resources for
solutions for tackling biodiversity loss and climate food and agriculture (SOLAW 2021) report was
change. The two organizations from India out of the released by the Food and Agriculture Organization of
ten winners globally are- Aadhimalai Pazhangudiyinar the United Nations (FAO). From 8 to 9 December 2021,
Producer Company Limited and Snehakunja Trust. the Land and Water Days are hosted by the Food and
168. (c) Agriculture Organization of the United Nations (FAO).
169. (e) Indian Farmers Fertiliser Cooperative Limited (IFFCO) The central theme of SOLAW 2021 is "Systems at
has introduced the world's first Nano Urea Liquid for breaking point".
farmers during its 50th annual general body virtual 180. (b)
meeting.The production of Nano Urea Liquid will 181. (b) India's leading public sector bank "bob World Wave"
commence by June 2021. has launched a solution for digital banking payments,
170. (b) named bob World Wave.The bob World Wave is a
171. (a) Two Indian firms, Reliance Industries' technology arm wearable device, which will allow customers to monitor
Jio Platforms and e-learning startup Byju's have their Sp02, body temperature, heart rate and blood
figured in the Time Magazine's first-ever list of 100 pressure. BoB has partnered with NPCI to offer the
most influential companies. wearable payment solution.
172. (e) 182. (b) 183. (c) 184. (c) 185. (b) 186. (b)
• India is ranked at 10th place in the 'Climate Change 187. (c)
Performance Index (CCPI) 2021', released on 7th 188. (a) Union Ministry of Statistics and Programme
December 2020.
Tg:- @NextGenBankers
• The overall score of India is 63.98. It is for the second
time in a row that India emerged among the top 10
Implementation (MoSPI) releases the Index of
Industrial Production (IIP). As per the recent edition
of the data, the IIP measure grew 1 per cent for
countries with higher climate performance from among December 2020. The Consumer Price Index inflation
58 economies. (CPI) decreased to 4.06 per cent in January 2021, due
• India was at 9th spot last year in the index. to the decline in vegetable prices.
173. (d) 189. (b) India has more than 63 million MSMEs. They account
174. (b) National Electronic Toll Collection is a program which for nearly 40 per cent of India's exports, about 6.11 per
uses FASTag device that employs Radio Frequency cent of the country's manufacturing GDP. MSMEs
Identification (RFID) technology, for making toll contribute to 24.63 per cent of the GDP from services
payments directly from the prepaid account linked to sector.
it. 190. (b) Uzbekistan's President Shavkat Mirziyoyev has been
175. (b) 176. (a) re-elected for a second five-year term, according to
177. (b) Indravati Tiger Reserve is located in the state of the preliminary results of a poll. Mirziyoyev won the
Chhattisgarh with a total area of approximately 2799.08 elections with 80.1 percent of the vote as per
square km. Recently, tiger counting was carried out in Uzbekistan's Central Election Commission. The 64-
the reserve covering more than 400 square kilometers, year-old leader took office in 2016 following the death
which is the first time covering such a large area of the of long-time President Islam Karimov. Mirziyoyev is
reserve. As per the latest data, the reserve had 3 tigers. known for relaxing many policies of his predecessor.
SBI Clerk (Junior Associates) Prelims Solved Paper-2021
(Based on Memory)
2. How many persons were seated between J and the one who
(e) More than four
DIRECTIONS (Qs. 11-15) : Answer the following questions
faces X? referring to the symbol-letter-number sequence given below:
(a) Three (b) More than three S @ N 1 Q 3YW8 N O 5 C Z 2 $ P*AB 9 L6 M 4 # F X 7 I
(c) None (d) One 11. How many symbols are there in the above sequence which
(e) Two are immediately preceded by a number and immediately
3. The number of persons seated between Z and E was the followed by a consonant?
same as the number of persons seated between ______ (a) None (b) Two (c) One (d) Three
(a) P and T (b) J and I (e) None of these
(c) I and T (d) T and U 12. What should come in place of the question mark (?) in the
(e) U and S following sequence?
13@ 8OY ? A9P
4. Who is sitting between E and Z in Row-1?
(a) Z2C (b) C2N (c) Z$C (d) Z$5
(a) X and Y (b) D and X
(e) None of these
(c) F and X (d) T and U
13. Which of the following is exactly middle between the
(e) I and T seventh element from the right and sixth element from the
5. Who among the following sits 3rd to the left of J in Row-2? left end?
(a) U (b) P (c) S (d) T (a) Z (b) 2 (c) $ (d) P
(e) I (e) None of these
DIRECTIONS (Qs. 6-9) : Study the following information 14. Which of the following is the fifth element to the left of the
carefully and answer the questions given below. ninth element from the right end in the above sequence?
(a) 2 (b) P (c) $ (d) *
Eight friends K, J, I, H, T, U, V and W are seated in a straight line,
(e) None of these
facing north, but not necessarily in the same order. There are four
15. If first and second halves of the above sequence are written
persons in between I and W. J and U is immediate neighbour of I,
in reverse order, then which will be the sixth element to
who is second to the right of T. U is not an immediate neighbour the right of the fourteenth element from the right end?
of T. H is not an immediate negihbour of W. V sits second to the (a) L (b) # (c) 6 (d) 9
left of K. K does not sit on the immediate left of H.
(e) None of these
2021-30 SBI Clerk (Junior Associates) Prelims Solved Paper-2021
DIRECTIONS (Qs. 16-17) : Study the information carefully 23. The number of boxes placed above X is same as the number
and answer the questions given below. of boxes placed below the box ______?
(a) Box Y (b) Box Z
Gopal starts walking from point Z towards north east directions (c) Box U (d) Box X
after walking some distance reached point Y and starts moving
(e) None of these
to east direction and walks 6 m. and reached point X from there
24. Four of the following five are alike in a certain way and
he takes a left turn and walks 3 m to reach point W, from there
so form a group. Find the one who does not belong to that
take a right turn and walk 5 m to reach point V.
group?
16. What is the shortest distance between point V and X?
(a) Y (b) T (c) W (d) V
(a) 3 34 (b) 25 (c) 34 (d) 2 8 (e) X
(e) None of these 25. Which of the following box is placed exactly between Y
17. In which direction is Z with respect to X? and S?
(a) North east (b) South West (a) Z (b) T (c) W (d) X
(c) East (d) West (e) None of these
(e) South
DIRECTIONS (Qs. 26-29) : In each question below are given
DIRECTIONS (Qs. 18-20) : In these questions, relationship some statements followed by some conclusions. You have to
between different elements is show in the statements. The take the given statements to be true even if they seem to be at
statements are followed by conclusions. Study the conclusions variance with commonly known facts. Read all the conclusions
based on the given statements and select the appropriate and then decide which of the given conclusions logically
answer: follows from the given statements, disregarding commonly
(a) If only conclusion I follows. known facts.
(b) If only conclusion II follows. (a) If only conclusion I follows.
(c) If either conclusion I or II follows. (b) If only conclusion II follows.
(d) If neither conclusion I nor II follows. (c) If either conclusion I or II follows.
(e) If both conclusions I and II follow. (d) If neither conclusion I nor II follows.
18. Statements : (e) If both conclusion I and II follow.
Conclusions :
Tg:- @NextGenBankers
D > Q ≥ P < M ≤ A; M > R = X 26. Statements :
Only a few students are Educator
I. A > X II. Q > M All Educator are Adults.
19. Statements : No Adults are Children.
E≤B≤M=T≥F≥G=S Conclusions :
Conclusions : I. Some Students are Children
I. M > G II. M = S II. No Children are Students
20. Statements : 27. Statements :
L≤X≥F>K≥D>N Only a few Classes are Schools.
Conclusions : All Schools are Colleges.
I. L < F II. X > D All Colleges are Universities.
DIRECTIONS (Qs. 21-25) : Study the following information Conclusions :
carefully and answer the questions given below. I. Some Schools are Universities
II. Some Schools can be Colleges
Eight boxes i.e. Z, Y, X, W, V, U, T and S are placed one above
the another but not necessarily in the same order. Three boxes 28. Statements :
are plaed between T and V. No box is placed between W and Some Registers are not Book
T. The number of boxes placed above the box W is same as Only a few Book are Copies.
the number of boxes placed below the box S. Box S is exactly Every Copies are Notebooks.
between the box T and V. Box U which is at top most position Conclusions :
is placed just above the box Y. Box Y is placed above the box I. Some Registers can be Notebooks
W. Box Z is placed above the box X, which is not placed at the II. Some Book are not Copies
bottom most position. 29. Statements :
21. How many boxes are placed below the box T? Only a few Phones are Laptops.
(a) Two (b) Four (c) Three (d) One Only Laptops are Computers.
(e) None of these Few Laptops are Smartphones.
22. How many boxes are placed between Y and T? Conclusions :
(a) None (b) Two (c) Three (d) Five I. Some Phones are not Computers
(e) One II. Some Laptops are not Smartphones
SBI Clerk (Junior Associates) Prelims Solved Paper-2021 2021-31
30. The position of how many digits in the number 38. 22, ___, 41, 56, 76, 102
“2451479638” will remain same when the first half and the (a) 23 (b) 30 (c) 31 (d) 26
second half of the digits are arranged in ascending order (e) 48
separately? 39. 1, 2, 6, 21, ___, 445
(a) One (b) Two (c) Three (d) Four (a) 44 (b) 94 (c) 66 (d) 88
(e) None (e) 90
DIRECTIONS (Qs. 31-35) : Study the following information 40. 99, 105, 112, ___, 137, 159
carefully and answer the questions given below – (a) 94 (b) 97 (c) 114 (d) 113
Z, Y, X, W, V, U, T and S are eight friends sitting in a circle facing (e) 122
the centre. U is on the immediate right of W while Y is on the DIRECTIONS (Qs. 41-50) : Simplify the following Questions.
immediate left of V. Z is neither sitting opposite to X nor S. And
41. x % of 270 = 564.3 – 540
Z is not the neighbour of S. V and S are sitting adjacent to X.
(a) 7 (b) 11 (c) 9 (d) 8
31. Which of the following is wrong?
(e) 6
I. U is to the immediate left of T.
4 1 1 1 1
II. Z is to the immediate right of W. 42. = + + +
x 400 300 200 100
III. S and W are sitting opposite each other.
(a) Only I (b) Only II (a) 192 (b) 292 (c) 392 (d) 194
(c) Only III (d) Only II and III (e) 198
(e) All I, II and III 43. ?2 = 729 ÷ 72 ÷ 81 × 512
(a) 32 (b) 512 (c) 256 (d) 128
32. Which of the following are two pairs of adjacent members?
(a) Z U and Y X (b) U S and Y T (e) 64
(c) Z T and X V (d) Y T and Z T 5 7 13 8 17
44. + + =?+ +
(e) None of these 2 2 2 5 10
33. What is the position of T? (a) 11.5 (b) 10.2 (c) 9.2 (d) 4.6
(a) Fourth to the left of U. (e) 2.3
45. 4x– 2 = 45 ÷ 163 × 642 ÷ 256
Tg:- @NextGenBankers
(b) Third to the left of Y.
(c) Third to the right of Y. (a) 1
(d) –2
(b) 3 (c) 2 (d) 0
(d) Second to the left of W.
(e) None of these 46. ? × 130 ÷ 144 = 390 × 704 ÷ 384
34. Which of the following is / are correct? (a) 738 (b) 594 (c) 1188 (d) 792
I. W is third to the left of V. (e) 752
1 1
II. W is fifth to the right of V.
47. 400 × (8000) 3 =
? × (4096) 4
III. Z and U are adjacent to each other.
(a) Only I (b) Only II (a) 15 (b) 24 (c) 50 (d) 20
(c) Only III (d) Only I and II (d) 30
(e) All I, II and III 1
35. If Z and V interchange places and so do U and S, then 48. 70% of 90 + 92 = (32) 5 + ?
(a) T and X are adjacent to U.
(a) 151 (b) 149 (c) 143 (d) 144
(b) W and Y are adjacent to Z
(e) 142
(c) V is not sitting opposite Z.
418
(d) S is not sitting opposite U. 49. × 324 − 14 × 20 =
?
(e) S is fourth to the left of X. 352
(a) 116 (b) 58 (c) 29 (d) 87
Numerical Ability (e) 145
50. ? × 2 – 202 = 142 – 18 × 22
DIRECTIONS (Qs. 36-40) : What will come in place of (a) 150 (b) 100 (c) 180 (d) 120
question mark (?) in the following series?
(d) 195
36. 10, 30, 68, ___, 222, 350
DIRECTIONS (Qs. 51-55) : Bar-chart given below shows
(a) 25 (b) 125 (c) 130 (d) 520 students registered for three different exams in five different
(e) 512 years. Study the data carefully and answer the following
37. 10, 12, 22, 34, ___, 90 questions.
(a) 56 (b) 52 (c) 50 (d) 46
(e) 32
2021-32 SBI Clerk (Junior Associates) Prelims Solved Paper-2021
35
(a) 1540000 (b) 2530000
30
(c) 2120000 (d) 1980000
25
(e) 1760000
20
58. A can do a work in 20 days. B is 25% more efficient than
15 A. Both worked together for 4 days. C alone completed
10 the remaining work in 4 days. A, B and C together will
5 complete the same work in?
0 (a) 5 days (b) 4 days
2015 2016 2017 2018 2019
1 1
MTS CGL CHSL (c) 5 days (d) 4 days
2 2
51. Total number of students registered for CGL in all the five (e) 3 days
years is what percent more than total number of students 59. Naveen and Naman invested `6000 and `8000 for
registered for CHSL in all the five years together?
12 months and 6 months respectively. Find profit share
1 1 of Naman is how much percent more or less than profit
(a) 50% (b) 37 % (c) 25% (d) 33 %
2 3 share of Naveen?
2
(e) 66 % (a) 10% more (b) 11.11% less
3
52. Find the ratio of total students registered for all the three (c) 11.11% more (d) 33.33% more
exams in 2015 and 2016 together to total students registered (e) 33.33% less
for all the three exams in 2017 and 2018 together? 60. P, Q and R invested in a ratio of 7 : 8 : 5 in a business. They
(a) 2 : 3
(c) 9 : 17
Tg:- @NextGenBankers
(b) 10 : 17
(d) 3 : 2
got an annual profit of ` 244900. If P and R withdrew their
amount at the end of 3 months and 7 months respectively.
(e) 5 : 8 Then find the difference between P and R share of profit?
53. Average number of students registered for MTS exam in (a) 58900 (b) 57000
all the five years together in how much less/more than (c) 38000 (d) 76000
average number of students registered for CHSL exam in (e) 85500
all the five years together?
DIRECTIONS (Qs. 61-65): The data given below shows the
(a) 8 (b) 10 (c) 4 (d) 3
information of total students in class P, Q and R which divided
(e) 2
in three group A, B and C.
54. Out of total students registered for all the exams in 2019,
80% appeared for exam. Students appeared in Clerk, Class P: Total no. of students are 180. 60 students are in group
CGL and CHSL are in the ratio 3 : 3 : 1. Find how much A. Ratio of number of students in group B and C is 1 : 2.
percent of students appeared for exam MTS out of students Class Q: Total no. of students are 160. The ratio of no. of
registered for MTS exam in that year? students in group A, B and C is 3 : 5 : 8.
(a) 60% (b) 70% (c) 80% (d) 90% Class R: The no. of students in group B are half of that in group
(e) 75% A. The no. of students in group A of class R are 20 more that
55. In 2016, out of total students registered for all the three in group A of class P no. of students in group C of class R are
exams, 80% appeared out of which only 25% qualified the 20% less than that in group A of same class.
exams. Find the number of students appeared in exam but 61. No. of students in group A of class Q are what percentage
disqualified. (in Lakh) of no. of students in groups B of class R.
(a) 63 (b) 54 (c) 45 (d) 36
(a) 90% (d) 100% (c) 85% (d) 75%
(e) 72
(e) 120%
56. Simple interest on a sum of `12000 at the end of 5 years
62. Find the respective ratio between average students in group
is `6000. What would be the compound interest on the
same sum at the same rate for 3 years when compounded A in class P and Q to total students in class R.
annually? (a) 7 : 32 (b) 7 : 12
(a) 2520 (b) 3972 (c) 3600 (d) 3970 (c) 7 : 23 (d) 35 : 184
(e) 4200 (e) 45 : 184
SBI Clerk (Junior Associates) Prelims Solved Paper-2021 2021-33
63. There is total 34 girls in class P. If 12 girls are in group 72. The program helps people (A)/recent released from prison
A, then find the no. of boys in group B and C together in (B)/figure out how they can play(C)/ a constructive role in
class A. society. (D)/ No Error (E)
(a) 108 (b) 50 (c) 60 (d) 98 (a) A (b) B (c) C (d) D
(e) 118 (e) No Error
64. What are the total no. of students in group B of all the 73. I will be leaving on vacation in (A)/two days and will not
classes? be (B)/able to do much else accept put down (C)/ideas in
(a) 85 (b) 80 (c) 70 (d) 130
my traveling notebook. (D)/ No Error (E)
(d) 100
(a) D (b) C (c) B (d) A
65. Total no. of students in group A is how much more or less
(e) No Error
than that in group C.
(a) 44 (b) 54 (c) 64 (d) 84 74. The issue of making the Internet (A)/safety for kids has
(e) 74 become(B)/ a bigger one as the Web (C)/ becomes ever
66. Area of square is how much percent it’s perimeter of side more ubiquitous. (D)/ No Error (E)
8 m. (a) A (b) D (c) C (d) B
(a) 100 (b) 200 (c) 150 (d) 50 (e) No Error
(e) 20 75. Chile happen to be one of the most (A)/highly vaccinated
67. A shopkeeper sold an article K at 25% gain and another countries in the (B)/world, with more than 90% of its (C)/
article P at 20% loss. Find her overall gain or loss percent population fully vaccinated as of January. (D)/ No Error
if S.P. of both articles were same? (E)
(a) 3.5% gain (b) 3.5% loss (a) D (b) C (c) B (d) A
(b) 4.5% gain (d) 2.44% loss (e) No Error
(e) 2.44% gain
68. Find the probability of selecting two black honor cards DIRECTIONS (Qs. 76-80): In each of the questions given
form a pack of 52 cards? below a sentence is given with three words in bold. Choose the
14 61 132 28 option which gives the correct sequence of these words to make
(a) (b) (c) (d) the sentence grammatically and contextually correct.
663 663 663
(e) None of these
Tg:- @NextGenBankers
663
76. The changes relating (A) in the Central GST Act sections
proposed (B) to input tax credits aim to restrict such credits
69. Aman is 20% more efficient than Ankit while Mohan is unless suppliers have remitted (C) their share of taxes.
40% less efficient than Aman. If Ankit alone can complete
(a) ACB (b) BAC (c) CAB (d) BCA
the work in 24 days then in how many days Aman and
Mohan together can complete the work. (e) No rearrangement required
(a) 10 days (b) 12.5 days 77. With coal stocks running (A) ‘critically low’, the
(c) 15 days (d) 20 days aluminium industry has precarious (B) urgent government
(e) 24 days intervention to address the sought (C) situation.
70. A boat travels 32 km downstream and 20 km upstream in (a) ACB (b) BAC (c) CAB (d) BCA
9 hrs. The same boat travels 64 km, downstream and 24 (e) No rearrangement required
km upstream in 14 hrs. Find the ratio between speed of 78. Ms. Sitharaman said to proliferation (A) affordable
boat in still water and speed of current? broadband and mobile service remote (B) in rural and
(a) 3 : 5 (b) 5 : 3 enable (C) areas.
(c) 2 : 5 (d) 5 : 2 (a) ACB (b) BAC (c) CAB (d) CBA
(e) 3 : 1 (e) No rearrangement required
English Language
79. The government seems to have prioritised (A) meeting
its fiscal deficit targets rather than using this inclusive (B)
DIRECTIONS (Qs. 71-75): Read each sentence to find out to signal a path of employment-centred and opportunity
whether there is any grammatical or idiomatic error in it. The (C) growth.
error, if any, will be in one part of the sentence. The number (a) ACB (b) BAC (c) CAB (d) BCA
of that part is the answer. If there is no error the answer is (e). (e) No rearrangement required
(Ignore errors of punctuation, if any.) 80. With the country yet again unemployment (A) with
71. Despite of light rainfall in the (A)/national capital, the air another wave of the COVID-19 pandemic, there are
(B)/quality remained in the ‘very poor’ category with (C)/an concerns about grappling (B) growth and increasing
overall Air Quality Index (AQI) at 318. (D)/ No Error (E) faltering (C).
(a) C (b) B (c) A (d) D (a) ACB (b) BAC (c) CAB (d) BCA
(e) No Error (e) No rearrangement required
2021-34 SBI Clerk (Junior Associates) Prelims Solved Paper-2021
DIRECTIONS (Qs. 81-88): Given below is a paragraph that DIRECTIONS (Qs. 89-93): In the following question,
has blank spaces. Corresponding to each blank, five options sentences are given with a part in bold. The given phrase in the
are given, out of which only one is appropriate. Choose the bold may or may not contain an error. The options following
option that fits most suitably in the given blank making sentence can replace the incorrect phrase. The correct phrase that is to
grammatically and contextually correct. be replaced will be your answer. If the sentences are correct
The Budget proposals seek to ___________ (81) the economy then select ‘No improvement required’ as your answer.
by stepping up public investments, which will create demand 89. The petition contended that the establishments and factories
for industrial inputs like cement, steel and capital goods, and were operates in violation of environmental norms and
generate jobs, Economic Affairs Secretary Ajay Seth said. affected public health.
Finance Minister Nirmala Sithaharaman in her Budget 2022-23 (a) has been operating in violation of
___________ (82) public investment by as much as 35.4 per (b) were operating in violation of
cent to Rs 7.5 lakh crore or 2.9 per cent of the GDP. Observing (c) were operates in violation on
that direct support measures have only limited multiplier effect,
(d) was operating in violation of
Seth said steps which can have long to medium term impact
(e) No improvement required
are needed to ___________ (83) the economy in a sustained
manner. “When we are looking at economic management, it is 90. The Delhi Commission for Women (DCW) was issued
not a one-year affair. One has to look at short, medium or long a notice to the police regarding the same and sought a
term. In the short term, what was needed has been ___________ detailed action taken report.
(84). “When we come to the medium and long term, we find (a) had issued a notice to the police
that a direct income support that simulates consumption demand (b) have issue a notice to the police
has a very limited ___________ (85) effect, whereas capital (c) has been issuing a notice to the police
investment has a much larger, very strong multiplier effect, (d) has issue a notice to the police
which lasts more than one year. (e) No improvement required
“How it helps? It generates demand for the inputs which get 91. Ravi was sent to Taloja Jail, where some of his co-accused
into the ___________ (86) — cement, steel, capital goods, have now spend more than three years without bail or
construction machinery and so on,” Seth told PTI in an interview. trial.
In view of the ___________ (87) caused by the COVID-19 (a) have now spent more than three year
pandemic, the government provided direct support to people (b) have now spend more then three years
Tg:- @NextGenBankers
through various schemes like Jan Dhan, PM KISAN and Pradhan
Mantri Garib Kalyan Yojana, he said, adding “in year two the
(c) have now spent more then three years
(d) have now spent more than three years
need moderated and year three we expect that the need may not
be there.” By increasing public investment, the government has (e) No improvement required
provided the signal that it is ready to ___________ (88) funds 92. With technology becoming smarter, there has been a
into growth-oriented activities. corresponding increase in cyber fraud, harassments and
81. (a) Obliterate (b) Quash threats.
(c) Annihilate (d) Apprehend (a) On technology becoming smarter
(e) Stimulate (b) At technology becoming smarter
82. (a) Accepted (b) Hiked (c) On technology became smarter
(c) Egressed (d) Confronted (d) With technology has become smarter
(e) Exacerbated (e) No improvement required
83. (a) Dawdle (b) Pillage 93. A good coach had the skills of his entire team accounted
(c) Boost (d) Supervene for, and knows where to put them for the best results.
(e) Flout (a) have the skills (b) have a skill
84. (a) Provided (b) Deployed (c) has the skills (d) had skills
(c) Waned (d) Fainted (e) No improvement required
(e) Resolved DIRECTIONS (Qs. 94-100): Read the following passage
85. (a) Myriad (b) Plausible and answer the given questions below it with the help of the
(c) Multiplier (d) Multiplicity information provided in it.
(e) Exclusive
Screaming is a unique noise. Research suggests when we scream
86. (a) Investment (b) Misadventure
in fear, the noise serves the dual purpose of sharpening our own
(c) Ubiquitous (d) Delinquent focus in the face of a threat as well as of warning others. In fact,
(e) Extraneous our brains process screams in a unique way. Most noises we hear
87. (a) Manoeuvre (b) Resolve are delivered from our ear to an area of the brain devoted to
(c) Hardship (d) Reticent analyzing a sound and breaking it into its component parts, such
(e) Strife as gender, age, and tone; high or low; a brass, stringed or electric
88. (a) Discord (b) Oblivious instrument — or several combined; a ribbit or a coo, and what
(c) Impropriety (d) Amity animal makes that sound. This indicates that we differentiate
(e) Pump all the noises we hear every day, even subconsciously.
SBI Clerk (Junior Associates) Prelims Solved Paper-2021 2021-35
A scream is different though, according to David Poepple, PhD, 96. Among the following statements, choose option(s) which
a professor of psychology and neural science at New York are FALSE in accordance with the information given in
University. Poepple is the lead author of a 2015 study that looked the passage.
into what happens in the body when people scream in fear. A (I) The part of the brain that understands the fear of the
scream goes straight from the ear to the amygdala, the part of the situation and sends us into a full blown fight/flight
brain that processes fear and kickstarts the body’s fight-or-flight response is called amygdala.
response. The sound jolts our brains into increased alertness and
(II) The number of muscles used in the process of speaking
analysis. This is especially true for screams when we’re scared.
are over 100.
These have a unique sound signature encompassed by the term
“roughness.” This roughness is what serves to alert others to (III) The purpose of a scream is to vent out your anger and
danger. Humans and other animals scream for many reasons release the emotions a person has been withholding
— in joy, in surprise, in fear. But we’ve evolved to be able to inside.
produce different shrieks and to be able to discern the difference (a) Both I and II (b) Only III
when we hear them. (Incidentally, roughness is also why we (c) Only II (d) Only I
find the sound of a baby crying and fingernails on chalkboards (e) Both II and III
disturbing; both sounds have high roughness.) If we focus on 97. Choose the option which states the reason for why some
the sounds that gain our attention, we will be able to tell which people go mute when they are scared.
one is rough and which is not.
(a) The brain finds it hard to concentrate between how to
But what about people who have the opposite reaction when
combat the dangerous situation and figuring out how
they’re scared? What about the people who don’t scream in fear,
to talk which leaves a person mute.
but become speechless in the face of a threat? This has been less
studied, but it’s likely a part of the third, overlooked response (b) Some people find that they don’t want to express their
to terror: paralysis. Contrary to pop medicine, fight and flight feelings of terror and fright which is why they choose
aren’t the only two reactions to something scary. Humans, like to remain quiet in those moments.
other animals, also evolved the option to freeze, or “play dead” (c) A certain part of the brain responsible for overlooking
in the face of a threat. This happens involuntarily, when the muscle activity takes over other responses of the brain
cerebellum, the part of the brain that regulates muscle activity, and forces every movement of the body to shut down,
overpowers other terrified brain responses and basically shuts including speech.
Tg:- @NextGenBankers
down all movement. Since screaming takes quitfiocue a few
muscles (probably a little less than the 100 used for speaking),
a person frozen in fear is rendered mute as well as immobile.
(d) Being quiet has been found to help do away with the
dangerous and harmful factors that a person might be
facing at that exact moment.
94. Choose the statement(s) which is/are TRUE based on the
(e) None of the above
information given in the passage.
(I) Our systems are built in a way that only allows us to 98. Among all the three responses to fear, how does a scream
scream in dangerous or threatful situations. help us in possible dangerous situations?
(II) One of the ways humans tackle fear is by going into a (a) The brain processes the scream in a part which helps
freeze mode and acting dead which is a similar action activate our fight or flight mode which then helps us
to animals. become aware and scan the situation.
(III) The brain sorts the noises we hear regularly on the (b) Scream can notify the people in your surroundings like
basis of factors such as gender or high or low tone, your neighbours or anybody in close vicinity to you.
age, etc. (c) When you scream, your vocal cords stretch and
(a) Both II and III (b) Both II and I become flexible enough to scream louder and warn
(c) Only III (d) Both I and III others of the situation.
(e) Only I (d) None of the above
95. According to the article, what contributes to alerting (e) All of the above
someone of the danger a person is in? 99. Select the word which is similar in meaning to the
(a) The gestures and expressions we make when we are highlighted word INDICATES.
confronted with a seemingly dangerous situation. (a) announces (b) signals
(b) The scream one lets out when faced with danger holds
(c) bargains (d) divides
an element of roughness which is crucial for letting
others know of the possible danger. (e) flows
(c) Secretion of certain fluids and juices in our bodies by our 100. Choose the phrase which will replace the phrase ‘focus
organs play a part in accomplishing the above said thing. on’ as highlighted in the passage.
(d) The frequency of our screams, i.e., how often we (a) drain out (b) tear apart
shout in the given amount of time can alert others of (c) fire away (d) concentrate on
the presence of danger. (e) mix into
(e) None of the above
2021-36 SBI Clerk (Junior Associates) Prelims Solved Paper-2021
ANSWER KEY
1 (b) 11 (b) 21 (b) 31 (e) 41 (c) 51 (d) 61 (d) 71 (c) 81 (e) 91 (d)
2 (e) 12 (d) 22 (e) 32 (c) 42 (a) 52 (b) 62 (e) 72 (b) 82 (b) 92 (e)
3 (d) 13 (b) 23 (a) 33 (d) 43 (e) 53 (e) 63 (d) 73 (b) 83 (c) 93 (c)
4 (a) 14 (b) 24 (d) 34 (d) 44 (c) 54 (d) 64 (d) 74 (c) 84 (a) 94 (a)
5 (b) 15 (c) 25 (b) 35 (a) 45 (b) 55 (b) 65 (b) 75 (d) 85 (c) 95 (b)
6 (c) 16 (c) 26 (c) 36 (c) 46 (d) 56 (b) 66 (b) 76 (b) 86 (a) 96 (e)
7 (c) 17 (b) 27 (a) 37 (a) 47 (c) 57 (c) 67 (d) 77 (a) 87 (c) 97 (c)
8 (e) 18 (a) 28 (e) 38 (b) 48 (e) 58 (b) 68 (a) 78 (d) 88 (e) 98 (a)
9 (d) 19 (c) 29 (d) 39 (d) 49 (a) 59 (e) 69 (b) 79 (a) 89 (b) 99 (b)
10 (e) 20 (b) 30 (e) 40 (e) 50 (b) 60 (a) 70 (e) 80 (d) 90 (a) 100 (d)
T J I U V H K W
Tg:- @NextGenBankers X
V
6. (c) 7. (c) 8. (e) 9. (d)
21. (b) 22. (e) 23. (a) 24. (d) 25. (b)
Sol. (26-29) :
10. (e)
D I A G O N A L 26. (c) Adults
Students Educator Children
Sol. (11-15) :
11. (b) 2 $ P, 4 # F 12. (d) Z$5 13. (b) ‘2’
27. (a)
14. (b) ‘P’ 15. (c) ‘6’
Schools
Sol. (16-17) : Classes
16. (c) College
W 5m. V
3m. Universities
Y 6m.
X 28. (e)
Notebooks
Z Registers Book Copies
25 + 9 =34
17. (b) W 5m.
V 29. (d)
3m. Laptops Smartphones
Y 6m.
Phones Computers
X
Z 30. (e) After arranging, we get;
Sol. (18-20):
2 4 5 1 4 7 9 6 3 8
18. (a) I. A > X (True) II. Q > M (False)
1 2 4 4 5 3 6 7 8 9
19. (c) I. M > G (False) II. M = S (False)
20. (b) I. L < F (False) II. X > D (True)
SBI Clerk (Junior Associates) Prelims Solved Paper-2021 2021-37
X ⇒ 20 × 20 = x × (84) 4 ⇒ 400 = x × 8
31. (e) 32. (c) 33. (d) 34. (d) 35. (a) ⇒ x = 50
Sol. (36-40): 1
36. (c) Pattern 23 + 2, 33 + 3, 43 + 4, 53 + 5, 63 + 6, 73 + 7 48. (e) 70% of 90 + 92 = (32) 5 + x
130, 222, 350
1
37. (a) Pattern of series 70 × 90 + 81
10 + 12 = 22 ; 12 + 22 = 34 = (25 ) 5 + x ⇒ 144 = 2 + x ⇒ x = 142
100
22 + 34 = 56 ; 4 + 56 = 90
38. (b) Pattern of series 418 19 × 22
49. (a) × 324 − 280 = x ⇒ × 324 − 280 = x
352 19 × 18
⇒ 396 – 280 ⇒ x = 116
50. (b) x × 2 – 202 = 142 – 18 × 22 ⇒ 2x = 400 – 200
⇒ 2x = 200 ⇒ x = 100
Sol. (51-55) :
Double difference 51. (d) Total no. of students registered for CGL
39. (d) Pattern of series = 25 + 35 + 40 + 55+ 45 = 200
1×1+1=2 ;2×2+2=6 Total no. of students registered for CHSL
6 × 3 + 3 = 21 ; 21 × 4 + 4 = 88 = 20 + 30 + 35 + 45 + 20 = 150
88 × 5 + 5 = 445
40. (e) Pattern of series
Tg:- @NextGenBankers Required % =
200 − 150
150
× 100
=
50
150
× 100 = 33 %
1
3
Take difference :- 52. (b) Total students registered for all the three exams in
99 105 112 122 137 159 2015 and 2016 together
= 15 + 25 + 20 + 25 + 35 + 30 = 150
6 7 10 15 22 Total students registered for all the three exams in
2017 and 2018 together
1 3 5 7 = 35 + 40 + 35 + 45 + 55 + 45 = 255
Double difference 150 10
Required ratio = =
Sol. (41-45): 255 17
x × 270 27 x 24.3 53. (e) Average number of student registered for MTS
= 564.3 − 540 ; =
41. (c)
100 10 10 ⇒ x = 9 exam
15 + 25 + 35 + 45 + 40 160
= = = 32
4 3 + 4 + 6 + 12 5 5
42. (a) =
x 1200 Average number of students registered for CHSL exam
4 × 1200 20 + 30 + 35 + 45 + 20 150
4 25 = = = 30
= =⇒ x = 192 5 5
x 1200 25
Required difference = 32 – 30 = 2
729 729
43. x2
(e) = × 512 ⇒ × 512 = 64 54. (d) Total students registered for a all the three exams in
72 72 × 81 2019 = 40 + 45 + 20 = 105
81 80
Number of appeared students = × 105 = 84
5 + 7 + 13 33 25 100
44. (c) = x+ ⇒ = x + 3.3 84
2 10 2 Students appeared for MTS exam = × 3 = 36
7
⇒ 12.5 – 3.3 = x ⇒ 9.2 = x
36
45 (43 )2 Required % = × 100 =90%
45. (b) 4x–2 = 2 3 × 4 40
(4 ) 4 55. (b) Total number of students register for all exams
4x–2 = 45–4 = 41 ⇒ x – 2 = 1 ⇒ x = 3 = 25 + 35 + 30 = 90 Lakh
2021-38 SBI Clerk (Junior Associates) Prelims Solved Paper-2021
Required number of students 62. (e) Average students in group A of class P and Q
80 75 60 + 30
=90 × × = 54 Lakh = = 45
100 100 2 45
Principal × rate × time Required ratio =
56. (b) Simple Interest = 184
100 63. (d) Total girls in group B and C together = 34 – 12 = 22
12000 × r × 5 Total boys in group B and C together on class P
6000 = ⇒ r = 10% = (180 – 60) – 22 = 98
100
64. (d) Total students in group B of all the classes = 130
Compound Interest for 3yrs on same rate 65. (b) Required difference = 224 – 170 = 54
t 3
r 10 8×8
A P 1 +
= =
; A 12000 1 + 66. (b) Required percentage = × 100 = 200%
100 100 8× 4
11
3 67. (d) Let S.P. of each article be ` 100.
A = 12000 ⇒ A = 15972 Total S.P. = ` 200
100 100
\ C.P of article K = × 100 = ` 80
C. I. = A – P = 15972 – 12000 = ` 3972 125
57. (c) Population of city in 2016
100 100 100 100
C.P. of article P = × 100 = ` 125
= 2332000 × × × = 2120000 80
110 80 125
125 Total C.P. = ` 205
58. (b) B = A Overall gain or loss percent
100
B:A 205 − 200 100
= × 100 = = 2.44% loss
Efficiency = 125 : 100 205 41
5:4 8
C2 14
A can do a work in 20 days 68. (a) Required probability = =
52 663
Total work = Efficiency × time = 4 × 20 = 80 C2
(A + B) work for 4 days = (4 + 5) × 4 = 36 69. (b) Efficiency :- Aman : Ankit
Tg:- @NextGenBankers
Remaining = 80 – 36 = 44
Complete remaining work in 11 days
44
6 : 5
Efficiency :- Mohan : Aman
3 : 5
= = 11 (efficiency) Combined efficiency
4
80 80 Aman Ankit Mohan
(A + B + C) = = = 4 days.
(4 + 5 + 11) 20 30 25 18
59. (e) Ratio between profit share of Naveen to Naman Total work = 25 × 24 = 600
= 6000 × 12 : 8000 × 6 600
Aman + Mohan = = 12.5 days
Let profit of Naveen and Naman are ` 3x and 2x respectively 48
3x − 2 x 70. (e) Let speed of boat in still water = x kmph
Required percentage = × 100 = 33.33% less and speed of current = y kmph.
3x
60. (a) Ratio of their profit sharing 32 20
+ =9 . . . (i)
P : Q : R = 6 × 3 : 7 × 12 : 8 × 7 = 9 : 42 : 28 x+ y x− y
Annual profit = 244900
Difference b/w P and R share of profit 64 24
+ 14
= . . . (ii)
19 x+ y x− y
= × 244900 = ` 58900
79 Solving (i) & (ii),
Sol. (61-65) : x = 6 km/h, y = 2 km/h
Group Group Group Required ration = 3 : 1
Total 71. (c) The correct choice is option (c). In part (A), ‘despite
A B C
of’ is incorrect as ‘despite’ is never followed by
P 60 40 80 180 ‘of’. Hence, option (c) is the right answer choice.
72. (b) The error lies in part (B) of the sentence. Here
Q 30 50 80 160 ‘recently’ should be ‘recent’ as we need adjective
R 80 40 64 184 here while ‘recently’ is adverb. Hence, option (b) is
the right answer choice.
Total 170 130 224 524 73. (b) The error lies in part (C). Here ‘accept’ should be
‘except’. Accept is contextually incorrect in the given
30
61. (d) Required percentage = × 100% =
75% sentence. accept means to agree to take something
40 that somebody offers you except means not including
SBI Clerk (Junior Associates) Prelims Solved Paper-2021 2021-39
somebody/something; apart from the fact that hence, 83. (c) ‘Boost’ fits most suitably in the given blank making
option (b) is the right answer choice. sentence grammatically and contextually correct.
74. (c) The error lies in part (C). Here ‘safety’ should be Hence, option (c) is the right answer choice.
‘safe’ as we need an adjective here for the noun Dawdle: waste time; be slow.
‘place’. Hence, option (c) is the right answer choice. Pillage: rob a (place) using violence, especially in
75. (d) The error lies in part (A). Here ‘happen’ should be wartime.
replaced by ‘happens’ as to comply with the rule of Boost: help or encourage (something) to increase or
subject verb agreement, we need singular verb for improve.
the singular subject (Chile). Supervene: occur as an interruption or change to an
76. (b) The correct sequence of words is BAC. Therefore, existing situation.
the correct sentence will be Flout: openly disregard (a rule, law, or convention).
“The changes proposed (B) in the Central GST 84. (a) ‘Provided’ fits most suitably in the given blank
Act sections relating (A) to input tax credits aim to making sentence grammatically and contextually
restrict such credits unless suppliers have remitted correct. Hence, option (a) is the right answer choice.
(C) their share of taxes.” Provided: made available for use; supply.
Hence, option (b) is the right answer choice. Deployed: brought into effective action.
77. (a) The correct sequence is ACB. Therefore, the correct Waned: (of the moon) have a progressively smaller
sentence will be, “With coal stocks running (A) part of its visible surface illuminated, so that it
‘critically low’, the aluminium industry has sought appears to decrease in size.
(C) urgent government intervention to address the Fainted: lost consciousness for a short time because
precarious (B) situation.” of a temporarily insufficient supply of oxygen to the
Hence, option (a) is the right answer choice. brain.
78. (d) The correct sequence is CBA. Therefore, the Resolved: firmly determined to do something.
sentence will be, Ms. Sitharaman said to enable 85. (c) ‘Multiplier’ fits most suitably in the given blank
(C) affordable broadband and mobile service making sentence grammatically and contextually
proliferation (A) in rural and remote (B) areas. correct. Hence, option (c) is the right answer choice.
Hence, option (d) is the right answer choice. Myriad: Innumerable
79. (a) The correct sequence is ACB. Therefore, the Plausible: Credible
sentence will be, Multiplier: a person or thing that multiplies.
Tg:- @NextGenBankers
“The government seems to have prioritised (A)
meeting its fiscal deficit targets rather than using this
Multiplicity: Abundance, scores, mass
Exclusive: Complete
opportunity (C) to signal a path of employment- 86. (a) ‘Investment’ fits most suitably in the given blank
centred and inclusive (B) growth.” making sentence grammatically and contextually
Hence, option (a) is the right answer choice. correct. Hence, option (a) is the right answer choice.
80. (d) The correct sequence is BCA. Therefore, the correct Investment: the action or process of investing
sentence will be, “With the country yet again money for profit.
grappling (B) with another wave of the COVID-19 Misadventure: death caused by a person accidentally
pandemic, there are concerns about faltering (C) while performing a legal act without negligence or
growth and increasing unemployment (A).” intent to harm.
Hence, option (d) is the right answer choice. Ubiquitous: present, appearing, or found everywhere.
81. (e) ‘stimulate’ fits most suitably in the given blank Delinquent: (typically of a young person) tending to
making sentence grammatically and contextually commit crime, particularly minor crime.
correct. Hence, option (e) is the right answer choice. Extraneous: irrelevant or unrelated to the subject
Obliterate: destroy utterly; wipe out. being dealt with.
Quash: reject as invalid, especially by legal procedure. 87. (c) ‘Hardship’ fits most suitably in the given blank
Annihilate: Destroy utterly, obliterate making sentence grammatically and contextually
Apprehend: arrest someone for a crime correct. Hence, option (c) is the right answer choice.
Stimulate: raise levels of physiological or nervous Manoeuvre: a movement or series of moves
activity in (the body or any biological system). requiring skill and care.
82. (b) ‘Hiked’ fits most suitably in the given blank making Resolve: settle or find a solution to (a problem or
sentence grammatically and contextually correct. contentious matter).
Hence, option (b) is the right answer choice. Hardship: severe suffering or privation.
Accepted: generally believed or recognized to be Reticent: not revealing one’s thoughts or feelings
valid or correct. readily.
Hiked: increased (something, especially a price) sharply. Strife: angry or bitter disagreement over fundamental
Egressed: gone out of or leave (a place). issues; conflict.
Confronted: came face to face with (someone) with 88. (e) ‘Pump’ fits most suitably in the given blank making
hostile or argumentative intent. sentence grammatically and contextually correct.
Exacerbated: made (a problem, bad situation, or Hence, option (e) is the right answer choice.
negative feeling) worse. Discord: disagreement between people.
2021-40 SBI Clerk (Junior Associates) Prelims Solved Paper-2021
Oblivious: not aware of or concerned about what is 96. (e) The correct choice for the answer is option (e): Both
happening around one. II and III
Impropriety: failure to observe standards of honesty Statement II is false as the passage says: Since
or modesty; improper behaviour or character. screaming takes quite a few muscles (probably a
Amity: friendly relations. little less than the 100 used for speaking), a person
Pump: move vigorously up and down. frozen in fear is rendered mute as well as immobile.
89. (b) The given highlighted phrase is incorrect. The Statement III is also false because it can be found
sentence is in past continuous tense hence ‘operates’ in the article that: Research suggests when we
in statement is incorrect hence option (c) is also scream in fear, the noise serves the dual purpose of
incorrect. In option (a), ‘has’ is incorrect because sharpening our own focus in the face of a threat as
subject (establishments and factories) is plural here well as of warning others.
therefore option (d) is also incorrect. Hence correct The facts present in the passage help us figure out
choice is option (b). why the statements II and III are false.
90. (a) The highlighted phrase in the statement is incorrect Hence, option (e) is the correct choice for the answer.
as sentence is in Past indefinite. In option (b), ‘have’ 97. (c) The correct choice for the answer is option (c): A
is incorrect as subject (DCW) is singular. Option (c) certain part of the brain responsible for overlooking
is in Perfect continuous hence incorrect. In Option muscle activity takes over other responses of the
(d), ‘issue’ is incorrect as ‘has+v3’ is correct. brain and forces every movement of the body to
91. (d) In the highlighted phrase ‘spend’ is incorrect because shut down, including speech.
sentence is in past tense and third form of spend is The answer is found towards the end of the last
‘spent’ therefore option (b) is also incorrect. In option paragraph: This happens involuntarily, when the
(a), ‘year’ should be ‘years’. In option (b), ‘then’ cerebellum, the part of the brain that regulates
should be ‘than’ hence option (c) is also incorrect. muscle activity, overpowers other terrified brain
92. (e) The highlighted phrase in the sentence is correct responses and basically shuts down all movement.
hence no improvement required is correct choice. Since screaming takes quite a few muscles (probably
93. (c) The correct replacement of the highlighted phrase is a little less than the 100 used for speaking), a person
“has the skills” as the sentence follows the structure of frozen in fear is rendered mute as well as immobile.
present tense. Moreover, the subject of the sentence is All the other options are nowhere to be found in the
singular. Hence, option (c) is the correct choice. above passage.
94.
(a)
II and III. Tg:- @NextGenBankers
The correct choice for the answer is option (a): Both
Reasoning Ability & Computer Aptitude DIRECTIONS (6-10): In the following questions, the symbols
@, #, %, $ and * are used with the following meaning as
DIRECTIONS (Qs. 1-5): Study the following information illustrated below-
carefully and answer the questions given below: M@N – M is neither greater than nor equal to N
Thirteen friends I, J, K, L, M, N, O, P, Q, R, S, T and U are M%N – M is neither smaller than nor equal to N
playing cards. They are sitting in such a way that some are sitting M#N – M is not greater than N
around a square and some are sitting around a circle and circle M$N – M is not smaller than N
is inscribed in a square. Some of them are seated at the corner M*N – M is neither smaller than nor greater than N
of the table while some of them have seated at the centre of the Now in each of the following questions assuming the given
table. The friends who are sitting at the corner of the table facing statement to be true, find which of the two conclusions I and
outside the centre and those who are sitting at the middle facing II given below them is/are definitely true and give your answer
the centre. Those who are sitting around the circle face inside accordingly.
the centre. They like different cloth brand viz. Puma, Reebok, 6. Statements: – A@Y, Y%X, X*W, W$V
Gucci, Prada, Arrow, Flying Machine, Chanel, Allen solly and Conclusions: – (I) Y%V (II) A%V
Zara but not necessarily in the same order. Only two persons like (a) Only conclusion I follows
the same cloth who is sitting around the square. N is third to the (b) Only conclusion II follows
right of the one who likes Puma. Q sits two seats to the left of
(c) Either conclusion I or II follows
S. The person who likes Puma is sitting immediate neighbour
(d) Neither conclusion I nor II follow
of the person sits in the front of the one who likes Puma, same
Tg:- @NextGenBankers
for Prada, Gucci and Reebok. R who likes Flying Machine sits
second to the right of S. Q is sitting around the circle. Only two 7.
(e) Both conclusion I and II follows
Statements: – A%Y, Y*X, X#W, W*V
persons sit between the one who likes Puma and O. O does not Conclusions: – (I) A%X (II) V$Y
like Puma. M likes Gucci is sitting third to the right of P who is (a) Only conclusion I follows
an immediate neighbour of the person who likes Puma. P does (b) Only conclusion II follows
not sit at any of the corners and does not like Reebok. U is not (c) Either conclusion I or II follows
an immediate neighbour of Q. The one who likes Reebok sits (d) Neither conclusion I nor II follow\
immediate left of I, who does not sits any of the corners. The (e) Both conclusion I and II follows
one who likes Arrow and Chanel are sitting next to each other. 8. Statements: – A*Y, Y$X, X#W, W@V
3 likes Gucci and immediate left of N. K does not like Reebok. Conclusions: – (I) V*A (II) X%A
T neither likes Chanel nor Arrow. The one who likes Zara sits (a) Only conclusion I follows
second to the right of the one who likes Arrow. (b) Only conclusion II follows
1. P likes which of the following cloth brand? (c) Either conclusion I or II follows
(a) Puma (b) Reebok (d) Neither conclusion I nor II follow
(c) Gucci (d) Allen solly (e) Both conclusion I and II follows
(e) Prada 9. Statements: – A$Y, Y$X, X*W, W@V
2. Four of the following are alike in a certain way and hence Conclusions: – (I) A$V (II) V%X
form a group. Which of the following does not belong to
(a) Only conclusion I follows
that group?
(b) Only conclusion II follows
(a) O (b) N (c) M (d) K
(e) L (c) Either conclusion I or II follows
3. Who among the following likes Allen solly? (d) Neither conclusion I nor II follow
(a) S (b) U (c) T (d) O (e) Both conclusion I and II follows
(e) Q 10. Statements: – A%V, V@X, X%Y, Y*W
4. How many persons are sitting between N and L? Conclusions: – (I) X%W (II) Y@A
(a) None (b) One (c) Two (d) Three (a) Only conclusion I follows
(e) Four (b) Only conclusion II follows
5. Who among the following likes the same cloth brand? (c) Either conclusion I or II follows
(a) K - P (b) L - M (c) K - M (d) P - N (d) Neither conclusion I nor II follow
(e) U - I (e) Both conclusion I and II follows
GP_4458
2020-2 SBI Clerk (Junior Associates) Mains Solved Paper-2020
Tg:- @NextGenBankers
There are six student of different types subject viz., Biology,
Physics, Chemistry, Mathematics, Computer and English in the
The one who goes on 20th September like Gaya. V goes on
25th of a month which having less than 31 days but not in
February. Three persons are going in between V and W who like
Sharda university but not necessarily in same order. They attend Faridabad. X are not going in September. Y and S are going in
their seminar at different timing in a single day. The duration of the same month. Y like Swimming. U goes immediate after V.
their seminar with the seminar is different. There is no gap between Three persons are going in between U and T who neither like
the seminar time of all student. The duration of seminar either full Agra nor Bareilly. One of them who goes in June like Pune. W
hour or half an hour but not in one third or one fourth of the hour. are not going for trip on last day. The one who like Lucknow and
The Physics attend the seminar after the seminar time of the one who like Agra are not going for trip on same date. Two
persons are going in between R and P who like Goa. R goes for
Biology. The time duration of attending their seminar of the
trip before P and does not like Bareilly. Three persons are going
Biology and the English is same. The duration of seminar with in between the one who like Pune and the one who like Meerut.
seminar of Physics is one hour less than the duration of seminar Two persons are going in between X and Q and none of them are
with seminar of English. The Biology attends his seminar going for trip in January and February. Y are not going for trip
immediately after or immediately before Chemistry attends his on an odd date. The one who like Lucknow are not going for trip
seminar. The timing of English seminar is after 7:00 am. The on 20th of any month. T does not like Pune.
seminar duration of Mathematics is half an hour more than the 31. Who among the following like Meerut?
Physics. The Computer attends his seminar immediately before (a) V (b) T (c) P (d) S
the Physics attend the seminar. The Mathematics attends the (e) None of these
seminar before Biology but not immediately before. The total 32. The one who goes on 25th February like which of the
duration of the seminar with the seminar by all students is of following city?
(a) Pune (b) Goa
9:30 hours. The English seminar time is before Chemistry. The
(c) Lucknow (d) Bareilly
seminar time of Computer is 2:30 pm to 4:00 pm. The time (e) None of these
duration of seminar of Chemistry and Mathematics is same. 33. Which of the following is true about U?
The seminar time of English is two hours. The seminar time of (a) 25th -Lucknow (b) December-Swimming
Mathematics is after 9:00 am in the morning. (c) September- Gaya (d) 20th -Agra
26. Who among the following Student attend the seminar (e) June- Pune
between seminar hours 11:00 am to 12:30 pm? 34. X goes which of the following month?
(a) Biology (b) Physics (a) January (b) February
(c) Chemistry (d) Mathematics (c) June (d) September
(e) None of these (e) December
GP_4458
2020-4 SBI Clerk (Junior Associates) Mains Solved Paper-2020
35. How many persons are going in between T and Q? (a) Prakhar The one who likes white colour
(a) Two (b) Four (c) Six (d) Three (b) Quan, Shyam
(e) More than Six (c) Tanu, The one who likes blue colour
36. If ‘A × B’ mean ‘B is the sister of A’, ‘A + B’ means ‘B is (d) Ram, The one who likes white colour
the daughter of A’ , ‘A ÷ B’ means ‘A is the wife of B’, and (e) None of these
‘A-B’ means ‘A is brother of B’ then how will ‘Q is mother DIRECTIONS (Qs. 43-45): Each of the questions below
of P’ be denoted? consists of a question and two statements numbered I and II
(a) J – Q ÷ P + L (b) Q + P + M × J given below it. You have to decide whether the data provided in
(c) J – Q ÷ L + P (d) Q – J ÷ L + P the statements are sufficient to answer the question.
(e) None of these
37. In which of the following expression will the expression (a) If statements I alone is sufficient to answer the
‘C3000>A1000’ holds true? question, but statement II alone is not sufficient to
(a) C3000 ≤ E5000 = F6000 > A1000 ≤ D4000 answer the question.
(b) F6000 > E5000 = D4000 < A1000 ≥ B2000 = C3000 (b) If statement II alone is sufficient to answer the
(c) A1000 < B2000 = D4000 ≥ E5000 = C3000 question, but statement I alone is not sufficient to
(d) E5000 < C3000 ≥ D4000 > B2000 ≥ A1000 = F6000 answer the question.
(e) C3000 ≤ D4000 < E5000 ≤ A1000 < B2000. (c) If statement either I or II is sufficient to answer the
question.
DIRECTIONS (Qs. 38-42): Study the following information (d) If both the statements I and II taken together are not
carefully and answer the questions given below: sufficient to answer the questions.
(e) If both the statements I and II taken together are
Seven employees Prakhar, Quan Ram ,Shyam, Tanu, Uday sufficient to answer the questions.
and Vijay had their offices situated along a straight row facing 43. Six girls DD, UU, KK, LL, MM and NN are sitting in
the north. They like different colours among Red, Blue, While, linear row and facing north. Who among sits second to the
Black, Yellow, Pink and Green but not necessarily In the same left of KK?
order. The distance between the neighbouring offices was a I. Two girls sit between LL and MM. One person sits
successive integral multiple of 5km. The distance increased between KK and LL. DD is not immediate neighbor of
from left to right. LL.
Prakhar’s office was third to the left of the official who likes pink II. DD sits third to the left of KK. UU is immediate neighbor of
Tg:- @NextGenBankers
colour. Quan’s office was 135km lo the right of the official who
likes black colour. Ram’s office was exactly between Vijay’s
office and official who likes green colour. The official who likes
44.
DD. KK does not sit extreme end of the row.
How many children are there between Priya and Gargi in a
row of children?
yellow colour was to the immediate left of the official who likes I. Priya is tenth from the right end in the row of 40 children.
white colour. The official who likes white colour was 115km to II. Gargi sits twelfth from left end of the row.
the left of Shyam’s office. The official who likes pink colour was 45. How is Fiza related to Ankit?
the neighbour of Shyam’s office. The official who likes black I. Ankit is son in law of Deep. Bikki is daughter of Fiza.
colour was 85km away from Tanu’s office. The official who II. H is brother in law of Bikki.
likes red colour had office at one of the extreme ends. Vijay did 46. If in the number 57672154, 2 is multiplied to each of the
not like blue colour. The distance between any two offices was digit which is less than 4 and 3 is subtracted from each
less than 80 km. of the digit which is more than 4 and equal to 4 then how
38. Which colour does Uday likes? many digits are repeating in the number thus formed?
(a) Red (b) Blue (c) Yellow (d) Green (a) None (b) Four (c) One (d) Three
(e) Pink (e) Two
39. What is the distance between Ram and the employee who 47. Statement: A slump in home sales has pushed builders’
likes white colour? inventory to “unsustainable levels” in the National Capital
(a) 95km (b) 85km (c) 170km (d) 135km Region of Delhi, creating conditions for a real price
(e) 150km correction that developers have so far avoided.
40. How many offices are there between Prakhar and the Which of the following statements is definitely true on the
official who likes blue colour? basis of the given statement?
(a) None (b) One (c) Two (d) Three (a) Demand for houses is quite low in most of the big
(e) Four cities.
41. Who among the following sits to the immediate left of (b) The high inventory level will hamper the ability of
Tanu? launching new projects by the builders.
(a) Prakhar (c) Builders in the market who have shown a good track
(b) The one who likes white colour record of delivering on time and on their promises have
(c) Vijay been able to garner sales even in this slow market.
(d) Unless this unsold stock gets absorbed, it will be
(d) The one who likes blue colour
difficult for consumer confidence to come back in this
(e) Quan
market.
42. Who among the Following has distance of 165km between
(e) High prices have pushed most of the on-sale
them?
apartments beyond the reach of average home buyers.
SBI Clerk (Junior Associates) Mains Solved Paper-2020 2020-5
48. Statement: According to the manpower Employment 51. If the expenditure of Scientist on Health and Food is 10%
Outlook Survey, globally, Indian employers are the more than the expenditure on Health and Food by Doctor.
most upbeat on hiring plans in the fourth quarter ending Then, find amount of expenditure on Food and health by
December 2015. Scientist.
Which of the following can be an assumption in the (a) ` 12000 (b) ` 11000
given information? (An assumption is something that is (c) ` 15000 (d) ` 25000
supposed or taken for granted) (e) ` 13000
(A) Decline in the profit margins of the company 52. If saving of Doctor is (2/5) of the income left after spend
(B) Establishment of more branches by the MNCs in on Health and Food. Then, find the value of x.
India (a) ` 10,000 (b) ` 12,000
(c) ` 16,000 (d) ` 25,000
(C) Maintenance of positive growth rate by the MNCs in
(e) ` 20,000
India. 53. If the saving of Scientist is 45% of his monthly income,
(D) Increase in the growth of service sector in June 2015. then, find the monthly income of Scientist.
(a) Only A and C (b) Only B and D (a) ` 22,000 (b) ` 15,000
(c) Only A and B (d) Only C and D (c) ` 50,000 (d) ` 40,000
(e) Only B and C (e) ` 20,000
49. Statement: Persistent weakness in global demand and 54. If the saving of Scientist is 45% of his monthly income,
the lower value of oil products led India’s merchandise find the sum of the monthly income of Doctor, Engineer
exports to fall for the sixth straight month in May, while a and Scientist
decline in gold imports helped the trade deficit narrow to a (a) ` 1,42,000 (b) ` 1,62,000
three-month low. (c) ` 1,20,000 (d) ` 2,00,000
Which of the following inference drawn is probably false (e) ` 1,25,000
55. After spending the monthly income on Health and Food,
according to the above statement?
remaining income of Engineer is his saving. Then, saving
(a) There will be softening of oil products in the global of Engineer is what percent of saving of Scientist.
market. (a) 704% (b) 625% (c) 500% (d) 729%
(b) Rupee will appreciate against the dollar. (e) 539%
(c) There will be an increase in the collection of indirect
DIRECTIONS (Qs. 56-60): Read the given graphs carefully
Tg:- @NextGenBankers
taxes in the month of April and May.
(d) There will be an increase in the domestic demand of and answer the questions that follow:
gold and jewellery. During the winter season, people from Mumbai are travelling
(e) Only (b) and (d) to different holiday locations by booking tickets from
50. Statement: The water situation is grim as the total storage travel the world.com. The total revenue generated by the website
in 91 main reservoirs across the country has gone below is ` 60,000.
the last 10-year average, says the report of the Central
Water Commission (CWC). Manali
Which of the following may be the most probable reason 18% Dehradun
for the fall in the level in reservoirs? 21% Dehradun, 21%
Quantitative Aptitude
Haridwar
15%
DIRECTIONS (Qs. 51-55) : Read the table carefully and City Tickets Cancelled
answer the questions given below :
Dehradun 3
Table represents the expenditure, saving and monthly Shimla 12
income of Doctor, Engineer and Scientist.
Haridwar 10
Persons/ Items Doctor Engineer Scientist
Rishikesh 13
Expenditure on 20% of 12% of
Health and Food monthly monthly Manali 15
income income 56. If the cost of each ticket on travelling to Haridwar is ` 600,
Saving `x – ` 9000 then what will be the number of people who travelled?
(Cancel tickets are not refundable)
Monthly income ` 50,000 ` 72,000 (a) 10 (b) 12 (c) 15 (d) 7
(e) 8
GP_4458
2020-6 SBI Clerk (Junior Associates) Mains Solved Paper-2020
57. The total number of people that travelled to Shimla was 30 DIRECTIONS (Qs. 66-70): The bar graph shown below
and to Rishikesh was 26 then calculate the average price indicate the number of books owned and the number of books
per ticket issued by the 5 different libraries P, Q, R, S and T. Based on the
(a) ` 340.50 (b) ` 520.75 given bar graph answer the questions given below:
(c) ` 500 (d) ` 620.75
(e) ` 700 Book owned Book issued
58. The cost of each ticket on travelling to Manali is ` 720 and 1200
no ticket was cancelled, then what will be the number of 1040
960
1080
72. Quantity I : Two dice are thrown simultaneously. What is 79. What is the value of the two-digit number?
the possibility of getting odd number on both the dice? Statement I : The sum of the digits at unit's place and
1 tenth's place is 12.
Quantity II : Statement II : When the digits are interchanged then the
5
number obtained is 36 larger than the original number.
73. Quantity I : P 2 − 39 P + 378 = 0
2 80. What is the sum of m and n?
Quantity II : Q − 24Q + 108 = 0 Statement I : m2 – 10m + 25 = 0
2
74. Quantity I : P − 11P + 14 = 0 Statement II : n2 – 14n + 49 = 0
2 81. What is the area of a right-angled triangle PQR?
Quantity II : Q − 20Q + 96 = 0
75. Quantity I : A downstream speed is 20 Km/h and upstream Statement I : The lengths of sides PQ and QR of the
speed is 10 Km/h. Distance travelled by a boat is 30 Km. triangle is 16 cm and 12 cm respectively.
Find the time taken by a boat to cover the distance. Statement II : The inradius of triangle PQR is 4 cm and
Quantity II : 4 hours. the circumradius is 10 cm.
82. A box contains 21 mugs numbered 1 to 21. A mug is drawn
DIRECTIONS (Qs. 76-81): Each question below is followed by
and then another mug is drawn without replacement. What
two statements I and II. You have to determine whether the data
is the probability that both mugs are odd numbered?
given in the statements are sufficient for answering the question.
You should use the data and your knowledge of Mathematics to 2 8 3 5
choose the best possible answer. (a) (b) 21 (c) (d)
7 14 21
(a) The data in statement I alone is sufficient to answer (e) None of these
the question, while the data in statement II alone is 83. The average temperature from Monday to Wednesday is
not sufficient to answer the question. 36°C. When data for Thursday is included, the average
(b) The data in statement II alone is sufficient to answer increases by 1°C. What was the temperature on Thursday?
the question, while the data in statement I alone is not (a) 37°C (b) 38°C (c) 39°C (d) 40°C
sufficient to answer the question. (e) None of these
(c) Statement I or Statement II alone is sufficient to 84. By how much is 3/4th of 52 is less than 2/3rd of 99?
answer the question. (a) 27 (b) 33 (c) 39 (d) 29
Tg:- @NextGenBankers
(d) The data in both the statements I and II are not
sufficient to answer the question. 85.
(e) 55
The monthly incomes of Mohit and Sohit are in the ratio
(e) The data in both the statements I and II together are of 4 : 3. Their expenses are in the ratio of 3 : 2. If each
necessary to answer the question. saves 6,000 at the end of the month, their monthly incomes
76. Auto A is moving behind Auto B in the same direction and respectively are (in `)
the distance between them initially is 40 km and the speed (a) 24,000 and 18,000 (b) 28,000 and 21,000
of Auto B is 50 km/h, then how long would Auto A take to (c) 32,000 and 24,000 (d) 34,000 and 26,000
cross Auto B. (e) 30,000 and 28,000
Statement I : The time taken by Auto B to cover a distance 86. Neelam's mathematics test had 70 problems carrying equal
of 200 km is 4 hours. marks i.e., 10 Arithmatic, 30 Algebra and 30 Geometry.
Statement II : If the Auto were moving in the opposite Although she answered 70% of the Arithmatic, 40% of the
directions, towards each other, the relative speed of Auto Algebra and 60% of the Geometry problems correctly. She
A with respect to Auto B would have been 120 km/hr. did not pass the test because she got less than 60% marks.
77. A shopkeeper sold two items of same marked price of ` The number of more questions she would have to answer
3000 at a profit of 25% and 20% respectively. Find the correctly to earn a 60% passing marks is :
selling price of both the items together. (a) 1 (b) 5 (c) 7 (d) 9
Statement I : The selling price of item A and selling price (e) None of these
of item B are in the ratio of 17 : 23 respectively. 87. If the cost of a Banana is equal to the cost of a Mango.
Statement II : The discount offered by the shopkeeper for If the cost of a Banana increases by 4% and the cost of
item A is 25% and the ratio of the cost price of item A to
a Mango is increases by 5% then what percent more one
the cost price of item B is 11 : 10 respectively.
should have to pay to purchase 5 Mangoes and 4 Bananas?
78. Find the value of "a".
(a) 4% (b) 4.55% (c) 5.55% (d) 5%
Statement I : Rahul and Shivam started a business with
(e) 6.25%
the investment of money in the ratio 5 : a respectively.
88. 8 members are to be selected from a group of 9 Boys and
After 2 months, Garvit joined them in such a way that the
7 Girls. In how many ways will the members with at most
ratio of the initial investment of Shivam and Garvit is 7 : 8
respectively. After 2 years, they received a total profit of 3 Girls and atleast 4 Boys be selected?
` 16800. (a) 6435 ways (b) 6225 ways
Statement II : The profit share of Garvit after 2 years in ` (c) 6472 ways (d) 7320 ways
6000 out of total profit of ` 16800. (e) None of these
GP_4458
2020-8 SBI Clerk (Junior Associates) Mains Solved Paper-2020
89. In a family of 4 persons, P and Q are husband and wife. R DIRECTIONS (Qs. 96-100): Study the following information
and S are daughter and son of P and Q respectively. The carefully and answer the questions given below:
ratio of the present ages of mother and daughter is 9 : 1. S's
age 7 years ago was 3 years. The sum of the present ages In online products industry, the growth of the industry is driven
of P, Q and R is 100 years. At the time of the birth of R, the by the increase in the number of people buying online and the
age of P was equal to the present age of Q. Find the present increase in the number of people selling online.
age of P. In 2019, it was expected that total 200 million people would
(a) 35 years (b) 40 years buy products online in India that would be 25% of the total
population of India and 4% of the total population of India would
(c) 50 years (d) 45 years
sell products online. If in 2020, the population of India was
(e) None of these
increased by 10% over the previous year together with the total
90. The average weight of a office's 60 employees are 60 Kg. number of people who bought products online was increased by
If two of the employee with weight 40% and 50% of 'a' 25% over the previous year and the number of sellers remained
leave the office and two new employees with weight a and constant then in the year 2020 the industry revenue was $100
a-34 join the office, then the average weight of the office's billion.
employees are increased by 2 Kg. Find the value of a. 96. In 2019, what was the total number of people from India
(a) 90 Kg (b) 140 Kg who sold the products online?
(c) 100 Kg (d) 170 Kg (a) 1 million (b) 100 million
(e) 78 Kg (c) 50 million (d) 32 million
91. A vessel contains mixture of Juice and Water in the ratio (e) None of these
of 5 : 2 respectively. If 35 liters mixture taken out from 97. If the revenue per seller was same in 2019 as compared to
the vessel and now the difference between Juice and Water 2020 then what was the revenue per seller (in $) in 2019?
in the remaining mixture is 141 liters, then find initial (one billion is equal to 1000 millions)
mixture in vessel? (a) 40 million (b) 200 million
(a) 364 liters (b) 200 liters (c) 5 million (d) 2 billion
(c) 150 liters (d) 120 liters (e) None of these
(e) None of these 98. If in 2021, the number of people who will buy products
92. Rahul sold his article at 25% profit. If he sold article for ` online will increased by 40% over the previous year then
Tg:- @NextGenBankers
60 more, he would have gained 35%; if he wants a profit
of 70% then what should be the marked price of the article
in 2021, total how many people in million will buy product
online?
(in `) (a) 244 (b) 175 (c) 132 (d) 250
(e) None of these
(a) 1750 (b) 1020 (c) 1680 (d) 2000
99. In 2021, the population of India was 1000 million then
(e) 1875
what was the percentage growth of India over the period
93. A shopkeeper prepared a list of 30 article's amount and 2019 to 2021?
average of their amount was 136. But amount of one (a) 20% (b) 40% (c) 25% (d) 20%
article was written as 70 instead of 106. Find the correct (e) None of these
average amount? 100. It is assumed that in 2021, because of internet, 50% of the
(a) 128.4 (b) 140.4 (c) 131.2 (d) 130.02 total population of India will buy products online. If in
(e) 137.2 2021, the population of India was increased by 10% over
94. The length of sides PQ, PR and QR of the triangle is 20, previous year then in 2021, total how many people will
25 and 15 cm respectively, then inradius of the triangle is buy product in India?
what percent less than the circumradius of the triangle? (a) 528 million (b) 543 million
(a) 60% (b) 40% (c) 37½% (d) 70% (c) 539 million (d) 433 million
(e) 65% (e) None of these
95. Two cars C1 and C2 are going towards x to y and y to x
respectively. After sometime they meet anywhere between English Language
x and y and after meeting car C1 reaches y in 1 hrs and C2
DIRECTIONS (Qs. 101-107): Read the following passage and
at 4 hrs. If the speed of car C2 is 60 Km/h then find the answer the following questions.
speed of car C1 in m/sec.
150 Remote work is shaping up to be a major trend that will likely
(a) 100 m/sec. (b) m/sec. keep growing even after the pandemic ends. Remote working has
7
several benefits, including flexibility, better diversity, location
50 independence, no commute time, and more. But what about
(c) 20 m/sec. (d) m/sec.
3 the drawbacks? Earlier, employees could disconnect from their
work when they stepped out of the office. Now, the boundaries
100 between personal and professional lives have blurred. Juggling
(e) m/sec.
3 between professional and personal tasks with no leisure time
SBI Clerk (Junior Associates) Mains Solved Paper-2020 2020-9
can stress employees out and affect their mental health. If (I) Establish a connection with the employees
left unchecked, this stress can, in some cases, also increase (II) Hear and understand employees perspective
an employee’s risk for heart diseases. As the stress levels of (III) Varied employee centric policies like wellness leaves
individual employees increase, it becomes a ______________ keeping in mind their preferences
(A) effect that can lead to an obstructive work environment and (a) Only II (b) Only I
weak professional relationships. (c) Both I and II (d) Both I and III
Work-life balance is crucial for employee satisfaction and overall (e) All of the above
business success as well. Organizations that neglect this aspect 104. Which of the following cannot be inferred from the given
will have a hard time attracting and retaining talent. Individuals, passage?
especially Millennials and Gen Z, expect companies to respect (I) Neglecting employees stress management will have
their time and mental health. They want to be able to balance no adverse effect on the businesses
their family time, health, social life, hobbies, and more. It’s even (II) Significant measures can improve work-life balance
been shown that they prefer work-life balance over any financial of the employees
(III) Companies haven’t acquired needed initiatives for the
incentives when choosing a company to work for.
remote employees
To stay ahead of expectations, many organizations have begun
(a) Only II (b) Both I and III
initiatives to provide remote employees a smooth working
(c) Only III (d) Both II and III
experience. According to a recent research conducted by the (e) None of these
University of Birmingham and the University of Kent, 73% 105. Which of the following words is most similar in meaning
of the employees surveyed either agreed or strongly agreed with the given word as highlighted in the passage?
that their managers cared about the impact of work demands Significant
on their personal life during the pandemic. We will continue to (a) Auditory (b) Unnecessary
see more organizations taking significant measures to improve (c) Amazing (d) Outer
their employees’ work-life balance and enhance employee (e) Momentous
management. 106. Which of the following can fill in the blank (A) to make
It starts with creating a more empathetic work environment. the sentence grammatically and contextually correct.
Every workforce is unique, and it’s up to the management to (a) Practice (b) Intend
connect with their employees and hear their perspective before (c) Ripple (d) Concern
Tg:- @NextGenBankers
implementing necessary measures. Hybrid work, flexible holiday
policies, floating holidays, wellness leaves, mental health days— 107.
(e) Approached
Which of the following words is most opposite in meaning
these are all excellent places to start, but the degree to which with EMPHATIC as highlighted in the passage?
an organization adopts them will depend on their employees’ (a) Unconditional (b) Hesitant
preferences. Empathetic leadership practices like these will also (c) Outright (d) Definite
help employees adapt to the new normal working conditions and (e) Conclusive
deliver their best. DIRECTIONS (Qs. 108-115): Read the following passage and
101. Which of the following can be inferred from the passage? answer the following questions.
(I) Employees are getting bored from the remote working
so it will not be continued post-pandemic A gig economy consists of people carving a career path through
(II) Remote working has made it difficult to differentiate freelance and contractual work – wherein the individual is paid
between professional and personal life per job, or ‘gig’, as opposed to receiving a regular salary or
(III) People hardly get time to relax thus they begin getting wage from an employer. While the term ‘gig economy’ came
stressed out into being very recently, there has always been a freelance
(a) Only II (b) Both I and III industry shrouded by mammoths of the business world. As a
(c) Only III (d) Both II and III matter of fact, before the industrial revolution and the rise of the
employee-employer contract, this arrangement was prototypical.
(e) None of these
Today, most people look at it as the much desired antidote to a
102. Why author is of the opinion that work-life balance is
monotonous nine-to-five existence, allowing them to work with
crucial?
freedom and flexibility. There also exists a certain section who
(a) The work-life balance is necessary for satisfied
despises the uncertain nature of gig work and its potential for
employees as well as business success exploitation. That being said, currently the world is steadily
(b) Neglecting the balance will result in hard-time to moving towards the gig economy and while it seems like the
attract and retain talents in the organizations ideal contingency plan to tackle anything remotely similar to
(c) People can balance their professional and personal the ominous pandemic that brought the world to a standstill,
life we can’t help but wonder, what does the future hold for a gig
(d) Both (a) and (b) workforce.
(e) People deny work-life balance and go for financial The rise of the gig economy has made it easier for people to take
incentives charge of their work lives, giving them the freedom to work the
103. How to create an emphatic work environment according to way they want to. This very characteristic of the gig economy
the author of the given passage? can be greatly tempting for creative or skilled professionals such
GP_4458
2020-10 SBI Clerk (Junior Associates) Mains Solved Paper-2020
as content writers, web developers, etc. who can maximize their 110. Why the author is of the opinion that full-time employment
skills through an unrestrained, unrestricted work set up. Further, is not for everyone?
if we try to understand basic human nature, it becomes evident (a) The employer establishes its authority over
that the lack of autonomy, ownership and flexibility can lead to employees’ skills
people feeling disgruntled and detached from their work. This (b) The employer thinks that he gets the right to mend
is a clear indicator of the fact that full-time employment isn’t and use the employees’ skills as per his needs and
for everyone. For most people, the deal-breaker in a full-time desires
employment set up would be that the employer principally owns (c) Both (a) and (b)
the employee’s skills; employers believe they have the right (d) The monthly fees which we receive as a salary gives
the employer the mandate to buy our skills
to use them as they desire and as in line with the employment
(e) All of the above
contract and job designation. In essence, the employer buys the
111. How the writer sees future of the Gig work?
rights to an individual’s skills for a monthly fee which is the (a) The gig work is more meaningful and rewarding
salary. However, on the flipside, the gig economy allows an irrespective of the financial earning
individual to take absolute charge of their skills and knowledge. (b) One will build his own recognition rather than
The future of Gig work holds great potential because it tends someone else’s
to be more meaningful and rewarding in more than just the (c) It is more focused on outcomes than rigorous
monetary aspect. The simple fact that you are building your processes
own brand instead of someone else’s makes this type of work (d) All of the above
intrinsically motivating in the long run. Unlike traditional (e) None of these
employment, gig work focuses on results instead of embroiling 112. In what manner, gig work will open up an opportunity rich
processes. Gig workers or freelancers are evaluated and paid world for freelance workers?
based on their ability to deliver results. How they achieve their (a) It integrates better with technology and digital
targets is completely up to them. This allows freelancers to tap innovations
into their natural reserves of perception, imaginative thinking (b) The digital marketplaces are expanding their
and self-reliance. functioning on contractual basis
Also, the biggest advantage of gig work lies in its integration (c) Digitized work platforms will be advantageous for
with technology because needless to say, the future of India’s corporate giants
(d) The service providers can directly contact with
start-ups and its gig economy lies in digital innovations and a
Tg:- @NextGenBankers
robust digitized ecosystem. A major part of the gig economy,
digital marketplaces are now signing up demand-driven service
consumers via mobile apps
(e) all (a), (b), and (d)
providers skilled in niche areas such as beauty, fitness, plumbing, DIRECTIONS (Qs. 113-114): Which of the following is most
electrical repairs, etc. and offer these services to consumers on similar in meaning with the given word as highlighted in the
a contractual basis as per requirements. This business model given passage?
allows for the service providers to communicate with the end 113. Contingency
users directly through the app according to their terms and (a) Instance (b) Possibility
conditions. This opens up a world of opportunities for the (c) Projection (d) Constant
freelance workforce. (e) Affected
108. Which of the following can be inferred about ‘gig 114. Disgruntled
economy’ as discussed in the given passage? (a) Straighten (b) Conceptualized
(I) People do freelancing and work on contractual basis (c) Courteous (d) Aggrieved
(II) The salary is not regular in a gig economy (e) Passages
(III) It helps people to escape from monotonous 9 to 5 115. Which of the following is the most opposite in meaning
work routine with INTRINSICALLY as highlighted in the given
(a) Only II (b) Both I and III passage?
(c) Only III (d) Both II and III (a) Established (b) Conducted
(e) All of the above (c) Acquired (d) Engaged
109. Which of the following correctly states the benefits of gig (e) Scarcity
economy? DIRECTIONS (Qs. 116-120): In the following passage, some
(I) People can manage their work life freely of the words have been left out, each of which is indicated by
(II) The work set up is unrestricted a letter. Find the suitable word from the options given against
(III) Freedom to work is favorable for skilled and creative each letter and fill up the blanks with appropriate words to make
professionals the paragraph meaningful.
(IV) The individuals entirely own their knowledge and Reversing two days of heavy losses, the Sensex posted its biggest
skills jump in almost four months on Tuesday as investors returned to
(a) I, III and II (b) I, III and IV buying mode amid expectations of fresh _________________
(c) I, II, and IV (d) all of the above (116) in the U.S. and other economies. A recovering rupee and
(e) Only I and IV persistent foreign capital inflows further _________________
(117) sentiment, traders said. The 30-share S&P BSE Sensex
SBI Clerk (Junior Associates) Mains Solved Paper-2020 2020-11
soared 834.02 points, or 1.72%, to close at 49,398.29. Only 123. (a) Prohibits (b) Persists
three Sensex components closed in the red — Tech Mahindra, (c) Pleads (d) Weighs
ITC and M&M, shedding up to 0.54%. Global equities were (e) Inclines
on a/an _________________ (118) after U.S. President-elect 124. (a) Attributed (b) Exerted
Joe Biden’s nominee for Treasury Secretary, former Federal (c) Oppressed (d) Barred
Reserve chair Janet Yellen _________________ (119) for (e) Contended
more measures to fight the recession and avoid an even worse 125. (a) Toil (b) Rail
downturn. In testimony prepared for her confirmation hearing (c) Hurt (d) Boast
on Tuesday before the Senate Finance Committee, Ms. Yellen (e) Lofty
said more aid is needed to get COVID-19 vaccines distributed
and help families struggling with job losses stay fed and DIRECTIONS (Qs. 126-130): In each of the following
_________________ (120). questions, choose the sentence/s that has/have the correct usage
116. (a) Render (b) Generate of the highlighted and underlined idiom.
(c) Stimulus (d) Range
(e) Labor 126. (I) The mugger showed a clean pair of heels, skittering
117. (a) Conferred (b) Granted down a side alley and escaping out of sight.
(c) Dwelled (d) Entertained (II) As makers of quality software, they’ve shown a
(e) Buoyed clean pair of heels to the rest of the industry
118. (a) Yield (b) Earnest (III) They ran after her, but she showed them a clean pair
(c) Upswing (d) Wandering of heels.
(e) Insisting (a) Only I (b) Both I and III
119. (a) Batted (b) Convinced (c) Both II and III (d) Only II
(c) Knighted (d) Inspired (e) All of these
(e) Conventional 127. (I) Can you please bury the hatchet and make up with
120. (a) Skilled (b) Harrying your sister already? I can’t take the constant fighting.
(c) Reflected (d) Housed (II) She thought she buried the hatchet when she saw
(e) Furnished John slinking away from the scene of the crime.
DIRECTIONS (Qs. 121-125): In the following passage, some (III) He gave a very diplomatic explanation, but if you
Tg:- @NextGenBankers
of the words have been left out, each of which is indicated by
a letter. Find the suitable word from the options given against
each letter and fill up the blanks with appropriate words to make
bury the hatchet, it seems like he was fired for
political reasons.
(a) Only I (b) Both I and II
the paragraph meaningful. (c) Both I and III (d) Only III
India is considering revising its foreign investment rules for (e) All of these
e-commerce, three sources and a government spokesman 128. (I) I had a chewing the cud this morning when a tractor
told Reuters, a move that could ____________ (121) players, trailer unexpectedly swerved into my lane.
including Amazon.com Inc., to restructure ties with some major (II) I’ve been chewing my cud for days, but I still haven’t
sellers. The government discussions ____________ (122) with decided whether I’m taking the job or not.
a growing number of complaints from India’s bricks-and-mortar (III Sometimes you just need to stop chewing the cud
retailers, which have for years accused Amazon and Walmart and make a decision.
Inc.-controlled Flipkart of creating complex structures to bypass (a) Only III (b) Only I
federal rules, allegations the U.S. companies deny. India only (c) Both II and I (d) Both II and III
allows foreign e-commerce players to operate as a marketplace (e) All of these
to connect buyers and sellers. It ____________ (123) them 129. (I) I tried to do well in this class, but I’ve been at sea
from holding inventories of goods and directly selling them on since we started.
their platforms. Amazon and Walmart’s Flipkart were last hit in (II) Bill was at sea over the calculus problem.
December 2018 by investment rule changes that ____________ (III) Mom sounded like she is at sea, so you better clean
(124) foreign e-commerce players from offering products from your room.
sellers in which they have an equity stake. Now, the government (a) Only III (b) Only II
is considering adjusting some provisions to prevent those
(c) Both II and III (d) Both I and II
arrangements, even if the e-commerce firm holds an indirect
(e) All of these
stake in a seller through its parent, the sources said. The changes
could ____________ (125) Amazon as it holds indirect equity 130. (I) Peter only comes out for a drink once in blue moon
stakes in two of its biggest online sellers in India. now that he has kids.
121. (a) Temper (b) Intimate (II) Sue’s daughter only visits her once in a blue moon.
(c) Compel (d) Undertake (III) Only once in a blue moon do properties of this
(e) Assert quality become available.
122. (a) Despair (b) Justify (a) Only III (b) Both I and III
(c) Manifest (d) Scale (c) Both II and III (d) Both I and II
(e) Coincide (e) All of these
GP_4458
2020-12 SBI Clerk (Junior Associates) Mains Solved Paper-2020
(c) Collation, Haggling (d) Cohort, Savant 144. The Serum Institute of India (SII) has signed an agreement
(e) Adjure, Unction with UNICEF & GAVI to supply Pneumococcal Conjugate
135. The __________ yardsticks to look at for investing include Vaccine (PCV) to low-income countries. Where is SII
the current valuations and future earnings __________ of located?
underlying companies. (a) Mumbai (b) New Delhi
(c) Pune (d) Kolkata
(a) Clarion, Acrimony (b) Turbid, Cupidity
(e) None of these
(c) Disaffected, Expatiating (d) Didactic, Sinuous
145. Rajat Bhatia, who announced his retirement, is associated
(e) Relevant, Trajectory
with which sports?
DIRECTIONS (Qs. 136-140): Read the sentence to find out (a) Football (b) Billiards
whether there is an error in it. The error, if any, will be in one (c) Cricket (d) Squash
part of the sentence. The number corresponding to that part will (e) None of these
be your answer. If the given sentence is correct as it is, mark the 146. Which day is observed across the globe on the Death
answer as ‘No error’. Ignore the errors of punctuation, if any. Anniversary of Mother Teresa?
(a) International Day of Charity
136. I myself am very proud (A)/and excited because (B)/our (b) World Peace Day
Kamala Harris is (C)/ becoming the first Vice-President of (c) International Day of Humanity
US.(D)/ No Error (E) (d) International Day of Compassion
(a) A (b) B (c) C (d) D (e) None of these
(e) No Error 147. Who has authored the book titled ‘10 Rules of Successful
137. The Port Authority Bus Terminal in Midtown (A)/has a Nations?
reputation of being the (B)/most squalid and overcrowd (a) Amit Shah (b) Ranjan Gogoi
(C)/transportation hub in New York.(D) (c) Ruchir Sharma (d) Roopa Pai
(a) A (b) B (c) C (d) D (e) None of these
(e) No Error 148. Which public sector bank has launched ‘Insta Click
138. The Budget for the next (A)/financial year is likely to laid Savings Account’, a 100 per cent paperless digital self-
(B)/down a roadmap to normalise fiscal (C)/deficit in the assisted online savings account?
next two to three years. (D)/ No Error (E) (a) IDBI (b) SIDBI
(a) A (b) B (c) C (d) D (c) Bank of Baroda (d) ICICI
(e) No Error (e) None of these
SBI Clerk (Junior Associates) Mains Solved Paper-2020 2020-13
149. Which insurance company has launched first-of-its-kind (a) Farmers (b) Swachhta Karmcharis
insurance product called ‘Shagun – Gift an Insurance”, (c) Rural Entrepreneurship (d) Civil services reforms
which is a unique gift of Personal Accident policy? (e) None of these
(a) SBI General Insurance 161. Where is the HQ of Federal bank?
(b) LIC (a) Uttar Pradesh (b) Bihar
(c) ICICI Lombard (c) Kerala (d) Madhya Pradesh
(d) General Insurance Company Ltd (e) None of these
(e) None of these 162. World Autism Awareness Day is celebrated on -----
150. Which of the following banks has launched ‘Home Utsav’, (a) 7 April (b) 8 April
a virtual property exhibition that digitally showcases real (c) 9 Apri (d) 2 April
estate projects by renowned developers from key cities (e) None of these
across the country? 163. The International Day of Charity is an international day
(a) IDBI (b) SIDBI (c) HDFC (d) ICICI observed annually on---
(e) None of these (a) 1 September (b) 5 September
151. Which of the following banks has launched ‘Shaurya KGC (c) 2 September (d) 3 September
(Kisan Gold Credit) Card’ for the armed forces? (e) None of these
(a) IDBI (b) SIDBI (c) HDFC (d) ICICI 164. What is the Share of Central Government in RRB?
(e) None of these (a) 60% (b) 50% (c) 70% (d) 80%
152. Harley-Davidson to shut down its manufacturing plant in (e) None of these
India______. 165. Book – ‘10 rules of successful nation’ is written by---
(a) Gurugram (b) NOIDA (a) Ruchir Sharma (b) Prachir Sharma
(c) Pune (d) Hyderabad (c) Ranveer (d) Chetan Sharma
(e) None of these (e) None of these
153. What is the stake of State Bank of India in the Jio Payments 166. Titan Pay is launched by which bank?
Bank? (a) SBI (b) PNB
(a) 10 per cent (b) 25 per cent (c) ICICI Bank (d) HDFC
(c) 30 per cent (d) 49 per cent (e) None of these
(e) None of these 167. Who is the new Chairman of TRAI?
154. On 8th October Indian Air Force celebrated its _____ (a) PD Vaghela (b) AD Vaghela
anniversary?
(a) 88th
Tg:- @NextGenBankers
(b) 70th (c) 77th (d) 73rd
(c) BD Vaghela
(e) None of these
(d) ND Vaghela
(e) None of these 168. Minimum Paid up capital………. for Retail Banking
155. How many deputy governors are there in RBI? under New Umbrella Entity (NUE).
(a) 2 (b) 3 (c) 4 (d) 5 (a) ` 800 crore (b) ` 700 crore
(e) None of these (c) ` 600 crore (d) ` 500 crore
156. The Central Board of Secondary Education (CBSE) has (e) None of these
partnered with which social media platform to launch a 169. Where is the HQ of UNDP?
free and comprehensive training program for teachers and (a) New York (b) Maxico
students of its affiliated schools in the first phase? (c) New Delhi (d) London
(a) Google (b) Facebook (e) None of these
(c) Microsoft (d) Twitter 170. Company which manages Assets is called ..
(e) None of these (a) AMC (b) DMC (c) CMC (d) VMC
157. As per the “Framework for authorisation of pan-India (e) None of these
Umbrella Entity for Retail Payments” released by RBI 171. Atal Pension Yojana is regulated by ---
what should be the minimum paid-up capital of the (a) PFRDA (b) SBI (c) RBI (d) LIC
umbrella entity? (e) None of these
(a) Rs. 100 crore (b) Rs. 300 crore 172. Mudra is the subsidiary of which organization?
(c) Rs. 500 crore (d) Rs. 1000 crore (a) RBI (b) SBI (c) SIDBI (d) LIC
(e) None of these (e) None of these
158. The PM KUSUM scheme has been extended to allow how
173. Who is the Current head of Tesla?
much farmers to set up standalone solar pumps?
(a) Elon Reeve Musk (b) Daniel Zhang
(a) 10 lakh (b) 20 lakh
(c) Jeff Bezos (d) Lisa Su
(c) 30 lakh (d) 40 lakh
(e) None of these
(e) None of these
174. Retail inflation was ----- percent in rural India
159. India has been ranked at________position on the Global
(a) 5.4 (b) 6.7 (c) 7.2 (d) 8.1
Economic Freedom Index 2020.
(e) None of these
(a) 105th (b) 100th
175. Oriental Bank of Commerce and United Bank of India
(c) 95th (d) 110th
merged with---
(e) None of these
(a) RBI (b) SBI (c) PNB (d) LIC
160. The ‘Mission Karmayogi’ which was recently in news is
related to (e) None of these
GP_4458
2020-14 SBI Clerk (Junior Associates) Mains Solved Paper-2020
176. Which Ministry represented India in the 2+2 dialogue 184. Rani Lakshmi Bai Central Agricultural University is
Between USA & India? central agricultural university located in----
(a) Ministry Of Home Affairs (a) Lucknow (b) Kanpur
(b) Defence Ministry (c) Jhansi (d) Varanasi
(c) EAM (e) None of these
(d) EAM & Defence Ministry 185. Atal Mission for Rejuvenation and Urban Transformation
(e) None of these (AMRUT) was launched in –
177. What is the Currency of South Korea? (a) January 2015 (b) July 2015
(a) Yen (b) Rial (c) Dinar (d) won (c) August 2015 (d) June 2015
(e) None of these (e) None of these
178. The Prime Minister launched India Post Payments Bank 186. The Ranapratap Sagar Dam is a gravity masonry dam of
(IPPB) on….. 53.8 metres height built on -------
(a) September 1, 2018 (b) September 3, 2018 (a) Yamuna river (b) Chambal River
(c) September 4, 2018 (d) September 5, 2018 (c) Ganga River (d) Gomati River
(e) None of these (e) None of these
179. The third Richest Person in India is ---- 187. When is the World Tribal Day observed?
(a) Hinduja brothers (b) Cyrus Poonawalla (a) August 10 (b) August 4
(c) Shiv Nadar (d) Gautam Adani
(c) August 9 (d) August 5
(e) None of these
(e) None of these
180. First White Label ATM was launched by----?
(a) Indian Oil Corporation Lt(d) 188. Total expenditure in budget 2020-21 is expected to
(b) Tata Communications Payment Solutions Limited be-----
(c) Reliance Industries Ltd (a) Rs. 50,42,230 crore (b) Rs. 30,42,000 crore
(d) State Bank of India (c) Rs. 40,42,230 crore (d) Rs. 30,42,230 crore
(e) None of these (e) None of these
181. What is the Stake of RIL in Jio Payment bank? 189. Dandiya Raas is the socio-religious folk dance originating
(a) 70% (b) 60% (c) 50% (d) 80% from-----
(e) None of these (a) Gujarat (b) Uttar Pradesh
182. Karmyogi scheme is launched for--- (c) Madhya Pradesh (d) Karnataka
(a) Doctors (b) Civil Servants (e) None of these
(c) Engineers
(e) None of these
Tg:- @NextGenBankers
(d) Skilled labourers 190. STARS Programme is related with—
(a) Education
(c) Film
(b) Game
(d) Tourism
183. The Dudhwa Tiger Reserve is a protected area in----
(a) Uttar Pradesh (b) Madhya Pradesh (e) None of these
(c) Bihar (d) Jharkhand
(e) None of these
ANSWER KEY
1 (e) 21 (e) 41 (a) 61 (b) 81 (b) 101 (d) 121 (c) 141 (d) 161 (c) 181 (a)
2 (a) 22 (a) 42 (c) 62 (c) 82 (e) 102 (d) 122 (e) 142 (a) 162 (d) 182 (b)
3 (c) 23 (b) 43 (e) 63 (e) 83 (d) 103 (e) 123 (a) 143 (c) 163 (b) 183 (a)
4 (d) 24 (d) 44 (e) 64 (b) 84 (a) 104 (b) 124 (d) 144 (c) 164 (b) 184 (c)
5 (d) 25 (c) 45 (d) 65 (e) 85 (a) 105 (e) 125 (c) 145 (c) 165 (a) 185 (d)
6 (a) 26 (c) 46 (e) 66 (b) 86 (b) 106 (c) 126 (e) 146 (a) 166 (a) 186 (d)
7 (e) 27 (a) 47 (e) 67 (c) 87 (b) 107 (b) 127 (a) 147 (c) 167 (a) 187 (c)
8 (d) 28 (e) 48 (e) 68 (d) 88 (a) 108 (e) 128 (d) 148 (c) 168 (d) 188 (d)
9 (b) 29 (b) 49 (d) 69 (d) 89 (c) 109 (d) 129 (d) 149 (a) 169 (a) 189 (a)
10 (a) 30 (d) 50 (a) 70 (a) 90 (b) 110 (c) 130 (e) 150 (d) 170 (a) 190 (a)
11 (b) 31 (d) 51 (b) 71 (d) 91 (a) 111 (d) 131 (c) 151 (c) 171 (a)
12 (d) 32 (b) 52 (c) 72 (b) 92 (b) 112 (e) 132 (b) 152 (a) 172 (c)
13 (c) 33 (c) 53 (e) 73 (c) 93 (e) 113 (b) 133 (d) 153 (c) 173 (a)
14 (b) 34 (c) 54 (a) 74 (a) 94 (a) 114 (d) 134 (a) 154 (a) 174 (c)
15 (c) 35 (b) 55 (a) 75 (d) 95 (e) 115 (c) 135 (e) 155 (a) 175 (c)
16 (c) 36 (c) 56 (c) 76 (a) 96 (d) 116 (c) 136 (a) 156 (b) 176 (d)
17 (d) 37 (d) 57 (b) 77 (b) 97 (d) 117 (e) 137 (c) 157 (c) 177 (d)
18 (c) 38 (e) 58 (e) 78 (e) 98 (b) 118 (c) 138 (b) 158 (b) 178 (a)
19 (d) 39 (d) 59 (c) 79 (e) 99 (c) 119 (a) 139 (b) 159 (a) 179 (c)
20 (b) 40 (d) 60 (a) 80 (e) 100 (a) 120 (d) 140 (e) 160 (a) 180 (b)
SBI Clerk (Junior Associates) Mains Solved Paper-2020 2020-15
20. (b) In step IV the elements “48 25 situation” found in the 27. (a) Chemistry Student attend the seminar immediately
same order. after Mathematics.
21. (e) Step II: 17 23 Set 48 pack situation 61 25 16 puzzle 28. (e) Mathematics student attends seminar from 9:30 am
bad
-11:00 am.
In step II, the sum of fourth element from left end and
third element from right end is 48 +16 = 64 29. (b) Computer student attends seminar just after the
22. (a) Step III: 17 23 61 Set 48 situation 25 16 pack puzzle bad Biology.
Element would be at the third from the left of the 30. (d) The duration (in hours) of the Mathematics’ seminar
element which is sixth from the right end in step III is is 1.5 hour
ninth from the right end i.e., ‘61’
Sol. (31-35) :
Sol. (23-25):
10 m Date
AAA GGG FFF 20th 25th
Month
Person City Person City
5m January R Agra W Faridabad
February T Banglore P Goa
10 m June X Bareilly V Pune
6m EEE September U Gaya Q Lucknow
DDD
December Y Srinagar S Meerut
5m 31. (d) S likes Meerut.
20 m 32. (b) One who goes on 25th February Like Goa.
BBB CCC 33. (c) U goes Gaya in September.
23. (b) Point FFF is in north east direction with respect to 34. (c) X goes in June month.
BBB. And north east is coded as @&. 35. (b) Four persons are going in between T and Q.
24.
Tg:- @NextGenBankers
(d) If point PPP is & 8m of point BBB then the shortest
distance between PPP and DDD
36. (c)
J(+) Q(–) L(+)
10 m
AAA GGG FFF
5m P(–)
10 m DDD 6m By option (c) we can denote ‘Q is mother of P’
37. (d) By option (d) E5000 < C3000 ≥ D4000 > B2000
5m ≥ A1000 = F6000
8m 12 m ‘C3000 > A1000’ holds true.
BBB PPP
Sol. (38-42):
52 + 122 = 13m
Vijay Ram Prakhar Tanu Quan Uday Shyam
25. (c) The direction of point EEE with respect to AAA is
south east which is coded as #&
35km 40km 45km 50km 55km 60km
Sol. (26-30):
Student Timing of Meeting Red Black Green Yellow White Pink Blue
English 7:30 am to 9:30 am
38. (e) Uday likes Pink colour.
Mathematics 9:30 am to 11:00 am
39. (d) Distance between Ram and the official who likes
Chemistry 11:30 am to 12:30 pm white colour is 135km.
Biology 12:30 pm to 2:30 pm 40. (d) Three offices are there between Prakhar and the
Computer 2:30 pm to 4:00 pm official who likes blue colour.
Physics 4:00 pm to 5:00 pm 41. (a) Prakhar sits to the immediate left of Tanu.
26. (c) Chemistry student attend the seminar between 42. (c) Tanu and the one who likes blue colour have distance
seminar hours 11:00 am to 12:30 pm. of 165km between them.
SBI Clerk (Junior Associates) Mains Solved Paper-2020 2020-17
(c) m2 – 12m + 36 = 0
62. r2 – 10r + 25 = 0
2 r = 5 meter
⇒ (m − 6) =0 ⇒ m =6 m The length of the side of cube C = 5 m.
The length of the side of cube A = 6 m. \ The length of the side of cube R = 5 + 3 = 8 m.
\ The length of the side of cube P = 6 + 1 = 7 m. The required ratio
n2 + 21n – 46 =0 = (6 × 62) : (6 × 82) {Total surface area of cube
2
⇒ n + 23n − 2n − 46 = 0 = 36 : 64 = 6 × (side)2}
⇒ n ( n + 23) − 2( n + 23) = 0 = 9 : 16
65. (e) The length of the side of cube A = 6 m
⇒ (n − 2)(n + 23) = 0 \ The length of the side of cube P = 6 + 1 = 7 m.
⇒ n = 2 m, n = −23 (Not possible) The length of the side of cube B = 2 m.
The length of the side of cube B = 2 m. \ The length of the side of cube Q = 2 + 2 = 4 m.
\ The length of the side of cube Q = 2 + 2 = 4 m. The length of the side of cube C = 5 m
2 \ The length of the side of cube R = 5 + 3 = 8 m.
r − 10r + 25 = 0
\ No cube has equal side length.
2
⇒ r − 5r − 5r + 25 = 0 66. (b) Books available in library P = (880 – 720) = 160.
Books available in library Q = (1040 – 880) = 160
⇒ r (r − 5) − 5(r − 5) =0
Books available in library R = (960 – 840) = 120
2
⇒ (r − 5) =0 Books available in library S = (840 – 640) = 200
Books available in library T = (1080 – 760) = 320.
⇒ r = 5 m
Total books in all the 5 libraries
The length of the side of cube C = 5 m. = (160 + 160 + 120 + 200 + 320) = 960
\ The length of the side of cube R = 5 + 3 = 8 m. 960
Average side length of cube A, Q and R Required average = = 192.
5
(6 + 4 + 8) 18 67. (c) Books available in library P, R and S together
= = = 6
3 3 = (160 + 120 + 200) = 480.
Average side length of cube P, B and C Books available in library Q and T together
= =
(7 + 2 + 5) 14
3 3
Tg:- @NextGenBankers = (160 + 320) = 480.
\ Required ratio = 480 : 480 = 1 : 1.
68. (d) Books available in library P and S together
Required percentage = (160 + 200) = 360
14 4 Books available in library Q and T together
6 − 200 4 = (160 + 320) = 480
3
= × 100 = 3 × 100 = = 28 % more. Required percentage
14 14 7 7
480 − 360
3 3 = = × 100 25% less.
480
63. (e) m 2 − 12m + 36 = 0
2 69. (d) Books available in library S = 200
⇒ (m − 6) ⇒ m = 6 m.
Books owned by library Q and R together
The length of the side of cube A = 6 meter = (1040 + 960) = 2000
So the length of the side of cube P = 6 + 1 = 7 meter 200
Volume of cube P and A together Required percentage = × 100= 10%.
2000
= 63 + 73 = 216 + 343 = 559 m3
2 70. (a) Total books owned by all the libraries together
n + 21n − 46 = 0
= (880 + 1040 + 960 + 840 + 1080) = 4800
2
⇒ n + 23n − 2n − 46 = 0 Total books issued by all the libraries together
⇒ n ( n + 23) − 2( n + 23) = 0 = (720 + 880 + 840 + 640 + 760) = 3840
Required Ratio = 4800 : 3840 = 5 : 4.
⇒ (n + 23)(n − 2) = 0
71. (d) Quantity I :
⇒ n = 2 m, n = −23 (Not possible) Average of four numbers = 28
The length of the side of B = 2 meter. \ Sum of four numbers = 28 × 4 = 112.
So, the length of the side of cube Q = 2 + 2 = 4 meter Let the number removed is a.
Volume of cube B and Q together Then new average = (28 – 2) = 26
3 3 3 \ Sum of three numbers = 26 × 3 = 78
=2 + 4 =8 + 64 =72 m 112 – a = 78
\ Required ratio = 559 : 72. \ a = 112 – 78 = 34.
64. (b) m2 – 12m + 36 = 0 Quantity II : 36
m = 6 meter. So, Quantity I < Quantity II.
The length of the side of cube A = 6 m.
SBI Clerk (Junior Associates) Mains Solved Paper-2020 2020-19
79. (e) Let the two digit number = ab = 10a + b 83. (d) The average temperature from Monday to Wednesday
Statement I : = 36°C.
a + b = 12 ...(1) Total temperature from Monday to Wednesday
Statement II : = 36 × 3 = 108°C.
10b + a = 10a + b + 36 Average temperature from Monday to Thursday
9 (a – b) = 36 = 36 + 1 = 37°C
a – b = 4 ...(2) Total temperature from Monday to Thursday
From eqn. (1) and (2), = 37 × 4 = 148°C
a + b = 12 So, temperature of Thursday = 148 – 108 = 40°C.
a–b=4 3 3
\ a = 8, b = 4. 84. (a) of 52 = 52 × = 39.
So, the value of two digit number is 84. 4 4
Hence both the statements are required to answer the 2 2
questions. of 99 = 99 × = 66.
3 3
80. (e) Statement I :
∴ 66 − 39 = 27.
2
m − 10m + 25 = 0 85. (a) Let the income of Mohit and Sohit is 4x and 3x
2 respectively and their expenditures are 3y and 2y
⇒ m − 5m − 5m + 25 = 0
⇒ ( m − 5)( m − 5) = 0 respectively.
\ 4x – 3y = 6000
2
and 3x – 2y = 6000
⇒ (m − 5) =0 ⇒ m =5.
Statement II : After solving it, x = 6000
2 Their income are 4 × 6000 = 24,000
n − 14n + 49 = 0 and 3 × 6000 = 18,000 respectively.
2 86. (b) Let each question carry 1 mark, so total mark = 70
⇒ n − 7 n − 7 n + 49 =
0
2 And passing marks = 60% of 70 = 42 marks.
⇒ (n − 7) = 0⇒n= 7 Number of questions answer correctly
So, both the statements are required to answer the = 70% of 10 + 40% of 30 + 60% of 30
question.
81. (b) Statement I :
Tg:- @NextGenBankers = 7 + 12 + 18 = 37 marks.
So, he has to answer 42 – 37 = 5 more questions
PQ = 16 cm, QR = 12 cm. correctly to get pass.
We can not conclude that which of the angle of triangle 87. (b) Let, cost of Banana and Mango = x / Banana or
is right angle by alone statement I. Mango
Statement II : Cost of Banana later = x × 1.04 / Banana
Let the sides of triangle are a, b and c. Cost of Mango later = x × 1.05 / Mango
a+b−c Initial value of 5 Mango and 4 Banana = 9x
= = 4 cm.
Inradius of triangle
2 Final value of 5 Mango and 4 Banana
Hypotenuse C = 5 (1.05x) + 4 (1.04x)
Circumradius of triangle= = = 10
C = 20 cm. 2 2 = 5.25x + 4.16x = 9.41x.
a + b = 28. So, percentage increase
We can calculate the value of a and b by pythagorean (9.41x − 9 x) 41
theorem, then we can calculate the area of right-angled = × 100 = = 4.55%.
9x 9
triangle.
88. (a) Case I :
So, statement II alone is sufficient to answer the
5 Boys and 3 Girls can be selected.
question but only statement I alone is not sufficient to
answer the question. Number of ways of selection
9 7
82. (e) There are 11 odd numbers in the group 1 - 21. = C5 × C3 = 126 × 35 = 4410.
The probability that the first mug is odd numbered Case II :
11 7 Boys and 1 Girl can be selected.
= .
21 Number of ways of selection
Since the mug is not replaced there are now 20 mug 9 7
left, of which 10 are even numbered. = C7 × C1 = 36 × 7 = 252.
The probability that the second mug is odd numbered Case III :
10 1 6 Boys and 2 Girls can be selected.
= ⇒ . Number of ways of selection
20 2 9 7
11 1 11 = C6 × C2 = 84 × 21 = 1764.
Required probability = × = .
21 2 42
SBI Clerk (Junior Associates) Mains Solved Paper-2020 2020-21
100. (a) Let the total population of India in 2019 = a million, Statement (III): To stay ahead of expectations,
then, 25% of a million = 200 million many organizations have begun initiatives to provide
a = 200 × 4 = 800 million remote employees a smooth working experience….
The population of India in 2020 Here, it is clear from the above quoted lines that
= 120% of 800 = 960 million statement I and III cannot be inferred from the given
The population of India in 2021 passage while statement II can be inferred. Hence,
= 110% of 960 million = 1056 million option (b) is the right answer choice.
In 2021 because of Interest 50% of the total population 105. (e) Significant: important or large enough to be noticed
of India will buy products online -having a particular meaning
= 50% of 1056 Momentous: Important, serious
50 × 1056 Hence, option (e) is the right answer choice.
= = 528 million. 106. (c) A ripple effect is the effect or influence of a situation,
100 action, event, etc. that does not stop but is experienced
Hence, option (a) is correct. on a series of things one after the other
101. (d) Refer to the first paragraph, the hint can be drawn Hence, option (c) is the right answer choice.
from the lines, 107. (b) Emphatic: expressing something forcibly and clearly.
Remote work is shaping up to be a major trend that will Hesitant: slow to speak or act because you are
likely keep growing even after the pandemic ends. uncertain
Now, the boundaries between personal and Hence, option (b) is the right answer choice.
professional lives have blurred. Juggling between 108. (e) Refer to the first paragraph, the hint can be drawn
professional and personal tasks with no leisure time from the lines,
can stress employees out and affect their mental A gig economy consists of people carving a career
health. If left unchecked, this stress can, in some path through freelance and contractual work –
cases, also increase an employee’s risk for heart wherein the individual is paid per job, or ‘gig’, as
diseases. opposed to receiving a regular salary or wage from
Here, Both (II) and (III) statement can be inferred an employer.
from the above quoted lines. Hence, option (d) is the Today, most people look at it as the much desired
right answer choice. antidote to a monotonous nine-to-five existence,
allowing them to work with freedom and flexibility.
from the lines.Tg:- @NextGenBankers
102. (d) Refer to the second paragraph, the hint can be drawn
111. (d) Refer to the third paragraph, the hint can be drawn 120. (d) “Housed” will fill up the blank with appropriate
from the lines, words to make the paragraph meaningful. Hence,
The future of Gig work holds great potential because option (d) is the right answer choice.
it tends to be more meaningful and rewarding in more Housed: to provide with living quarters or shelter
than just the monetary aspect. The simple fact that 121. (c) “Compel” will fill up the blank with appropriate
you are building your own brand instead of someone words to make the paragraph meaningful. Hence,
else’s makes this type of work intrinsically motivating option (c) is the right answer choice.
in the long run. Unlike traditional employment, 122. (e) “coincide” will fill up the blank with appropriate
gig work focuses on results instead of embroiling
words to make the paragraph meaningful. Hence,
processes. Gig workers or freelancers are evaluated
and paid based on their ability to deliver results. option (e) is the right answer choice.
With the above quoted lines, (a), (b), and (c) are 123. (a) “Prohibits” will fill up the blank with appropriate
justified. Hence, option (d) is the right answer choice. words to make the paragraph meaningful. Hence,
112. (e) Refer to the last paragraph, the hint can be drawn option (a) is the right answer choice.
from the lines, 124. (d) “Barred” will fill up the blank with appropriate
Also, the biggest advantage of gig work lies in its words to make the paragraph meaningful. Hence,
integration with technology because needless to say, option (d) is the right answer choice.
the future of India’s start-ups and its gig economy 125. (c) “Hurt” will fill up the blank with appropriate words
lies in digital innovations and a robust digitized to make the paragraph meaningful. Hence, option (c)
ecosystem. A major part of the gig economy, digital is the right answer choice.
marketplaces are now signing up demand-driven 126. (e) Show a clean pair of heels: To run or move away
service providers skilled in niche areas such as beauty, from someone at great speed.
fitness, plumbing, electrical repairs, etc. and offer
1. run away
these services to consumers on a contractual basis as
per requirements. This business model allows for the 2. get ahead of somebody in a competition
service providers to communicate with the end users Hence, option (e) is the right answer choice.
directly through the app according to their terms and 127. (a) Bury the hatchet: End the quarrel and make peace
conditions. This opens up a world of opportunities for In II, “smelled a rat” will be suitable which means
the freelance workforce. Suspect something foul
Tg:- @NextGenBankers
From the above quoted lines, only (a), (b), and (d) are
justified. Hence, option (e) is the right answer choice.
113. (b) Contingency: a future event or circumstance which is
In III, “read between the lines” will be suitable
which means Understand the hidden meaning.
Hence, option (a) is the right answer choice.
possible but cannot be predicted with certainty. 128. (d) Chew the cud: To contemplate something. (“Cud”
Possibility: the fact that something might exist or is partially digested food that is regurgitated to be
happen, but is not likely to chewed again, a common behavior of cows.)
Hence, option (b) is the right answer choice. In I, “A close shave” will be suitable which means A
114. (d) Disgruntled: Disappointed and appointed narrow escape from or avoidance of a situation, often
Aggrieved: upset or angry at being treated unfairly
an unfavorable or dangerous one.
Hence, option (d) is the right answer choice.
115. (c) Intrinsic: belonging naturally; essential. Hence, option (d) is the right answer choice.
Acquired: to obtain or buy something 129. (d) At sea: Puzzled, perplexed, or completely confused
Hence, option (c) is the right answer choice. (about a subject or some task at hand).
116. (c) “stimulus” will fill up the blank with appropriate In III, “meaning business” will be suitable. Mean
words to make the paragraph meaningful. Hence, business- To be grave and resolute; to be serious
option (c) is the right answer choice. about what one is promising or proposing to do.
Stimulus: something that causes activity, development Hence, option (d) is the right answer choice.
or interest 130. (e) Once in a blue moon: very rarely.
117. (e) “Buoyed” will fill up the blank with appropriate Hence, option (e) is the right answer choice.
words to make the paragraph meaningful. Hence, 131. (c) “Overall, Substantially” is the set of words for each
option (e) is the right answer choice. blank that best fits in the context of the sentence.
Buoyed: kept (something) at a high level Hence, option (c) is the right answer choice.
118. (c) “Upswing” will fill up the blank with appropriate
132. (b) “Contractual, Applications” is the set of words for
words to make the paragraph meaningful. Hence,
each blank that best fits in the context of the sentence.
option (c) is the right answer choice.
Upswing: an increase in strength or quantity; an Hence, option (b) is the right answer choice.
upward trend. 133. (d) “Abolition, Solitary” is the set of words for each
119. (a) “Batted” will fill up the blank with appropriate words blank that best fits in the context of the sentence.
to make the paragraph meaningful. Hence, option (a) Hence, option (d) is the right answer choice.
is the right answer choice. 134. (a) “Comparative, Inoculation” is the set of words for
Batted: to deliver a blow to (someone or something) each blank that best fits in the context of the sentence.
usually in a strong vigorous manner Hence, option (a) is the right answer choice.
GP_4458
2020-24 SBI Clerk (Junior Associates) Mains Solved Paper-2020
135. (e) “Relevant, Trajectory” is the set of words for each 146. (a) The International Day of Charity was established
blank that best fits in the context of the sentence. with the objective of sensitizing and mobilizing
Hence, option (e) is the right answer choice. people, NGOs, and stakeholders all around the world
136. (a) In the given sentence, the error is in part (A). While to to help others through volunteer and philanthropic
introducing oneself, possessive pronoun “myself” activities.
should not be mixed up with subject pronoun “I”. The date of 5 September was chosen in order to
Hence, option (a) is the right answer choice. commemorate the anniversary of the passing away of
137. (c) In the given sentence, the error is in part (C). Use Mother Teresa of Calcutta, who received the Nobel
‘overcrowded ‘ in place of ‘overcrowd’ as we need an Peace Prize in 1979 “for work undertaken in the
adjective to qualify the noun ‘transportation’. Hence, struggle to overcome poverty and distress, which also
option (c) is the right answer choice. constitute a threat to peace”.
138. (b) In the given sentence, the error is in part (B). Use ‘lay 147. (c) A wake-up call to economists who failed to foresee
down’ in place of ‘laid down’ as V1 will be used here every recent crisis, including the cataclysm of 2008,
to make the sentence grammatically correct. Hence, 10 Rules is full of insights on signs of political,
option (b) is the right answer choice. economic, and social change Sharma explains, for
139. (b) In the given sentence, the error is in part (B). ‘whose example, why autocrats are bad for the economy;
design’ is incorrect, it must be changed to ‘the design robots are a blessing, not a curse; and consumer prices
of which’ as whose is never used for non-living don’t tell you all you need to know about inflation.
things. Hence, option (b) is the right answer choice. He shows how currency crises begin with the flight of
140. (e) The given sentence is correct and there is no error in knowledgeable locals, not evil foreigners; how debt
any of its parts. Hence, option (e) is the right answer crises start in private companies, not government;
choice. and why the best news for any country is none at all.
141. (d) Axis Bank in association with Google Pay launched Rethinking economics as a practical art, 10 Rules
ACE Credit Card today. Axis Bank ACE Credit Card is a must-read for business, political and academic
is an entry level card which provides accelerated leaders who want to understand the most important
rewards on bill pay and recharge at Google Pay. This forces that shape a nation’s future.
card provides the reward in the form of cashback 148. (c) Bank of Baroda (BoB), India’s third-largest public
sector bank, has launched ‘Insta Click Savings
making it simple and hassle free.
Tg:- @NextGenBankers
142. (a) Airtel Payments Bank Ltd has collaborated with
National Skill Development Corporation (NSDC) to
Account’ for all its customers.
149. (a) SBI General Insurance, one of the leading general
insurance companies in India, announced the
train and skill young people in rural areas on financial
launch of first-of-its-kind offering, ‘Shagun – Gift
services, enabling them to find jobs and become
an Insurance”, a unique gift of Personal Accident
entrepreneurs in the sector, the payments bank said
policy. This product was filed by SBI General under
in a statement. Currently only six payments banks
Insurance Regulatory and Development Authority’s
are operational in India, including Paytm Payments (IRDAI) Sandbox regulations. The key differentiating
Bank, Airtel Payments Bank, India Post Payments feature of the policy is that it can be gifted to anyone
Bank, Fino Payments Bank, Jio Payments Bank and you want, which means it is not necessary for the
NSDL Payments Bank. policy buyer to be related to the insured1
143. (c) PD Vaghela was appointed as Chairman of TRAI 150. (d) ICICI Bank announced the launch of ‘Home Utsav’,
(Telecom Regulatory Authority of India). He has a virtual property exhibition that digitally showcases
replaced R S Sharma real estate projects by renowned developers from key
144. (c) Serum Institute of India Pvt. Lt (d) is now the cities across the country. The exhibition is available
world’s largest vaccine manufacturer by number for everyone, including ICICI Bank’s customers and
of doses produced and sold globally (more than those who are not customers of the Bank.
1.5 billion doses) which includes Polio vaccine as 151. (c) India’s largest private sector lender HDFC Bank
well as Diphtheria, Tetanus, Pertussis, Hib, BCG, recently launched “Shaurya KGC Card” aimed at
r-Hepatitis B, Measles, Mumps and Rubella vaccines. serving the armed forces. Unveiling the new product
It is estimated that about 65% of the children in the through the virtual platform, HDFC Bank managing
world receive at least one vaccine manufactured director (MD) and chief executive officer (CEO)
by Serum Institute Vaccines manufactured by the Aditya Puri said it is a first-of-its-kind product which
Serum Institute are accredited by the World Health is designed specifically for the armed forces.
Organization, Geneva and are being used in around 152. (a) American cult bike manufacturer Harley-Davidson
170 countries across the globe in their national recently said it is discontinuing its current business
immunization programs, saving millions of lives model in India. As part of the process, the company is
throughout the world. planning to close its manufacturing facility in Bawal
145. (c) Rajat Bhatia, a veteran in Indian domestic cricket, (Haryana) and significantly reduce the size of its
has called it quits from all forms of the game to bring sales office in Gurgaon, the bike manufacturer said in
down the curtains on a career spanning 20 years. a statement.
SBI Clerk (Junior Associates) Mains Solved Paper-2020 2020-25
153. (c) Jio Payments Bank Limited is a joint venture between 2012. The prime purpose of the International Day of
the Reliance Industries and the State Bank of India Charity is to raise awareness and provide a common
with the stake of 70:30. platform for charity related activities all over the
154. (a) Indian Air Force Day is celebrated every year by the world.
Indian Air Force on 8th of October. This year Indian 164. (b) In RRBs, 50% share shall be held by the central
Air Force celebrated its 88th anniversary. Indian Air government, 15% by the concerned state government
Force was officially established on 8 October 1932 and 35% by the sponsor bank. The RRB amendment
by the British Empire as the Royal Indian Air Force. act 2014 has allowed the RRBs to raise their
The name was changed to Indian Air Force in 1950. capital from sources other than the central and state
155. (a) The RBI, headed by Governor Shaktikanta Das, can governments, and sponsor banks.
have a maximum of four deputy governors. 165. (a) Ruchir Sharma is an Indian investor and fund
156. (b) The Central Board of Secondary Education (CBSE) manager who has written widely on global economics
and Facebook have partnered to launch curriculum and politics. A longtime columnist for newspapers
on "digital safety and online well-being" and
and magazines around the world, he is the author of
"Augmented Reality" for students and educators.
--The 10 Rules of Successful Nations.
157. (c) The Reserve Bank of India (RBI) came up with
166. (a) Titan Pay is powered by YONO SBI. Therefore, with
a framework for pan-India Umbrella Entity for
this launch, SBI account holders can tap their Titan
Retail Payments, wherein the central bank has said
that an entity must have a minimum paid-up capital Pay watch on contactless payment POS machine
of Rs 500 crore for setting up an umbrella entity without the need of swiping or inserting their SBI
for retail payment. bank card. Payments up to Rs. 2000 can be made
158. (b) Finance Minister Nirmala Sitharaman on February 1, without entering a PIN.
2019, while presenting the budget has announced the 167. (a) The Appointments Committee of the Cabinet has
expansion of Pradhan Mantri Kisan Urja Suraksha approved the appointment of PD Vaghela as the
Utthan Mahabhiyan (PM KUSUM) Scheme under new Chairman of Telecom Regulatory Authority of
which 20 lakh farmers will now be provided with India (TRAI). He will succeed Rs Sharma, who will
funds to set up standalone solar pumps. complete his tenure as the head of the telecom.
159. (a) India has been ranked 105th position “Global 168. (d) New Umbrella Entity (NUE) shall have a minimum
Tg:- @NextGenBankers
Economic Freedom Index 2020 Annual Report” by
Canada›s Fraser Institute, which has been released
paid-up capital of ` 500 crore (to support/address
the need of capital for managing risks, invest in
in India in conjunction with Center for Civil Society, technological infrastructure, for business operations,
New Delhi-based think tank. This is the 24th edition etc). No single promoter/promoter group shall have
of Economic Freedom of the World. more than 40% investment in the capital of the
160. (a) Dubbed as the biggest bureaucratic reform initiative, NUE.
the Union Cabinet approved ‘Mission Karmayogi’, 169. (a) UNDP has its headquarters in New York City,
a new capacity-building scheme for civil servants but works primarily through its offices in about
aimed at upgrading the post-recruitment training 170 countries and territories, helping to eradicate
mechanism of the officers and employees at all levels. poverty, reduce inequalities and exclusion, and build
Prime Minister Narendra Modi said this exercise resilience so that countries can sustain progress. As
will “radically” improve the government’s human the UN’s development agency, UNDP plays a critical
resource management practices and asserted it will role in helping countries achieve the Sustainable
use state-of-the-art infrastructure to augment the Development Goals.
capacity of civil servants. 170. (a) An asset management company (AMC) is a firm that
161. (c) Federal Bank Limited is a Private sector scheduled
invests pooled funds from clients, putting the capital
commercial Bank. It was founded in 23 April 1931
to work through different investments including
by Mr. K.P Hormis. Federal Bank main office as
stocks, bonds, real estate, master limited partnerships,
Registered and Head office is in Aluva, Kochi,
and more a long with high-net-worth individual
Kerala, India.
Federal Bank Provides Retail Banking, Wholesale portfolios.
Banking, Finance and Insurance, Mortgage loans. 171. (a) Atal Pension Yojana is regulated by the Pension Fund
162. (d) World Autism Awareness Day is celebrated on 2 April Regulatory & Development Authority Act (PFRDA).
every year, encouraging Member States of the United The Pension Fund Regulatory & Development
Nations to take measures to raise awareness about Authority Act was passed on 19th September, 2013
people with autistic spectrum disorders including and the same was notified on 1st February, 2014.
autism and Asperger syndrome throughout the world. PFRDA is subscribed by employees of Govt. of
163. (b) The International Day of Charity is an international India, State Governments and by employees of
day observed annually on 5 September. It was private institutions/organizations & unorganized
declared by the United Nations General Assembly in sectors.
GP_4458
2020-26 SBI Clerk (Junior Associates) Mains Solved Paper-2020
172. (c) The Micro Units Development & Refinance Agency Building (NPCSCB), in a bid to transform status-
Ltd (MUDRA) was set up by the Government of India quoist and rule-obsessed civil servants working in
(GoI). MUDRA has been initially formed as a wholly silos into “experts”.
owned subsidiary of Small Industries Development The scheme attempts to challenge an oft-repeated
bank of India (SIDBI) with 100% capital being criticism against civil servants of clearing an exam in
contributed by it. their 20s, and then doing little to build on their skills.
173. (a) Elon Reeve Musk is a business magnate, industrial 183. (a) The Dudhwa Tiger Reserve is a protected area
designer and engineer. He is the founder, CEO, CTO in Uttar Pradesh. It covers the Lakhimpur Kheri
and chief designer of Space X; early investor, CEO and Bahraich districts and comprises the Dudhwa
and product architect of Tesla. National Park, Kishanpur Wildlife Sanctuary and
174. (c) The NSO data revealed that retail inflation was 7.2 Katarniaghat Wildlife Sanctuary.
percent in rural India and 6.73 per cent in urban areas, 184. (c) Rani Lakshmi Bai Central Agricultural University
taking the combined CPI-based inflation to 6.93 is central agricultural university located in Jhansi
percent. district of Uttar Pradesh. The university was
175. (c) The merger of Oriental Bank of Commerce and established by the government of India through the
United Bank of India with Punjab National Bank Rani Lakshmi Bai Central Agricultural University
became effective from April 1, 2020. Act - 2014, passed by parliament in 2014.
176. (d) US secretary of state and defence secretary took part 185. (d) Atal Mission for Rejuvenation and Urban
in the two-plus-two dialogue with External Affairs Transformation (AMRUT) was launched by Prime
Minister S. Jaishankar and Defence Minister Rajnath Minister Narendra Modi in June 2015 with the focus
Singh. to establish infrastructure that could ensure adequate
A host of crucial bilateral, regional and global issues robust sewage networks and water supply for urban
were included -- China’s efforts to expand influence transformation by implementing urban revival
in the Indo-Pacific region as well as its aggressive projects.
behaviour in eastern Ladakh was to figure in the talks. 186. (d) The Ranapratap Sagar Dam is a gravity masonry
177. (d) The Korean won (KRW) is the national currency of dam of 53.8 metres height built on the Chambal
South Korea Its users denote the won by using the River at Rawatbhata in Rajasthan. It is part of
symbol “₩”. Since 1950, it has been administered by integrated scheme of a cascade development of the
Tg:- @NextGenBankers
the nation’s central bank, the Bank of Korea.
178. (a) The Prime Minister launched India Post Payments
river involving four projects starting with the Gandhi
Sagar Dam in the upstream reach in Madhya Pradesh
Bank (IPPB) on September 1, 2018. and the Jawahar Sagar Dam with a terminal structure
It is an initiative of the government aimed at making of the Kota Barrage in Rajasthan for irrigation.
banking services available at people’s doorstep. 187. (c) World Tribal Day is observed on August 9 every year
IPPB is a wholly-owned subsidiary of Deparment of from 1994.
Post, with 100 percent Government of India equity. The day is aimed at promoting and protecting the
179. (c) Indian IT pioneer Shiv Nadar cofounded HCL rights of the world’s indigenous population.
in a garage in 1976 to make calculators and The date recognizes the first meeting of the United
microprocessors. He chairs HCL Technologies, a Nations Working Group on Indigenous Populations
$9.9 billion (revenue) company. He is third richest in Geneva in 1982.
person in India . 188. (d) Total expenditure in 2020-21 is expected to be
180. (b) Automated Teller Machines (ATMs) set up, owned `30,42,230 crore, which is 12.7% higher than the
and operated by non-bank entities are called “White revised estimate of 2019-20.
Label ATMs” (WLAs). 189. (a) Dandiya Raas is the socio-religious folk dance
Tata Communications Payment Solutions Limited originating from Indian state of Gujarat and popularly
(TCPSL) was the first company authorized by the performed in the festival of Navaratri. The dance is
Reserve Bank of India (RBI) to open White Label performed in the Marwar region of Rajasthan too.
ATMs in the country. It got launched under the brand 190. (a) The World Bank has approved the Strengthening
name ‘Indicash’. Teaching-Learning and Results for States (STARS)
181. (a) Jio Payments Bank Limited is a joint venture between Programme. It will improve the quality and
the Reliance Industries and the State Bank of India governance of school education in six Indian states
with the stake of 70:30. of Himachal Pradesh, Kerala, Madhya Pradesh,
182. (b) The Narendra Modi government launched the Maharashtra, Odisha and Rajasthan through the
‘Karmayogi Yojana’ for Civil Services Capacity Samagra Shiksha.
SBI Clerk (Junior Associates) Prelim Solved Paper-2020
(Based on Memory)
Reasoning Ability 7. If Himani is 16m west of Ajay then, what is the shortest
distance between Himani and Edinit?
DIRECTIONS (Qs. 1-5): Study the following information care- (a) 11m (b) 12m
fully and answer the questions given below. (c) 13m (d) 14m
(e) None of these
Eight persons in XYZ company have different designations i.e,
8. Four of the following five are alike in certain way and hence
Chairman (CHM), General Manager (GM), Assistant General
form a group, find the one which does not belong to that
Manager (AGM), Deputy General Manager (DGM), Manager,
group?
Assistant Manager, Probationary Officer, Clerk in a company.
(a) Ajay-Faryad (b) Balaji-Edinit
The order of seniority is the same as given above i.e, CHM is the
(c) Dimple-Ajay (d) Chandra-Balaji
senior-most designation and Clerk is the junior-most designation.
(e) Dimple-Faryad
Only two persons are junior than Joy. Only one designation is
there between Joy and Omkar. Only two designation is there DIRECTIONS (Qs. 9-13): Study the following information care-
between Omkar and Mohan . Mohan is senior than Omkar. fully and answer the questions given below:
Number of persons who are senior than Omkar is same as the Eight cousins were born in four different months i.e. January,
number of persons who are junior than Prem. Nisha is senior March, June and September and on two different dates i.e., 8th
than Prem but junior than Lakshya. Kishore is not in the junior- and 15th of the same year.
most post. Qazi is not the PO. Shivam was born on 8th in one of the months having 30 days.
1. How many persons are junior than Omkar? Only three cousins were born between Shivam and Payal. Raj
(a) Three
(c) Two
(e) Four
Tg:- @NextGenBankers
(b) One
(d) None
and Tarun were born in the same month. Raj was born after Payal.
Uchit was born immediately before Payal. Number of cousin were
born before Raj is same as number of cousin were born after
2. How many designations are there between Nisha and Prem? Vinit. Qasim was born before Wasim.
(a) One (b) Two 9. Who among the following was born on 8th of January?
(c) Three (d) Four (a) Qasim (b) Raj
(e) Five (c) Payal (d) Tarun
3. Number of persons who are junior than Joy is the same as (e) None of these
the number of persons who are senior than ______ ? 10. How many cousins were born before Vinit?
(a) Mohan (b) Lakshya (a) One (b) Two
(c) Kishore (d) Nisha (c) Three (d) Four
(e) Omkar
4. Which of the following designations is exactly between (e) More than four
the designations of Qazi and Omkar? 11. Who among the following was born in the same month as
(a) Assistant Manager (b) Manager Uchit?
(c) PO (d) CHM (a) Wasim (b) Qasim
(e) AGM (c) Payal (d) Vinit
5. Who among the following is the Manager of the company? (e) None of these
(a) Omkar (b) Prem 12. Four of the following five are alike in a certain way and
(c) Joy (d) Lashya hence form a group which of the following does not belong
(e) Nisha to the group?
(a) Tarun-Vinit (b) Payal-Tarun
DIRECTIONS (Qs. 6-8): Study the following information care-
(c) Wasim-Tarun (d) Shivam-Raj
fully and answer the questions given below:
(e) Vinit-Uchit
Ajay is 14m west of Balaji. Chandra is 17m south of Ajay. Dimple 13. Who among the following was born between Tarun and
is 16m west of Chandra. Edinit is 30m north of Dimple. Gagan is Vinit?
23m south of Balaji. Faryad is 30m east of Edinit. (a) Raj (b) Qasim
6. What is the shortest distance between Faryad and Gagan? (c) Payal (d) Shivam
(a) 25m (b) 27m (e) None of these
(c) 36m (d) 28m
(e) None of these
GP_4458
2020-28 SBI Clerk (Junior Associates) Prelim Solved Paper-2020
14. If in the word “RESISTER” all vowels are changed by next 21. Statements:
letter and all consonants are changed by previous letter of A > B < C ³ M > E, E > G = I > N
English alphabetical series. Then. How many letters are
Conclusions:
repeated?
(a) None (b) One I. M > I
(c) Two (d) Three II.B ³ N
(e) None of these
DIRECTIONS (Qs. 22-24): Study the following series carefully
DIRECTIONS (Qs. 15-19): Study the following information care- and answer the questions given below.
fully and answer the questions given below:
657 824 523 487 348
Eight candidates are sitting around a circular table facing towards 22. The product of the first and third digit of which of the
the center of the table in front of the GD constructor in a bank for following number is the second highest?
the post of Deputy Manager. Priyam sits second to the left of (a) 487 (b) 657
Kajal. Only two candidate sit between Priyam and Usha. Tanvi
(c) 523 (d) 824
faces Usha. Rachit sits third to the right of Tanvi. Suraj sit third (e) 348
to the left of Wazir. Vandana is an immediate neighbor of Wazir. 23. The sum of all the digits of how many numbers is divisible
15. Who among the following candidate sits third to the left of by 3?
Suraj? (a) Two (b) One
(a) Rachit (b) Tanvi (c) Three (d) Four
(c) Priyam (d) Kajal
(e) Five
(e) Usha
24. If we arrange all the digits in decreasing order from left to
16. Four of the following five are alike in a certain way and
right within the number, then which of the following is
hence form a group find the one which does not belong to
third lowest number thus formed?
that group?
(a) Usha-Tanvi (b) Priyam-Rachit (a) 657 (b) 824
(c) Vandana-Suraj (d) Wazir-Kajal (c) 523 (d) 487
(e) Wazir-Suraj (e) 348
Tg:- @NextGenBankers
17. Which of the following statement is true regarding Wazir?
(a) Wazir faces Kajal
(b) Wazir sits immediate left of Vandana
DIRECTIONS (Qs. 25-29): In each of the questions below. Some
statements are given followed by conclusions/group of
conclusions. You have to assume all the statements to be true
(c) Vandana and Usha are immediate neighbours of Wazir even if they seem to be at variance from the commonly known
(d) Wazir sits second to right of Rachit facts and then decide which of the given conclusions logically
(e) All are true follow from the information given in the statements:
18. How many candidate sits between Priyam and Usha when
counted clockwise direction from Priyam? (a) If only conclusion I follows
(a) One (b) Two (b) If only conclusion II follows
(c) Three (d) None (c) If either I or II follows
(e) More than three (d) If neither I nor II follows
19. Who among the following candidate sits second to the (e) If both I and II follows
right of the one who faces Rachit? 25. Statement:
(a) Kajal (b) Tarun Only a few store is glass.
(c) Priyam (d) Suraj No glass is paint.
(e) Usha Only a few paint is carpet.
Conclusions:
DIRECTIONS (Qs. 20-21): In each of the following questions
I. Some carpet is not glass.
assuming the given statements to be true, find which of the two
II. All store can be paint.
conclusions I and II given below them is/are definitely true and
26. Statement:
give your answer accordingly.
Only a few press is pencil.
(a) If only conclusion I is true Some pencil is pen.
(b) If only conclusion II is true Only a few easy is pen.
(c) If either conclusion I or II is true Conclusions:
(d) If neither conclusion I nor II is true I. Some easy is not pen.
(e) If both conclusions I and II are true II. All pen can be pencil.
20. Statements:
27. Statement:
J ³ P > R = T, T < U ³ X Only a few angel is girl.
Conclusions: All girl is gang.
I. X > P Some gang is fruit.
II. J > T
SBI Clerk (Junior Associates) Prelim Solved Paper-2020 2020-29
DIRECTIONS (Qs. 46-50): Read the given information carefully becomes one-fifth then each students gets 25 candies. Find
and answer the following questions. the total number of candies distributed in the class?
The following bar graph shows the cost price (in Rs.) of four (a) 177 (b) 175
different products in 2018. (c) 196 (d) 174
(e) 198
53. Two trains running at 72 km/h and 36 km/h respectively in
opposite direction cross each other in 9 seconds then find
170 the time taken to cross each other when running in same
directions?
160
2
150 (a) 27 sec (b) 4 sec
25
140 4
(c) 18 sec (d) 4 sec
130 25
120 (e) 12 sec
P Q R S 54. P and Q entered into a partnership by investing Rs. 21600
Cost Price and Rs. 37800 respectively. After 6 months, R joined them
with an amount of Rs. 32400 and at the end of the year
46. Find the difference between sum of cost price of products profit share of R is Rs. 42000 then find the total profit?
P and Q together and that of R and S together? (a) Rs. 112000 (b) Rs. 168000
(a) Rs 20 (b) Rs 15 (c) Rs. 105000 (d) Rs. 182000
(c) Rs 10 (d) Rs 20 (e) Rs. 196000
(e) Rs 25 55. Five years ago, age of A’s father is 2.5 times of the age of A
47. Find the ratio of sum of cost price of P and S together to at that time and 5 years hence ratio of age of A’s father to
the cost price of P? that of A is 2:1. Find the sum of present age of A and his
(a) 56: 27 (b) 27: 56 father?
(c) 29: 56
(e) 56: 29
Tg:- @NextGenBankers
(d) 29: 27 (a) 65 years
(c) 80 years
(b) 50 years
(d) 70 years
48. If the cost price of product Q is increased by 40% in 2019 (e) 40 years
with respect to that of in 2018 then find the increment in the DIRECTIONS (Qs. 56-60): Each of the following question is
cost price of Q? followed by two equations I & II which you have to solve and
(a) Rs 90 (b) Rs 80 mark answer accordingly.
(c) Rs 60 (d) Rs 56
(a) if P £ Q
(e) Rs 50
(b) if P > Q
49. Sum of cost price of products Q and S together is approxi-
(c) if P = Q or no relation can be established
mately what percent more than the cost price of product Q?
(d) if P < Q
(a) 96% (b) 135%
(c) 125% (d) 90% (e) if P ³ Q
(e) 80% 56. I. 4x2 + 8x + 3 = 0
50. Average of the cost price of products Q and R is how much II. 2y2 + 3y + 1 = 0
more or less than the average of the cost price of products 57. I. 16x2 + 16x + 3 = 0
P and S? II. 9y2 + 27y + 20 = 0
(a) Rs 20 (b) Rs 10.5 58. I. 9x2 – 9x + 2 = 0
(c) Rs 15 (d) Rs 13 II. 9y2 – 18y + 8 = 0
(e) Rs 12.5 59. I. 49x2 + 14x – 3 = 0
51. Area of a square is equal to the area of a rectangle and II. 49y2 + 56y + 15 = 0
length of the rectangle is 4m more than the side of the 60. I. x3 = 512
square. Find the perimeter of the rectangle if the breadth of II. 2Q2 = 128
the rectangle is 3 m less than the side of the square? 61. Amount received on a certain sum when compounded an-
(a) 55 m (b) 50 m nually for 2 years at R% per annum is Rs 3240 and that of in
(c) 60 m (d) 40 m three years at the same rate of interest is Rs 3888. Find the
(e) 45 m sum?
52. There are some students in the class and number of candies (a) Rs. 2000 (b) Rs. 2250
received by each student is one-seventh of the total (c) Rs. 2400 (d) Rs. 2500
students in the class. If the number of students in the class (e) None of these
SBI Clerk (Junior Associates) Prelim Solved Paper-2020 2020-31
62. A alone can do a work in 30 days and ratio of efficiency of B 71. Trump has falsely asserted that (A)/he won the 2020 presi-
to that of A is 5: 3. If A started and worked for 10 days and left dential (B)/ election and repeatedly seek to (C)/ overturn
then in how many days will B do the remaining work? the results of the election. (D)
(a) 13 days (b) 24 days (a) A (b) D
(c) 12 days (d) 25 days (c) B (d) C
(e) 15 days (e) E
63. In an election of two candidates, 20% of votes cast were 72. UMI Blockchain is convenient if the (A)/ transfer needs to
declared invalid. The winner got 70% of the valid votes be (B)/sent urgent, and the (C)/ Internet is temporarily not
and defeats the loser by 3200 votes. How many votes were accessible. (D)
cast in total? (a) D (b) A
(a) 10000 votes (b) 13000 votes (c) E (d) C
(c) 12000 votes (d) 11000 votes (e) B
(e) None of these 73. They visited the site to gather (A)/first hand information
64. In an alloy ‘A’, tin & steel is in the ratio of 1 : 2. In the about the (B)/ incident and was informed that the (C)/fire
second alloy ‘B’, the same element are in the ratio 2 : 3. If was not related to Dzukou fire. (D)
these two alloys mixed to form a new alloy in which tin and (a) B (b) E
steel is in the ratio 3 : 5, find the ratio in which alloy ‘A’ and (c) C (d) A
alloy ‘B’ are mixed? (e) D
(a) 2 : 3 (b) 3 : 2 74. Having to repeal a law is not so (A)/easy especially for a
(c) 3 : 5 (d) 4 : 1 government (B)/which is extremely proud of (C)/its numeri-
(e) 3 : 1 cal majority in the Lok Sabha. (D)
65. A sells his phone at 25% profit to B while B sales it to C at (a) C (b) B
a loss of 20%. If C pays Rs. 3000. Find at what price A sold (c) E (d) A
phone to B? (e) D
(a) ` 2900 (b) ` 3200 75. A meeting of board of directors of the company (A)/ is
(c) ` 3600 (d) ` 3750 scheduled to be held in Tuesday, (B)/to consider and
(e) ` 3700 approve unaudited (C)/financial results of the company. (D)
(a) D (b) A
Tg:- @NextGenBankers
DIRECTIONS (Qs. 66-70): What should come in place of ques-
tion mark (?) in the following series questions?
(c) E
(e) C
(d) B
66. 9, 18, 32, 56, 95, ? 76. Once he wins, he has to figure out what (A)/he stands in,
(a) 153 (b) 148 gain the respect of his (B)/biggest critic, while trying to get
(c) 162 (d) 154 anything (C)/right for America's second weirdest city. (D)
(e) 143 (a) E (b) D
(c) B (d) C
67. 3, 10, 31, 94, 283, ?
(e) A
(a) 850 (b) 840
77. The agency also seems to be aware off the (A)/impact dam-
(c) 739 (d) 735
aged consumer trust can (B)/ have on personal health apps,
(e) 832 and said in the notice (C)/that its eye is on the rest of the
68. 4, 3, 4, 7, ? 38.5 industry as well. (D)
(a) 17 (b) 15 (a) D (b) C
(c) 13.75 (d) 22.25 (c) B (d) E
(e) 15.75 (e) A
69. 3, 11, 27, 59, ?, 251
(a) 159 (b) 118 DIRECTIONS (Qs. 78-82): In each of the following questions, a
(c) 115 (d) 175 statement with a single blank is given followed by five options.
(e) 123 Choose the most suitable word from the given options to com-
70. 12, 13, 40, 165, 508, ? plete the sentence meaningfully.
(a) 1328 (b) 1545 78. The primary purpose of a/an ___________ trust is that it
(c) 1237 (d) 1337 moves assets out of your estate, reducing your possibility
(e) 1431 of bumping up against the estate tax.
(a) Inoperable (b) Irrevocable
English Language (c) Reasonable (d) Usable
(e) Available
DIRECTION (Qs. 71-77): Read each sentence to find out whether 79. Health officials in the capital have warned that high levels
there is any grammatical error in it. The error, if any, will be in of pollution will _________ the effects of the pandemic.
one part of the sentence. Choose the option with that part as (a) Exert (b) Oppress
your answer. If there is no error, mark (E). (c) Contend (d) Exacerbate
(e) Toil
GP_4458
2020-32 SBI Clerk (Junior Associates) Prelim Solved Paper-2020
80. The Ghaziabad Traffic Police has launched operation 87. Which among the following will be the third sentence of the
"Nakel" in the district for disciplining errant motorists and paragraph after the rearrangement?
restore a ______________ of normalcy on district road. (a) F (b) D
(a) Disposition (c) E (d) B
(b) Notwithstanding (e) C
(c) Semblance DIRECTIONS (Qs. 88-92): In each question below, four words
(d) Esteem
printed in bold type are given. One of these words printed in
(e) Ascertain
bold might either be wrongly spelt or inappropriate in the con-
81. In recent years, _________________ has been the path-
text of the sentence. Find out the word that is inappropriate or
way used to enact or pursue highly partisan legislation
wrongly spelt, if any. The number of the word is your answer. If
(a) Reconciliation (b) Directions
the words printed in bold are correctly spelt and appropriate in
(c) Inclination (d) Conditioned
the context of the sentence then mark (e), i.e. 'All Correct', as
(e) Reaction
your answer.
82. A flood in my social housing block lays bare the
___________ of placing profit over people. 88. It has token no less than the highest court of the land to
(a) Early (b) Effectively reiterate the fact that the value of a woman's work at home
(c) Indefinitely (d) Folly is no less than that of her office-going husband.
(e) Legally (a) Token (b) Reiterate
(c) Woman's (d) Than
DIRECTIONS (Qs. 83-87): The question consists of six statements
(e) All are correct
labelled A, B, C, D, E and F which when logically ordered form
89. There are significant challenges faced by senior citizens in
a coherent passage. Choose the option that represents the most
the form of chronic illnesses and mental conditions in addi-
logical order.
tion to frequent ill-treatment by family members or
(A) It has professionally managed residences, golf courses, ten- caregivers.
nis courts and 10 miles of beach along the island. (a) Significant (b) Chronic
(B) That reputation only grew in the first nine months of the (c) Conditions (d) Members
virus. (e) All are correct
Tg:- @NextGenBankers
(C) The retreat has long been regarded a safe destination.
(D) In recent years, Kiawah Island has built a reputation as a
90. Learning may be priceless, but education for all cannot be
achieved by skimping on funds and if miserliness also
mecca for well-heeled families nationally. wishes to appear benevolent, the combination can be
(E) The distinctive features of Kiawah Island itself have also seriously damaging.
contributed to boosting its already-strong appeal during (a) Priceless (b) Cannot
the novel coronavirus period. (c) Skimming (d) Benevolent
(F) Collectively, it contributed to making social distancing readily (e) All are correct
manageable even before the term "social distancing" was 91. Public objection to the Jaitley statue was intermittent as a
invented. challenge to the ruling dispensation's higher standards of
83. Which among the following will be the fourth sentence of stadium nomenclature.
the paragraph after the rearrangement? (a) Objection (b) Intermittent
(a) D (b) C (c) Higher (d) Nomenclature
(e) All are correct
(c) E (d) A
92. The drama at the Capitol in Washington obscured the
(e) B resumption of the intra-Afghan dialogue with the second
84. Which among the following will be the second sentence of round between the Taliban to the government of
the paragraph after the rearrangement? Afghanistan beginning last week.
(a) B (b) C (a) Obscured (b) Dialogue
(c) E (d) F (c) To (d) Beginning
(e) A (e) All are correct
85. Which among the following will be the fifth sentence of the
DIRECTIONS (Qs. 93-100): Read the following passage and
paragraph after the rearrangement?
answer the following questions based on the passage. Some
(a) A (b) C
words are highlighted to help you locate while answering the
(c) F (d) B
(e) E questions.
86. Which among the following will be the FIRST sentence of Almost all sections of people including farmers agree that the
the paragraph after the rearrangement? Agricultural Produce Market Committee (APMC)-mandi policies
(a) B (b) D for agricultural marketing, initiated in the 1960s for a few crops,
(c) C (d) F have outlived their utility and the system needs a new policy in
(e) A the face of the agricultural sector's growth slowdown, the crop-
composition not widening, and investments in land not
SBI Clerk (Junior Associates) Prelim Solved Paper-2020 2020-33
happening. Recently, the government of the day has opened up rice belts in northern India are not protesting. Evidently so, since
the output market with the aim to let market forces improve the country is diverse with some 15 agroclimatic zones and has
efficiency and create more value for farmers and the economy. over 50 crops grown.
These laws state that farmers are now free to sell all their products 93. Why there is a need of new policy according to the given
anywhere and to anyone beyond the physical premises of APMC passage?
markets. Additionally, the laws promote contract farming through (i) Slowdown in the growth of agriculture sector
establishing partnerships between farmers and food-processing (ii) The composition of crop is not widening
companies, and also permit unlimited hoarding of food except in (iii) The APMC policies are no longer effective
special circumstances. (iv) There is a lack of investment on agricultural lands
Three main suggestions were put forth by farmers when we (a) i, ii, iv
recently made enquiries with them: one, their produce prices (b) ii, iii, iv
should be the cost of production plus a reasonable mark-up; two, (c) i, iii, iv
fluctuations in prices should be minimal; and three, there should (d) i, ii, iii
be little or no interface with legal or administrative officials - they (e) All of the above
are not comfortable dealing with the "sahibs and the police". All 94. What are the aspects the vegetable market should
address if government encourages farmers to move from
these farmer concerns have been ignored in the way the current
wheat to vegetable markets?
laws are drafted. Additionally, as the old laws are being repealed, (i) The production prices should include production cost
they said that there is a need for a wider view of the sector to with reasonable gain
include more crops. Thus, if the government encourages farmers (ii) Minimized fluctuations in prices
to move from wheat to vegetables, markets for the latter should (iii) Minimum or no dealing with legal or administrative
address all the above three aspects. officials
The first law of the Minimum Support Price-mandi is a known (iv) The farmers do not want to talk with the government
devil, but the new markets will be an unknown ghost with no (a) i, ii, iv
control over them by anyone. Thus, while "malpractices" in mandis (b) ii, iii, iv
are known and local leaders (Members of Parliament, Members of (c) i, iii, iv
the Legislative Assembly, panchayats) are often brought in to (d) i, ii, iii
____________ (I) farmers' anger or arbitrate in difficult situations, (e) None of these
malpractices in the new systems are neither forecast-able nor is 95. Why the author termed new markets 'an unknown ghost' as
Tg:- @NextGenBankers
there any authority to report to. Next, while the government says
that the mandi-MSP system will continue, the question is, for
mentioned in the given passage?
(a) The malpractices of mandis are known but there will
how long? If the alternative traders offer better prices, farmers be no one to control the new market
will go there and not to the mandis. What happens after two to (b) One cannot forecast malpractices of the new markets
three years when the regulated mandis become weaker or begin (c) Both (a) and (b)
to shut down due to lack of business? (d) The government claims that MSP will be revised
There are many issues here. Traders could reduce the prices on without anymore suffering of farmers
more than one pretext, such as finding faults with the product; (e) The alternative traders will not offer better prices for
declining to buy on the pretext of glut (a wait and watch strat- the produce
egy); defaulting on payments, and so on. Since traders are few (at 96. Which issues the author discusses if the regulated mandis
least locally) they can form cartels, while farmers many: this is shut down or get weaker?
imminently possible. The farmers are further handicapped by the (a) Traders can diminish the pricing on different aspects
fact that they come from long distances with loads of several like poor quality produce
quintals/tonnes of produce on hired tractors; going back owing (b) Payment defaulting can occur
to the transport cost incurred is not an option for them. Their (c) Few traders are likely to form associations and farmers
situation worsens when their cash needs are immediate, which is can't because they are large in number
the case for the small farmers who constitute 90-plus% of those (d) The farmers can't afford the cost spent in bringing the
who sell at the mandis at MSP. loads of several tones produce
The second law has somewhat similar issues. The corporate- (e) All of the above
buyers might just not buy the full quantity of the product on one 97. What are the issues in the second law?
or another pretext or delay payments; and if farmers complain, the (a) The corporate buyers are free to not buy the entire
corporates have access to a battery of lawyers, the fine print in quantity of the produce
contracts, the advantage of language, and, above all, the capacity (b) They have the privilege of finely printed contracts,
to wait it out. In both the above cases, the problem is of contracts language and huge pockets
between unequals: whether it is traders or corporates, they are far (c) The outcomes are uneven because the contract is be-
fewer and with deeper pockets, and they will deal with (poor/ tween unequal i.e poor farmers and affluent buyers
little-educated) small farmers (about 85% have two or less hectares (d) All of the above
of land), resulting in unequal outcomes. The other issue is in (e) None of these
regard to different regions and crops. Many proponents of the
agri-marketing laws maintain that farmers from outside the wheat-
GP_4458
2020-34 SBI Clerk (Junior Associates) Prelim Solved Paper-2020
98. Which of the following words can fit in the blank (I) to make it 100. Which of the following is most similar in meaning with IM-
a grammatically correct and contextually meaningful sentence? MINENTLY as highlighted in the given passage?
(a) Fare (b) Vain (a) Clearly (b) Computing
(c) Vent (d) Tyro (c) Approaching (d) Anxiously
(e) Foist (e) Impending
99. Which of the following is must opposite in meaning with
ALTERNATIVE as highlighted in the given passage?
(a) Substitute (b) Unusual
(c) Reserve (d) Compulsion
(e) Replacement
ANSW ER KEY
1 (e) 11 (b ) 21 (a) 31 (e) 41 (b ) 51 (b) 61 (b) 71 (d) 81 (a) 91 (b)
2 (a) 12 (b ) 22 (d ) 32 (b ) 42 (e) 52 (b) 62 (c) 72 (d) 82 (d) 92 (c)
3 (d ) 13 (a) 23 (a) 33 (c) 43 (d ) 53 (a) 63 (a) 73 (c) 83 (b) 93 (e)
4 (a) 14 (d ) 24 (b ) 34 (a) 44 (a) 54 (e) 64 (c) 74 (c) 84 (a) 94 (d)
5 (b ) 15 (c) 25 (a) 35 (d ) 45 (b ) 55 (c) 65 (d) 75 (d) 85 (a) 95 (c)
6 (c) 16 (e) 26 (b ) 36 (d ) 46 (b ) 56 (c) 66 (d) 76 (c) 86 (b) 96 (e)
7 (c) 17 (e) 27 (b ) 37 (e) 47 (e) 57 (b) 67 (a) 77 (e) 87 (c) 97 (d)
8 (b ) 18 (b ) 28 (c) 38 (b ) 48 (d ) 58 (a) 68 (b) 78 (b) 88 (a) 98 (c)
9 (a) 19 (a) 29 (b ) 39 (a) 49 (a) 59 (e) 69 (e) 79 (d) 89 (e) 99 (d)
10 (c) 20 (b) 30 (e) 40 (c) 50 (e) 60 (e) 70 (c) 80 (c) 90 (c) 10 0 (e)
Tg:- @NextGenBankers
ANSWERS & EXPLANATIONS
Sol. (1-5): Sol. (6-8):
Designations Persons
CHM Mohan Edinit 30m
Faryad
GM Laksya
13m
AGM Nisha
14m
DGM Omkar Himani Balaji
Ajay
Manager Prem
Assistant Manager Joy 17m
23m
PO Kishore 16m
Clerk Qazi Chandra
Dimple
1. (e) Four persons are junior than Omkar.
2. (a) Only one designations are there between Nisha and Gagan
Prem.
3 (d) Number of persons who are junior than Joy is the 6. (c) The shortest distance between Faryad and Gagan
same as the number of persons who are senior than (23m + 13m = 36m)
Nisha.
7. (c) If Himani is 16m west of Ajay then, the shortest
4. (a) Joy’s designations is exactly between the
distance between Himani and Edinit is 13m.
designations of Qazi and Omkar.
8. (b) Balaji-Edinit does not belong to that group because
5. (b) Prem is the Manager of the company. Edinit is north-west of Balaji while in other first is
south west of second.
SBI Clerk (Junior Associates) Prelim Solved Paper-2020 2020-35
Sol. (9-13): 24. (b) Given -657 824 523 487 348
Months Dates Persons After applying operation- 765 842 532 874 843
Hence, 824 is third lowest number thus formed.
January 8 Qasim
15 Uchit 25. (a) Store Glass paint carpet
March 8 Payal
15 Vinit Conclusions:
I. Some carpet is not glass. (True)
June 8 Raj II. All store can be paint. (False)
15 Tarun Hence, only conclusion I follows
September 8 Shivam
15 Wasim 26. (b) Press Pencil Pen Easy
9. (a) Qasim was born on 8th of January.
10. (c) Three cousins were born before Vinit. Conclusions:
11. (b) Qasim was born in the same month as Uchit. I. Some easy is not pen. (False)
12. (b) Except Payal-Tarun there is only one cousin is born II. All pen can be pencil.(True)
between the them. While there is two cousins born
between the Payal-Tarun. Hence, only conclusion II follows
13. (a) Raj was born between Tarun and Vinit. 27. (b) Gang
14. (d) Given Word- RESISTER
Fruit
After applying operation- QFRJRSFQ Angel
Q, F and R are repeated letters. Girl
Sols. (15-19):
Wazir Conclusions:
I. Some girl is fruit. (False)
Vandana
Tg:- @NextGenBankersUsha II. All fruits can be angel. (True)
Hence, only conclusion II follows
desk Wood
Tanvi Suraj
Kajal
Conclusions:
15. (c) Priyam sits third to the left of Suraj. I. Some wood is desk. (False)
16. (e) Except Wazir –Suraj all are sitting opposite to each other. II. No wood is desk. (False)
17. (e) All statement are true here regarding Wazir. But both are complimentary pair so, our answer is either I
18. (b) Two candidate sits between Priyam and Usha when or II follows .
counted clockwise direction from Priyam.
19. (a) Kajal sits second to the right of the one who faces 29. (b)
Rachit.
20. (b) I. X > P as, P > R = T < U ³ X so, (false) Risk Mute Case Toy
II. J > T as, J ³ P >R = T so, (true)
21. (a) I. M > I as, M > E > G = I > N so, (true)
II. B ³ N as, B < C ³ M > E > G = I > N so, (false)
22. (d) Given -657 824 523 487 348
In 824, 8 × 4 = 32 is the second highest. Conclusions:
23. (a) Given -657 824 523 487 348 I: No Risk is Toy. (False)
Sum is-18 14 10 19 15 II: Some Toy are Risk. (True)
Hence, 2 numbers are divisible by 3. Hence, only conclusion II follows.
GP_4458
2020-36 SBI Clerk (Junior Associates) Prelim Solved Paper-2020
30. (e) 15 31 a
45. (b) + + 8 = + 14
C O M P E T I T I V E 7 7 7
46 a
= +6
There are eight pairs CI, PT, PV, EI, TV, EI, IM, EM. 7 7
31. (e) There are five letters in the series i.e, ESQ, ERP, UTR,
AES , IUT 4 a
= , a =4
32. (b) E is 9th from left of ‘U’ letter. 7 7
33. (c) If all vowels are eliminated from the series then, 46. (b) Required difference = (165 + 135) – (145 + 140)
letter ‘X’ is 11th from right end. = 300 – 285 = 15
34. (a) None letters are there in the series like this.
145 + 135 280
47. (e) = = 56 : 29
35. (d) 145 145
A B E/F D F/E C
140
Hence, E is either third or fifth position left end so; our 48. (d) Required increment in cost price of Q = 40 ´
100
answer is can’t be determined. = Rs. 56 Þ 56 – 40 = Rs. 16
a (140 + 135) - 140
36. (d) ´ 700 + 249 = 760 49. (a) Required % = ´ 100 » 96%
100 140
7a = 760 – 249
(165 + 140) (145 + 135)
7a = 511 50. (e) Required difference = -
a = 73 2 2
37. (e) 121 – 81 + 37 = a + 27 = 152.5 –140 = Rs. 12.5
40 + 37 = a + 27 51. (b) Let the side of the square be x m
a = 50 Then the length of the rectangle = (x + 4) m
Breadth of the rectangle = (x – 3) m
512 40 a
38. (b) -
8 100
64 – 40 = a ´ 8
Tg:- @NextGenBankers
´ 100 = ´ 24
3
ATQ
x2 = (x + 4) ´ (x – 3)
x = 12 m
24 = a ´ 8 Length of the rectangle = 16 m
a=3 and breadth of the rectangle = 9 m
39. (a) a = 20,960 – 7960 perimeter of the rectangle = 2 (1 + b) = 2(16 + 9) = 50 m
= 13000 52. (b) Let the number of students in the class be x
60 324 x x
40. (c) 46 ´ ´ 60 - ´3 = a ATQ, x ´ = ´ 25
100 18 7 5
x2
46 ´ 36 – 18 ´ 3 = a = 5x
7
a = 1602
41. (b) a2 = 477 – 252 x = 35
= 225 35
a = ±15 Required number of candies = ´ 25 = 175
5
42. (e) 121 = a ´ 11 53. (a) Sum of the length of both the trains
11 = a ´ 11 5
= (72 + 36) ´ ´ 9
a=1 18
60 40 5
43. (d) ´a - ´ 500 = 400 = 108 ´ ´ 9 = 270 m
100 100 18
6 270
´ a = 400 + 200 Required time = = 27 sec
10 5
(72 - 36) ´
18
600 ´10 54. (e) Profit share ratio
a=
6 P Q R
a = 1000 21600 ´ 12 : 37800 ´ 12 : 32400 ´ 6
44. (a) a4 + 4 + 4a2 = 4a2 + 85 4 : 7 : 3
a4 = 81
a = ±3
SBI Clerk (Junior Associates) Prelim Solved Paper-2020 2020-37
1
Tg:- @NextGenBankers
(2Q + 1) (Q + 1) = 0 \ Time taken by B to that of A = 3 : 5
Q = - , -1 ì 1 ü
2 íefficiency µ ý
î Time þ
Clearly, no relation can be established. A alone can do the work = 30 days
57. (b) 16P2 + 12P + 4P + 3 = 0 30
Time taken by B to do the same work = ´ 3 = 18 days
(4P + 1) (4P + 3) = 0 5
1
A’s one day work =
1 3 30
P= - , - 1
4 4 B’s one day work =
18
II. 9Q2 + 15Q + 2Q + 20 = 0 10 1
A’s 10 days work = Þ
(3Q + 4) (3Q + 5) = 0 30 3
1 2
4 5 Remaining work = 1 - =
Q= - ,- 3 3 2
3 3 2
B will do the remaining work = 3 Þ ´18
Clearly, P > Q 1 / 18 3
58. (a) I. 9P2 – 6P – 3P + 2 = 0 Þ 12 days
(3P – 1) (3P – 2) = 0 63. (a) Let total votes cast be 100x
1 2 Valid votes = 100x – 20x = 80x
P= , Winner votes = 70% of 80x = 56x
3 3
II. 9Q2 – 12Q – 6Q + 8 = 0 Loser votes = 80x – 56x = 24x
ATQ
(3Q – 2) (3Q – 4) = 0
56x – 24x ® 3200
2 4 32x ® 3200
Q= ,
3 3 3200
100x ® ´ 100 = 10, 000
Clearly, P £ Q 32
GP_4458
2020-38 SBI Clerk (Junior Associates) Prelim Solved Paper-2020
3 ´2 ´2 ´2 ´2
8 70. (c) Missing number = 1237
12 + 13 = 13
13 + 33 = 40
2 3 3 1
- - 40 + 53 = 165
5 8 8 3 165 + 73 = 508
508 + 93 = 1237
1 1 71. (d) The error lies in part C. The sentence is in past tense.
Þ : Þ 3:5
40 24 The main verb should be in past participle or third
form. Instead of "seek", "sought" should be used. The
65. (d) Let C.P. of phone for A be Rs. 100 correct sentence will be,
Trump has falsely asserted that he won the 2020
æ 125 ö
ç Amount paid by B = 100 ´ = Rs.125 ÷ presidential election and repeatedly sought to
è 100 ø overturn the results of the election.
Amount paid by C Hence, option (d) is the right answer choice.
72. (d) The error is in part C. Here, the sentence talks about
80 transfer to be sent on an urgent basis and Internet is
= 125 ´
100 not accessible temporarily. In order to modify a verb, an
= Rs. 100 adverb is used (urgently) and not an adjective (urgent).
"urgently" should be used instead of "urgent". The
ATQ,
CP of phone for A =
Tg:- @NextGenBankers
3000
´ 100 = Rs.3000
correct sentence will be,
UMI Blockchain is convenient if the transfer needs to be
sent urgently and Internet is temporarily not accessible.
100 Hence, option (d) is the right answer choice.
Required price at which A sold to B 73. (c) The error is in part (C). The subject verb agreement of
this sentence is not correct. With plural noun 'they',
125 plural verb "were" should be used instead of "was".
= 3000 ´ = Rs. 3750
100 The correct sentence will be,
66. (d) Missing number = 154 They visited the site to gather first hand information
about the incident and they were informed that the
9 18 32 56 95 154 fire was not related to Dzukou fire.
Hence, option (c) is the right answer choice.
9 14 24 39 59 74. (c) There is no error in any part of the given sentence.
Hence, option (c) is the right answer choice.
5 10 15 20 75. (d) The error is in part B of the sentence. To specify a day
of the week, preposition "on" should be used instead
67. (a) Missing number = 850 of "in" in the given sentence. The correct sentence
3 ´ 3 + 1 = 10 will be,
10 ´ 3 + 1 = 31 A meeting of board of directors of the company is
31 ´ 3 + 1 = 94 scheduled to be held on Tuesday to consider and
approve unaudited financial results of the company.
94 ´ 3 + 1 = 283 Hence, option (d) is the right answer choice.
283 ´ 3 + 1 = 850 76. (c) The error is in part B of the given sentence. stand in
68. (b) Missing number = 15 means somebody is replacing someone on a tempo-
4 ´ 0.5 + 1 = 3 rary basis and this is not what the sentence intends to
3´1 +1=4 mean. Here, the sentence implies that he has to figure
out what he stands "for". The correct phrasal verb will
4 ´ 1.5 + 1 = 7 be "stand for".
7 ´ 2 + 1 = 15
15 ´ 2.5 + 1 = 38.5
SBI Clerk (Junior Associates) Prelim Solved Paper-2020 2020-39
The correct sentence will be, Interpreted: explain the meaning of (information or actions).
Once he wins, he has to figure out what he stands for, Hence, option (b) is the right answer choice.
gain the respect of his biggest critic, while trying to 92. (c) "and" should be used instead of "to". It is clear from
get anything right for America's second weirdest city. the word "between" which indicates that there should
Hence, option (c) is the right answer choice. be two parties or people for the "second round dia-
77. (e) The error is in part A of the sentence. "Aware off" is logue" and "to" is not giving any sense here.
the incorrect phrase, and it should be "aware of". The Hence, option (c) is the right answer choice.
correct sentence will be, 93. (e) Refer to the first paragraph, the hint can be drawn from
The agency also seems to be aware of the impact the lines,
damaged consumer trust can have on personal health Almost all sections of people including farmers agree
apps, and said in the notice that its eye is on the rest that the Agricultural Produce Market Committee
of the industry as well. Hence, option (e) is the right (APMC)-mandi policies for agricultural marketing,
answer choice. initiated in the 1960s for a few crops, have outlived
78. (b) Irrevocable: Not able to be changed, reversed, or re- their utility and the system needs a new policy in the
covered; final. face of the agricultural sector's growth slowdown,
Irreversible, Irremediable, Irreparable the crop-composition not widening, and investments
79. (d) Exacerbate: Make (a problem, bad situation, or nega- in land not happening.
tive feeling) worse. With the above quoted lines, read carefully, all the
Aggravate, Worsen, Inflame, Compound, Intensify statements are justified with the above lines. Hence,
80. (c) Semblance: The outward appearance or apparent form option (e) is the right answer choice.
of something, especially when the reality is different. 94. (d) Refer to the second paragraph, the hint can be drawn
Appearance, Approximation, Show from the lines, Three main suggestions were put forth
81. (a) Reconciliation: The restoration of friendly relations. by farmers when we recently made enquiries with
Reuniting, Reunion, Conciliation them: one, their produce prices should be the cost of
82. (d) Folly: Lack of good sense; foolishness. production plus a reasonable mark-up; two,
Evil, Wickedness fluctuations in prices should be minimal; and three,
83. (b) The correct sequence of the given sentences is there should be little or no interface with legal or
DBECAF.
Tg:- @NextGenBankers
Hence, option (b) is the right answer choice.
administrative officials - they are not comfortable
dealing with the "sahibs and the police".
84. (a) The correct sequence of the given sentences is Thus, if the government encourages farmers to move
DBECAF. from wheat to vegetables, markets for the latter should
Hence, option (a) is the right answer choice. address all the above three aspects.
85. (a) The correct sequence of the given sentences is All the three statements, i, ii, and iii are justified with
DBECAF. the above quoted lines. Hence, option (d) is the right
Hence, option (a) is the right answer choice. answer choice.
86. (b) The correct sequence of the given sentences is 95. (c) Refer to the third paragraph, the hint can be drawn
DBECAF. from the lines,
Hence, option (b) is the right answer choice. The first law of the Minimum Support Price-mandi is
87. (c) The correct sequence of the given sentences is a known devil, but the new markets will be an
DBECAF. unknown ghost with no control over them by anyone.
Hence, option (c) is the right answer choice. Thus, while "malpractices" in mandis are known and
88. (a) "Token" is wrong here. It should be "taken" instead. local leaders (Members of Parliament, Members of
Hence, option (a) is the right answer choice. the Legislative Assembly, panchayats) are often
89. (e) All the highlighted words are correct. Hence, option brought in to vent farmers' anger or arbitrate in
(e) is the right answer choice. difficult situations, malpractices in the new systems
90. (c) "Skimming" is wrong here. It's a reading technique are neither forecast-able nor is there any authority
meant to look for main or general ideas in a text, to report to.
without going into detailed and exhaustive reading. Both options (a) and (b) are justified with the above
The correct word will be "Skimping" which means quoted lines. Hence, option (c) is the right answer choice.
expending or using less time, money, or material on 96. (e) Refer to the fourth paragraph, the hint can be drawn
something than is necessary in an attempt to econo- from the lines, There are many issues here. Traders
mize. could reduce the prices on more than one pretext,
Hence, option (c) is the right answer choice. such as finding faults with the product; declining to
91. (b) "Intermittent" is wrong here in the context of the buy on the pretext of glut (a wait and watch strategy);
given sentence. instead, it should be "interpreted". defaulting on payments, and so on. Since traders are
Intermittent: occurring at irregular intervals; not con- few (at least locally) they can form cartels, while
tinuous or steady. farmers many: this is imminently possible. The farmers
GP_4458
2020-40 SBI Clerk (Junior Associates) Prelim Solved Paper-2020
are further handicapped by the fact that they come above quoted lines. Hence, option (d) is the right an-
from long distances with loads of several quintals/ swer choice.
tonnes of produce on hired tractors; going back 98. (c) "Vent" will fit in the blank (I) to make it a grammatically
owing to the transport cost incurred is not an option correct and contextually meaningful sentence. Hence,
for them. option (c) is the right answer choice.
Here, all the options (a), (b), (c), and (d) justify the Vent:
above quoted lines. Hence, option (e) is the right an- (a) give free expression to (a strong emotion).
swer choice. (b) the release or expression of a strong emotion, en-
97. (d) Refer to the fifth paragraph, the hint can be drawn ergy, etc.
from the lines, (c) an opening that allows air, gas, or liquid to pass
The corporate-buyers might just not buy the full out of or into a confined space.
quantity of the product on one or another pretext or Meaning given in (a) makes the word correct to fit in
delay payments; and if farmers complain, the suitably in blank (I)
corporates have access to a battery of lawyers, the Tyro : someone new to a field or activity
fine print in contracts, the advantage of language, Foist: Force into another
and, above all, the capacity to wait it out. In both the Hence, option (c) is the right answer choice.
above cases, the problem is of contracts between 99. (d) Alternative: (of one or more things) available as
unequals: whether it is traders or corporates, they another possibility or choice.
are far fewer and with deeper pockets, and they will Compulsion: the action or state of forcing or being
deal with (poor/little-educated) small farmers (about forced to do something; constraint.
85% have two or less hectares of land), resulting in Hence, option (b) is the right answer choice.
unequal outcomes. 100. (e) Imminently: Very soon
All the options (a), (b), and (c) are justified with the Impending: (of an event regarded as threatening or
significant) about to happen; forthcoming.
Hence, option (e) is the right answer choice.
Tg:- @NextGenBankers
SBI Clerk (Junior Associates) Mains Solved Paper-2019
(Based on Memory)
Reasoning Ability & Computer Aptitude DIRECTIONS (Qs. 6-10): Study the following information
carefully and answer the given questions.
DIRECTIONS (Qs. 1-5): Study the following information and A word and number arrangement machine when given an input
of words and numbers rearranges them following a particular rule.
answer the questions given below:
The following is an illustration of input and rearrangement.
There are three rows i.e. row (a), row (b) and row (c) such that Input: 44 74 TBEF WRAK 97 NLDG 31 67 KEOR SXCP
row (b) is in the north of row (c) and row (a) is in the north of row Step I: 92 44 74 TBEF NLDG 31 67 KEOR SXCP AKRW
(b). There are 4 writers sitting in row (a) and 8 writers are sitting Step II: 79 92 44 NLDG 31 67 KEOR SXCP AKRW BEFT
in the row (b) and 4 writers are sitting in the row (c). Step III: 62 79 92 44 NLDG 31 KEOR AKRW BEFT CPSX
Step IV: 49 62 79 92 31 KEOR AKRW BEFT CPSX DGLN
Writers sitting in the row (c) faces north. Writers sitting in the Step V: 26 49 62 79 92 AKRW BEFT CPSX DGLN EKOR
row (a) faces south. First 4 writers sitting from west to east in row (b) Step V is the last step of the rearrangement. As per the rules
faces north and last four writer sitting from west to east in row followed in the above steps, find out in each of the following
(b) faces south. questions the appropriate steps for the given input.
Note: All the writers sitting in the row (a) and row (c) are facing Input: IKGE 42 71 RTBC 86 PKCT 25 KDSM 59 VATW
the writers sitting in the row (b). 6. How many steps would be needed to complete the
E faces the one who sits second to the right of P. No one sits on arrangement?
(a) V (b) VII
the left of E. Only one writer sits between P and R. Only two
(c) VI (d) IV
writer sits between R and the one who faces F. D sits immediate (e) None of these
Tg:- @NextGenBankers
right of F. D does not sits at the end of the row. Q sits second to
the right of the one who faces D. A face the one who sits on the
7. Which step number would be the following output?
47 54 66 91 IKGE 25 ATVW BCRT CKPT DKMS
immediate left of Q. G faces S but does not sits at the end of the (a) II (b) III
row. P is not the immediate neighbour of G. Only one writer sits (c) V (d) IV
between K and S. K faces the one who sits third to the right of N. (e) None of these
J and M are immediate neighbours. J does not face D. Only two 8. Which of the following would be Step III?
(a) 54 66 91 IKGE 42 25 KDSM ATVW CKPT BCRT
writers sits between M and L. More than two writer sits between (b) 54 66 91 IKGE 42 25 KDSM ATVW BCRT CKPT
B and C, who does not face L. C does not face south. (c) 66 54 91 IKGE 42 25 KDSM ATVW BCRT CKPT
1. How many writers sit between A and G? (d) 54 91 66 IKGE 42 25 KDSM ATVW BCRT CKPT
(a) One (b) None (e) None of these
(c) Three ` (d) Two 9. Which of the following element would be the 4th to the left
(e) None of these of the one which is 8th from the left in the step IV?
2. Who among the following sits second to the right of C? (a) 91
(b) IKGE
(a) F (b) D (c) 66
(c) G (d) L (d) CKPT
(e) None of these (e) None of these
3. Four of the following five belongs to a group following a 10. In step V, which of the following element would be on eighth
certain pattern find the one that does not belong to that position from the right end?
group. (a) ATVW (b) BCRT
(a) ML (b) CB (c) 66 (d) 54
(c) RF (d) ED (e) None of these
11. In the word ‘RELATIONSHIP’ all consonants are written
(e) KS as their preceding letter and all vowels are written as their
4. Which among the following pairs sits at the ends of the following letters. Now all letters are arranged in alphabetical
rows? order from left to right and all the repeated letters are
(a) BS (b) EQ eliminated. Then, how many such pairs of letters are there,
(c) KG (d) RM each of which have as many letters between them in the
(e) None of these word (in both forward and backward direction) as they have
5. How many writers sit on the right of L? between them in the English alphabetical series?
(a) Seven (b) Six
(a) Three (b) One (c) Four (d) Three
(c) No one (d) Four (e) More than eight
(e) None of these
GP_4458
2019- 2 SBI Clerk (Junior Associates) Mains Solved Paper-2019
12. If we form the word by 1 st , 3 rd and 5 th letter of DIRECTIONS (Qs. 21-25): Study the following information
‘CELEBRATE’ and 1st and 4th letter of ‘ALIKE’ then, what carefully and answer the questions given below:
will be the 3rd letter from right?
(a) B (b) A Six students are sitting in a row. Some of them are facing North
(c) L (d) E and some are facing South. They are of different ages. Student
(e) None of these whose age is even numbered doesn’t sit immediate right of the
student whose age is even numbered.
DIRECTIONS (Qs. 13-15): Study the following information Two students sit between A and E and one of them sits at extreme
carefully and answer the questions given below. end. Three students are sitting between the students whose age is
P%Q (32)- A (42 m) is in north of Q 16 and 44. Student whose age is 17 sits 2nd to the left of E. There
P$Q (17)- A (27 m) is in south of Q are as many students sit between E and student whose age is 17 as
P#Q (18)- A (28 m) is in east of Q between the students whose ages are 17 and 30. B sits immediate
P&Q (19)- A (29 m) is in west of Q right of the student whose age is 17. One student sits between D and
R#S (20), U%R (21), B&U (23), P$B (41), Q#P (48), J%Q (16) F and neither of them sit at extreme end. B is older than C. E whose
13 U is in which direction with respect to Q? age is even numbered is older than F who faces North. Student whose
(a) North (b) West age is 22 sits 3rd to the left of the student whose age is 21.
(c) North-west (d) East 21. How many students are sitting between C and the student
(e) South-east whose age is 22?
14. If Z is the midpoint of the line formed between R and J, (a) One (b) Three
then what is the distance between S and Z? (c) Two (d) Four
(a) 35.5m (b) 34m (e) None
(c) 42.5m (d) 37.5m 22. Who among the following sits immediate right of F?
(e) None of these (a) D
15. What is the distance between S and J? (b) Student whose age is 21
(a) 50m (b) 51m (c) E
(c) 46m (d) 48m (d) Student whose age is 44
(e) None of these
(e) 55m
23. What is the position of B with respect to the 2nd youngest
DIRECTIONS (Qs. 16-20): In the following questions, the student?
as illustrated below-
Tg:- @NextGenBankers
symbols %, ®, #, × and @ are used with the following meaning (a) 2nd to the right
(c) Immediate right
(b) Immediate left
(d) 2nd to the left
‘A#B’ means ‘A is not greater than B’ (e) None of these
‘A×B’ means ‘A is neither equal to nor smaller than B’ 24. What is the age of the student who sits immediate left of D?
‘A%B’ means ‘A is not smaller than B’ (a) 16 (b) 17
‘A@B’ means ‘A is neither smaller than nor greater than B’ (c) 29 (d) 22
‘A®B’ means ‘A is neither greater than nor equal to B’ (e) None of these
Now in each of the following questions assuming the given 25. How many students are facing North?
statement to be true, find which of the conclusions given below (a) Two (b) Three
them is/are definitely true and give your answer accordingly. (c) One (d) None
(a) If only conclusion I follows. (e) More than three
(b) If only conclusion II follows. DIRECTIONS (Qs. 26-30): Study the following information and
(c) If either conclusion I or II follows. answer the given questions:
(d) If neither conclusion I nor II follows. In alphabetical series each consonant is assigned a different
(e) If both conclusions I and II follow. number from 1-6 (for ex- B is coded as 1, C-2……….H-6) and
16. Statements: U × V % W % X ® Y @ Z again those numbers get repeated (for ex- J-1, K-2…….so on).
Conclusions: I. U % X And for the codes of vowels is starts from the numeric code of Z
II. X ® Z i.e. if the code of Z is 2 then the code of vowel A is 2 and code of
17. Statements: A # B × C × D ® E E is 3 and so on till U which is coded as 6.
Conclusions: I. A × D Note: The code of vowels lies in the range of 1 to 7.
II. D ® B Besides the above information, following operations are to be
18. Statements: M % N × O % P; O × R % S applied for coding the words given in the questions below.
Conclusions:I. Q ® M If the two immediate digits are same (in the code) then the digit
II. S × N of the letter having higher place value in the English alphabet
19. Statements: B # D ® F # G × H % C will be changed to ‘®’ i.e. If the code of a word is ‘225’ hence the
Conclusions:I. B ® G code will be changed to ‘®25’ and if the two immediate digits (in
II. G @ B the code) are in the form such that the preceding digit is one more
20. Statements: J % K ® L # M; K × O @ P than its succeeding digit then the higher digit will be changed to
Conclusions:I. P ® M ‘©’ i.e. If the code of a word is ‘657’ hence the code will be
II. J × K changed to ‘©57’.
SBI Clerk (Junior Associates) Mains Solved Paper-2019 2019- 3
26. If the code for the word ‘FO_ _ED’ is coded as ‘462®©3’ 33. To provide proper knowledge to the student-in the education
then what letters will come in the fill in the blanks to make sector, we need to enhance our education system. To get
it a meaningful word? better education system, we need good teachers. To provide
(a) RM (b) AL good teachers, we again need good education.
(c) EM (d) AD In India there are some good institutes too but most of the
(e) Either (b) or (d) students go for a 9:00 to 5:00 job rather than trying the
27. Which of the following word will be coded as ‘426©2©3’? career in teaching field which requires post-graduation as
I. FLECKED II. TROAKED minimum eligibility criteria because they do not get a
III. FROSKED satisfied salary there.
(a) Only I (b) Only II Course of Action-
(c) Either II or III (d) Either I or III (I) The salary of the teachers should be raised to boost up
(e) All of the three them for opting teaching as career.
28. Which of the following combination of the words and codes (II) The minimum eligibility criterion to be a teacher should
is/are true? be graduate rather than post-graduation and PhD to
(a) BANKING - 13525®5 become a teacher as students can get a job even after
(b) ACHIEVE- ©2©54©4 graduation.
(c) BURNING- 172®5®5 (a) Only II follows (b) Only I follows
(d) DISPUTE- 35367®4 (c) Either I or II follows (d) Neither I nor II follows
(e) All are true (e) Both I and II follows
29. If the code for the words ‘HOPE YOU _____’ is coded as
‘6®®42676©3®462’ in the given coded pattern, then what DIRECTIONS (Qs. 34-38): Study the following information
will be the missing word? carefully and answer the questions given below:
(a) GOOD (b) HEALTHY Six persons are working in a Bank. Their designations are Client
(c) HAPPY (d) FINE Service Manager (CSM), Customer Loan Processor (CLP),
(e) Either (b) and (d) Mortgage Consultant (MC), General Manager (GM), Chief
30. What is the code for ‘PERFECTION’? Finance Officer (CFO), Finance Officer (FO). Sequence of the
(a) 5®425424©5 (b) 642®4245©5 posts are as above i.e. the post of Client Service Manager (CSM)
(c) 4264®2245©
(e) None of these
Tg:- @NextGenBankers
(d) 54©622®54® is higher than Customer Loan Processor (CLP), the post of
Customer Loan Processor (CLP) is higher than Mortgage
DIRECTION (Q. 31): Study the following information care- Consultant (MC) and so on the post of Chief Finance Officer
fully and answer the questions given below: (CFO) is higher than Finance Officer (FO). They have different
years of experiences. They get different annual salaries.
Eight girls are sitting in a row and all are facing North. Only two
Only two person’s designation is lower than the designation of
girls sit to the right of Aaliya. Two girls sit between Aaliya and
the person whose annual salary is 11 lakhs. There are as many
Bindu. Three girls sit between Cherry and Gauri. Erwin and Gauri
posts above the post of the person whose annual salary is 18 lakhs
are immediate neighbors of Bindu. Deepa sits immediate left of
as below the post of the person who have 5-year experience.
Erwin. Fida sits 2nd to the right of Hema.
31. How many girls are sitting right of Gauri? Annual salary of CSM is 10 lakhs. There are three posts in between
(a) Two (b) Three the post of the persons whose experience are 2 years and 3 years.
(c) Four (d) Five Person whose annual salary is 18 lakhs have just higher post than
(e) None of these the post of the person who have 4 years’ experience and just lower
32. Rooftop solar power growth has demonstrated an overall post than the post of the person who have 6 years’ experience.
positive trend, But this will need to be scaled up massively Two posts are between the posts of the person who have 1 years
to achieve the national target. and 6 years’ experience. Person whose annual salary is 9 lakhs
Assumption: have higher post than the person whose annual salary is 13 lakhs
(I) With ongoing improvements to solar cell efficiency and lower post than the person whose annual salary is 8 lakhs.
and battery technology, rooftops will only get more Person whose salary is 13 lakhs doesn’t have 2 years’ experience.
attractive in the future. 34. Who among the following has lowest salary?
(II) Major solar projects that connect to the grid often face (a) Person who is CLP
the challenge of land acquisition and transmission con- (b) Person who have 4 years’ experience
nectivity. (c) Person who is GM
(III) A survey helps determine usable rooftops, separating (d) Person who have lowest experience
them from green spaces, and analyses the quality of (e) None of these
the solar resource. 35. Person who is GM have how many years’ experience?
(a) Only II follows (b) Both II and III follows (a) 1 years (b) 2 years
(c) Both I and III follows (d) Only I follows (c) 3 years (d) 6 years
(e) Both I and II follows (e) None of these
GP_4458
2019- 4 SBI Clerk (Junior Associates) Mains Solved Paper-2019
36. How many persons are senior than the person whose annual Toffees and container P. container X is kept at the odd number
salary is 18 lakhs? position but immediately below container U. There is only one
(a) None (b) One container is kept between container T and container having 25
(c) Two (d) Three Toffee. container S is kept immediately above the container
(e) More than three containing 63 Toffees. container T does not contain 63 Toffees.
37. Which of the following is the annual salary of the person The number of Toffees in container V is equal to the difference
who is CFO? between the number of Toffees in container S and container X.
(a) 8 lakhs (b) 9 lakhs container Q is kept above the container having 8 Toffees. container
(c) 11 lakhs (d) 13 lakhs V contains 21 less Toffees than container P contains. Only three
(e) None of these containers are kept between container S and the container
38. Which of the following pair of combination is/are true? containing 12 Toffees. Only two containers are kept between
(a) CLP - 8 lakhs (b) GM – 5 years container W and the container having 12 Toffees. More than three
(c) MC – 6 lakhs (d) CFO – 9 lakhs containers are kept between Box-container R and container W.
(e) None is true The container having Toffees which is a perfect square of 3 is
DIRECTIONS (Qs. 39-40): There are some criteria which will kept immediately above container R. The number of pens in
be considered during appraisal of employee-Statement by container S is equal to the sum of the number of Toffees in
manager of a company. container W and the container which is placed at 2nd position.
41. Which among the following box-container 42 Toffees?
• Punctuality and sincerity are one of the key points which (a) Container S (b) Container W
will be surely reviewed. (c) Container Q (d) Container T
• Hard work and dedication towards work will be applauded. (e) Container X
• Just to quantity an amount in the salary package of employee 42. How many containers are kept above container U?
with no reason will not be entertained. (a) Three (b) Two
39. Which of the following can be inferred from the give (c) Four (d) Five
statement? (e) None of these
I. Punching machine of office will be evaluated by HR. 43. Number of container between X and the container having
II. There will be minimum 10% increment for deserving 63 Toffees is same between the container Q and the
employee based on their performance. container?
III. Some of employees of company have done a tremen-
(a) Only I
Tg:- @NextGenBankers
dous job with complete determination and enthusiasm.
(b) Both II and III
(a) containers V
(b) 42 Toffees
(c) 81 Toffees
(c) Both I and III (d) Only III
(d) containers R
(e) None of these
(e) Both (a) and (c)
40. Which of the following undermines the statement given by
44. Container U contains how many Toffees?
Manager?
(a) 25 (b) 42
I. Increment of 15% has been done for all the employees
whose 1 year is completed. (c) 63 (d) 81
II. Performance chart has been prepared by leaders of (e) None of these
different department based on their proficiency. 45. Total number of Toffees in the container P, V and X is?
III. Relaxation time for late coming of 240 minutes in a (a) 153 (b) 149
year i.e. of 10 minutes twice in a month is given to all (c) 155 (d) 151
employees. (e) None of these
(a) Only II (b) Both II and III 46. Begusarai, a district in Bihar which has seen the phase of
(c) Both I and III (d) Only I kidnapping, extortion, murder to “The Industrial city of
(e) None of these Bihar”. Nowadays there are 4 major industries in Begusarai.
Growth rate of Begusarai is top amongst all districts of Bihar
DIRECTIONS (Qs. 41-45): Study the following information and and 5 th among all districts of India in year 2017.
answer the questions given below: Which of the following can be inferred from the above
There are nine container which are kept one above the other such statement?
that the container which is placed at the bottom most position is I. Opportunities of the employment has been increased
numbered 1 and so on till the container which is placed at the top in Begusarai.
position is numbered 9. All the Containers contain different II. There is no case of murder, kidnapping and extortion
number of Toffees in it. in 2017.
Note: The number of Toffees in a container is equal to the multiple III. People of Begusarai are hardworking and keen to go
of the place number of the container which is kept forward.
immediately above it i.e. The number of Toffees in the IV. Top industrialists of India are desirous to have their
container which is kept at the bottom is equal to the multiple of is industry in Begusarai.
2, 4, 6… and so on and the number of Toffees in the container (a) Only II (b) Only I and III
which is placed at the top (9th position) is 10, 20, 30 and so on. (c) Only I (d) Only III and IV
Only two containers are kept between the container having 42 (e) None of these
SBI Clerk (Junior Associates) Mains Solved Paper-2019 2019- 5
47. Statement: Start to think of travelling by train for a holi- Total fare = Base fare + Taxes/Surcharge + Convenience fee
day. A train journey can give one a better view of places on Taxes/Surcharge = 25% of Base fare
the way which an air journey cannot give. You can walk Convenience fee for one – way = 10% of Base fare + 20%
around whenever you want, meet other travelers and lo- of Taxes/Surcharge
cals, relax and watch the landscape go peacefully by. Train Base fare for round trip = One – way base fare + `908
travel is both a beautiful and affordable way to see the coun- Convenience fee for round trip = 15% of Base fare + 30%
try, and no trip is more scenic than the trip by trains. of Taxes/Surcharge
Conclusions: Base fare for each airways is different and based on the distance
I. While going for a holiday, people want to enjoy the between origin and destination. Table given below shows the
view of the places on the way. base fare calculator of those different airways:
II. People should not travel by air when they are going
for a holiday. Base fare (If distance between origin
Airways
Which of the following can be concluded from the given and destination is D)
statement? AA 2D + 500
(a) Only I follows (b) Only II follows BB 3D – 1000
(c) Both I and II follow (d) None follows
CC 36% of (2D + 3500)
(e) Either I or II follows
DD 30% of [3.5D + (2D/3)]
DIRECTIONS (Qs. 48-50): Study the following information 2
carefully and answer the questions which follow– EE 80% of (0.01D – 0.75D)
‘E @ F’ means ‘E is parent of F (either mother or father)’ 51. If Akshay goes from Delhi to Patna by airways AA and
‘E # F’ means ‘E is sister of F’ comes back by airways BB and distance between Delhi and
‘E $ F’ means ‘F is grandchild of E’ Patna is 1200 km, then what is difference between his total
‘E % F’ means ‘E is brother of F’ fare from Delhi to Patna and that from Patna to Delhi?
‘E & F’ means ‘E is the son-in-law of F’ (a) `420 (b) `380
‘E * F’ means ‘E is the wife of F’ (c) `640 (d) `560
‘E © F’ means ‘F is the mother of E’ (e) None of these
There are some members in a family having three generation. 52. Two persons Manu and Sindhu goes from Bhopal to Pune
defined as follows.
Tg:- @NextGenBankers
The relation between the different members of the family are for some office meeting and travel expense is given by the
company. Manu goes by BB and Sindhu goes by CC, then
what is the total reimbursement made by the company to
Q@U#T
X&S both the persons if the distance between Bhopal to Pune
V%W*X is 750 km?
R@W (a) `3580 (b) `4270
U*V%W©S (c) `4830 (d) `3670
T©P$M (e) None of these
R$Y 53. What is the difference between total fare amount for Sapna
48. If M is the only child of V, then how is M related to Q? and that for Rajat when both travels round trip between
(a) Son (b) Grandson Chennai and Bangalore. Sapna goes by AA and comes
(c) Daughter (d) Grand daughter back by BB while Rajat goes by CC and comes back by
(e) Can’t be determined the same airways? Assume distance between Chennai and
49. If P has only one son, then how is T related to V? Bangalore is 600 km.
(a) Son (b) Brother-in-law (a) `225 (b) `445
(c) Brother (d) Father (c) `335 (d) `285
(e) None of these
(e) None of these
54. Salman goes from Shrinagar to Ajmer for vacation by
50. If M has no sibling and R has no granddaughter, then how
airways CC and comes to Shrinagar by airways DD. If
is Y related to X?
distance between Shrinagar to Ajmer is 2000 km, then fare
(a) Daughter (b) Wife
from Shrinagar to Delhi is what per cent more than that
(c) Nephew (d) Son
from Ajmer to Shrinagar?
(e) Cannot be determined
(a) 12% (b) 8%
Quantitative Aptitude (c) 16%
(e) None of these
(d) 18%
DIRECTIONS (Qs. 51 - 55): Study the following data carefully 55. Uday goes from Kolkata to Hyderabad by airways EE and
and answer the questions: total fare amount for Uday is `3528, then what is the
distance between Hyderabad and Kolkata?
There are total five airways AA, BB, CC, DD and EE by which
(a) 700 km (b) 550 km
a person can book his flight ticket. The fare of a airways
(c) 625 km (d) 600 km
depends on base fare, taxes/surcharge and convenience fee as
(e) None of these
given below:
GP_4458
2019- 6 SBI Clerk (Junior Associates) Mains Solved Paper-2019
56. Arka Pratap divides a certain number by 5, 7 and 8 Total marks in the exam = Normalized marks in Math + Normalized
successively, the remainders are 2, 3 and 4 respectively. marks in Science + Normalized marks in English
What would be the remainder if the order of the division Total questions in the test are 100 out of which 35 are from Math
is reversed? section, 35 from Science section and remaining 30 are from
(a) 6, 5 and 2 (b) 5, 5 and 2 English section.
(c) 4, 5 and 2 (d) 2, 5 and 4 Table given below shows the ratio of number of Right answers to
(e) None of These the number of Wrong answers marked by five different students
57. Vijay, Ajay and Sanjay enter into a partnership. Vijay Abha, Banty, Charak, Debu and Enna in all the three sections:
1 Ratio (Rig ht Ratio (Right Ratio (Right
contributes of the capital for half of the time. Ajay
4 S tudents : W rong) in : W rong ) in : W rong) in
1 1 Math S cience Eng lis h
advances of the capital for of the time. Sanjay
5 4 A bh a 6:1 5:3 3:1
contribute the remaining capital for the whole time. Find Ban ty 15:1 5:1 7:3
the share of Ajay and Sanjay together in the profit of Ch arak 5:2 3:2 4:3
` 17,400?
(a) ` 14,400 (b) `16,500 Debu 1:1 4:1 17:7
(c) ` 6,000 (d) ` 14,700 Enn a 2:1 5:3 3:1
(e) None of These 61. If Abha attempts 80 questions and ratio of attempted
58. The ratio between the present ages of Sanjeet and Manjeet questions in Math, Science and English is
is 5:3 respectively. The ratio between Sanjeet’s age 4 years 7 : 8 : 5 respectively, then what is the total mark obtained
ago and Manjeet’s age 4 years hence is 1:1. What is the
by her in the test?
ratio between Sanjeet’s age 4 years hence and Manjeet’s
age 4 years ago? (a) 252.8 (b) 272.9
(a) 1:3 (b) 2:1 (c) 264.7 (d) 246.4
(c) 3:1 (d) 4:1 (e) None of these
(e) None of These 62. If Banty left 19 questions in Math, 23 questions in Science
Tg:- @NextGenBankers
59. What is the probability that when 2 dice and 4 coins are
thrown simultaneously, there is a sum of 7 on the dice and
at least 2 heads on the coins?
and 20 questions in English un-attempted, then what will
be the ratio of sections marks of him in Math, Science and
English respectively?
13 11 (a) 127: 69: 38 (b) 87: 39: 28
(a) (b) (c) 27: 9: 13 (d) 135: 89: 58
96 96
(e) None of these
12 11
(c) (d) 63. If Charak attempted, 60% of total Math questions in Math,
169 125
3
(e) None of These 71 % of total questions in Science and 70% of total
60. Two passanger trains of length 500 m and 750 m have the 7
questions in English, then total right answers are what
speeds of 33 km/hr and 60 km/hr respectively. In what time
will the trains be able to completely pass each other, given percent more than total wrong answers marked by him?
that the trains are moving in the same direction with the faster (a) 56% (b) 73%
train approaching the slower? (c) 64% (d) 68%
(a) 200 sec. (b) 3 min. 2 sec. (e) None of these
(c) 180 sec. (d) 166.67 sec. 64. Students Debu marked 15 questions in Math, 20 questions
(e) None of These in Science and 17 questions in English correctly, then what
DIRECTIONS (Qs. 61 - 65): Study the following data carefully will be the ratio of normalized marks obtained by him in
and answer the questions: Math, Science and English respectively?
An exam in conducted that consists of three sections: Math, (a) 3: 2: 4 (b) 1: 3: 2
Science and English. Sectional marked can be calculated as (c) 3: 1: 2 (d) 1: 2: 3
mentioned below: (e) None of these
Sectional marks = 5 *Right answers – 2* Wrong answers – 0.5 * 65. If the ratio of number of attempted questions by Enna in
Un-attempted questions Math, Science and English is 6: 4: 5 respectively and total
Normalized marks in any section (If Right answers are ³ 80%) marks obtained by him in the test is 147, then how many
= 1.2 * Sectional marks did he left unattempted in the test?
Normalized marks in any section (If Right answers are ³ 60% (a) 50 (b) 40
and < 80%) = 1 * Sectional marks (c) 45 (d) 75
Normalized marks in any section (If Right answers are < 60%)
(e) None of these
= 0.8 * Sectional marks
SBI Clerk (Junior Associates) Mains Solved Paper-2019 2019- 7
66. Kalraju can complete a piece of work in 30 days. 70. There are total ________ employees in a office. Some
Efficiency of J. Murthy is _______ times the efficiency of employes work only in Hindi, some work only in English
Kalraju. Kalraju and Mithun together can complete the and some work on both 80% of the employees work in
work in 20 days. If efficiency of Riyas is 1.5 times the Hindi, 45% of the employees work in English. Number of
efficiency of Mithun, number of days taken by J. Murthy employees who work in both the subjects is _________.
and Riyas to complete the work is __________. Which of the following option/options satisfy the given
Which of the following option/options satisfy the given condition?
condition? (a) 800, 200 (b) 1200, 400
I. 1.5, 40/3 (c) 1000, 200 (d) All of the above
II. 2, 15 (e) None of the above
III. 3, 8
DIRECTIONS (Qs. 71- 75): Study the following information
(a) Only I and II
(b) Only I and III carefully and answer the questions given below.
(c) Only II and III Market share of various companies in different product categories
(d) All I, II and III (sales value) in 2018
(e) None of these Washing Machine Audio System Other Products
67. Arjun and Banita together can complete the work in 24 Akai 30% HTC 19 % Sony 20%
days. Arjun and Chahak together can complete a piece of BPL 26% BPL 38% BPL 31%
work in 32 days and with the help of Dablu they can IFB 44% Xiomi 3% Samsung 15%
complete the work on 24 days. Efficiency of Banita is two —— —— Godrej 40% IFB 34%
times the efficiency of Dablu.
LG sales in product categories 2018
From the above statement which of the following can be
determined. Category Sales (Rupees in crores)
(I) Arjun alone complete the work Washing Machine 337
(II) Chahak alone complete the work Audio System 190
(III) Chahak and Dablu together can complete the work Others 388
(IV) Banita alone complete the work 71. By what percentage was the sales turnover of Akai greater
(a) All I, II, III and IV than Godrej in 2018? Assume the companies do business
only in the product categories shown in the chart.
(b) Only I and III
(c) Only II Tg:- @NextGenBankers
(d) Only I, III and IV
(a) 60%
(b) 72%
(e) Cannot be determine (c) 80%
68. Two trains Taj mail and JP mail in the length of 350 m (d) 95%
and 250 m respectively and crosses the pole in 21 seconds (e) None of These
and 10 seconds respectively. 72. If in 2017 the market share of the various brands in all the
From the above statement which of the following can be product categories was exactly similar to the market share
determined. in 2018, then what was the turnover of IFB in 2017?
(I) Time taken by train Taj mail crosses the train JP mail (Rupees in crores)
running in opposite direction (a) ` 1300 (b) `1925
(II) Length of the bridge, If Train Taj mail crosses the (c) `1309 (d) `1280
bridge in 36 seconds (e) Cannot be determined
(III) Time taken by Train JP mail crosses a man running in 73. The sale of BPL Audio systems increased by 20% every
same direction at the speed of 30 kmph. year since 2015, but its market share declined by 1% point
(IV) Ratio of speed of train Taj mail to train JP mail every year since 2015. What was the total Audio system
(a) All I, II, III and IV (b) Only II and IV market worth in 2015?
(c) All I, II and IV (d) Only II (a) 260 crore (b) 274 crore
(e) Cannot be determine (c) 268 crore (d) 245 crore
69. If the volume of the cuboid is 2016 cm3 and the radius and (e) None of These
height of the cone is equal to the breadth and length of the 74. If the sales of the BPL Washing machine market grow by
cuboid. The ratio of the length, breadth and height of the 10% every year since 2015, what were the sales of BPL
cuboid is 24:7:12. from Wahing machine market in 1995 (in rupees in
From the above statement which of the following can be millions)?
determined. (a) `2500 million (b) `2000 million
(I) Slanting height of the cone (c) `243 million (d) `253 million
(II) Diagonal of the cuboid (e) None of These
(III) Volume of the cone 75. Find the market share of MI Wahing machine in the
(IV) Surface area of the cuboid Wahing machine category in the year 2018?
(a) All I, II, III and IV (b) Only I (a) 500 crore (b) 400 crore
(c) Only I and IV (d) Only IV (c) 700 crore (d) Data Inadequate
(e) Cannot be determine (e) None of These
GP_4458
2019- 8 SBI Clerk (Junior Associates) Mains Solved Paper-2019
DIRECTIONS (Qs.76-80): Following questions contain two 83. (i) 14, 17, 15, 18, 16, 19, (a)
statements as statement I and statement II. You have to (ii) 32, 33, 68, 207, (b)
(iii) 5, 11, 25, 51, (c) , 155
determine which statement/s is/are necessary to answer the
(a) c £ a ³ b (b) c > a > b
question and give answer as, (c) a < b > c (d) c > a £ b
(a) The data in statement I alone is sufficient to answer the (e) None of these
question, while the data in statement II alone is not DIRECTIONS (Qs. 84 - 85): A number is bolded in the series I,
sufficient to answer the question II and III. Find the wrong series which does not satisfies the
(b) The data in statement II alone is sufficient to answer the given number and choose the options accordingly.
question, while the data in statement I alone is not
sufficient to answer the question 84. (i) 9, 19, 59, 239, 1199, 7200
(ii) 32, 16, 24, 60, 212, 945
(c) The data either in statement I alone or in statement II alone
(iii) 72, 77, 87, 102, 122, 145
is sufficient to answer the question (a) None (b) Only (i)
(d) The data given in both statements I and II together are not (c) Only (iii) (d) Only (ii)
sufficient to answer the question and (e) All the three
(e) The data given in both statements I and II together are 85. (i) 78, 156, 468, 2340, 16380
necessary to answer the question. (ii) 128, 65, 68, 141, 571, 4576
76. What is the common ratio of the geometric progression? (iii) 64, 33, 34, 52, 105, 264
Statement I: Sum of the infinite terms of the progression (a) Both (ii) and (iii) (b) Only (i)
is 2. (c) Only (iii) (d) Both (i) and (iii)
Statement II: Sum of the first m terms of the progression (e) All the three
is 2m – 1/2m. DIRECTIONS (Qs. 86-88) : There are three quantities provided
77. What is the area of the triangle ABC? in the questions. You have to find out the values of quantities and
Statement I: ABC is an isosceles righ-tangled triangle. compare them according to the given questions.
Statement II: The length of the largest side of the triangle
ABC is 15cm. Give answer :
(a) Quantity – I > Quantity – II > Quantity – III
78. How many people watch English movies assuming that all
(b) Quantity – I = Quantity – II > Quantity – III
people watch atleast one movie. (c) Quantity – I < Quantity – II > Quantity – III
only, is 100. Tg:- @NextGenBankers
Statement I: Number of people watching Hindi movies
DIRECTIONS (Qs. 89 - 90): Following questions have two 94. A container contains 120 litres mixture of Alcohol and
quantities as Quantity I and Quantity II. You have to determine water which contains Alcohol and water in the ratio 7:3.
the relationship between them and give answer as, __________ litres of the mixture is used in a party and
8 litre of Alcohol and 2 litres of water are added to the
89. Quantity I: Tomy and Jacky can do a job in 8 days and remaining mixture. Concentration of Alcohol in the final
12 days respectively. If they work on alternate days with
Tomy beginning, in how many days will the work be mixture is ___________%.
finished? Which of the following option/options satisfy the given
Quantity II: A sum of money is sufficient to pay Uday’s condition?
wages for 21 days and Muday’s wages for 20 days. It is I. 40, 78
then sufficient to pay the wages of both for II. 60, 84
(a) Quantity I > Quantity II III. 30, 71
(b) Quantity I ³ Quantity II (a) Only I and II
(c) Quantity I = Quantity II (b) Only II and III
(d) Quantity I < Quantity II (c) Only I and III
(e) Quantity I £ Quantity II (d) Only III
90. Quantity I: Gopal sold two Air Purifier for ` 12900 each, (e) None of these
neither losing nor gaining in the deal. If he sells one 95. Pramod Das a shopkeeper marked the price of a
commodity at a gain of 29%, the other commodity is sold shirt___________% above the cost price. He sold the shirt
at a loss of at a discount of 5% on the marked price. If selling price
Quantity II: If the sales turnover of a company increases of the shirt is `912, percent profit earned by the
from ` 100 crore to ` 300 crore in 3 years, what is the
shopkeeper is ___________.
compounded annual growth rate of sales (approximately)
for the company? Which of the following option/options satisfy the given
(a) Quantity I < Quantity II condition?
(b) Quantity I ³ Quantity II (a) 20, 14
(c) Quantity I = Quantity II (b) 25, 18.75
(d) Quantity I > Quantity II (c) 50, 42.5
(e) Quantity I £ Quantity II (d) All of the above
Tg:- @NextGenBankers
91. A boat running upstream takes 5 hours 36 min to cover a
certain distance, while it takes 3 hours to cover the same
distance running downstream. What is the ratio between
(e) None of the above
DIRECTIONS (Qs. 96 - 100): Study the following information
carefully and answer the given questions.
the speed of the boat and speed of the water current
respectively? Arka Pratap travels (A) km distance at the speed of x km/hr and
(a) 32:15 (b) 15:32 reaches his office in 2 hours. If he increases his speed by 25%, he
(c) 33:10 (d) 28:15 reached (B) hours less than the previous. Rudra Pratap travels
(e) None of These twice the distance travelled by Arka Pratap and reached his
92. Doon Express can cross a man running in the direction of office in 6 hours at the speed of 40 km/hr. Arka carries a bag
the train with the speed of 4 Km/h in 32.4 seconds. It can which contains (x) green ball, 5 black ball and 10 pink balls. He
also cross a platform of length 740 m in 90 seconds. Time takes one ball randomly and the probability of getting a pink ball
taken by the train to cross Moon Express of length
________ m coming from the opposite direction with the 1
is . Rudra also carries a bag which contains (x + 1) red caps,
speed of 36 Km/h is ___________ seconds. 2
Which of the following option/options satisfy the given (x – 1) orange caps and 5 brown caps. He also takes one cap and
condition? the probability of getting a brown cap is (C).
I. 540, 40.5 Arka’s mother bought a Bike with a discount of 20% on labeled
II. 420, 36.3 price and he again marks up 25%. She offers at 15% discount
III. 440, 36 and sold it to Arka’s friend and gained (D) %, if the labeled price
(a) Only I and II (b) Only III of the Bike is ` 40000.
(c) Only I and III (d) All I, II and III 96. Find the value of (A)
(e) None of these (a) 240 (b) 180
93. A, B and C entered into a partnership with investment of (c) 120 (d) 160
`2P, `1.5P and `___________ respectively. After one (e) 150
year, A made his investment 1.5 times. After one more year, 97. Find the value of (B)
B doubled his investment. At the end of three years, they
(a) 24 mins (b) 20 mins
earned a total profit of `11,5000. 10% of the profit goes
(c) 12 mins (d) 36 mins
to a charity. Share of C in the profit is `____________.
Which of the following option/options satisfy the given (e) 40 mins
condition? 98. Find the value of (x)
(a) 3P, 45000 (b) 2P, 34500 (a) 3 (b) 4
(c) P, 21000 (d) Only (a) and (b) (c) 2 (d) 5
(e) All of the above (e) 6
GP_4458
2019-10 SBI Clerk (Junior Associates) Mains Solved Paper-2019
99. Find the value of (C) a great change is being monitored in the society. We can also
1 1 understand ATM in this regard.
(a) (b) 101. What is the main business of the banking industry?
2 3
(a) To give the interest on deposits & distribute the amount,
1 1 so received, as loan and earn interest.
(c) (d)
5 4 (b) Banks have to make themselves competitive to remain
1 in the Industry and forget better profitability.
(e) (c) Banks are providing information technology.
6
100. Find the value of (D) (d) Banks are now computerizing their branches in rural
(a) 8.75% (b) 13.5% area also.
(c) 5% (d) 12.5% (e) None of these
(e) 6.25% 102. What is retail banking?
English Language
(a) In which all type of categories of population get ser-
vices/benefits from banking industry.
DIRECTIONS (Qs. 101-105): Read the passage carefully and (b) Retail banking is a customer oriented service which
answer the questions given below it. increases clientele.
(c) In this type of banking, possibilities of losses are very
The main business of the banking industry is to give the interest less and comparatively banks can get better produc-
on deposits & distribute the amount, so received, as loan and tivity and profitability.
earn interest. After Nationalization of banks in 1969, personalise (d) ATM is one of their retail banking product.
banking is converted into social banking, in which banks used (e) All the above
their limited resources for economical and social development of 103. Which of the following is the similar to the word
the country. This chapter successfully ran for about two decades. ‘stagnant’?
After that, process of economic liberalisation started and a new (a) flowing (b) Customise
era started in the Indian banking industry and economic progress (c) Particular (d) Static
became stagnant. In the present scenario, banks have to make (e) None of these
themselves competitive to remain in the Industry and forget 104. Which of the following is the similar of the word
‘oriented’?
Tg:- @NextGenBankers
better profitability. Every bank wants to become leader in the
Industry. Banks are now bound to computerize their branches in
rural area also. Efforts are being made to provide better services
(a) Steered
(c) Regressed
(b) intended
(d) Decayed
by using computers. Retail Banking is also a part of this developing (e) Both (a) and (b)
chain in the banking Industry. 105. Which of the following word is suitable to replace the
Retail Banking means a type of banking, in which all type of blank ‘A’?
categories of population get services/benefits from banking (a) period (b) Scenario
industry. Retail banking is a customer oriented service which (c) Scene (d) Years
increases clientele. In this type of banking, possibilities of losses (e) All of these
are very less and comparatively banks can get better productivity DIRECTIONS (Qs. 106-110): Read the passage carefully and
and profitability. If we compare, we found that in the answer the questions given below it.
present....A......., banks are behaving like multinational No distinction was drawn between economic growth and
companies, who are presenting their products with full facilities development in the beginning of the evolution of economics of
and comparative rates. Banks are, therefore, forced to be development. However, since the seventies it has been thought
practical about their marketing policies. ATM is one of their necessary to distinguish between economic growth and economic
retail banking product. development. There are two views even about the concept of
Before knowing about ATM, it is very essential that we should economic development. The traditional view has been to interpret
know well about information technology. Information and it in terms of planned changes in the structure of
society are now related with each other up to good extent. One national product and the occupational pattern of labour force and
cannot think about development of society, without information also the institutional and technological changes that bring about
technology. Even one can say that information is the origin point such changes or accompany such changes.
of the society. Information technology has made possible to join It may be noted that Kuznets in his study of Modern Eco-nomic
different types of people in the society that is why present society Growth interpreted the process of modern economic growth which
is also called Information society. If we think about the starting involves these structural changes. In this view during the process
of computers, we find that initially computer was used only for of economic growth share of agriculture in both national product
mathematical calculations, but today it is a machine of unlimited and employment of labour force declines and that of industries
capacity. Today with the help of computers human control over and services increases. Vari-ous strategies of development which
aeroplane, missiles etc. have become very easy. Computer has were suggested until seventies generally focused on rapid
made possible the invention of technique like Internet. Today with industrialization so that structural transformation could be
the help of Internet, doors of development are opened. Facilities achieved.
like e-business, e-banking, e-education are the facilities by which
SBI Clerk (Junior Associates) Mains Solved Paper-2019 2019-11
For this purpose appropriate institutional and technological 109. Which of the following is the opposite of the word
changes were recommended to bring about such structural ‘interpret’?
changes. Thus C.P. Kindle Berger writes, Economic growth means (a) misunderstand (b) fathom
more output and economic development ......A......both more (c) discern (d) grasp
output and changes in the technical and institutional arrangements (e) all the above
by which it is produced. 110. Which of the following word can replace the blank ‘a’?
Thus, according to traditional view, economic development (a) understand (b) find
implies growth plus structural change. Structural change refers (c) implies (d) regulates
to changes in technological and institutional factors which cause (e) All of these
shift of labour from agriculture to modern manufacturing and DIRECTIONS (Qs. 111-115): Read the passage carefully and
services sectors and also generate self-sustaining growth of output. answer the questions given below it.
An aspect of structural change which is of special mention is that
Financial management refers to the efficient and effective
during the process of economic development there occurs a shift
management of money (funds). Financial Management is about
of working population from low productivity employment in
preparing, directing and managing the financial activities of a
agriculture to the modern industrial and services sectors having
company such as buying, selling and using money to its best results
higher levels of productivity of labour.
to maximize wealth or produce best value for money. It is basically
That is, during the process of economic development percentage
applying general management concepts to the cash of the company.
share of working population in agriculture sharply falls whereas
Financial Management can also be defined as – The management
percentage shares of working population employed in modern
of the finances of a business / organization in order to achieve
industrial and services sectors substantially increase. Along with
financial objectives.
this change in sectoral distribution of labour force there occurs a
According to Dr. S. N. Maheshwari “Financial management is
change in sectoral composition of national in-come in which
concerned with raising financial resources and their effective
percentage contribution of agriculture to national income and
utilization towards achieving the organizational goals.”
declines and percentage contributions to national income of
According to Richard A. Brealey “Financial management is the
industrial and services sectors increase. This occurs due to the
process of putting the available funds to the best advantage from
change in pattern of consumption of the people as economy grows
the long term point of view of business objectives”
and people’s income increases as well as due to the changes in
It is crucial for both public and private sector organizations.
Tg:- @NextGenBankers
levels of productivity in the different sectors of the economy.
106. Study the following statements
Financial management is one of the important aspects in finance.
Nobody can ever think to start a business or a company without
(1) Since the seventies it has been thought necessary to
financial knowledge and management strategies.
distinguish between economic growth and economic The scope of financial management has increased greatly now. It
development. is important to carry out the financial analysis for a company and
(2) In this view during the process of economic growth this analysis helps in decision making process. Investment
share of agriculture in both national product and decision of the company comes under the scope of financial
employment of labour force declines and that of management.
industries and services increases. The investment decision involves the evaluation of risk,
(a) 1 is true, 2 is false (b) 1 is false, 2 is true measurement of cost of capital and estimation of expected benefits
from a project. Capital budgeting and liquidity are the two major
(c) Both are true (d) Both are false
components of investment decision. Capital budgeting is
(e) All of these
concerned with the allocation of capital and commitment of funds
107. What happened during the process of economic growth in permanent assets which would yield earnings in future.
share of employment of labour force of industries and Capital budgeting also involves decisions with respect to
services and agriculture? replacement and renovation of old assets. The finance manager
(a) share of agriculture in both national product and em- must maintain an appropriate balance between fixed and current
ployment of labour force declines assets in order to maximize profitability and to maintain desired
(b) The share of employment of labour to industries and liquidity in the firm.
services increases. Capital budgeting is a very important decision as it affects the
(c) The share of employment of labour to industries and long-term success. At the same time it is a very difficult decision
services decreases. because it involves the estimation of costs and benefits which are
(d) Both (a) and (b) are true uncertain and unknown.
(e) Both (a) and (b) are false While the investment decision involves decision with respect to
108. Which of the following is/are the similar to the word composition or mix of assets, financing decision is concerned
‘Substantially’? with the financing mix or financial structure of the firm and it
(a) Considerably (b) significantly also comes under the range of financial management. The raising
of funds requires decisions regarding the methods and sources of
(c) slighly (d) both (a) and (b)
finance, relative proportion and choice between alternative
(e) All of these
sources, time of floatation of securities, etc.
GP_4458
2019-12 SBI Clerk (Junior Associates) Mains Solved Paper-2019
The finance manager must develop the best finance mix or 113. According to the passage, who contributes the funds for
optimum capital structure for the enterprise so as to maximize the medium term financial requirement of an organisation?
the long- term market price of the company’s shares. A proper (a) Debenture holders (b) Financial institutions
balance between debt and equity is required so that the return to (c) Co-operative societies (d) Commercial bank
equity shareholders is high and their risk is low. Use of debt or (e) Shareholders
financial leverage affects both the return and risk to the equity 114. According to the passage, which of the following decisions
shareholders. The market value per share is maximized when risk does not come under the scope of financial management?
and return are properly matched. (a) Working capital decision
In order to achieve the wealth maximization objective, an (b) Dividend decision
appropriate dividend policy which also comes under the span of (c) Acquisition decision
financial management, must be developed. One aspect of dividend (d) Investment decision
policy is to decide whether to distribute all the profits in the form (e) Financing decision
of dividends or to distribute a part of the profits and retain the 115. Which of the following statements is true in the context
balance. of the passage?
Working capital decision also comes under the extent of financial (a) Working capital decision is related to the income and
management, is related to the investment in current assets and expenditure of business.
current liabilities. Current assets include cash, receivables, (b) Current assets are those assets which are convertible
inventory, short-term securities, etc. Current liabilities consist of into cash within a year.
creditors, bills payable, outstanding expenses, bank overdraft, (c) Capital budgeting is a very important decision as it
etc. Current assets are those assets which are convertible into affects the short-term success.
cash within a year. Similarly, current liabilities are those liabilities, (d) None of the above
which are likely to mature for payment within an accounting year. (e) All are true
Financial planning is the key element of the financial management. DIRECTIONS (Qs. 116-120): Given sentences are not in their
Financial Planning is process of framing objectives, policies, exact position. Rearrange them to make a coherent paragraph
procedures, programmes and budgets regarding the financial and then answer the questions given below.
activities of a concern.
(A) This type of ancient Hindu school in India was residential
The financial planning estimates the precise requirement of funds
in nature with the Shishyas or students and the Guru or
Tg:- @NextGenBankers
which means to avoid wastage and over-capitalization situation.
Funds can be arranged from various sources and are used for
teacher living in proximity within the same house.
(B) In addition to that ancient Indian education achieved a
long term, medium term and short term requirement. Financial noticeable position in the early Vedic period beginning in
planning is necessary for tapping appropriate sources at the 1200 B.C.
appropriate time as long term funds are generally contributed by (C) The students resided together irrespective of their social
shareholders and debenture holders, medium term by financial standing.
institutions and short term by commercial banks. (D) Education in Ancient India originated with
Financial planning suggests how the funds are to be allocated for the Gurukul system.
various purposes by comparing various investment proposals. It (E) However, several temples and community centres regularly
helps in ensuring a reasonable balance between outflow and inflow took the role of schools.
of funds so that stability is maintained. It also helps in reducing 116. Which of the following should be the FOURTH sentence
the uncertainties which can be a hindrance to growth of the after the rearrangement?
company and this helps in ensuring stability and profitability in (a) A (b) B
concern. (c) C (d) D
111. According to the passage, which of the followings cannot (e) E
be considered as an advantage of financial planning? 117. Which of the following should be the SECOND sentence
(a) Helps in estimating the requirement of funds after the rearrangement?
(a) A (b) B
(b) Helps in proper allocation of funds
(c) C (d) D
(c) Helps in maintaining the balance between inflow and
(e) E
out flow of fund 118. Which of the following should be the THIRD sentence
(d) Helps in growth and expansion programmes after the rearrangement?
(e) Helps in reducing the uncertainties (a) A (b) B
112. According to the passage, why should the finance manager (c) C (d) D
develop the optimum capital structure? (e) E
(a) For maintaining the liquidity in the firm 119. Which of the following should be the FIRST sentence
(b) For maximizing the value of the firm after the rearrangement?
(c) For maximizing the return to equity shareholders (a) A (b) B
(d) For reducing the cost of capital (c) C (d) D
(e) For maximizing the market price of share (e) E
SBI Clerk (Junior Associates) Mains Solved Paper-2019 2019-13
120. Which of the following should be the FIFTH sentence 125. the most important step towards the haven of security is
after the rearrangement? to _______in the senses, which, if not kept in control, carry
(a) A (b) B off even the wisest to the _______. Physical starving of
(c) C (d) D the senses works but as a temporary purpose; it is only
(e) E when the intellect realises its moorings in the Highest and
prevents the mind from _______that one can feel secure.
DIRECTIONS (Qs. 121-125): In each question below, a para- (a) Rein, abyss, briskly
graph is given, containing 3 blanks. Four options are given with (b) Rein, abyss, wandering
three words. Choose the correct combination of words from the (c) Palliative, recur, briskly
options, which can fit in the blanks correctly. If none of the four (d) Palliative, recur, wandering
options fit correct then choose (e) as the answer. (e) None of these
121. The apex court has described corruption as a serious DIRECTIONS (Qs. 126-130): In the following question, out of
_______and one impinging on the economy. In 2013, the the four alternatives, choose the one which best expresses the
protection given to sitting legislators from immediate meaning of the highlighted word given in the sentence.
_______was removed. Further, common sense would 126. A few of the modern Jewish houses have been embellished
suggest that disqualification should be more strictly at an enormous cost, but they are wanting in taste.
applied to those _______for corruption. (a) Simplified (b) Decorated
(a) Malady, disqualification, repulsed (c) Dilapidated (d) Tarnished
(b) infatuated, enamoured, repulsed (e) None of these
(c) Malady, disqualification, convicted 127. While the Deans were pleased that Martha had confided
(d) infatuated, enamoured, convicted in them about her gruesome discovery, her pending exit
(e) None of these remained an ever-present pall that hung over the remainder
122. Ascetic and _______, Gandhi changed the face of civil of the evening like a chilly fog.
disobedience around the world. Martin Luther King, Jr. (a) Sullen (b) Exhausting
drew on his tactics during the Civil Rights Movement, and (c) Hideous (d) Insulting
the Dalai Lama was inspired _______his teachings, which (e) None of these
are still _______by those who seek to inspire change 128. The vivacious lilac still grows, unfolding its sweet-scented
Tg:- @NextGenBankers
without inciting violence.
(a) Unflinching, by, akin
flowers each spring.
(a) Crestfallen
(c) Vibrant
(b) Languid
(d) Effete
(b) Cognizant, from, akin
(e) None of these
(c) Cognizant, from, heralded
129. His talent is quite prodigious and we are looking forward
(d) Unflinching, by, heralded
seeing him at this level where I am sure he will be very effective.
(e) None of these (a) Paltry (b) Phenomenal
123. Those were _______days as the whole nation, and Delhi (c) Mediocre (d) Nugatory
in particular, was engulfed in communal riots _______from (e) None of these
the Partition. Today, however, the serenity of the place and 130. The articulate speaker intrigued the audience with his
its surroundings _______the anger, the hurt, and the guilt message.
experienced by all those who came to bid the Mahatma (a) Impotent (b) Frigid
adieu at this place on January 30, 1948. (c) Eloquent (d) Hesitant
(a) Tumultuous, emanating, betrays (e) None of these
(b) cluttered, kempt, shroud
DIRECTIONS (Qs. 131-133): In the following questions,
(c) Cluttered, emanating, betrays choose the word opposite in meaning of the highlighted word
(d) Tumultuous, kempt, shroud given in the sentence.
(e) None of these
124. Attitude determines the lifestyle of a person and it 131. The filibuster began to irk everyone in the senate.
develops when _______is followed by projection. Positive (a) Vex (b) Pique
and negative attitudes are _______by an individual over (c) Delight (d) Exasperate
(e) None of these
a period of time and according to their development,
132. Alas for the vanity of man’s judgement and man’s
human beings fall into three categories: there are those who
prescience!
imagine and project difficulties _______any undertaking
(a) Humility (b) Haughtiness
even before they start work and end up doing nothing. They (c) Self-admiration (d) Ostentation
are like the car that will not start unless given a push. (e) None of these
(a) Vaunt, brandish, to 133. Alone in the middle of the ocean, the survivor could think
(b) Vaunt, acquired, in of nothing that would make the insipid days seem shorter.
(c) Perception, brandish, to (a) Mundane (b) Exhilarating
(d) Perception, acquired, in (c) Vapid (d) Prosaic
(e) None of these (e) None of these
GP_4458
2019-14 SBI Clerk (Junior Associates) Mains Solved Paper-2019
DIRECTIONS (Qs. 134-135): In the sentences given below, the General/Financial Awareness
parts marked a), b), c) and d) may contain errors. Choose the
part which is error-free. If the sentence is error free, choose e) 141. 19th August is celebrated as-
no error. (a) International Holocaust Day
134. The CBI have sought his custodial interrogation and (a)/ (b) World AIDS Day
claimed that he responded either evasive (b)/ or arrogantly (c) World Photography Day
during an apex court-ordered questioning(c)/ in Assam (e) None of these
recently. It said his arrest was being required. (d) 142. The First Indian State to issue Masala Bond-
(a) a and b (b) Only b (a) Maharashtra (b) Kerala
(c) Only c (d) a and d (c) Madhya Pradesh (d) Goa
(e) No error (e) Meghalaya
135. Manohar Sharma, Director of the journalism fellowship (a)/ 143. Which state is set to host the 21st Commonwealth Table
program at the Reuters Institute for the Study of Journalism, Tennis Championship at the Jawaharlal Indoor Stadium?
University of Oxford, explains (b)/ that Sri Lanka has a (a) Pune, Maharashtra (b) Guwahati, Assam
“long history of censoring the press, by killing journalists, (c) Jaipur, Rajasthan (d) Cuttack, Odisha
(c)/ blocking websites and using draconian laws to fine and (e) Indore, Madhya Pradesh
imprison reporters”. (d)
144. Commercial paper is an unsecured promissory note with
(a) a and b (b) Only b
(c) Only c (d) a and d a fixed maturity of not more than-
(e) No error (a) 300 days (b) 365 days
(c) 235 days (d) 290 days
DIRECTIONS (Qs. 136-140): In the given below questions a (e) 270 days
sentence is given with an idiom/phrase in bold words and four 145. Male is the capital city of-
options are given below it. Choose the right option which is show-
(a) Maldives (b) Singapore
ing the meaning of the given words. If out of four none is correct
the answer choose (e) as the right answer. (c) Malaysia (d) Tanzania
(e) Bhutan
136. John keeps drawing on fancy about the banking job in 146. Who is the present Minister of Chemicals and Fertilizers
government sector. in the Government of India?
(a) Use imagination
Tg:- @NextGenBankers
(b) Do the things beautifully
(c) Make clear conclusion about anything or anyone
(a) Shri Ramvilas Paswan
(b) Shri Ravi Shankar Prasad
(c) Shri Thaawar Chand Gehlot
(d) Likes government jobs
(e) None of these (d) Smt. Smriti Zubin Irani
137. After reading the story time and again we are finally reading (e) D V Sadananda Gowda
between the lines. 147. Who wins the silver medal in World Youth Cup 2019 from
(a) Reading to entertain others India?
(b) Understanding the hidden meaning (a) Arinjeeta Dey
(c) Reading only to show someone else about your style (b) Aronyak Ghosh
of reading. (c) Sreeshwan Maralakshikari
(d) Reading to clear the exam (d) Rakshitta Ravi
(e) None of these (e) None of these
138. Failing in exam again and again even after putting all the 148. Damnganga-Pinjal river linking project will fulfill water
hard work is bad blood. needs till 2060 in-?
(a) Consume time and affects the health (a) Ujjain, Madhya Pradesh
(b) The aspirants become unhealthy (b) Mumbai, Maharashtra
(c) The aspirants do not take interest (c) Bengaluru, Karnataka
(d) Create angry feeling
(d) Kurnool, Andhra Pradesh
(e) None of these
(e) None of these
139. Alice is very good at quant, but when she was watching
a teacher’s video she was taking it with a grain of salt. 149. The Shanghai Cooperation Organisation (SCO 2019) 19th
(a) She was enjoying the video summit was held in-
(b) To listen to something with considerable doubt (a) Nur Sultan, Kazakhstan
(c) She was listening with full attention (b) Dushanbe , Tajikistan
(d) She was not listening with full attention (c) Seoul, South Korea
(e) None of these (d) Bishkek, Kyrgyzstan
140. The boy broke the window and took to his heels. (e) None of these
(a) The boy ran into the window. 150. War Over Words: Censorship in India, 1930-1960 Book
(b) The boy broke the window and ran away. has been authored by-
(c) The boy broke the window and fell on his heels. (a) Devika Sethi (b) Deepak Nayyar
(d) The boy broke the window with his heels. (c) Meira Kumar (d) Kapil Isapuari
(e) None of these (e) Poonam Surie
SBI Clerk (Junior Associates) Mains Solved Paper-2019 2019-15
151. Which state government would set up a first-of-its-kind 162. Who is the present speaker of Lok Sabha?
elephant rehabilitation center here at an outlay of Rs 105 (a) Om Birla
crore? (b) Pralhad Joshi
(a) Sikkim (b) Kerala (c) Mahendra Nath Pandey
(c) West Bengal (d) Odisha (d) Giriraj Singh
(e) Arunachal Pradesh (e) Kiren Rijiju
152. Amrabad Tiger Reserve located in- 163. RBI has initiated steps to set up a wide-based DPCR to
(a) Kerala (b) Maharashtra capture details of all borrowers. DPCR stands for-
(c) Chhattisgarh (d) Telangana (a) Digital Public Cash Rupee
(e) Tamil Nadu
(b) Digital Plan of Cash Registry
153. According to some newspaper reports, one-third of the
(c) Digital Public Credit Registry
total wilful defaulter is which bank?
(a) Punjab National Bank (d) Detailed Public Credit Rate
(b) State Bank of India (e) None of these
(c) Karnataka Bank 164. The National Institute of Food and Agriculture (NIFA)
(d) Bank of Baroda based in-
(e) Central Bank of India (a) USA (b) Chine
154. RBI has formed a committee to review regulatory (c) UAE (d) UK
guidelines and supervisory framework applicable to core (e) None of these
investment companies (CICs). This committee is headed 165. Kawal Tiger Reserve was located at-
by- (a) Karnataka (b) Assam
(a) Bimal Jalan (b) Smt. Usha Thorat (c) Gujarat (d) Madhya Pradesh
(c) Tapan Ray (d) Shri T. N. Manoharan (e) Telangana
(e) None of these 166. Balphakram National Park was located at-
155. Kaziranga National Park is a protected area in- (a) Sikkim (b) Assam
(a) Sikkim (b) Assam (c) Meghalaya (d) Mizoram
(c) Arunachal Pradesh (d) Uttarakhand (e) None of these
(e) None of these
in Japan?
Tg:- @NextGenBankers
156. Who has lead Indian delegation for G-20 meeting on trade
167. Hanoi is the capital of-
(a) Brunei
(c) Malaysia
(b) Cambodia
(d) Vietnam
(a) Shri Nitin Jairam Gadkari
(e) Laos
(b) Shri Raj Nath Singh
(c) Piyush Goyal 168. What is the first instalment amount for PM Matru Vandana
(d) Smt. Harsimrat Kaur Badal Yojana?
(e) None of these (a) ` 500 (b) `1000
157. The surcharge was levied on individuals with a total (c) `1200 (d) `1500
income more than `1 crore at the rate of- (e) `2000
(a) 10% (b) 15% 169. Who is the AIFF women’s Player of 2019?
(c) 20% (d) 25% (a) Maymol Rocky
(e) 30% (b) Ashalata Devi
158. LIC has how much crore in unclaimed money? (c) Ngangom Bala Devi
(a) `96,000 crores (b) `10,000 crores (d) Sangita Basfore
(c) `10,500 crores (d) `11,200 crores (e) None of these
(e) `11,500 crores 170. Soyuz 2.1a Launched by which agency?
159. Where is the headquarters of National Institute of (a) NASA (b) Roscosmos
Agricultural Economics and Policy Research? (c) JAXA (d) UK Space Agency
(a) Mumbai (b) New Delhi (e) None of these
(c) Indore (d) Jaipur 171. In which stadium, Cricket World Cup final match was
(e) Kochi held?
160. Which Indian city made its entry into the UNESCO World
(a) Lord’s, London.
Heritage Site list recently?
(b) Headingley, Leeds.
(a) Goa (b) Jaipur
(c) Ranchi (d) Raipur (c) Ageas Bowl, Southampton.
(e) None of these (d) Edgbaston, Birmingham.
161. Deposits in bank accounts opened under Jan Dhan scheme (e) The Oval, London.
have crossed the Rs __________ lakh crore mark. 172. Who won men’s single Wimbledon 2019?
(a) 1 lack crore (b) 2 lack crore (a) Roger Federer (b) Rafael Nadal
(c) 3 lack crore (d) 4 lack crore (c) Novak Djokovic (d) Andy Murray
(e) 5 lack crore (e) None of these
GP_4458
2019-16 SBI Clerk (Junior Associates) Mains Solved Paper-2019
173. Which sport is associated with Ajay Singh? 182. Pradhan Mantri Karam Yogi Maandhan (PM-KYM)
(a) Cricket (b) Tennis Scheme All PM Karam Yogi Maan Dhan Scheme
(c) Boxing (d) Hockey beneficiaries must have annual turnover less than _____.
(e) Football (a) ` 1 crore (b) ` 1.2 crore
(c) ` 1.5 crore (d) ` 1.8 crore
174. What is the constituency of Arjun Munda?
(e) `2 crore
(a) Garhwa (b) Latehar
183. Every woman in Self-Help Group (SHG) was eligible for
(c) Giridih (d) Khunti a loan up to ______ under the MUDRA Scheme.
(e) Dhanbad (a) ` 0.5 lakh (b) `1 lakh
175. What is the Headquarters of National Cricket Academy? (c) `1.5 lakh (d) `2 lakh
(a) Bengaluru (b) Chennai (e) `2.5 lakh
(c) Mumbai (d) New Delhi 184. BRICS Summit 2019 was held at –
(e) Kolkata (a) Canada (b) Brazil
176. T.N. Manoharan Committee was launched by? (c) Chile (d) Argentina
(a) RBI (e) None of these
(b) Ministry of Finance 185. The Solar Charkha Mission comes under which ministry?
(a) Ministry of Micro Small & Medium Enterprises
(c) SIDBI
(b) Ministry of Home Affairs
(d) SEBI (c) Ministry of Consumer Affairs, Food and Public Dis-
(e) None of these tribution
177. Civil Aviation Training Center (CATC) was located at? (d) Ministry of Electronics and Information Technology
(a) Iran (b) USA (e) None of these
(c) France (d) UK 186. Name the first foreign bank to set up unit in Gujarat
(e) None of these International Finance Tec-City (GIFT).
178. Anshula Kant is the MD and CFO of? (a) Citi Bank (b) DBS Bank
(a) State Bank of India (c) HSBC Bank (d) Barclays Bank
(b) Asian Development Bank (e) Standard Chartered Bank
187. Which of the following bank has reportedly drawn up plans
(d) World Bank
(e) None of these
Tg:- @NextGenBankers
(c) International Finance Corporation
for a radical restructuring that will involve the creation of
a “bad bank”?
(a) Industrial and Commercial Bank of China
179. India has signed a deal of Rs.200 Crore to acquire Strum (b) Deutsche Bank
Ataka with which of the following countries? (c) HSBC Bank
(a) Russia (b) Brazil (d) Barclays Bank
(c) Canada (d) South Africa (e) None of these
(e) Oman 188. The 2023 ICC Cricket World Cup will be held at-
180. Which of the following has launched STRIDE scheme (a) Australia (b) India
recently, aimed at strengthening research culture and (c) England (d) South Africa
(e) None of these
innovation in universities and colleges of the country?
189. Name the organization works to eradicate poverty and
(a) National Human Rights Commission
reduce inequalities through the sustainable development of
(b) National Commission for Women nations.
(c) University Grants Commission (a) UNDP (b) UNESCO
(d) Competition Commission of India (c) UNCTAD (d) UNIDO
(e) None of these (e) None of these
181. Dutee Chand becomes the first Indian to win a gold medal 190. Budget 2019 has proposed to levy tax deduction at source
at the Summer Universiade which was held at- (TDS) of 2 per cent on the cash withdrawal of more than
(a) Ireland (b) Italy _______ from a bank account.
(c) Brazil (d) Austria (a) `0.5 crore (b) `1 crore
(e) None of these (c) `1.5 crore (d) `1.8 crore
(e) `2 crore
SBI Clerk (Junior Associates) Mains Solved Paper-2019 2019-17
ANSW ER KEY
1 (c) 21 (d) 41 (c) 61 (c) 81 (b) 101 (a) 121 (c) 141 (c) 161 (a) 181 (b)
2 (b) 22 (d) 42 (d) 62 (a) 82 (a) 102 (e) 122 (d) 142 (b) 162 (a) 182 (c)
3 (c) 23 (c) 43 (c) 63 (d) 83 (c) 103 (d) 123 (a) 143 (d) 163 (c) 183 (b)
4 (d) 24 (a) 44 (a) 64 (b) 84 (e) 104 (e) 124 (d) 144 (e) 164 (a) 184 (b)
5 (c) 25 (b) 45 (a) 65 (b) 85 (a) 105 (b) 125 (b) 145 (a) 165 (e) 185 (a)
6 (a) 26 (a) 46 (b) 66 (b) 86 (a) 106 (c) 126 (b) 146 (e) 166 (c) 186 (e)
7 (d) 27 (c) 47 (d) 67 (a) 87 (a) 107 (d) 127 (c) 147 (a) 167 (d) 187 (b)
8 (b) 28 (e) 48 (e) 68 (a) 88 (b) 108 (d) 128 (c) 148 (b) 168 (b) 188 (b)
9 (a) 29 (b) 49 (b) 69 (a) 89 (d) 109 (a) 129 (b) 149 (d) 169 (b) 189 (a)
10 (d) 30 (b) 50 (d) 70 (a) 90 (a) 110 (c) 130 (c) 150 (a) 170 (b) 190 (b)
11 (e) 31 (c) 51 (a) 71 (d) 91 (d) 111 (d) 131 (c) 151 (b) 171 (a
12 (b) 32 (d) 52 (b) 72 (e) 92 (c) 112 (e) 132 (a) 152 (d) 172 (c)
13 (c) 33 (e) 53 (c) 73 (c) 93 (d) 113 (b) 133 (b) 153 (b) 173 (c)
14 (c) 34 (b) 54 (b) 74 (a) 94 (d) 114 (c) 134 (c) 154 (c) 174 (d)
15 (e) 35 (a) 55 (d) 75 (d) 95 (d) 115 (b) 135 (e) 155 (b) 175 (a)
16 (b) 36 (b) 56 (b) 76 (a) 96 (c) 116 (e) 136 (a) 156 (c) 176 (a)
17 (b) 37 (b) 57 (a) 77 (e) 97 (b) 117 (a) 137 (b) 157 (a) 177 (a)
18 (a) 38 (d) 58 (c) 78 (d) 98 (d) 118 (c) 138 (d) 158 (c) 178 (d)
19 (a) 39 (a) 59 (b) 79 (d) 99 (b) 119 (d) 139 (b) 159 (b) 179 (a)
20 (a) 40 (c) 60 (d) 80 (c) 100 (e) 120 (b) 140 (b) 160 (b) 180 (c)
ANSWERS
Tg:- & EXPLANATIONS
@NextGenBankers
Sol. (1-5) rearranging the numbers add 5 in the even number and
subtract 5 in the odd numbers.
R S P K
Row (a): Input - IKGE42 71 RTBC 86 PKCT 25 KDSM 59 VATW
Step I: 91 IKGE 42 71 RTBC PKCT 25 KDSM 59 ATVW
D A N B Step II: 66 91 IKGE 42 PKCT 25 KDSM 59 ATVW BCRT
Row (b): Step III: 54 66 91 IKGE 42 25 KDSM ATVW BCRT CKPT
E G C F Step IV: 47 54 66 91 IKGE 25 ATVW BCRT CKPT DKMS
Step V: 20 47 54 66 91 ATVW BCRT CKPT DKMS EGIK
Row (c): M J Q L Step VI: is the last step of the rearrangement.
1. (c) 2. (b) 3. (c) 4. (d) 5. (c) 6. (a) 7. (d) 8. (b) 9. (a) 10. (d)
Sol. (6-10): Logic: As a first step let’s first understand 11. (e) RELATIONSHIP
the logic behind the Output. Words are arranged from Step I: QFKBSJPMRGJO
right end and numbers are arranged from left end. In each Vowel ® following letters
step one word and one number are arranged. Consonent® Preceding letters
Word – Words are arranged in ascending order from left
to right according to the English alphabetical order. In Step II: BFG J J KMOPQRS
the first step the word starting with the letter having higher (Alphabetical order arrangement)
place value in the English alphabet is arranged first on the
Step III: BFGKMOPQRS
right end and also the letters of the word are arranged in
increasing alphabetical order within the word. (Eliminate repeated letters JJ)
Number- Numbers are arranged in ascending order from
left to right. In the first step the highest number is arranged
first and then in the second step 2nd highest numbered gets B F G K M O P Q R S
Step IV:
arranged and so on till the last step and also after
GP_4458
2019-18 SBI Clerk (Junior Associates) Mains Solved Paper-2019
Sol. (13-15) 21. (d) 22. (d) 23. (c) 24.(a) 25. (b)
Sol. (26-30)
N In this new pattern coding decoding each letter,
B 33m U except vowel, is assigned a number from 1-6 So,
W E B-1, C-2, D-3, F-4, G-5, H-6, J-1, K-2, L-3, M-4,
31m
N-5, P-6, Q-1, R-2, S-3, T-4, V-5, W-6, X-1, Y-2,
Z
B' J Z-3. Also, each vowel is assigned different digits
S 30m R S
starting from the digit code of Z. So, for vowels the
digit codes are - A-3, E-4, I-5, O-6, U-7.
26m
51m 26. (a) 27. (c) 28.(e) 29. (b) 30. (b)
31. (c)
P 58m Q
13. (c) U is in north-west with respect to Q. Deepa Erwin Bindu Gauri Hema Aaliya Fida Cherry
14. (c) As, BU = 33m 32. (d) Only (I) can be assumed from the given statement
So, B' R = 33m as it is mentioned in the given statement that rooftop
we have PQ = 58m solar power technology is showing growth.
So, B' J = 58m
\ RJ = B' J– B' R
Tg:- @NextGenBankers 33. (e) Both I and II follows as salary is an important
factor for encouraging students to opt teaching as a
career option rather than job and changing the
= 58–33 eligibility criterion to graduation for being a
RJ = 25m teacher as generally the students choose their
As Z is mid point of RJ career option after graduation.
25 Sol. (34-38)
So, RZ = = 12.5 m 6yrs 2yrs 4yrs 1yrs 5yrs 3yrs
2
So, distance between S and Z is CSM > LP > MC > GM > CFO > FO
Þ SR+RZ 10 lakh 18 lakh 8 lakh 11 lakh 9 lakh 13 lakh
Þ 30m+12.5m 34. (b) 35. (a) 36. (b) 37. (b) 38. (d)
39. (a) For I: Yes, it can be inferred from the given
Þ 42.5m
statements as it is clearly mentioned punctuality
15. (e) From the previous question we get
and sincerity are one of the key points which will
RJ = 25m surely reviewed.
Hence, Distance between S and J is For II: No, it is clear from the given statements
Þ SR + RJ that there will be increment but percentage cannot
Þ 30m + 25m be inferred.
= 55m For III: No, as it is mentioned by manager that hard
16. (b) I. U % X (False) work and dedication towards work will be applauded.
II. X ® Z (True) But, Is there some employee who have done their
17. (b) I. A × D (False) work with complete determination and enthusiasm, it
cannot be inferred. We can assume it but it cannot be
II. D ® B (True)
inferred from it.
18. (a) I. O ® M (True) 40. (c) For I: Yes, it weakens the statement of manager as it
II. S × N (False) is said by manager that increment will be based on
19 (a) I. B ® G (True) performance i.e. hard work and determination. But
II. G @ B (False) 15% for all those employees who have completed
1 year undermines the statement by manager.
SBI Clerk (Junior Associates) Mains Solved Paper-2019 2019-19
Base fare of Chennai to Bangalore = 1692 + 908 = `2600 58. (c) Let the Present age is = x
Total fare of Sapna when she goes from Chennai to Sanjeet Manjeet
Bangalore Past (5x – 4)
= 1700 + 25% of 1700 + 10% of 1700 + 20% of 25% Present 5x 3x
of 1700
Future (3x + 4)
= 1700 + 425 + 170 + 85 = ` 2380
According to question,
Total fare of Sapna when she goes from Bangalore to
Chennai (5x – 4) / (3x + 4) = 1:1
= 800 + 25% of 800 + 10% of 800 + 20% of 25% of 800 So, (5x – 4) = (3x + 4)
= 800 + 200 + 80 + 40 = `1120 Solving we get 2x = 8
Total fare of Rajat when he goes from Chennai to x=4
Bangalore and comes back to Chennai. Hence, Sanjeet’s present age = 5x = 5 × 4 = 20 and
= 2600 + 25% of 2600 + 15% of 2600 + 30% of 25% after 4 years it will be 24 years.
of 2600 Manjeet’s present age = 3x = 3 × 4 = 12 and before
= 2600 + 650 + 390 + 195 = `3835 4 years it was 8 years
Required difference ratio = (2380 + 1120) – 3835 = `335 Thus Ratio will be 24:8 = 3:1
54. (b) Base fare from Shrinagar to Ajmer by airways CC
59. (b) Sum of 7 can be achieved in 4 ways (6, 1), (1, 6), (5,
= 36% of (2D + 3500) = `2700
2), (2, 5) , (3, 4) and (4, 3)
Base fare from Ajmer to Shrinagar by airways DD
Probability of a sum of 7 on the dice = 6/36 = 1/6
= 30% of [3.5D + (2D/3)] = `2500
Probability of at least 2 coins showing on head
Total fare from Shrinagar to Ajmer
= 2700 + 25% of 2700 + 10% of 2700 + 20% of 25% 4 4 4
æ1ö æ1ö æ1ö
of 2700 =4 C 2 ç ÷ + 4 C 3 ç ÷ + 4 C 4 ç ÷
è2ø è2ø è2ø
= 2700 + 675 + 270 + 135 = `3780
Total fare from Ajmer to Shrinagar 4
æ1ö 11
of 2500 Tg:- @NextGenBankers
= 2500 + 25% of 2500 + 10% of 2500 + 20% of 25%
11 1 11
Required per cent = [(3780 – 3500)/3500] × 100 = 8% Thus, the required probability = ´ =
16 6 96
55. (d) Let the distance between Hyderabad and Kolkata
= ‘D’ km 60. (d) When the trains are moving in the same direction, the
Base fare from Kolkata to Hyderabad by airways faster train crosses the slower when the tail of the faster
EE = 80% of (0.01D2 – 0.75D) = (0.008D2 – 0.6D) just passes the front end of the slower train.
Total fare from Kolkata to Hyderabad Hence, the distance that the faster train has to cover
= (0.008D2 – 0.6D) + 25% of (0.008D2 – 0.6D) before crossing the slower train
+ 10% of (0.008D 2 – 0.6D) + 20% of 25% of = Sum of the length of the trains = (500 + 750) m = 1250 m.
(0.008D2 – 0.6D) = 3528 Relative speed of the faster train with respect to the
140% of (0.008D2 – 0.6D) = 3528 slower = (60 – 33) km/hr
(0.008D2 – 0.6D) = 2520 ; D = 600 km = 27 km/hr = 7.5 m/s
56. (b) Let the certain number
Thus, time taken for the faster train to cross the slower
N = 5 [7(8x + 4) + 3] + 2 = 35(8x) + 140 + 17 = 280x + 157
When this is divided by 8, quotient = 35x + 19 and 1250
= sec = 166.7 sec.
remainder = 5 7.5
When (35x + 19) is divided by 7, quotient 61. (c) Total attempted questions in Math = 80 × (7/20) = 28
= 5x + 2 and remainder = 5 Total attempted questions in Science = 80 × (8/20) = 32
When (5x + 2) is divided by 5, quotient = x and Total attempted questions in English = 80 × (5/20) = 20
remainder = 2. Normalized marks in Math = 1.2 × [{5 × 28 × (6/7)}
57. (a) Contribution of Vijay and Ajay = 1/4 × 1/2 : 1/5 × 1/4 – {2 × 28 × (1/7))} – {0.5 × (35 – 28)}] = 130.2
= 1/8 : 1/20 Normalized marks in Science = [{5 × 32 × (5/8)}
Contribution of Sanjay = 1 – (1/4 + 1/5) = 11/20 – {2 × 32 × (3/8))} – {0.5 × (35 – 32)}] = 74.5
Share of Vijay, Ajay and Sanjay = 1/8 : 1/20 : 11/20 Normalized marks in English = [{5 × 20 × (3/4)}
on solving we get Ratio of profit sharing = 5 : 2 : 22 – {2 × 20 × (1/4))} – {0.5 × (30 – 20)}] = 60
Profit share of Ajay and Sanjay Total marks in the exam = 130.2 + 74.5 + 60 = 264.7
= 17,400 × (2 + 22) / 29 = 14,400
SBI Clerk (Junior Associates) Mains Solved Paper-2019 2019-21
Chahak+ Arjun =
Tg:- @NextGenBankers
1
= 125 – 100 = 25%
(a) Students (Hindi and English) =
25
´ 800 = 200
32 100
= Satisfies the given condition
1 1 25
Chahak = – (b) Students (Hindi and English) = × 1200
32 48 100
1 = 300 ¹ 400
Chahak = = Doesn’t satisfy the given condition.
96
Chahak can complete the work in 96 days. 25
(c) Studnets (Hindi and English) = × 1000
Work done by Chahak, Dablu in one day, 100
1 1 2 1 = 250 ¹ 200 = Doesn’t satisfy the given condition.
Chahak + Dablu = + = = 71. (d) Akai’s turnover:
96 96 96 48
Chahak and Dablu can complete the work in 48 days. BPL sales in Washing Machine = 337 26% of market
Akai’s share = 30% of market
5 = (30 × 337) ÷ 26 = 390 approx
68. (a) 350 = speed of train Taj Mail × ×21
18 Godrej turnover:
Speed of train Taj Mail = 60 kmph BPL sales in Audio System = 190 = 38% of market
5 Godrej share = 40% of market = (40 × 190) ÷ 38 = 200
250 = speed of train JP Mail × × 10 Percentage by which Akai’s turnover is greater than
18
Speed of train JP Mail = 90 kmph Godrej = (390 – 200) ÷ 200 × 100 = 95% Approx.
72. (e) Though we know the market share, we do not know
5 the value of the market as no value nor is percentage
(i) 350 + 250 = (60 + 90) × × required time
18 growth related to 2018 given. So we cannot calculate.
Required time = 14.4 seconds 73. (c) In 2018, market share = 38%, it was 41% in 2015.
5 Sales S = 190 / [1 + 20/100]3 = 109.95
(ii) 350 + length of the bridge = 60 × × 36 Total refrigerator market in 2015 = (100/41) × 109.95
18
= 268 crore.
Length of the bridge = 250 m
SBI Clerk (Junior Associates) Mains Solved Paper-2019 2019-23
74. (a) In 2018 BPL’s sales = 337 80. (c) From Statement I, Champa got 8000 so 4x = 8000
In 2015 BPL’s sales = S So x = 2000
337 = S [1 + 10/100]3 = S × (1.1)3 So total profit = 6x + 3x + 4x = 13x
= (337/1.331) × 107 => S = 2500 × 106 = 13×2000 = 26,000
= 2500 million. So profit% = 26000/52000 × 100 = 50%
75. (d) Data inadequate is the answer because we don’t have From II, 6x – 3x = 6000
any additional information regarding the sales and
So x = 2000
market share of the MI in the Category of Washing
Machine. Further same as in I.
76. (a) From statement II 81. (b) Series – I pattern:
Sm = 1 – 1 / 2m The pattern is, +(1×3), +(2×6), +(3×9), +(4×12),..
Sm – 1 = 1 – 1/2m – 1 The answer is, 78
Þ tm = (1 – 1/2m – 1 ) – (1 – 1/2m) = 1/2m Series – II pattern:
Þ Common ratio = 1 / 2
7 13 24 40 61
From statement I:
a/1–r=2 +6 +11 +16 +21
Þ a = 2(1 – r)
+5 +5 +5
æ 2–a ö The difference of difference is 5.
Þr= ç ÷
è 2 ø The answer is, 87
We cannot have the value of r. So only statement I, Series – III pattern:
alone give the answer.
5 + (42 – 1) = 20
77. (e) Statement I: A 20 + (62 – 3) = 53
Here DABC is a isosceles
53 + (82 – 5) = 112
right-angle triangle with
AB = BC = X cm Tg:- @NextGenBankers
Area = 1/2 x2 ; but we
X
112 + (102 – 7) = 205
205 + (122 – 9) = 340
don’t know the length of The answer is, 340
B C
x. So area cannot be X Hence, a < b < c
found out by I. 82. (a) Series – I pattern:
Statement II:We have only
10 The pattern of the series is 14 , 24 , 34 , 44 , 54 ……..
one length = 15 cm. So we
X The answer is, 2401
can’t find the area from II,
Series – II pattern:
alone
Combining I and II: 1335 444 147 48 15 4
X
–891 –297 –99 –33 –11
225
Area = 1/2x2 and 2x2 = 152 => x2 =
2 ¸3 ¸3 ¸3 ¸3
Þ Area = 56.25 sq. units The answer is, 48.
Þ So, I and II, combined solve this problem.
Series – III pattern:
78. (d) Statement I: Gives no information about n(E) or n(E Ç H)
The pattern is, ×2 – 32 , ×3 + 42, ×4 – 52, ×5 + 62, ×6 –
Statement II: Gives n(E È H) but we cannot count n(E)
72 ,..
From both I and II we get n(E È H) = n(E) + [n(H) – n(E Ç H)]
The answer is, 1011
or 300 = n(E) + 100
=> n(E) = 200 Hence, a > b < c
Hence, from both statements is necessary to give the 83. (c) Series – I pattern:
answer. The pattern of the series is +3, –2, +3, –2………..
79. (d) Statement I provide the car load for onset of roads. No The answer is, 17
data is given for the other set. Series – II pattern:
Statement II only defines the logic for movement of traf- The pattern is, ×1+1, ×2+2, ×3+3, ×4+4, ×5+5,..
fic but no data on the time it would take to wait on each
The answer is, 832
road.
GP_4458
2019-24 SBI Clerk (Junior Associates) Mains Solved Paper-2019
Series – III pattern: 86. (a) Let the weight of P, Q, R and S are 9x, 7x, 4x and 5x
respectively.
5 11 25 51 93 155 Quantity – I : Total weight of P, Q and R = 225 kg
+6 +14 +26 +42 +62 9x + 7x + 4x = 225
20x = 225
+8 +12 +16 +20 x = 11.25
\ Weight of P = 9x = 101.25 kg
+4 +4 +4 Quantity – II : Total weight of Q, R and S = 180 kg.
The difference of difference is, 8, 12, 16, 20,.... 7x + 4x + 5x = 180
16x = 180
The answer is, 93
x = 11.25
Hence, a < b > c \ Weight of Q = 7x = 78.75 kg.
84. (e) Series – I pattern: Quantity III :
9 × 2 + 1 = 19 Weight of S = 5x
19 × 3 + 2 = 59 = 5 × 11.25 = 56.25 kg
59 × 4 + 3 = 239 Hence, Quantity – I > Quantity – II > Quantity – III
239 × 5 + 4 = 1199 87. (a) From the question,
1199 × 6 + 5 = 7199
The bolded number is not followed the given series.
A D
Series – II pattern:
32 × 0.5 = 16 98.q cm
16 × 1.5 = 24 B C
24 × 2.5 = 60
60 × 3.5 = 210
Area of the square = 98 sq. cm
210 × 4.5 = 945 \ Side of the square = AB = AC
The bolded number is not followed the given series.
98 = 7 2 = cm
Series – III pattern:
72 + 5 = 77
Tg:- @NextGenBankers In ABC, AC = ( AB )2 + ( BC )2
77 + 10 = 87 From the question,
87 + 15 = 102
102 + 20 = 122
AC = (7 2) 2 + (7 2)2
122 + 25 = 147 = 196 = 14
The bolded number is not followed the given series. Diameter of the circle AC = 14 cm
85. (a) Series – I pattern: Quantity – I : Area of the circular region P
78 × 2 = 156 2
æ 14 ö
156 × 3 = 468 = p ç ÷ = 154 sq. cm
è 2ø
468 × 5 = 2340
Quantity – II : Area of the circular region P outside the
2340 × 7 = 16380
square ABCD = 154 – 98 = 56 sq. cm
The bolded number is followed the given series. Quantity – III : Perimeter of the circular region P
Series – II pattern: = p (14) = 44 cm
128 × 0.5 + 1 = 65 So, Quantity – I > Quantity – II > Quantity – III.
65 × 1 + 3 = 68 88. (b) From question Dipak’s investment = `1000
68 × 2 + 5 = 141 Let Panas’s investment is ` x
141 × 4 + 7 = 571 Sanjeet’s investment is ` 2x
571 × 8 + 9 = 4577 Then, 10000 + 2x + x = 22,000
The bolded number is not followed the given series. 3x = 12000
Series – III pattern: x = 4000
\ Panas’s investment = ` 4000
64 × 0.5 + 1 = 33
Sanjeet’s investment = ` 8000
33 × 1 + 1 = 34
and Dipak’s investment = ` 10000
34 × 1.5 + 1 = 52 Ratio of investment = 4000 : 8000 : 10000 = 2 : 4 : 5.
52 × 2 + 1 = 105 Quantity – I : Profit received by Panas in 3 years
105 × 2.5 + 1 = 263.5 2
The bolded number is not followed the given series. = ´ 2750 ´ 3 = ` 1500
(2 + 4 + 5)
SBI Clerk (Junior Associates) Mains Solved Paper-2019 2019-25
Quantity II : Profit received by Snajeet in 18 months. 91. (d) Let speed of boat in still water = x km/hr
4 Speed of current = y km/hr
= ´ 2750 ´ 1.5 = ` 1500 (x – y) × 336 = (x + y) × 180
(2 + 4 + 5)
Quantity III : Profit received by Dipak in 1 year Solving we get the ratio of speed of boat and speed of
current = 28:15
5
= ´ 2700 = ` 1250. 92. (c) Let, length of train Doon Express be l metres
(2 + 4 + 5) And speed of train Doon Express be s Km/h.
\ Quantity I = Quantity II > Quantity III
89. (d) Quantity I:
5
l = (s – 4) × × 32.4
Assume units of work done = 24 [LCM (8, 12)] 18
Þ l = (s – 4) x 9
24 Þ l = 9s – 36 ...(i)
Units of work done by Tomy in a day = =3
8
5
l + 740 = s × × 90
24 18
Units of work done by Jacky in a day = =2
12 Þ l + 740 = 25s
2 days units done = 3 + 2 = 5 Þ l = 25s – 740 ...(ii)
8 days units done = 20 From (i) and (ii)
9 days units done = 23 9s – 36 = 25s – 740
On 10th day work to be done = 24 – 23 = 1 unit Þ 25s – 9s = 740 – 36
Þ 16s = 704
On 10th days as Jacky working, time required = ½ day.
Total time = 9 ½ days. 704
Þs=
Quantity II: 16
Let the total amount be ` x Þ s = 44 Km/h
x From (i)
Udya’s wage per day is l = 9 × 44 – 36
21
Tg:- @NextGenBankers
Muday’s wage per day is
x
20
Þ l = 396 – 36
Þ l = 360 m
Let, required time taken = t seconds
When working together wage paid per day to them is
(I) (360 + 540) = (44 + 36) × 5/18 × t
x x x ´ 41 Þ 900 = 80 × 5/18 × t
+ =
20 21 420
900 18
x ´ 420 Þt= ×
Number of days = x ´ 41 = 10.24 days 80 5
Þ t = 40.5 seconds = 40.5
Hence, Quantity I < Quantity II Þ Satisfies the given condition.
90. (a) Quantity I: 5
(II) (360 + 420) = (44 + 36) × ×t
129 18
SP = × CP1 = ` 12900; CP1 = 10000
100 5
CP of Air Purifier = 12900 × 2 – 10000 = 15800 Þ 780 = 80 × ×t
18
He sold the Air purifier for `12900 780 18
(15800 –12900) Þt= ×
Loss = × 100 80 5
15800 Þ t = 35.1 seconds ¹ 36.3
2900 = Doesn’t satisfy the given condition.
= × 100 = 18.35%
15800 5
Quantity II: (III) (360 + 440) = (44 + 36) × ×t
18
Suppose the initial turnover is x and the compounded 5
annual growth rate is r. Then we have Þ 800 = 80 × ×t
18
3
æ 1+ r ö æ 1+ r ö 3 800 18
3x = x ç ÷ or ç 100 ÷ = 3 = 1.4422 Þt= ×
è 100 ø è ø 80 5
Þ t = 36 seconds = 36
Or r = 44.22%
= Satisfies the given condition.
Hence, Quantity I < Quantity II
GP_4458
2019-26 SBI Clerk (Junior Associates) Mains Solved Paper-2019
the fourth sentence. Sentence (b) is beginning with ‘in addition’ (C) is incorrect as palliative means (of a medicine or
it means another quality of the gurukul system should be the last medical care) relieving pain without dealing with the
sentence. hence the coherent sequence is DACEB. cause of the condition which does not fit here.
116. (e) 117. (a) 118. (c) 119. (d) 120. (b) (D) is incorrect as ‘recur’ does not make any sense here.
121. (c) In the given sentence, only ‘malady, 126. (b) the word ‘embellished’ means ‘make something more
disqualification, convicted’ makes it grammatically attractive by the addition of decorative details or
as well as contextually correct. features’. ‘Simplified’ means ‘make something simpler
(A) is incorrect because ‘repulsed’ does not fit here. or easier to do or understand’; decorated means ‘make
(B) is incorrect as ‘infatuated’ means possessed with an something look more attractive by adding extra items
intense but short-lived passion or admiration for or images to it’; dilapidated means ‘cause something
someone.
to fall into disrepair or ruin’ and tarnished means ‘lose
(C) is the correct alternative among the following as
or cause to lose lustre, especially as a result of
‘malady’ means a disease or ailment, ‘disqualification,
exposure to air or moisture’. Thus, it can be clearly
convicted’ fit here both grammatically and
contextually. be ascertained that the words embellished and
(D) is incorrect as enamoured means be filled with love decorative are synonymous to one another; making
for which does not fit here contextually option (b) the right answer.
122. (d) In the given sentence, only ‘unflinching, by, heralded’ 127. (c) the word gruesome means disgusting or extremely
make it grammatically as well as contextually correct. unpleasant; Sullen means ‘bad-tempered and sulky’
(A) is incorrect because ‘akin’ does not fit here. exhausting means ‘making one feel very tired’;
(B) is incorrect as ‘cognizant’ means having knowledge hideous means ‘extremely ugly or unpleasant’ and
or awareness which does not make any sense here. insulting means to speak to or treat with disrespect or
(C) is incorrect as ‘from’ preposition does not fit here. scornful abuse’. Hence, option (c) is the right answer.
(D) is the correct alternative among the following as 128. (c) the word ‘vivacious’ means ‘lively’ or ‘bright’; Vibrant
‘unflinching, by, heralded’ fit here both grammatically and vivacious are synonymous to each other; hence,
and contextually. option (c) is the right answer.
123. (a) In the given sentence, only ‘tumultuous, emanating, 129. (b) the word ‘ prodigious’ means remarkably or
betrays’ make it grammatically as well as contextually
correct. Tg:- @NextGenBankers
(A) is the correct alternative among the following as
impressively great in extent, size or degree’ is the
synonymous to the word ‘phenomenal’ which means
‘remarkable or exceptional, especially exceptionally
‘emanating’ means (of a feeling, quality, or sensation) good.’ Thus, option (b) is the right answer.
issue or spread out from, ‘tumultuous, betrays’ fit here 130. (c) the word ‘articulate’ means ‘having or showing the
both grammatically and contextually. ability to speak fluently and coherently’, impotent
(B) is incorrect as kempt means (of a person or a place) means ‘unable to take effective action; helpless or
maintained in a neat and clean condition; well cared
powerless’; eloquent means ‘fluent or persuasive in
for which does not make any sense here.
speaking or writing’ and the word ‘hesitant’ means
(C) is incorrect as cluttered means cover or fill (something)
‘tentative, unsure or slow in acting or speaking’; thus,
with an untidy collection of things which does not
make any sense here. ‘eloquent’ is the correct answer.
(D) is incorrect as ‘shroud’ does not fit here contextually 131. (c) the word ‘irk’ means ‘irritate or annoy’. Vex means
124. (d) In the given sentence, only ‘perception, acquired, in’ ‘make someone feel annoyed, frustrated, or worried,
make it grammatically as well as contextually correct. especially with trivial matters’; pique means ‘feel
(A) is incorrect because vaunt means boast about or praise irritated or resentful’; delight means ‘please someone
(something), especially excessively which does not greatly and EXASPERATE means ‘irritate and
fit here. frustrate someone intensely’; thus, option (c) is the
(B) is incorrect as ‘vaunt’ does not make any sense here. right answer.
(C) is incorrect as ‘to’ does not fit here. 132. (a) the word ‘vanity’, haughtiness, self-admiration and
(D) is the correct alternative among the following as ostentation are synonyms of one another which means
‘perception, acquired, in’ fit here both grammatically ‘excessive pride in or admiration of one’s own
and contextually. appearance or achievements’. Humility means ‘the
125. (b) In the given sentence, only ‘rein, abyss, wandering’ quality of having a modest or low view of one’s
make it grammatically as well as contextually correct. importance’; Thus, making option (a) the correct answer.
(A) is incorrect as ‘briskly’ does not make any sense here. 133. (b) insipid means ‘lacking vigour or interest or excitement;
(B) is the correct alternative among the following as dull’; exhilaration means ‘making one feel very happy,
‘abyss’ means a deep or seemingly bottomless chasm animated, or elated; thrilling ‘; vapid means ‘offering
and ‘rein, wandering’ fit here both grammatically and nothing that is stimulating or challenging’ and prosaic
contextually. means ‘lacking imaginativeness or originality’. Hence,
option (b) is the correct answer.
SBI Clerk (Junior Associates) Mains Solved Paper-2019 2019-29
134. (c) Use ‘has’ for a single entity. 152. (d) Amrabad Tiger Reserve spreads over 2,800 sq.kms
Use ‘evasively’ instead of ‘evasive’. in the districts of Mahabubnagar and Nalgonda. Earler,
Use only ‘was ‘instead of ‘was being’ it was part of ‘Nagarjunasagar-Srisailam Tiger reserve’
135. (e) The sentence is error-free and grammatically correct. but post-bifurcation, the northern part of the reserve
136. (a) 137. (b) 138. (d) 139. (b) 140. (b) is vested with Telangana state and renamed as
141. (c) August 19 is observed as World Photography Day, ‘Amrabad Tiger Reserve’. The southern portion
which aims to inspire photographers across the planet
continues to be ‘NSTR’ is with Andhra Pradesh.
to share a single photo with a simple purpose: to share
their world with the world. 153. (b) India’s state-owned banks had classified ` 1.50 trillion
142. (b) Recently, the state-owned Kerala Infrastructure worth of loans as “wilful defaults” in 2018-19, with
Investment Fund Board (KIIFB) debuted its ‘masala the biggest lender State Bank of India accounting for
bond’ issue of ` 2,150 crore on the London Stock nearly a third. The State Bank of India saw the highest
Exchange. KIIFB became the first sub-sovereign entity number of wilful defaults at ` 46,158 crore, while
in India to tap the offshore rupee international bond Punjab National Bank stood second at ` 25,090 crore,
market. with Bank of India at ` 9,890 crore.
143. (d) ‘Host India made most of home’ advantage and 154. (c) RBI has formed a committee to review regulatory
grabbed all 7 gold medals in 21st Commonwealth guidelines and supervisory framework applicable to
Table Tennis Championships 2019 which concluded core investment companies (CICs). This committee
at Jawaharlal Nehru Indoor Stadium in Cuttack, is headed by Tapan Ray.
Odisha. 155. (b) Kaziranga National Park is a national park in the
144. (e) The world of fixed-income securities can be divided Golaghat, Karbi Anglong and Nagaon districts of the
into two main categories. Capital markets consist of state of Assam, India. The sanctuary, which hosts two-
securities with maturities of more than 270 days, while thirds of the world’s great one-horned rhinoceroses,
the money market comprises all fixed-income is a World Heritage Site.
instruments that mature in 270 days or fewer. 156. (c) G-20 Ministerial meeting on Trade and Digital
145. (a) Male is the capital city of Maldives. This country is a Economy was held from 8 June- 9 June 2019 in
small island nation in South Asia, located in the Japanese city of Tsukuba, Ibaraki Prefecture, Japan.
Arabian Sea of the Indian Ocean.
Tg:- @NextGenBankers
146. (e) D V Sadananda Gowda is the present Minister of
Chemicals and Fertilizers in the Government of India.
Piyush Goyal to lead Indian delegation for G20 meet
on trade .
157. (a) The surcharge was levied on individuals with a total
147. (a) Arinjeeta Dey (Barasat, West Bengal) won a silver
income more than ` 1 crore at the rate of 10%.
medal under the 12 category in the World Youth Cup
2019 . The competition was held in Umag, Croatia 158. (c) India’s biggest life insurer LIC or Life Insurance
(from July 1 to July 5, 2019). It was organized by Corporation of India is sitting on over two-third of
World Karate Federation affiliated to the International that unclaimed money while the other 22 private life
Olympic Association. A total of 35 countries insurers account for the rest, according to a PTI report.
participated in this competition. LIC has ` 10,509 crore of unclaimed insurance money
148. (b) The Damanganga-Pinjal river linking project proposes while among private insurers, ICICI Prudential Life
to divert excess water from reservoirs in the Insurance Co. Ltd has ` 807 crore, followed by
Damanganga basin to Mumbai through the Pinjal dam, Reliance Nippon Life Insurance ( ` 696 crore), SBI
built on the Pinjal river in Vaitarna basin. Life Insurance Co ( ` 678 crore) and HDFC Standard
149. (d) The Shanghai Cooperation Organisation (SCO 2019) Life Insurance ( ` 659 crore), says the report.
19th summit was held in Bishkek, Kyrgyzstan. 159. (b) The National Centre for Agricultural Economics and
150. (a) The book titled “War over Words: Censorship in India, Policy Research (NCAP), is a premier agricultural
1930-1960” has been authored by Devika Sethi, who economics research institution in India. The Indian
teaches modern Indian history at IIT-Mandi. The book Council of Agricultural Research (ICAR) established
explores the diverse mechanisms and motivations for NCAP in 1991 with a view to strengthen agricultural
censorship and its role in the shaping of the modern economics research through integration of economics
Indian republic. It looks at state censorship in India input in planning, designing, and evaluation of
in the transition period from colonial to national rule, agricultural research programs and enhancing the
when historical actors behaved in unpredictable ways. competence in agricultural policy analysis within the
151. (b) Kerala government would set up a first-of-its-kind National Agricultural Research System. Its
elephant rehabilitation center here at an outlay of Rs headquarter is located in New Delhi.
105 crore to take care of jumbos, which were either
160. (b) The walled city of Jaipur, known for its iconic
orphaned or abandoned. The rehab centre would house
architectural legacy and vibrant culture, made its entry
an elephant museum, mahout training centre, a well-
equipped veterinary hospital and crematorium for the into the UNESCO World Heritage Site list, becoming
pachyderms. the second city of the country after Ahmedabad to get
the recognition.
GP_4458
2019-30 SBI Clerk (Junior Associates) Mains Solved Paper-2019
161. (a) Deposits in bank accounts opened under Jan Dhan 19 awards. Chhetri was named as Men’s footballer of
scheme have crossed the Rs 1 lakh crore mark. As per the year, while Ashalata Devi won the Women’s
the latest finance ministry data, the total balance in footballer of the year title. The AIFF Player of the
over ` 36.06 crore Pradhan Mantri Jan Dhan Yojana Year is an annual association football award presented
accounts at ` 1,00,495.94 crores The PMJDY to the best footballer in India over the previous year.
launched on August 28, 2014. It has been awarded by All India Football Federation
162. (a) Om Birla is the 17th and current Speaker of the Lok since 1992.
Sabha. He serves a Member of Parliament for the 170. (b) Russian space agency Roscosmos successfully
launched its Soyuz-2.1a carrier rocket on 5 july 2019
Kota-Bundi constituency in Rajasthan. Prior to
with a hydrometeorological satellite and 32 small
parliament, he was elected thrice to the assembly of
satellites.
Rajasthan. He belongs to the Bharatiya Janata Party.
171. (a The 2019 ICC Cricket World Cup was the 12th
163. (c) Reserve Bank of India (RBI) has initiated steps to set Cricket World Cup, a quadr ennial One Day
up wide-based Digital Public Credit Registry (DPCR) International (ODI) cricket tournament contested by
to capture loan information of individuals and men’s national teams and organised by the
corporate borrowers. In this regard, RBI has invited International Cricket Council (ICC). It was hosted
expression of interest (EOI) for developing PSC from between 30 May to 14 July across eleven venues in
companies with turnover of over Rs 100 crore in last England and Wales. The 2019 Cricket World Cup Final
three years. Setting up of PCR assumes significance played at Lord’s in London, England, on 14 July 2019.
amidst rising bad loans in financial system. England became Cricket World Cup winners for the
164. (a) NIFA, a federal agency within the United States first time.
Department of Agriculture (USDA), is part of USDA’s 172. (c) The 2019 Wimbledon Championships was a Grand
Research, Education, and Economics (REE) mission Slam tennis tournament that took place at the All
area. The agency administers federal funding to England Lawn Tennis and Croquet Club in
address the agricultural issues impacting people’s daily Wimbledon, London, United Kingdom. The defending
lives and the nation’s future. Gentlemen’s singles champion Novak Djokovic
165. (e) Kawal Tiger Reserve is located at Jannaram mandal retained his title, while the defending Ladies’ singles
champion Angelique Kerber lost in the second round
Tg:- @NextGenBankers
of Mancherial District (Old Adilabad district) in
Telangana state of India. Govt of India declared Kawal
wildlife sanctuary as Tiger Reserve in 2012. The
to Lauren Davis. Simona Halep won the Ladies’
Singles title.
173. (c) Ajay Singh is associated with boxing. He is currently
reserve is the oldest sanctuary in the northern
the president of the Boxing Federation of India (BFI).
Telangana region of the state. It is well known for its
174. (d) Arjun Munda is a Cabinet Minister of Government of
abundant flora and fauna. This sanctuary is catchment India. He was also Chief Minister of Indian state of
for the rivers Godavari and Kadam, which flow Jharkhand. Khunti is the constituency of Arjun Munda.
towards the south of the sanctuary. Khunti Lok Sabha constituency is one of the fourteen
166. (c) Balphakram National Park is a national park in the Lok Sabha (parliamentary) constituencies in
south of Garo Hills in Meghalaya, India, close to the Jharkhand state in Eastern India. This constituency is
international border with Bangladesh. It was reserved for the candidates belonging to the Scheduled
inaugurated in December 1987 and provides habitat tribes.
for barking deer, Asian golden cat, Bengal tiger, 175. (a) `The National Cricket Academy located in
marbled cat, wild water buffalo, red panda and Indian Chinnaswamy Stadium Bengaluru, Karnataka in India.
elephant. Academy was established in 2000 as a cricket facility
167. (d) Hanoi is the capital of Vietnam. It is the easternmost of the Board of Control for Cricket in India (BCCI) for
country on the Southeast Asian Indochinese Peninsula. training young cricketers who are identified as having
It is the 15th most populous country in the world. the potential to represent the Indian cricket team.
168. (b) On 31 December 2016, at the Prime Minister’s address 176. (a) The Reserve Bank of India (RBI) constituted a task
to the nation, a scheme to benefit pregnant women force to suggest policy and regulatory interventions
and lactating mothers known as the Pradhan Mantri required for development of secondary market in
Matru Vandana Yojana (PMMVY) was announced. corporate loans, including loan transaction platform
for stressed assets. The six-member body, headed by
The scheme came into effect from 1 January 2017 at
Canara Bank chairman T. N. Manoharan, has been
all the districts in India. The total amount payable set up to review the existing state of the market for
under this scheme is Rs 5000. Cash benefits are loan sale/transfer in India as well as international
provided in three instalments of Rs.1,000, Rs.2,000, experience in loan trading.
and Rs.2,000. 177. (a) Civil Aviation Training Center (CATC) is located in
169. (b) Indian men’s football team captain Sunil Chhetri and Iran and have bases in Tehran, Ahwaz, Mashhad,
women’s footballer Ashalata Devi have bagged top Shiraz, Esfahan and Tabriz.
honours at All India Football Federation (AIFF) 2018-
SBI Clerk (Junior Associates) Mains Solved Paper-2019 2019-31
178. (d) Anshula Kant is the chief financial officer and generating employment in rural areas. Under this
managing director of the World Bank Group, mission, Ministry of Micro Small and Medium
appointed on 12 July 2019. She is from Roorkee, India. Enterprise (MSME) will cover 50 clusters across the
179. (a) India has signed Rs.200 crore deal with Russia for country including in the Northeast and each cluster
acquiring Strum Ataka anti-tank missile for its fleet will employ 400 to 2,000 artisans.
of Mi-35 attack choppers of Indian Air Force (IAF). 186. (e) Standard Chartered Bank will become the first foreign
This deal comes in wake of an IAF’s attempt to keep bank to launch operations at International Financial
itself battle-ready in situations like post-Balakot aerial Services Centre (IFSC) located in Gujarat
strikes. International Finance Tec City (GIFT City).
180. (c) University Grants Commission (UGC) has launched 187. (b) Deutsche Bank has reportedly drawn up plans for a
STRIDE scheme recently, aimed at strengthening radical restructuring that will involve the creation of
research culture and innovation in universities and a “bad bank” to hold tens of billions of euros of toxic
colleges of the country. assets and a round of severe cuts to its investment
181. (b) Dutee Chand has won the gold medal in women’s 100- banking operations.
metre sprint at the 30th Summer University Games in 188. (b) The 2023 ICC Cricket World Cup will be the 13th
Napoli, Italy becoming the first Indian to win do so. edition of the men’s Cricket World Cup, scheduled to
182. (c) Pradhan Mantri Karam Yogi Maandhan (PM-KYM) be hosted by India from 9 February to 26 March 2023.
Scheme All PM Karam Yogi Maan Dhan Scheme This will be the first time the competition is held
beneficiaries must have annual turnover less than Rs. completely in India.
1.5 crore. 189. (a) The United Nations Development Programme
183. (b) Every woman in Self-Help Group (SHG) was eligible (UNDP) is the United Nations’ global development
for a loan up to Rs. 1 lakh under the MUDRA Scheme. network. It advocates for change and connects
184. (b) The 2019 BRICS summit was the eleventh annual countries to knowledge, experience and resources to
BRICS summit, an international relations conference help people build a better life for themselves. It
to be attended by the heads of state or heads of provides expert advice, training and grants support to
government of the five member states Brazil, Russia, developing countries, with increasing emphasis on
India, China and South Africa. This summit was held assistance to the least developed countries.
at Brazil. 190. (b) Budget 2019 has proposed to levy tax deduction at
185. (a) President Ram Nath Kovind launched Solar Charkha source (TDS) of 2 per cent on the cash withdrawal of
Tg:- @NextGenBankers
Mission under which Government will disburse
subsidy of Rs 550 crore to thousands of artisans,
more than Rs 1 crore from a bank account.
GP_4458
2019-32 SBI Clerk (Junior Associates) Prelim Solved Paper-2019
Reasoning Ability sit at the extreme end. Two singers sit between Shreya Ghosal
and Quan. Richa Sharma sits immediate left of Shreya Ghosal.
Chitra sits next to Bella Shende.
DIRECTION (Qs. 1-5): Study the following information
6. Who among the following faces Quan?
carefully and answer the questions given below:
(a) Bella Shende (b) Deepa
Eight girls are sitting around a circular table facing to the center (c) Arjit (d) Chitra
but not necessarily in the same order. (e) None of these
Two girls sit between Quan and Priya (either from left or right). 7. Who among the following sits at the extreme end of the row?
Riya sits immediate to the right of Quan. One girl sits between (a) Ed Sheeran (b) Tom
Riya and Suchi, who faces to Tiya. Quan and Tiya are not (c) Chitra (d) Richa Sharma
immediate neighbors of each other. Wahida sits 2nd to the left of (e) Palak
Vinny. Three girls sit between Urkush and Vinny. 8. Four of the following five are alike in a certain way and
1. Four of the following five are alike in a certain way and hence they form a group. Which one of the following does
hence they form a group. Which one of the following does not belong to that group?
not belong to that group? (a) Quan (b) Arjit
(a) Quan-Wahida (b) Priya-Urkush (c) Tom (d) Deepa
(c) Suchi-Wahida (d) Vinny-Tiya (e) Palak
(e) Quan-Priya 9. Who among the following sits 2nd to the right of Palak?
2. Who among the following sits immediate right of Urkush? (a) Shreya Ghosal (b) Richa Sharma
(a) Wahida
(c) Tiya
Tg:- @NextGenBankers
(b) Riya
(d) Suchi
(c) Quan
(e) None of these
(d) Tom
(e) Vinny 10. What is the position of Arjit with respect to Ed Sheeran?
3. The number of girls sit between Quan and Tiya, when counted (a) 3rd to the left (b) Immediate to the left
to right of Quan is same as the number of girls sit between (c) 2nd to the left (d) 3rd to the right
Wahida and ___, when counted to the left of ___? (e) 2nd to the right
(a) Priya (b) Suchi 11. In the word ‘COMMUNICATION’, how many pairs of the
(c) Tiya (d) Urkush letters have the same number of letters between them in the
(e) None of these given word as they have in the English alphabet series?
4. Who among the following faces Riya? (a) Four (b) Two
(a) Urkush (b) Vinny (c) One (d) Three
(c) Priya (d) Wahida (e) More than five
(e) Quan
DIRECTION (Qs. 12-15): In each of the questions below
5. Who among the following sits 3rd to the right of Suchi?
are given some statements followed by some conclusions.
(a) Priya (b) Urkush
You have to take the given statements to be true even if they
(c) Quan (d) Tiya
seem to be at variance with commonly known facts. Read all
(e) None of these
the conclusions and then decide which of the given conclusions
DIRECTION (Qs. 6-10): Study the following information logically follows from the given statements disregarding com-
carefully and answer the questions given below monly known facts.
Ten singers are sitting in two parallel rows containing five singers 12. Statements: All Cherry are Toy. No Toy is Mango. Only a
in each row such a way that there is an equal distance between few Mango are Dew.
adjacent singers. In the first row, Arjit, Belashende, Chitra, Deepa Conclusions: I. No Cherry is Mango
and Ed Sheeran are seated and all of them are facing north. In the II. Some Toy can never be Dew.
second row, Palak, Quan, Richa Sharma, Shreya Ghosal and Tom (a) If only conclusion II follows.
are seated and all of them are facing south. Therefore, in the given (b) If only conclusion I follows.
seating arrangement, each singer seated in a row faces another (c) If neither conclusion I nor II follows.
singer of the other row. Ed Sheeran sits 2nd from one of the extreme (d) If either conclusion I or II follows.
end of the row. Palak faces the one who sits 2nd to the right of Ed (e) If both conclusions I and II follow.
Sheeran. Deepa sits 2nd to the left of Bela Shende, who does not
SBI Clerk (Junior Associates) Prelim Solved Paper-2019 2019-33
13. Statements: No Piano is Ghazal. All Song are Ghazal. Only DIRECTION (Qs. 21-23): Study the following information care-
a few Ghazal are Society. fully and answer the questions given below:
Conclusions: I. Some Society can be Song.
II. Some Piano can be Society. Point D is 30m west of point C. Point C is 60m south of point B.
(a) If either conclusion I or II follows. Point A is 40m west of point B. Point G is 30m south of point A.
(b) If only conclusion I follows. Point E is 45 north of point D.
(c) If neither conclusion I nor II follows. 21. If point F is exactly between point B and C, then how far
and in which direction is point G with respect to F?
(d) If only conclusion II follows.
(a) 15m, North-East (b) 15m, East
(e) If both conclusions I and II follow.
(c) 10m, North-west (d) 40m, West
14. Statements: No Money is House. Only a few House are
Royal. Only a few Royal is Varanda. (e) 20m, North-East
Conclusions: I. Some House are Varanda. 22. Four of the following are alike in a certain way, so form a group.
II. Some Money can never be Royal. Which of the following does not belong to that group?
(a) If only conclusion I follows. (a) A, E (b) G, B
(b) If neither conclusion I nor II follows. (c) C, A (d) D, B
(c) If either conclusion I or II follows. (e) G, E
(d) If only conclusion II follows. 23. If point H is in 5m east of point G, then what is the distance
(e) If both conclusions I and II follow. between point H and Point D?
15. Statements: Some Bat are Godown. All Godown are Table. (a) 10m (b) 30m
Only a few Table are Pipe. (c) 35m (d) 25m
Conclusions: I. Some Bat are Pipe. (e) 20m
II. Some Godown can be Pipe. DIRECTION (Qs. 24-26): Study the following information care-
(a) If only conclusion II follows. fully and answer the questions given below:
(b) If neither conclusion I nor II follows. A certain number of students are sitting in the row. All of them
(c) If either conclusion I or II follows. are facing towards north. B sits sixth from the right of D. E sits
(d) If only conclusion I follows. fourth to left of B. Only two students sit between B and A. C sits
Tg:- @NextGenBankers
(e) If both conclusions I and II follow.
DIRECTION (Qs. 16-20): Study the following information care-
fourth to the left of D. F sits between D and E. G sits second to
the right of F. H is third from any of the end. D is eight from the
fully and answer the questions given below: left end of the row. Six students sit between H and G.
24. How many numbers of students could sit in the row?
Seven women are buying different fruits. Only one women buy
(a) 14 (b) 18
fruits between T and S. U is buying fruits immediately before T.
Two women buy fruits between T and V. T buy fruits after V. W (c) 23 (d) 15
buys fruits immediately before V. More than three women buy fruits (e) 20
between W and X. Only three women buy fruits between V and Y. 25. What is the position of ‘H’ with respect to B?
16. How many women buy fruits between S and X? (a) Fourth to the right (b) Fifth to the right
(a) One (b) Two (c) Fourth to the left (d) Eighth to the right
(c) Three (d) Four (e) Sixth to the left
26. If J sits immediate right of G then how many students sit
(e) More than four
between J and A?
17. Who among the following buy fruit immediately after?
(a) Five (b) Six
(a) V (b) U
(c) Four (d) Three
(c) Y (d) S (e) None of these
(e) No one 27. If it is possible to make only one meaningful word with the 1st,
18. Who among the following buy fruit immediately before Y? 4th, 7th and 11th letters of the word ‘INTERPRETATION’ which
(a) V (b) U would be the second letter of the word from the right end? If
(c) T (d) S more than one such word can be formed give ‘Y’ as the answer.
(e) No one If no such word can be formed, give ‘Z’ as your answer.
19. How many women buy fruits before S? (a) Y (b) E
(a) One (b) Two (c) M (d) T
(c) Three (d) Four (e) Z
(e) More than four DIRECTIONS (Qs. 28-30): In each of the question, relationships
20. As many as women are buying fruits before V as after___? between some elements are shown in the statements. These
(a) W (b) U statements are followed by conclusions numbered I and II. Read
(c) X (d) S the statements and give the answer.
(e) Y
GP_4458
2019-34 SBI Clerk (Junior Associates) Prelim Solved Paper-2019
33. How many such numbers are there in the given series which are
20
10
Arka Billu Chitra Dhruv
immediately preceded by a symbol and followed by a letter? Pratap Sharma Pathak Mehta
(a) None (b) One
(c) Two (d) Three 41. Find average number of movie seen by Arka P., Chitra P. &
(e) Four Dhruv M. in 2017.
34. How many such letters are there in the given series which (a) 64 (b) 70
are immediately preceded by number and immediately (c) 75 (d) 60
followed by a symbol? (e) 56
(a) One (b) Two 42. Find ratio of movie seen by Billu S. & Chitra P. together in
(c) Three (d) More than three 2017 to movie seen by Arka P. & Dhruv M. together in
(e) None of these 2018.
35. Find the odd one out? (a) 15 : 16 (b) 5 : 6
(a) N64 (b) D86 (c) YØ8 (c) 1 : 5 (d) 4 : 7
(d) R27 (e) 2 : 3
(e) 8EL 43. Movie seen by Arka Pratap & Dhruv Mehta together in 2017
are what percent more than movie seen by Chitra Pathak in 2018?
Numerical Ability 2 1
(a) 46 % (b) 54 %
3 3
DIRECTIONS (Qs. 36-40): What will come in the place of 2 1
question mark (?) in the following number series: (c) 25 % (d) 33 %
3 3
36. 16, 19, ?, 32, 44, 61 2
(a) 22 (b) 24 (e) 66 %
3
(c) 25 (d) 23 44. Movie seen by Arka Pratap & Chitra Pathak together in
(e) 21 2017 are how much more or less than movie seen by Billu
37. 5, 9, 16, 29, ?, 103 Sharma & Dhruv Mehta together in 2018?
(a) 54 (b) 60 (a) 24 (b) 14
(c) 68 (d) 58 (c) 18 (d) 22
(e) 62
(e) 28
SBI Clerk (Junior Associates) Prelim Solved Paper-2019 2019-35
45. Movie seen by Billu Sharma & Chitra Pathak together in 56. Find the total distance covered by boat in each upstream
2018 are what percent of movie seen by Billu Sharma in and downstream in 5.5 hours if the speed of boat in still
2017? water and speed of current is 31.5 km/h and 4.5 km/h
(a) 100% (b) 120% respectively?
(c) 250% (d) 200% (a) 380 km (b) 341 km
(c) 315 km (d) 301 km
(e) 160%
(e) 322 km
DIRECTIONS (Qs. 46-55): What will come in place of (?) 57. If the difference of the compound interest received in first
question mark in the following questions? year and second year at 20% per annum at CI is ` 836 then
find the sum?
46. (36% of 630) ÷ 1.4 = ?
(a) ` 25,000 (b) ` 26,000
(a) 156 (b) 160 (c) ` 35,400 (d) ` 18,400
(c) 164 (d) 162 (e) ` 20,900
(e) 172 58. Panas alone can type a book in 12 days while Panas and
47. (2945 + 6295) ÷ ? = 165 Abdul together can type in 7.5 days. Find the time taken by
(a) 52 (b) 44 Rekha alone to do that typing if Rekha takes 3 days more
(c) 56 (d) 58 than that of Abdul alone to do that book typing?
(e) 36 (a) 33 days (b) 30 days
(c) 23 days (d) 27 days
48. 70% of 100 × 12 = ? – 25
(e) 28 days
(a) 865 (b) 932 59. Ratio of income of Akhil to that of Babli is 5:9. If
(c) 864 (d) 862 3
(e) None of these expenditure of Akhil is th of his income and expenditure
8
49. 0.8% of 5550 – 16.4 = ? 4
(a) 26.6 (b) 28 of Babli is th of his income and sum of their saving is
9
(c) 29.2 (d) 30.4 ` 1170 then find the difference between their income?
(e) 32 (a) ` 946 (b) ` 1000
50. 2992 ÷ 17 = ? % of 440
(a) 25
Tg:- @NextGenBankers
(b) 40
(c) ` 788
(e) ` 692
(d) ` 576
67. Raj, Shikha and Dixit invested in a ratio of 7: 8: 5 in a extrapolate the possible health impacts of climate change,
business. They got an annual profit of Rs. 91200. If Raj and WHO estimates that climate change is expected to cause
Dixit withdrew their amount at the end of 3 months and 7 approximately 250,000 additional deaths per year between 2030
months respectively. Then find the difference between Raj and 2050. Of these, 38,000 are likely to occur due to heat
and Dixit’s share of profit? exposure in elderly people 48,000 due to diarrhoea, 60,000 due
(a) ` 8400 (b) ` 9,500 to malaria, and 95,000 due to childhood malnutrition. Areas with
(c) ` 13,500 (d) ` 10,500 weak health infrastructure particularly in developing countries
(e) `.8600 are likely to be the hardest hit. Unfortunately, climate-based
1 healthcare planning ... nowhere in sight when it comes to India.
68. John sold his bycicle at 33 % profit and another at 100% We need to train health personnel on climate change and related
3
profit. Find his overall profit percentage if the selling price health topics and modify the curricula to include climate change
of both the bycicle is same? at the secondary and tertiary levels. Contingency plans should
(a) 60% (b) 58% be designed to deploy health personnel in events of outbreaks
2 and extreme weather events. Investment plans should be modified
(c) 67 % (d) 67% to incorporate potential losses caused due to climate change.
3
71. What are a series of devastating changes in the behaviour
2 of our atmosphere due to climate change?
(e) 56 %
3 (a) an increase in extreme weather events such as floods,
69. A milk tank contains mixture of milk and water in the ratio 4: drought and storms, a threat to biodiversity
1. When 50% of the mixture is taken out and replaced by (b) the possible sinking of coastal areas.
48 liters of water then the ratio of milk to water in the mixture (c) climate change has also been shown to impact mental
becomes 1: 1. Find initial quantity of mixture. health
(a) 180 liters (b) 145 liters (d) None of these
(c) 170 liters (d) 160 liters (e) All (a), (b) and (c)
(e) 175 liters 72. What are the estimates of WHO regarding the adverse
70. 4 years ago, ratio of Arka’s age to Deepak’s age was 2: 3 effects of climate change between 2030 and 2050?
and ratio of Arka’s age 4 years ago to Deepak’s age 5 years (a) WHO estimates that climate change is expected to
(a) 12 years
Tg:- @NextGenBankers
hence is 4 : 9. Find present age of Arka.
(b) 18years
cause approximately 250,000 additional deaths per
year between 2030 and 2050.
(c) 14 years (d) 16 years (b) Of these, 38,000 deaths are likely to occur due to heat
(e) 20 years exposure in elderly people
English Language (c) Areas with weak health infrastructure particularly in
developed countries are likely to be the hardest hit.
DIRECTIONS (Qs. 71-78): Read the following passage and (d) Only (a) and (b)
answer the following questions based on the given passage. Some (e) All the above
of the words are highlighted which would help you to answer 73. What can be inferred from this sentence “the connection
some of the questions given. between climate change and healthcare might seem
far-fetched?”
On a cursory look, the connection between climate change and (a) Climate change is impacting human health.
healthcare might seem far-fetched. Yet, when you look at it (b) The connections between climate change and
closely, you are bound to realize how climate change is impacting healthcare are disproportionate.
human health and how the continued shift will necessitate a (c) Climate-based healthcare planning ... nowhere in sight
change in healthcare delivery approaches. Climate change is also when it comes to India.
bringing in its wake a series of devastating changes in the
(d) The impact of climate change while planning our health
behaviour of our atmosphere. This includes an increase in extreme
strategies.
weather events such as floods, drought and storms, a threat to
(e) All the above
biodiversity as well as the possible sinking of coastal areas. Some
74. Study the following statements
of the major adverse effects of climate change on health include
direct effects like prolonged exposure to high temperatures, (1) Exposure to extreme weather events increases the risk
increase in the incidence of vector-borne diseases and effects of anxiety and depression
that result from social inequality, such as malnutrition. Similarly, (2) India is among the countries most vulnerable to
climate change has also been shown to impact mental health — climate change.
for instance, exposure to extreme weather events increases the (3) Investment plans should be modified to incorporate
risk of anxiety and depression, disproportionately affecting potential losses caused due to climate change
people who already suffer from pre-existing mental disorders. (a) Only (2) (b) Only (1)
This is why there is a need to include the impact of climate change (c) Only (3) (d) Both (1) and (2)
while planning our health strategies. While it is not easy to clearly (e) All the above
SBI Clerk (Junior Associates) Prelim Solved Paper-2019 2019-37
75. What is/are the meaning of the word ‘far-fetched’? (a) Preventing (b) Spread
(a) Unlikely (b) Credible (c) Grow (d) Curb
(c) Implausible (d) Both (a) and (c) (e) None of these
(e) None of these 89. Education is what ............ us from other living beings on earth.
76. Which of the followings is/are the most similar in meaning (a) Differ (b) Mingle
to ‘adverse’ as per the given passage? (c) Differentiates (d) Prevent
(a) Unfavourable (b) Inauspicious (e) None of these
(c) Pleasant (d) Both (a) and (b) 90. It is a doorway to success which requires hard work, dedication
(e) Both (b) and (c) and more after which can you open it ............
77. Which of the followings is/are the most similar in meaning (a) Successful
to ‘extrapolate’ as per the given passage? (b) Successfully
(a) Judge (b) Infer (c) Difficulty
(c) Deduce (d) Both (a) and (b) (d) Decoratively
(e) All (a), (b), (c) (e) None of these
78. Which of the followings is/are the most opposite in meaning
to ‘contingency’ as per the given passage? DIRECTIONS (Qs. 91-95): Given sentences are not in their
(a) Emergency (b) Eventuality exact position. Rearrange them to make a coherent paragraph
(c) Exigency (d) Improbability and then answer the questions given below.
(e) None of these A. It plays a prominent role in all-around development of
individual as well as society.
DIRECTIONS (Qs. 79-85): Read each sentence to find out if B. Education plays a key role in creating patriotic, disciplined
there is any error in it. The error, if any will be in one part of the and productive manpower.
sentence. The number of that part is the answer if there is no C. Educated manpower constitutes precious assets as well as
error; the answer is (e). agents for advancing the nation.
79. It is consider (a)/ that a man who is not (b)/ healthy can D. A large number of books have been written on the
live his life (c)/ in a normal way (d)/ No error. (e) importance of education.
80. I met (a)/ an old lady (b)/ when I will be (c)/ going to the E. Education is fundamental to human progress.
market. (d)/ No error. (e) 91. Which of the following should be the FOURTH sentence
No error. (e)
Tg:- @NextGenBankers
81. This is (a)/ one of the (b)/ most notorious (c) criminal (d)/ after the rearrangement?
(a) A (b) B
82. The world has come (a)/ around to the view that (b)/ (c) C (d) D
democracy is essential for (c)/ full human development (d)/ (e) E
no error (e). 92. Which of the following should be the SECOND sentence
83. Mohan is determined (a)/ to be success (b)/ in all fields after the rearrangement?
(c)/ he chooses. (d)/ No error. (e) (a) A (b) B
84. Democracy cannot be sustained (a) /unless the electorate (c) C (d) D
is well informed, (b) / chooses its leadership wisely, and (e) E
this leadership is intellectually (c)/empowered by a multi- 93. Which of the following should be the THIRD sentence
dimensional intelligence (d)/ no error (e). after the rearrangement?
85. Not only (a)/ Johan’s friends but also (b)/ Johan were(c)/ (a) A (b) B
laughing at the joke he cracked.(d)/No error (e) (c) C (d) D
(e) E
DIRECTIONS (Qs. 86-90): In each of the following sentences, 94. Which of the following should be the FIRST sentence after
there is a blank space. Below in each sentence, there are five the rearrangement?
words out of which one can be used to fill the blank to make the (a) A (b) B
sentence grammatically and coherently correct. Find the most (c) C (d) D
appropriate word that fit into the blank contextually. (e) E
86. We started noticing the climatic change and its .... on human life. 95. Which of the following should be the FIFTH sentence after
(a) Affect (b) Effect the rearrangement?
(c) Adverse (d) Loss (a) A (b) B
(e) None of these (c) C (d) D
87. People in India earn their ......... by involving themselves in (e) E
many of these activities. DIRECTIONS (Qs. 96-100): In the following questions, a
(a) Life (b) Lively sentence is divided into four parts consisting of a highlighted
(c) Livelihood (d) Income word in each part. Choose the option reflecting the word which
(e) Work is either misspelt or grammatically incorrect. If all the highlighted
88. The government identified these problems as hindering the words are correct, choose option (e) i.e. “all are correct” as
economic growth of the country and established policies to your answer choice.
...... them.
GP_4458
2019-38 SBI Clerk (Junior Associates) Prelim Solved Paper-2019
96. The average age for a child to be approapriate to work is 99. I could not wear to see her punished. It wasn’t the first
considered fifteen years and more. time she had seen a bear track, but it was the first time
(a) Average (b) Considered she had scene one that fresh.
(c) Approapriate (d) More (a) Wear (b) Bear
(e) All are correct (c) First (d) Scene
97. If we wish to iradicate child labour, we need to formulate (e) Both (a) and (d)
some very effective solutions which will save our children. 100. It seemed very doubtful whether he would accomplice his
(a) Iradicate (b) Formulate desire.
(c) Effective (d) Children (a) Seemed (b) Whether
(e) All are correct (c) Accomplice (d) Desire
98. In a democratic age, when the duty of forming clear (e) All are correct
opinions on practical and social problems devolves on all
of us, th e study of history should be regarded as
emperative.
(a) Democratic (b) Opinions
(c) Devolves (d) Emperative
(e) All are correct
ANSW ER KEY
1 (a) 11 (e) 21 (d ) 31 (d ) 41 (b ) 51 (e) 61 (d) 71 (e) 81 (d) 91 (b)
2 (c) 12 (b ) 22 (c) 32 (d ) 42 (a) 52 (d) 62 (e) 72 (d) 82 (e) 92 (a)
3 (b ) 13 (e) 23 (b ) 33 (b ) 43 (e) 53 (c) 63 (a) 73 (b ) 83 (b) 93 (d)
4 (c) 14 (b ) 24 (e) 34 (a) 44 (b ) 54 (a) 64 (c) 74 (e) 84 (e) 94 (e)
5 (b ) 15 (a) 25 (a) 35 (e) 45 (c) 55 (b) 65 (a) 75 (d) 85 (c) 95 (c)
6 (c) 16 (c) 26 (c) 36 (b ) 46 (d ) 56 (b) 66 (c) 76 (d) 86 (b) 96 (c)
7
8
(b )
(e)
17
18
(e)
(c)
Tg:- @NextGenBankers
27
28
(a)
(e)
37
38
(a)
(b )
47
48
(c)
(a)
57
58
(e)
(c)
67
68
(a)
(a)
77
78
(e)
(d)
87
88
(c)
(d)
97
98
(a)
(e)
9 (a) 19 (b ) 29 (a) 39 (e) 49 (b ) 59 (d) 69 (d) 79 (a) 89 (c) 99 (c)
10 (d ) 20 (e) 30 (b ) 40 (d ) 50 (b ) 60 (b) 70 (d) 80 (c) 90 (b) 10 0 (c)
Toy 30 m
G F 60 m
13. (e)
Song D C
Piano Society 30 m
22. (c) All except (c) is in north-east direction with respect to
Ghazal Ist element
E
14. (b)
45 m
A 40 m
Manoj House Royal Varanda B
30 m
15. (a)
G 60 m
Bat
Tg:- @NextGenBankers
Godown
Pipe
Table D C
30 m
23. (b) Take a point B', as BC = 60 m. So, B'D = 60 m
Sol. (16-20):
Women We have AG = 30 m
So, B'H = 30 m
W
Hence HD = B'D – B'H = 60 – 30 = 30 m
V
E
S
U 45m
T A 40m B' B
Y
X 30m
G H 60m
16. (c) 17. (e) 18. (c) 19. (b) 20. (e) 5m
D C
30m
Sol. (24-26)
C D F E G B A H
3
1 +1
2
2 +2 Tg:- @NextGenBankers
3 +3
3 2
4 +4 5 +5
3 2
6 +6 51. (e)
100
700
´ 2800 ´
1
16
= (?) 2
1
14 74. (e) All the above (all the options are given in the passage)
´ 91200 75. (d) Both (a) and (c) the word ‘far-fetched’ means ‘unlikely’
152
and ‘implausible’ but the word ‘credible’ is opposite
= ` 8400 in meaning.
68. (a) Let SP of both bycycle 76. (d) Both (a) and (b) the word ‘adverse’ mean s
ATQ, ‘unfavourable’ and ‘inauspicious’ but the word
bycycle 1 bycycle 2 ‘pleasant’ is opposite in meaning.
6x 4x 77. (e) All the given three words are similar to the word
CP = 10
´
3 1 ‘extrapolate’.
´
4 2 78. (d) The words emergency, eventually, and exigency are
SP 8x 8x = 16
synonymous to the word ‘contingency’ only the word
16 x –10 x ‘improbability’ is opposite in meaning.
Profit % = ´100 = 60% 79. (a) There is an error in part (a) of the sentence in place of
10 x
69. (d) Let initial quantity of milk and water in the mixture be “consider” we will use “considered”.
‘40x liters’ and ‘10x liters’ respectively. 80. (c) There is an error in part (c) of the sentence.
In place of “will be” we will use “was” because the
ATQ,
events of the sentences are in past.
æ 1ö 81. (d) The word ‘criminal’ should be replaced with
ç 40x ´ ÷ 1 ‘criminals’.
è 2ø 20x 1
=
1 Þ 5x + 48 = 1 82. (e) There is no error in the sentence.
10 ´ + 24 1 83. (b) There is an error in part (b) of the sentence. Use “to
2
succeed” in place of “to be success” because after
20x = 5x + 48 “determined” we use “to infinitive”.
15x = 48 84. (e) There is no error in the sentence.
x = 3.2 85. (c) Replace ‘were’ with ‘was’ as we are considering only
So, required quantity = 40x + 10x second subject which is singular in number.
Reasoning Ability & Computer Aptitude 5. Which of the following go to the industrial city?
(a) C and E (b) C and A
DIRECTIONS (Qs. 1-5): Study the information given below and (c) A and E (d) Either A or B
answer the questions based on it. (e) None of these
8-person A, B, C, D, E, F, G and H are sitting around a circle
facing the centre and having discussion to go on a trip during DIRECTIONS (Qs. 6-10): Study the following information and
answer the questions.
different months viz, January, march, may, June, August,
October, November and December respectively but not in the Eight friends Hemant, Charu, Nitesh, Gaurav, Bharat, Pravin,
same order. 3 of them want to go to hill stations, 3 to historical Deepak and Isha are sitting in a row facing north. All of them
places and 2 to industrial city. D sits second to the left of F, like different months, viz January, February, March, April, May,
who wishes to go to a hill station during a month which has June, July and August but not necessarily in the same order.
30 days. Exactly one person sit between G and H and G, H • There is only one person between Nitesh and the one
wants to go to an historical place in the month of June and who likes July.
August respectively. H, who is between D and F and the one • Deepak is neither an immediate neighbour of Nitesh nor
who is interested in going to industrial city sit diagonally
he likes April.
opposite. D and B neither wish to go to historical places nor
• Hemant sits fourth to the left of the one who likes
in the month of October and December. The one who goes in
Tg:- @NextGenBankers
June sits diagonally opposite to the one who wants to go in
the month of October. D is sitting opposite to the person who •
July but he does not like February.
The person who likes June is the third to the right of the
visits historical places. E is interested in going in the month of one who likes April and sits on the immediate right of
May. G is to the third to the left of F. B, who is immediate left Hemant.
of G, goes to hill station. Person going to visit industrial city • The one who likes April sits at one of the extreme ends
goes in October. of the row. Charu does not like April.
1. Who goes to Industrial city? • Pravin is an immediate neighbour of both Deepak and
(a) A (b) E Nitesh.
(c) C (d) Cannot be determined • Isha sits at one of the extreme ends of the row but she
(e) None of these. does not like April.
2. If C goes in October than in which month will A go? • The one who likes August sits second to the right of the
(a) December (b) January one who likes March.
(c) March (d) Either A or B • The one who likes June and February are immediate
(e) None of these neighbours.
3. Which of the following is definitely true? • Bharat sits third to left of Nitesh and likes May.
(a) E goes to historical place in may • There is only one person between the persons who like
(b) B goes to hill station in January May and June.
(c) F goes to hill station in November 6. How many persons are there between Gaurav and Charu?
(d) D goes to hill station in March (a) Two (b) Three
(e) None of these (c) Four (d) Five
4. Which of the combination is correct? (e) None of these
(a) D- march-historical place 7. Who among the following is an immediate neighbour of
(b) B-January-hill station the persons who like June and May?
(c) D-march-hill station (a) Deepak (b) Gaurav
(d) B-march-hill station (c) Hemant (d) Bharat
(e) None of these (e) None of these
GP_4458
2018-2 SBI Clerk (Junior Associates) Mains Solved Paper-2018
8. Who among the following likes March? 15. Who among the following doesn’t belong to the group?
(a) Nitesh (b) Pravin (a) O (b) P
(c) Deepak (d) Bharat (c) T (d) N
(e) None of these (e) M
9. Who among the following sits on the immediate left of
DIRECTIONS (Qs. 16-20): Study the information given below
the one who likes August?
and answer the questions based on it.
(a) The one who likes March
(b) Either Pravin or Nitesh Eight people M, N, O, P, Q, R, S and T are sitting around a
circular table. Four of them face towards the center and the rest
(c) The one who likes July
face outside the center. Each of them has different choice of
(d) The one who likes June
mobile brands- Apple, Sony, Nokia, HTC, LG, LYF, XOLO and
(e) None of these VIVO, but not necessarily in the same order. There are three
10. Who among the following sits third to the right of the male members and five female members amongst them.
one who likes January?
· Each male is sitting between two female members.
(a) Nitesh (b) Gaurav
(c) Deepak (d) Pravin · The One who likes HTC is sitting third to the right of O,
who is facing the center.
(e) None of these
· P is facing the same direction as O and also he is an
DIRECTIONS (Qs. 11-15): Study the information given below immediate neighbor of both M and the one who likes
and answer the questions based on it. LYF.
Eight persons M, N, O, P, Q, R, S and T are staying on an eight · T and N face in the same direction i.e., towards the
storey building. Lowermost floor is numbered 1 and topmost center.
floor is numbered 8.
· P is sitting exactly opposite to N, who likes Apple.
Note: Their ages are in increasing order from bottom to top
floor. The one who lives on even numbered floor his/her age · Both Q and R are immediate neighbors of N and neither
is an even number and the one who lives on odd numbered of them likes LYF.
Tg:- @NextGenBankers
floor his/her age is an odd number.
Q is 19 years old and two persons are staying between Q and
·
·
T is an immediate neighbor of Q and she likes Nokia.
O is a XOLO player and she is sitting third to the left of
the one who is 30 years old. R is the youngest person in the the one who likes Sony.
group and the one who is the eldest person is 46 years old. · P is the only male who is facing inside.
M is elder to Q and one person is between M and N. S is
35 years old. The difference of age between M and P is · The One who likes Sony is sitting second to the right of
10 years. Two persons are between T and P. The number of the one who likes VIVO.
person lives above S is same as below P. One person is 16 16. Who is sitting second to the left of the one who likes
years old. T is not 29 years old. The one who is youngest is HTC?
11 years old. (No two persons have the same age) (a) Q
11. What is the age of T? (b) S
(a) 29 years (b) 25 years (c) The one who likes Sony
(c) 27 years (d) 31 years
(d) The one who likes LYF
(e) 30 years
(e) Both (a) and (c)
12. Who among the following is the eldest person?
17. Four of the five are alike in a certain way. Find the odd
(a) O (b) S
one out?
(c) T (d) R
(a) MRQ (b) QSR
(e) M
(c) TOP (d) PQM
13. Which of the following combination is correct?
(e) OTN
(a) O-6th floor
18. Who likes LG?
(b) T-28 years old
(a) M (b) N
(c) R-3rd floor
(c) O (d) P
(d) N-6th floor
(e) Q
(e) P-18 years old
19. Which of the following combination is correct?
14. How many persons live between N and Q? (a) M-Nokia (b) S-LG
(a) 0 (b) 1 (c) R-VIVO (d) Q-LYF
(c) 2 (d) 3 (e) T-XOLO
(e) 4
SBI Clerk (Junior Associates) Mains Solved Paper-2018 2018-3
20. Who among the following is second to the left of the one 26. What is the code for ‘Graham’?
who likes XOLO? (a) !X3 (b) @X3
(a) M (b) O (c) #Y3 (d) #X3
(c) P (d) Q (e) None of these
(e) N
27. What is the code for ‘Firing not done’?
DIRECTIONS (Qs. 21-25): Read the following information (a) @G3 !E1 %K2 (b) !G3 %E1 @K2
carefully and answer the given questions.
(c) %G3 @E1 !K2 (d) @H3 !F1 %L4
Seven players A, B, C, D, E, F and G are playing for different (e) None of these
International team – India, Pakistan, Australia, Sri Lanka, South
Africa, New Zealand and West Indies. But not necessarily in 28. What is the code for ‘Jingoism’?
the same order. They are specialized in different forms, viz. All (a) @M4 (b) @K4
Rounder, Batsman and Bowler. At least two players are (c) #K4 (d) #M4
specialized in the same forms. F is playing for Sri Lanka and (e) None of these
he is not an All-Rounder. D is playing from Australia and he
is an All-Rounder. The one who is playing for New Zealand is 29. ‘%E5’ could be a code for which of the following?
not a bowler. A is not playing for West Indies and he is not (a) Forget (b) Ultraviolet
an All-Rounder. E is playing for Pakistan but he is not an (c) Letter (d) Ultracrepidarian
All-Rounder. G is playing for South Africa and he is a batsman (e) None of these
along with only B. No one playing for India and West Indies.
as is an All-Rounder. 30. ‘Sweet Raining’ could be a code for which of the
following?
21. Which of the following represent the group of players
that are bowlers? (a) %T2 !P3 (b) !T2 %P3
(a) C, F, G (b) A, F, G (c) %T3 !P2 (d) %T4 !P2
(c) A, F, E (d) A, B, D (e) None of these
(e) None of these
Tg:- @NextGenBankers
22. In which form does ‘C’ Specialized?
DIRECTIONS (Qs. 31-35): In each question below is given a
group of letters followed by four combinations of digits/
(a) All-Rounder (b) Bowler symbols numbered (a), (b), (c) and (d). You have to find out
(c) Batsman (d) Can’t be determined which of the combinations correctly represents the group of
letters based on the following coding system and mark the
(e) None of these
number of that combination as the answer. If none of the four
23. ‘B’ plays for which International team? combinations correctly represents the group of letters, mark
(a) New Zealand (b) Srilanka (e) i.e. ‘None of the above’, as the answer.
(c) West Indies (d) Australia
(e) India Letter M K H R B N E T D J I A Q F P
24. Which of the following combination is true? Digit / symbol 8 1 Ó 2 # £ 7 9 @ 6 % 3 4 $ *
(a) A – West Indies – Bowler
Conditions:
(b) B – New Zealand – Batsman
1) If the first and the last letters are consonants, both are
(c) C – New Zealand – Bowler
to be coded as the code for the first letter.
(d) E – Pakistan – Batsman
(e) None of these 2) If the first letter is a consonant and the last letter is as
25. Who among the following plays for New Zealand? vowel their codes are to be interchanged.
(a) D (b) C 3) If both the first and the last letters are vowels, both are
(c) B (d) A to be coded as the last letter.
(e) None of these 31. TABDRE
DIRECTIONS (Qs. 26-30): Study the following information (a) 93#2@7 (b) 73#@29
arrangement carefully and answer the questions. (c) 93#@27 (d) 73#@27
‘alarm forest cuddle moving’ is written as ‘%G3 !C3 #X2 @U3’, (e) None of the above
‘sight fire making criticism’ is written as ‘#P4 @G2 %R2 !N3’, 32. QHFKAN
‘raising centre recent alarm’ is written as ‘@K3 %V3 #X2 !P3’ (a) 4©$134 (b) 4©$138
‘strike arm ignoring slight’ is written as ‘!K4 %P3 @G3 #L1’ (c) £©$134 (d) 4©$431
(e) None of the above
GP_4458
2018-4 SBI Clerk (Junior Associates) Mains Solved Paper-2018
33. IAJEPI 40. Four of the following five are alike in a certain way and
(a) *%367$ (b) %367%* thus form a group. Which is the one that does not
belong to that group?
(c) %367*% (d) 6%367*
(a) C (b) D
(e) None of the above
(c) R (d) E
34. NR, JTBP
(e) P
(a) £26#9@ (b) £%269£
(c) £962#© (d) £269#£ DIRECTIONS (Qs. 41-45): Study the given information and
answer the questions:
(e) None of the above
When a word and number arrangement machine is given an
35. JATHQE
input line of words and numbers, it arranges them following a
(a) 739©46 (b) 639©47 particular rule. The following is an illustration of input and its
(c) 639©46 (d) 739©47 rearrangement.
(e) None of the above Input: what 42 has 54 also 18 not 12 gone 26
Step I: 12 what 42 has 54 18 not gone 26 also
DIRECTIONS (Qs. 36-40): Study the following information
Step II: 18 12 what 42 has 54 not 26 also gone
carefully and answer the questions given below:
Step III: 26 18 12 what 42 54 not also gone has
Ten persons are sitting in two parallel rows containing five
Step IV: 42 26 18 12 what 54 also gone has not
persons each in such a way that there is an equal distance
Step V: 54 42 26 18 12 also gone has not what
between adjacent persons. In the first row, A, B, C, D and E
are seated and all of them are facing south. In the 2nd row, P, Step V is the last step of the above arrangement as the
Q, R, S and T are seated and all of them are facing north. intended arrangement is obtained.
Therefore, in the given seating arrangement, each member As per the rules followed in the above steps, find out in each
seated in a row faces another member of the other row. S sits of the following questions the appropriate step for the given
third to the left of P. A faces an immediate neighbor of S. C sits input.
Tg:- @NextGenBankers
second to the right of A. Only one person sits between B and
D. Q and T are immediate neighbours. T does not face A and
Input: fly 92 high 48 bird 74 tree 62 green 56
41. Which element is exactly between ‘92’ and ‘74’ in Step
B. III?
36. How many persons are seated between B and E? (a) Green (b) Bird
(a) None (c) Tree (d) High
(b) One (e) None of these
42. What is the difference between the number which is
(c) Two
fourth from left end in Step III and the number which is
(d) Three second from the right end in Step I?
(e) Can’t be determined (a) 23 (b) 24
37. Who amongst the following faces D? (c) 29 (d) 25
(a) P (b) Q (e) 36
(c) R (d) S 43. How many steps are required to complete the above
(e) T arrangements?
38. Which of the following is true regarding Q? (a) Four (b) Seven
(a) P and R are immediate neighbour of Q. (c) Six (d) Five
(b) Q sits at one of the extreme ends of the line. (e) None of these
44. In which step the elements “48 92 tree” found in the same
(c) E is an immediate neighbor of the person who is
order?
facing Q.
(a) Step IV (b) Step V
(d) S sits on the immediate left of Q.
(c) Step II (d) Step I
(e) None of these
(e) Step III
39. Who amongst the following are sitting exactly in the
45. In step V, ‘92’ is related to ‘tree’ and ‘74’ is related to
middle of the rows?
‘high’. In the same way ‘62’ is related to?
(a) A and T (b) D and S
(a) High (b) Green
(c) D and P (d) P and C
(c) Bird (d) 56
(e) A and Q (e) 48
SBI Clerk (Junior Associates) Mains Solved Paper-2018 2018-5
DIRECTIONS (Qs. 46 and 47): Each question below is followed 48. Statement: Should all the electricity state boards be
by two assumptions numbered I and II. You have to decide which privatized in India?
of the assumption is implicit in the statement. Arguments
Give answer I. No, this will increase the grievances of the people.
(a) If only assumption I is implicit II. Yes, it will check the growing menace of power theft
which has resulted in annual pilferage of a huge
(b) If only assumption II is implicit
amount.
(c) If either I or II is implicit
49. Statement: Should the private companies be allowed to
(d) If neither I nor II is implicit operate passenger train services in India?
(e) If both I and II are implicit Arguments
46. Statement: Government should deploy army to rehabilitate I. Yes, this will improve the quality of service in Indian
the people displaced due to earthquake. Railways as it will have to face severe competition.
Assumptions II. No, the private companies may not agree to operate
I. Army can be used for purposes other than war also. in the non-profitable sectors.
II. Only army can rehabilitate the displaced victims
DIRECTION (Q. 50): In the question below, are given a
of earthquake.
statement followed by two courses of action numbered I and
47. Statement: A warning in a train compartment- “To II. You have to decide which of the suggested courses of
stop train, pull chain. Penalty for improper use ` 500.” action logically follow(s) for pursuing.
Assumptions Give answer:
I. Some people may misuse the chain. (a) if only I follows.
II. On certain occasion, people may want to stop a (b) if only II follows.
running train.
(c) if either I or II follows.
DIRECTIONS (Qs. 48 and 49): Each question below is (d) if neither I nor II follows.
Tg:- @NextGenBankers
followed by two arguments numbered I and II. You have to
decide which of the arguments is ‘strong’. 50.
(e) if both I and II follow.
Statement: A number of school children in the local
Give answer: schools have fallen ill after the consumption of their
(a) if only argument I is strong. subsidized tiffin provided by the school authority.
(b) if only argument II is strong. Course of action
I. The tiffin facility of all schools should
(c) if either I or II is strong.
be discontinued with immediate effect.
(d) if neither I nor II is strong.
II. The government should implement a system to
(e) if both I and II are strong. certify the quality of tiffin provided by the school.
Quantitative Aptitude
DIRECTIONS (Qs. 51-55): Study the given table carefully to answer the following questions.
Field Name Shape Side (in m) Base (in m) Height (in m) Radius (in m) Cost of flooring Cost of Fencing
(in ` Per sq meter) (in ` Per sq m)
A Triangle 16 12 50 20
B Rectangle 10 ´ 20 30 15
C Square 15 40 18
D Parallelogram 20 12 60 25
E Circle 10 45 22
51. What is the cost flooring of A 52. What is the difference between the cost of fencing of C
(a) ` 4000 (b) ` 4600 and that of B ?
(c) ` 4800 (d) ` 5000 (a) ` 180 (b) ` 120
(e) ` 4400 (c) ` 240 (d) ` 360
(e) ` 480
GP_4458
2018-6 SBI Clerk (Junior Associates) Mains Solved Paper-2018
53. What is the ratio of the cost of flooring to that of fencing (a) if a > b
of field D ? (b) if a > b
(a) 4 : 1 (b) 6 : 1 (c) if a = b or no relation can be established between
(c) 8 : 1 (d) 9 : 1 a and b.
(e) 5 : 1 (d) if a < b
54. The cost of fencing of field E is approximately what (e) if a < b
percent of the cost of flooring of field C ? 61. I. a² – 11a + 24 = 0
(a) 10.5% (b) 19.46% II. b² + 3b – 18 = 0
(c) 18.71% (d) 15.36% 62. I. 2a² + 17a + 35 = 0
(e) 13.82% II. 3b² + 17b + 24 = 0
55. The cost of fencing of field C is what percent of the cost 63. I. a² + 72 = 108
of flooring of field ? II. b³ + 581 = 365
(a) 54% (b) 12% 64. The concentration of Alcohol in 60L water and Alcohol
(c) 13% (d) 36% solution is 30%. If 6L of solution replaced by water and
(e) 5% 5L of resulting solution again replaced by water, then
56. A shopkeeper gives 10% discount on marked price of an find the concentration of Alcohol in final solution.
article and earns a profit of 25% on it. If cost price of the (a) 24.75% (b) 25%
article is ` 2160, then find the marked price of the article? (c) 23.5% (d) 22.2%
(a) ` 2500 (b) ` 2700 (e) None of these
(c) ` 3500 (d) ` 3000 65. Two friends P and Q are running in opposite direction
(e) ` 4500 from A to B and another from B to A. Speed of P is 8/
57. The ratio between present ages of Ritu and Priya is 3 : 5 of speed of Q. After meeting each other, if P takes 3.5
5. After 6 years, this ratio becomes 2 : 3. Find the present hours to reach B then what will be the time taken by Q
age of Priya ? to reach A after meeting?
(a) 25 years
(c) 30 years
Tg:- @NextGenBankers
(b) 32 years
(d) 27 years
(a) 8. 96 hours
(c) 9. 20 hours
(b) 9 hours
(d) 10 hours
(e) 10. 20 hours
(e) 35 years
66. The ratio between two numbers is 3 : 5. If the smaller
58. The speed of a boat in still water is 12 kmph. If the boat
number is increased by 20% and the bigger one is
covers 36 km upstream in 4 hours, what is the speed of
decreased by 25%, the new ratio of numbers (smaller:
stream?
bigger) will be –
(a) 3 km/hr (b) 4 km/hr (a) 25 : 24 (b) 24 : 25
(c) 5 km/hr (d) 2 km/hr (c) 23 : 24 (d) 24 : 23
(e) 6 km/hr (e) 11 : 13
59. The difference between C.I. and S.I. on a certain sum at 67. Mr. Aditya’s age is 120% of his wife Sunita’s age. They
the rate of 10% per annum for 2 years is ` 122. Find that have two children. The average age of family is 20 years.
sum (in `). If Sunita’s age is 25 years, what is the average age of
(a) 12,000 (b) 12,400 both children?
(c) 12,200 (d) 13,400 (a) 13 years (b) 12.5 years
(e) 14,600 (c) 14.5 years (d) 15 years
60. X and Y started a business with capitals ` 20000 and ` (e) None of these
25000. After few months Z joined them with a capital of 68. A man can swim at 8 kmph in still water. He covers an
` 30000. If the share of Z in the annual profit of ` 50000 equal distance of 18 km both in upstream and downstream
is ` 14000, then after how many months from the between two points in 6 hours. Find the speed of stream.
beginning did Z join? (a) 5 km/h (b) 6 km/h
(a) 7 (b) 6 (c) 7 km/h (d) 4 km/h
(c) 3 (d) 4 (e) None of these
(e) 5 DIRECTIONS (Qs. 69 & 70): Each question below is followed
DIRECTIONS (Qs. 61-63): In each of these questions, two by two statements A and B. You have to determine whether the
equations (I) and (II) are given. You have to solve both the data given in the statement is sufficient for answering the
equations and give answer accordingly. question.
SBI Clerk (Junior Associates) Mains Solved Paper-2018 2018-7
81. What is the number of male tourists who visited of Goa 87. What was the value of output per tree for coconuts?
and MP together? (a) ` 36 (b) ` 360
(a) 632 (b) 674 (c) ` 3,600 (d) ` 240
(c) 684 (d) 649 (e) None of these
(e) None of these
88. What was the amount received by Gopal in 1997?
82. The number of female tourists who visited MP are what
(a) ` 1.5 lakh (b) ` 3 lakh
per cent of the total number of tourists who visited all (c) ` 6 lakh (d) Cannot be determined
states together? (e) None of these
(a) 7 % (b) 5 % 89. What was the value of output per acre of the lemon tree
(c) 19 % (d) 15 % planted?
(e) 11 % (a) 0.24 lakh/acre (b) 2.4 lakh/acre
83. What is the respective ratio of the number of female (c) 24 lakh/acres (d) Cannot be determined
tourists who visited Uttarakhand and Jammu & Kashmir (e) None of these
together to the total number of tourist who visited above 90. What was the total output of coconuts?
mentioned two states together? (a) ` 24,000 (b) ` 36,000
(a) 35 : 33 (b) 23 : 51 (c) ` 18,000 (d) ` 48,000
(c) 22 : 31 (d) 3 : 7 (e) None of these
(e) None of these DIRECTIONS (Qs. 91-95): Study the bar graph given below
84. What is the approximate average no. of male tourists who and answer the following questions:
visited all the states together? The bar graph below shows the percentage break-up of the
(a) 212 (b) 210 number of students who visited various tourist spots on New
(c) 223 (d) 208 Year Evening.
(e) 206
Class X Class XI Class XII
85.
Tg:- @NextGenBankers
The number of male tourist who visited Goa are what per
cent of total number of tourist who visited Goa? 60
5 6 50
(a) 92 % (b) 92 %
7 7 40
6 6
(c) 94 % (d) 95 %
7 7 30
6
(e) 94 % 20
7
10
DIRECTIONS (Qs. 86-90): Answer the questions based of the Ooty Shimla Manali Goa Nainital
following information. (150) (200) (250) (350) (200)
Krishna distributed 10-acre land to Gopal and Ram who paid The table below shows the total number of students in
him the total amount in the ratio 2 : 3. Gopal invested a further classes X, XI and XII.
` 2 lakh in the land and planted coconut and lemon trees in
the ratio 5 : 1 on equal areas of land. There were a total of 100 Class No.of students
lemon trees. The cost of one coconut was ` 5. The crop took X 420
7 yr to mature and when the crop was reaped in 1997, the total XI 480
revenue generated was 25% of the total amount put in by
XII 400
Gopal and Ram together. The revenue generated from the
coconut and lemon trees was in the ratio 3 : 2 and it was shared
Note: No Student went to more than one place.
equally by Gopal and Ram as the initial amounts spent by them
91. If the ratio of girls and boys, who went to Manali from
were equal.
class X, was 7 : 8, then what percent of the total number
86. What was the ratio of yield per acre of land for coconuts of students from class X who went to a tourist spot is
and lemons (in terms of number of lemons and coconuts)? the number of girls who went to Manali? (approximate)
(a) 3 : 2 (b) 2 : 3 (a) 19% (b) 23%
(c) 1 : 1 (d) Cannot be determined (c) 25% (d) 15%
(e) None of these (e) 21%
SBI Clerk (Junior Associates) Mains Solved Paper-2018 2018-9
92. What is the difference between the number of students 100. Quantity I: A man on tour travels first 160 km at
who did not got to any tourist spotfrom class XI and that 64 km/hr and the next 160 km at 80 km/hr. The average
from class XII? speed of the tour is:
(a) 19 (b) 41
Quantity II: A went from P to Q with the speed of 60km/
(c) 27 (d) 21
hr. and return back with the speed of 90km/hr. Find the
(e) None of these
average speed.
93. If all the students from class XII who did not go to any
tourist spot, later changed their mind and went to
Nainital, then calculate the % mark-up in the number of
General English
students who visited Nainital? DIRECTIONS (Qs. 101-110): Read the passage given below
(a) 30% (b) 25.5% and answer the questions that follow based on the information
(c) 35.5% (d) 42.6% given in the passage.
(e) None of these
94. The number of students from class XI and XII together Right through history, imperial powers have clung to their
who visited Manali is what percent of the number of possessions to death. Why, then, did Britain in 1947 give up the
students from class X who visited Shimla, Ooty and jewel in its crown, India? For many reasons. The independence
Nainital together? (approximate) struggle exposed the hollowness of the white man's burden.
(a) 81% (b) 72% Provincial self-rule since 1935 paved the way for full self-rule.
(c) 75% (d) 77% Churchill resisted independence, but the Labour Government of
(e) 70% Atlee was anti-imperialist by ideology. Finally, the Royal Indian
95. Find the total number of students who didn’t visit any Navy Mutiny in 1946 raised fears of a second Sepoy Mutiny,
tourist spot? and convinced British waverers that it was safer to withdraw
(a) 152 (b) 165 gracefully. But politico-military explanations are not enough. The
(c) 105 (d) 150 basis of empire was always money. The end of empire had much
(e) None of these to do with the fact that British imperialism had ceased to be
DIRECTIONS (Qs. 96-100): In the following questions two profitable. World War II left Britain victorious but deeply indebted,
Tg:- @NextGenBankers
statements (Quantity -I and Quantity -II) are given. You have
to solve both the statements and give answer.
needing Marshall Aid and loans from the World Bank. This
constituted a strong financial case for ending the no longer-
(a) Quantity I > Quantity II profitable empire.
(b) Quantity I > Quantity II Empire building is expensive. The US is spending one billion
(c) Quantity II > Quantity I dollar a day in operations in Iraq that fall well short of fullscale
(d) Quantity II > Quantity I imperialism. Through the centuries, empire building was costly,
(e) Quantity I = Quantity II or Relation cannot be yet constantly undertaken because it promised high returns. The
established investment was in armies and conquest. The returns came through
96. Quantity I: The age of teacher, if the average age of 36 plunder and taxes from the conquered. No immorality was attached
students is 14. When teacher’s age is included the to imperial loot and plunder. The biggest conquerors were
average increases by 1.
typically revered (hence titles like Alexander the Great, Akbar the
Quantity II: The age of teacher, if the average age of 19
Great, and Peter the Great). The bigger and richer the empire, the
students is 35. When teacher’s age is included the
more the plunderer was admired. This mindset gradually changed
average increases by 0.5.
with the rise of new ideas about equality and governing for the
97. Quantity I: Profit Percentage, if Some articles were
bought at 6 articles for ` 5 and sold at 5 articles for ` 6. public good, ideas that culminated in the French and the American
Quantity II: Profit Percentage, if 100 toys are bought at Revolutions. Robert Clive was impeached for making a little
the rate of ` 350 and sold at the rate of ` 48 per dozen. money on the side, and so was Warren Hastings. The white
98. Quantity I: On selling 17 balls at ` 720, there is a loss man's burden came up as a new moral rationale for conquest. It
equal to the cost price of 5 balls. The cost price of a ball was supposedly for the The Princeton Review CAT sample paper
is: 12 good of the conquered. This led to much muddled hypocricy.
Quantity II: A man buys a cycle for ` 1400 and sells it On the one hand, the empire needed to be profitable. On the
at a loss of 15%. The selling price is: other hand, the white man's burden made brazen loot impossible.
99. Quantity I: A and B together can do a piece of work in An additional factor deterring loot was the 1857 Sepoy Mutiny.
4 days. If A alone can do the same work in 6 days, then Though crushed, it reminded the British vividly that they were a
B alone can do the same work in? tiny ethnic group who could not rule a gigantic subcontinent
Quantity II: A can do a piece of work in 4 hours; B and without the support of important locals. After 1857, the British
C together can do it in 3 hours, while A and C together can stopped annexing one princely state after another, and instead
do it in 2 hours. How long will B alone take to do it? treated the princes as allies. Land revenue was fixed in absolute
GP_4458
2018-10 SBI Clerk (Junior Associates) Mains Solved Paper-2018
terms, partly to prevent local unrest and partly to promote the 102. Which of the following best expresses the main purpose of
notion of the white man's burden. The empire proclaimed itself to the author?
be a protector of the Indian peasant against exploitation by Indian (a) To present the various reasons that can lead to the
elites. This was denounced as hypocrisy by nationalists like collapse of an empire and the granting of
Dadabhai Naoroji in the 19th century, who complained that land independence to the subjects of an empire.
taxes led to an enormous drain from India to Britain. Objective (b) To point out the critical role played by the 'white man's
calculations by historians like Angus Maddison suggest a drain burden' in making a colonizing power give up its claims
to native possessions.
of perhaps 1.6 percent of Indian Gross National Product in the
19th century. (c) To highlight the contradictory impulse underpinning
empire building which is a costly business but very
But land revenue was more or less fixed by the Raj in absolute attractive at the same time.
terms, and so its real value diminished rapidly with inflation in
(d) To illustrate how erosion of the financial basis of an
the 20th century. By World War II, India had ceased to be a profit empire supports the granting of independence to an
centre for the British Empire. Historically, conquered nations paid empire's constituents.
taxes to finance fresh wars of the conqueror. India itself was (e) None of these
asked to pay a large sum at the end of World War I to help repair
103. What was the main lesson the British learned from the Sepoy
Britain's finances. Mutiny of 1857?
But, as shown by historian Indivar Kamtekar, the independence (a) That the local princes were allies, not foes.
movement led by Gandhiji changed the political landscape, and
(b) That the land revenue from India would decline
made mass-taxation of India increasingly difficult. By World War dramatically.
II, this had become politically impossible. Far from taxing India to
(c) That the British were a small ethnic group.
pay for World War II, Britain actually began paying India for its
contribution of men and goods. Troops from white dominions (d) That India would be increasingly difficult to rule. The
Princeton Review CAT sample paper 13
like Australia, Canada and New Zealand were paid for entirely by
these countries, but Indian costs were shared by the British (e) None of these
government. Britain paid in the form of non-convertible sterling 104. Which of the following best captures the meaning of the
Tg:- @NextGenBankers
balances, which mounted swiftly. The conqueror was paying the
conquered, undercutting the profitability on which all empire is
'white man's burden', as it is used by the author?
(a) The British claim to a civilizing mission directed at
founded. Churchill opposed this, and wanted to tax India rather ensuring the good of the natives.
than owe it money. (b) The inspiration for the French and the American
Revolutions.
But he was overruled by Indian hands, who said India would
resist payment, and paralyze the war effort. Leo Amery, Secretary (c) The resource drain that had to be borne by the home
country's white population.
of State for India, said that when you are driving in a taxi to the
station to catch a life-or-death train, you do not loudly announce (d) An imperative that made open looting of resources
impossible.
that you have doubts whether to pay the fare. Thus, World War
(e) None of these
II converted India from a debtor to a creditor with over one billion
pound in sterling balances. Britain, meanwhile, became the biggest 105. Why didn't Britain tax India to finance its World War II
efforts?
debtor in the world. It's not worth ruling over people who are
afraid to tax. (a) Australia, Canada and New Zealand had offered to
pay for the Indian troops.
101. Which of the following was NOT a reason for the emergence (b) India had already paid a sufficiently large sum during
of the 'white man's burden' as a new rationale for empire World War I.
building in India? (c) It was afraid that if India refused to pay, Britain's war
(a) The emergence of the idea of the public good as an efforts would be jeopardised.
element of governance. (d) The British empire was built on the premise that the
conqueror pays the conquered.
(b) The decreasing returns from imperial loot and
(e) None of these
increasing costs of conquest.
(c) The weakening of the immorality attached to an DIRECTIONS (Qs. 106-108) : Choose the word which is opposite
in meaning to the word printed in bold as used in the passage.
emperor's looting behaviour.
106. proclaim
(d) A growing awareness of the idea of equality among
(a) declare (b) clarion
peoples.
(c) trumpet (d) predicate
(e) None of these (e) deny
SBI Clerk (Junior Associates) Mains Solved Paper-2018 2018-11
107. debtor growth, these are also pictures of inequality that point to how
(a) loanee (b) drawee well these children will be able to contribute to India's economic
(c) mortgagor (d) defaulter growth and their own prosperity. Their poor nutrition stunts more
(e) mortagagee than their bodies. It stunts their well-being, and, consequentially,
108. hypocrisy that of their home States and their nation. Two other things, both
related to inequality, stand out as well in the data. First, the data
(a) glibness (b) phoniness
point to tremendous variability across States in delivering what
(c) honesty (d) quackery
should be universal, rights-based and already mandated health
(e) deceit and nutrition services. For example, sample this for intra-State
DIRECTIONS (Qs. 109-110): Choose the word which is most disparity. Food provided by the Integrated Child Development
similar in meaning to the word printed in bold as used in the Services reaches barely 1 in 5 children in Uttar Pradesh but over
passage 90 per cent in Odisha. Less than 33 per cent of children in Nagaland
109. imperialism are fully immunised; in Goa, it is more than 90 per cent. Close to
(a) developement (b) quackery 80 per cent defecate in the open in Odisha; barely 2 per cent do
so in Kerala. Why?
(c) underprogress (d) failure
These are disparities across States that operate in the same
(e) None of these
national framework, and there is, let's face it, no good reason for
110. fresh this other than an inability or an unwillingness to invest in
(a) new (b) old changing ground realities, for everyone and everywhere. This is
(c) medium (d) light not an insurmountable challenge and it's certainly an area where
(e) None of these States can, if they want it, make dramatic change in short
timeframes. Examples abound from within India. The data in the
DIRECTIONS (Qs. 111-114): Read the following passage reports show that, clearly, the imperative for introspection, and
carefully and answer the questions given after the passage. looking within for solutions was never clearer. No child should
The Global Nutrition Report (GNR) and India Health Report on go without basic health care, food security and things like water
and a toilet. Indeed, no adult should either. No society should
Tg:- @NextGenBankers
Nutrition, 2015 (IHR), offer a critical analysis of the state of nutrition
in India. The first report, the India Health Report: Nutrition
2015(IHR), provides easy-to-understand, State-wise data
condone such inequalities in the basics. And no society has
progressed without addressing these basics.
111. What are the disparities that states are facing in the national
dashboards that give a comprehensive view of nutrition and its
framework?
determinants. It looks at disparities in these outcomes and their
I. Health and nutirition services
multiple determinants across geographical regions, socio-
II. Variable food services
economic classes, and demographic groups to help identify
III. Inequality in education
strategic choices for policy-making at the State level.
In turn, GNR assesses progress in reducing malnutrition for all (a) III (b) II & III
193 countries. It concludes that while India is on track to meeting (c) I & II (d) I,II & III
only two of the eight global targets on nutrition, it has significantly (e) none of the above
improved its nutrition performance in the past 10 years. GNR 112. The Global Nutrition Report focuses mainly on the
notes that there has been a big increase in the number of countries (a) Sustainable Development Goals
on track to meet global nutrition targets, and encourages (b) comprehensive view of nutrition and its determinants
countries, including India, to establish specific and time-bound (c) India's economic growth
targets for malnutrition reduction that are consistent with the (d) Intra-State variability
new Sustainable Development Goals. Together, these reports paint (e) Progress in reducing malnutrition
several pictures about India, a data-poor country. They portray 113. Which of the following would be the suitable title?
one of great progress in improving nutrition across India; stunting (a) State of nutrition in India
among children, a marker of the most persistent types of (b) Global nutrition targets
(c) Nutrition and its determinants
malnutrition, has declined rapidly in the last ten years. And this (d) Progressive India
decline has been faster than in many other countries. But as we (e) Inequality: A big challenge
dig deeper, there are diverse pictures about the life conditions of 114. According to author what is the biggest challenge that
Indian children - positive stories about children's lives and futures India is facing?
in Goa, Kerala, Manipur and Tamil Nadu, but dismal ones in Bihar, (a) Reducing progress in malnutrition
Jharkhand and Uttar Pradesh. What cannot be debated is the (b) Inability to invest in education and health services
reality of deep, systemic inequality; of inequality in the (c) unwillingness and inability to invest in basic facilities
(d) High global nutrition targets
circumstances that children are born into, that they live and grow
(e) None of these
in. For those of us who are worried about India's economic
GP_4458
2018-12 SBI Clerk (Junior Associates) Mains Solved Paper-2018
DIRECTIONS (Q.115): Each question below contains a 120. After the West Asia crisis, the World Bank conducted a
statement with three blanks followed by four options. Choose study on the major reasons for the crisis. It was found that
which words can fill the three blanks and mark it as your answer. at least a major part of the fundamental responsibility was
115. The creation of NITI Aayog was ______ to be a game on banks, which had understated their non performing ac-
changer, to ______ new vigour and rigour in the policy counts by as much as 47%. Since this was a study and not
planning process, involve key stakeholders, and address an investigation____________ Nevertheless, the Ramsel
the ______ of the previous body. committee on supervision did take cognizance, and issued
(a) proposed, absorb, challenges circulars and directives not only on supervision, but also
(b) proposed, address, success on Internal Functional Management. It will be remembered
(c) expected, infuse, failures by those interested that Basel committee had also acted
expeditiously after the Barring Bank's failure, to separate
(d) supposed, implant, accomplishment
treasury and lending operations from the decision making
(e) considered, deter, drawbacks
processes. Bank failures are nothing new in the world, al-
116. The Competition Commission with a _____ to protect the
consumer from industry _____ has been fully functional though we in India have been insulated from such traumas
for eight years now and has _____ a good reputation for for more than two decades.
itself. (a) The findings were not taken note of
(a) goal, monsters, performed (b) The findings were not taken seriously
(b) command, monopoly, priced (c) The findings were not legally binding on any one
(c) mandate, cartelization, earned (d) The fallout from this revelation was only taken note of
(d) score, growth, reached (e) The fallout from this revelation was seriously taken
(e) goal, threats, gained 121. Whether the Government is right in bailing out a private
117. The _____ for debt-waiver programmes comes from the sector bank is an issue that is decided more than by the
theoretical argument that a high level of outstanding debt long term social security policy of the Government, than
_____ the incentive for the debtor to ____ effort to repay.
by economic reasons alone ____________ Nevertheless,
(a) support, reduces, exert
Tg:- @NextGenBankers
(b) opposition, increases, increase
(c) disapproval, forces, minimize
in a situation of scarcity of resources, bailing out some-
body means the denial of resources to others. The irony of
it is that in performing its duties of proper governance to
(d) notice, moves, creditors
the larger society through the process of bailing out, Gov-
(e) help, hinder, toil
ernment excuses the lack of corporate governance in banks.
118. With weapons stolen from the government, a group of
(a) Economists world over learnt it hard way during the
______ begin to unleash _____on citizens worldwide and
plan to get rich in the process-until a young man Great depression
accidentally finds the kill switch to _____ their plans. (b) This is elementary principle of economics taught in
(a) attacker, lawfulness, devise schools
(b) gangsters, havoc, succeed (c) Reasons are not limited to these two but extend to
(c) warrior, freedom, withdraw debts, liquidity & credit ratings issues
(d) belligerents, anarchy, destroy (d) It was unexpected and came like a bolt from the blue
(e) hooligans, laxity ,ruin (e) Particularly true for the Asian countries like India and
119. Even now, after the chaos caused by India’s decision last China
November to _____ nearly 90% of its banknotes, few people 122. But no depreciation is allowed on Live Stock i.e. Horses.
would argue with the policy’s ______ assumption: Going Although the horses are in the nature of fixed assets in the
cashless is, if _____ well, a good thing.
hands of the owner, no depreciation is allowed under In-
(a) reduce, hidden, transferred
come Tax Act. Instead when the animal dies or becomes
(b) eliminate, underlying, handled
permanently useless the entire value of the horse can be
(c) find, dominating, performed written off as revenue loss in the year in which it dies or
(d) hide, valued, viewed becomes permanently useless. When the gross income
(e) defunct, inherent, wielded exceeds the total expenditure, it results in net profit which
DIRECTIONS (Qs. 120-124): In each of the following questions will be taxable at usual rates of tax applicable to the person.
a short passage is given with one of the lines in the passage ____________ Although the live stock is in the nature of
missing and represented by a blank. Select the best out of the fixed assets of the owners buy them, maintain them, train
five answer choices given, to make the passage complete and them, and participate in races and Sell them or send them
coherent. away to studs when they are useless.
SBI Clerk (Junior Associates) Mains Solved Paper-2018 2018-13
(a) But when the gross income is less than the expendi- DIRECTIONS (Qs. 125-129): For each of the following
ture, then results it in loss questions, a part or the whole of the original sentence has been
(b) But when the gross income is higher than the expen- bold. You have to find the best way of writing the bold part of
diture, then results in loss the sentence. If the sentence is grammatically correct, mark
(c) But when the gross income is equal to expenditure option (e) .i.e. "No correction required".
then result is loss
125. The concerned matter was referred back to the sports com-
(d) But when the gross income is there loss is the result mittee since the solution to the problem was different from
(e) But when the gross income is increasing then result is the one proposed earlier.
becoming evident (a) referred back to the sports committee since the solu-
123. Aggregation of risks is somewhat quite new to banks in tion in the problem was different from the one pro-
India. While some banks have started thinking in that line posed earlier.
by trying to put integrated limits framework and integrated (b) referred to the sports committee since the solution to
risk policies as well as using CBS solutions for technologi- the problem was different from the one proposed ear-
cal integration, the effort required is beyond such require- lier.
ment. Risk aggregation would mean aggregating the indi- (c) referred back to the sports committee since the solu-
vidual risk measures to decide most appropriate assets class tion to the problem was different than the one pro-
that would contain the risk to the desired level dictated by posed earlier.
the risk appetite. Capital allocation (about how much) would (d) referred to committee since the solution to the prob-
be based on such strategies ____________ lem was different than the one proposed earlier.
(a) Most banks are yet to conceptualize the same in their (e) No correction required
processes 126. Armed with the talents of a high intellect, an actor with
(b) Most banks have already integrated it in their func- extraordinary gifts and an ingenious criminal, Charles
tioning; it is working over the years satisfactorily. Sobhraj played an overbearing role in the sensationalisation
(c) Which would in long run prove to be the growth im- of crime during the later part of the 20th century in the
peding Indian sub continent.
(a) an actor with extraordinary gifts of an ingenious crimi-
Indian banks Tg:- @NextGenBankers
(d) Of risk aggregation which is really a new concept to
nal
(b) an ingenious actor and an extraordinarily gifted crimi-
(e) On expected lines of the regulation conditions laid
nal
down in the manual of the bank
(c) a gifted actor and an ingeniously criminal
124. However, it is possible that the non-resident entity may
(d) an extraordinarily gifted actor and an ingenious crimi-
have a business connection with the resident Indian en-
nal
tity. In such a case, the resident Indian entity could be (e) No correction required
treated as Permanent Establishment of the nonresident 127. The smaller firms in any industry sell either on a price or
entity. ____________. During the last decade or so, India quality-of-workmanship basis.
has seen a steady growth of outsourcing of business pro- (a) The smaller firms in any industry sells either on a price
cesses by non-residents or foreign companies to IT-en- or quality-of-workmanship basis.
abled entities in India. Such entities are either branches or (b) The smaller firms in any industry either sell on a price
or quality-of-workmanship basis.
associated enterprises of the foreign enterprise or an inde-
(c) The smaller firms in any industry sell on either a price
pendent India enterprise. The nonresident entity or for- or a quality-of-workmanship basis.
eign company will be liable to tax in India only if the IT- (d) The smaller firms in any industry sell on either a price
enabled BPO unit in India constitutes its Permanent Estab- or on a quality-of-workmanship basis.
lishment. (e) No correction required
(a) The tax treatment of the Permanent Establishment in 128. Current economic conditions demand that we not only cut
such a case is under consideration jobs and prices but also reduce the rate of interest on PF
(b) How would the profit would be shared is not decided deposits.
(a) that we not only cut jobs and prices but also
yet?
(b) not only cutting job and prices but also to
(c) A lengthy and cumber some process requiring a lot of (c) not only to cut jobs and prices but also
application of mind and revenue principles is ahead (d) not only a cut in jobs and prices but also to
for the tax department of India (e) No correction required
(d) A new trend is seen in last decade. 129. All-rounders in any cricket team, in theory, make good
(e) Indian companies have a lot on stake as competition sense; in actuality, however, they are normally difficult to
increases. discover.
GP_4458
2018-14 SBI Clerk (Junior Associates) Mains Solved Paper-2018
161. The 2026 FIFA World Cup will be the 23rd FIFA World Cup, 170. The Department of Telecom has approved merger of Telenor
the quadrennial international men's football championship India with which company?
contested by the national teams of the member associa- (a) Idea (b) Airtel
tions of FIFA. Which country will host the FIFA World (c) Vodafone (d) Jio
Cup 2026? (e) BSNL
(a) Japan, China, India 171. Shujaat Bukhari was a journalist and the editor of which
(b) France, Germany, UK Newspaper?
(c) Brazil, USA, Russia (a) The Times of India (b) The Hindu
(d) United States, Canada, Mexico (c) Hindustan Times (d) The Economic Times
(e) Italy, Canada, Scotland (e) Rising Kashmir
162. Bhumi Pednekar has received Dadasaheb Phalke awards 172. Which Institute has launched a Bharat Inclusion Initiative
2018, given by Dadasaheb Phalke Film Foundation for which to build knowledge and foster innovation and entrepre-
film? neurial activity across areas such as financial inclusion,
(a) Toilet: Ek Prem Katha livelihood, education and health?
(b) Shubh Mangal Saavdhan (a) IIM-Kolkata (b) Indian Institute of Science
(c) Lust Stories (c) IIT-Delhi (d) IIM-Ahmedabad
(d) Dum Laga Ke Haisha (e) IGNOU
(e) Sonchiraiya 173. International Conference on Telecommunications (ICT) was
163. Which Bank was takeover by LIC? held in which country?
(a) Punjab National Bank (b) State Bank of India (a) Nepal (b) UAE
(c) IDBI Bank (d) Bank of Baroda (c) India (d) China
(e) ICICI Bank (e) Russia
164. Government of India has designated _________________ 174. _____________ brand diminishes the value of the car for
as the "Statistics Day" in the category of Special Days to which the title is issued because the brand generally re-
be celebrated every year at the National level. flects its damaged condition, the cost of needed repairs
(a) 27th June (b) 08th June and its roadworthiness.
(c) 12th June (d) 21st June (a) Worthiness (b) Uneconomical
(e) 29th June (c) Salvage (d) Branding
165. In which city ASEAN India Film Festival was held recently? (e) None of the given options is true
(a) Bengaluru
(c) Mumbai
Tg:- @NextGenBankers
(b) New Delhi
(d) Goa
175. NABARD has released how much fund (amount) in 2017-18?
(a) Rs 40,000 crore (b) Rs 65,000 crore
(e) None of these (c) Rs 25,000 crore (d) Rs 80,000 crore
166. The United Nations Educational, Scientific and Cultural (e) Rs 10,000 crore
Organization (UNESCO) finally declared the city's Victo- 176. If someone cannot pat EMI, what is it for the Bank?
rian and Art Deco Ensembles a World Heritage Site. It is (a) Credit Risk (b) Market Risk
located in- (c) Bad Loans (d) NPA
(a) Hyderabad (b) Patna (e) Interest
(c) Mumbai (d) Kolkata
177. In which state, "BHAROSA" has launched?
(e) Guwahati
167. In which state, Pilibhit Tiger Reserve is located? (a) Rajasthan (b) Telangana
(a) Karnataka (b) Uttarakhand (c) Kerala (d) Madhya Pradesh
(c) Maharashtra (d) Odisha (e) Gujarat
(e) Uttar Pradesh 178. Hirakud Dam is built across the Mahanadi River in-
168. The two day 'Kabir Mahotsav' organized by the _________ (a) Rajasthan (b) Telangana
to commemorate the 500th year of death anniversary (punya (c) Odisha (d) Madhya Pradesh
tithi) of 15th century poet-saint Kabir concludes today with (e) Gujarat
much fanfare in Maghar, Sant Kabir Nagar District of Uttar 179. Where is the Headquarters of Asian Development Bank
Pradesh. (ADB)?
(a) Ministry of Culture
(b) Ministry of External Affairs (a) New Delhi, India
(c) Home Ministry (b) Tokyo, Japan
(d) Ministry for Development of North Eastern Region (c) Beijing, China
(e) Ministry of Human Resource Development (d) Manila, Philippines
169. Small Industries Development Bank of India (SIDBI) has (e) Jakarta, Indonesia
charted a plan to promote micro enterprises in _________ 180. Dudhwa National Park is a national park in the Terai of-
aspirational districts across the country to contribute in (a) Uttar Pradesh
their development. (b) Kerala
(a) 125 (b) 105 (c) Bihar
(c) 110 (d) 115
(e) 120 (d) Madhya Pradesh
(e) Assam
SBI Clerk (Junior Associates) Mains Solved Paper-2018 2018-17
181. 5th International Conference on Human Excellence through controversial deal on Assumption Island, where India wants
Yoga (2019) will be held in- to build a strategic facility.
(a) Karnataka (b) Kerala (a) $1500 million (b) $300 million
(c) Uttarakhand (d) Maharashtra (c) $500 million (d) $1000 million
(e) Himachal Pradesh (e) $100 million
182. World Micro, Small and Medium-sized Enterprises 187. How much amount will be expand by NABARD in the cur-
(MSMEs) Day is celebrated each year on- rent fiscal (2018-19) as part of its effort to improve the rural
(a) 27th June (b) 23rd June economy?
(c) 19th June (d) 12th June (a) ` 80,000 crore (b) ` 10,000 crore
(e) 05th June (c) ` 60,000 crore (d) ` 50,000 crore
183. What is the meaning of "L" in LSR? (e) ` 30,000 crore
(a) Legal (b) Ledger 188. The 2018 ICC Women's World Twenty20 will be hosted in-
(c) Limit (d) Liquidity (a) South Africa (b) Australia
(e) Least (c) West Indies (d) Sri Lanka
184. The Union Cabinet Chaired by Prime Minister Shri Narendra (e) England
Modi has approved the establishment of National Institute 189. SKOCH has conferred the 'Best performing Social Sector
of Mental Health Rehabilitation (NIMHR) at- Ministry' award on the Ministry of?
(a) Dispur (b) Bhopal (a) Ministry of Consumer Affairs, Food and Public Distri-
(c) Lucknow (d) Patna bution
(e) Jaipur (b) Ministry of Women and Child Development
185. Lusaka is the capital of- (c) Ministry of External Affairs
(a) Kenya (b) South Sudan (d) Ministry of Finance
(c) Yemen (d) Iraq (e) Ministry of Human Resource Development
(e) Zambia 190. The National Digital library of India is a project under Minis-
186. India has announced _______________ million line of
Tg:- @NextGenBankers
credit to Seychelles to strengthen its defence capabilities
and maritime infrastructure even as the two countries de-
try of Human Resource Development, India. It is located in-
(a) Ajmer (b) Surat
(c) Varanasi (d) Kharagpur
cided to work together in the interest of each other on the (e) Darbhanga
ANSW ER KEY
1 (d) 21 (c) 41 (d) 61 (b) 81 (c) 101 (a) 121 (a) 141 (d) 161 (d) 181 (c)
2 (a) 22 (a) 42 (e) 62 (e) 82 (e) 102 (d) 122 (a) 142 (a) 162 (a) 182 (a)
3 (c) 23 (c) 43 (d) 63 (b) 83 (c) 103 (c) 123 (a) 143 (c) 163 (c) 183 (d)
4 (e) 24 (e) 44 (a) 64 (a) 84 (c) 104 (a) 124 (a) 144 (e) 164 (e) 184 (b)
5 (e) 25 (b) 45 (b) 65 (a) 85 (b) 105 (c) 125 (b) 145 (b) 165 (b) 185 (e)
6 (a) 26 (d) 46 (a) 66 (b) 86 (d) 106 (e) 126 (d) 146 (e) 166 (c) 186 (e)
7 (c) 27 (b) 47 (e) 67 (b) 87 (b) 107 (e) 127 (c) 147 (d) 167 (e) 187 (a)
8 (b) 28 (c) 48 (b) 68 (d) 88 (a) 108 (c) 128 (e) 148 (c) 168 (a) 188 (c)
9 (c) 29 (b) 49 (a) 69 (d) 89 (a) 109 (a) 129 (c) 149 (b) 169 (d) 189 (b)
10 (d) 30 (a) 50 (b) 70 (a) 90 (b) 110 (a) 130 (e) 150 (d) 170 (b) 190 (d)
11 (c) 31 (b) 51 (c) 71 (b) 91 (a) 111 (c) 131 (c) 151 (b) 171 (e)
12 (a) 32 (a) 52 (a) 72 (c) 92 (b) 112 (e) 132 (a) 152 (d) 172 (d)
13 (d) 33 (c) 53 (d) 73 (d) 93 (c) 113 (a) 133 (d) 153 (a) 173 (a)
14 (c) 34 (d) 54 (d) 74 (b) 94 (d) 114 (c) 134 (e) 154 (e) 174 (c)
15 (c) 35 (a) 55 (b) 75 (c) 95 (d) 115 (c) 135 (b) 155 (c) 175 (b)
16 (e) 36 (c) 56 (d) 76 (d) 96 (a) 116 (c) 136 (b) 156 (a) 176 (a)
17 (d) 37 (e) 57 (c) 77 (b) 97 (a) 117 (a) 137 (b) 157 (b) 177 (b)
18 (d) 38 (d) 58 (a) 78 (c) 98 (c) 118 (d) 138 (c) 158 (c) 178 (c)
19 (c) 39 (e) 59 (c) 79 (b) 99 (a) 119 (b) 139 (c) 159 (b) 179 (d)
20 (e) 40 (b) 60 (e) 80 (d) 100 (c) 120 (d) 140 (b) 160 (a) 180 (a)
GP_4458
2018-18 SBI Clerk (Junior Associates) Mains Solved Paper-2018
53. (d) Required Ratio = (20 × 12) × 60 : 2 (20 + 12) × 25 61. (b)
= 14400 : 1600 = 9 : 1 I. a2 – 11a + 24 = 0
a2 – 8a – 3a + 24 = 0
22
54. (d) Cost of fencing of E = 2 × × 10 × 22 = ` 1382.85 a(a – 8) – 3 (a – 8) = 0
7
(a – 8) (a – 3) = 0
Cost of flooring of C = 15 × 15 × 40 = ` 9000.
a = 8, 3
1382.85 II. b2 + 3b – 18 = 0
Required % = × 100 = 15.36 %
9000 b2 + 6b – 3b – 18 = 0
b(b + 6) – 3 (b + 6) = 0
15 ´ 4 ´18 ´100
55. (b) Required % = = 12 % (b – 3) (b + 6) = 0
(40 ´15 ´ 15)
b = 3, – 6
56. (d) Let marked price be `x a ³ b
9x 62. (e)
\ Selling price = I. 2a2 + 17a + 35 = 0
100
2a2 +10a + 7a + 35 = 0
125 2a (a + 5) + 7 (a + 5) = 0
But selling price = 2160 × = ` 2700
100 (2a + 7) (a + 5) = 0
100 7
\ Marked price = 2700 × = ` 3000 a= - , –5
90 2
57. (c) Let present ages of Ritu and Priya are 3x and 5x II. 3b2 + 17b + 24 = 0
respectively. 3b2 + 9b + 8b + 24 = 0
3x + 6 2 3b (b + 3) + 8 (b + 3) = 0
ATQ, = Þ 9x + 18 = 10x + 12 (b + 3) (3b + 8) = 0
Þ
Tg:- @NextGenBankers
5x + 6 3
x = 6 years b = –3, –
8
\ Present age of Priya = 30 years 3
58. (a) Let speed of stream = S km/hr b>a
\ (12 – S) × 4 = 36 63. (b)
Þ S = 3 km/hr I. a2 + 72 = 108
59. (c) Let sum be ` P a2 = 108 – 72 = 36
a = +6
éæ 10 ö
2 ù P ´10 ´ 2 II. b3 + 581 = 365
\ P êç1 + ÷ - 1ú - = 122
êëè 100 ø úû 100 b3 = –216
b=–6
P (21 - 20) a ³ b
Þ = 122 Þ P = ` 12,200
100 64. (a) The concentration of Alcohol in final solution,
60. (e) Let the investments of X, Y and Z respectively be
3 æ 6 ö æ 5 ö
20000, 25000 and 30000 ´ ç1 - ÷ ´ ç 1- ÷ ´100 = 24.75 %
10 è 60 ø è 60 ø
Let the investment period of Z be x months.
Ratio of annual investments Hence, (A) is the correct option.
(20000 × 12): (25000 × 12) : (30000 × x) = 8 : 10 : x S1 t t
8
The share of Z in the annual profit of ` 50000 is 65. (a) We know that = 2 = = 2
S2 t1 5 3.5
` 14000
æ x ö 64 t2
çè ÷ × 50000 = 14000 = = Þ t2 = 8.96 hours
x + 18 ø 25 3.5
x = 7 months 66. (b) Let numbers be 3x & 5x
So, Z joined in the business after 12 – 7 = 5 months. ATQ,
SBI Clerk (Junior Associates) Mains Solved Paper-2018 2018-21
Solutions (86-90):
æ 50 50 30 40 30 ö
86. (d) Data are insufficient to determine the required ratio. = 480 – ç ´150 + ´ 200 + ´ 250 + ´ 350 + ´ 200 ÷
è 100 100 100 100 100 ø
87. (b) Ratio of number of coconut trees and lemon trees =
= 480 – (75 + 100 + 75 + 140 + 60)
5 : 1, therefore number of coconut trees is 500, since
revenue generated from coconut trees is ` 180,000. = 480 – 450 = 30
No of students from XII who didn’t visit any place
1,80, 000
Hence value per trees = = ` 360
= 400 – æç ö
500 30 20 10 34 50
´150 + ´ 200 + ´ 250 + ´ 350 + ´ 200 ÷
è 100 100 100 100 100 ø
88. (a) Since revenue of ` 3,00,000 is equally divided by
Gopal and Ram. Hence, amount received by Gopal in = 400 – (45 + 40 + 25 + 119 + 100)
= 400 – (329) = 71
1
1997 = × 3,00,000 = ` 1,50,000. Required Difference = 71 – 30 = 41
2
93. (c) Using solution of previous question,
89. (a) The value of lemon output per acre of land
71
1, 20, 000 Required % = ´100 = 35.5 %
= = 0.24 lakh/acre. 200
5
94. (d) No of students who visited Manali from class XI
90. (b) Let the amount invested by Gopal and Ram be 2x
and 3x respectively. 30 + 10
and XII = × 250 = 100
Gopal further invested ` 2 lakh. 100
Acc, to question No of students from class X who visited Shimla,
(2x + 2) = 3x or x = 2 lakh. Ooty and Nainital = 30 + 60 + 40 = 130
Hence, initial amount paid by Gopal and Ram to 100
Krishna is 4 lakh and 6 lakh respectively. Required % = ´100 » 77 %
130
Hence, total, money invested by them together
= (6 + 6) = 12 lakh. 95. (d) From Class X:
Tg:- @NextGenBankers
The total revenue generated = 12 × 25% = 3 lakh.
Also The ratio of revenue from coconut and lemon
æ 20
420 – ç
è 100
´ 150 +
30
100
´ 200 +
60
100
´ 250 +
26
100
´ 350 +
20
100
ö
´ 200÷
ø
trees are in the ratio 3 : 2, = 420 – 30 – 60 – 150 – 91 – 40 = 49
Hence, revenue from coconut = ` 1,80,000 and From class XI: 30
revenue from lemons = ` 1,20,000, so total output of From class XII: 71
1,80, 000 So required number = 49 + 30 + 71 = 150
coconut = = `36,000.
5 96. (a)
91. (a) No. of students from class X who visited Manali I. A = a + or – nd
14 + (37×1) =14 + 37 = 51yrs.
60
= × 250 = 150 II. 35 + (20×0.5) = 35 + 10 = 45yrs.
100
97. (a)
No. of girls from class X who visited Manali I. If the no of article bought is LCM of 6 and 5 is 30
7 CP of 30 articles = 5/6*30 = ` 25
= × 150 = 70
15 SP of 30 articles = 6/5*30 = ` 36
Total no. of students from class X who went to a Profit 36 – 25 = 11
tourist place Profit %ge = 11/25 *100 = 44%
II. CP of 1 toy = 350/100 = 3.50
20 30 60 26 20 SP of 1toy = 48/12 = 4.
=´150 + + 200 + ´ 250 + ´ 350 + ´ 200 Profit = 4 – 3.5 = 0.5.
100 100 100 100 100
Profit %ge (0.5/3.5) *100 = 14 2/7%
= 30 + 60 + 150 + 91 + 40 = 371
98. (c)
70 I. C.P. of 12 balls = S.P. of 17 balls = ` 720.
Required % = ´100 » 19 %
371 CP of 1 ball = 720/12 = ` 60.
92. (b) No of students from class XI who didn’t visit any II. SP = 85% of 1400 = 85/100*1400 = ` 1190.
place
SBI Clerk (Junior Associates) Mains Solved Paper-2018 2018-23
Reasoning Ability All the given members belong to the same the family. A is the
brother of L. A is the only son of R.W is the father-in-law of
L. D is the maternal grandfather of P, who is a male. Q is the
DIRECTIONS (Qs. 1-3): Read the information carefully and only son of W. W is the grandfather of N and C is the daughter
answer the questions: of N.
An Organisation Hare Krishna printed different number of 7. How L is related to C?
books in different years 1943, 1956, 1987, 1998, 2002 such that (a) Mother (b) Son
number of books printed are not same in any year. 66 books (c) Brother (d) Father
were printed in an odd numbered year which is not 1943. The
(e) None of these
number of books printed in 1943 is 10 less than that printed in
1987. 59 books were printed in an year before the year in which 8. How is P related to N?
61 books are printed but not immediate before. The number of (a) Mother (b) Son
books printed in 2002 is 2 more than that printed in 1998. (c) Brother (d) Father
1. How many books were printed in 1943? (e) None of these
(a) 56 (b) 66
(c) 63 (d) 61 DIRECTIONS (Qs. 9-11): Read the information carefully and
(e) None of these answer the question:
2. What is the difference between the number of books Point B is 10 m north of point Q. Point T is 10 m east of point
printed in 1956 and 2002? B. Point S is 15 m south of point T. Point P is 20 m south of
point Q. Point R is 25 m east of point P. Point L is 15 m east
(a) 7
(c) 8
(e) None of these
Tg:- @NextGenBankers
(b) 10
(d) 4 of point S. Point M is the midpoint of point B and P.
9. What is the distance between point L and R?
3. In how many years the number of books printed are more (a) 10 m (b) 15 m
than that printed in 1998? (c) 5 m (d) 20 m
(e) 25 m
(a) two (b) one
10. In which direction is point T with respect to P?
(c) none (d) three
(a) north-west (b) south-west
(e) four (c) south-east (d) north-east
4. How many words can be formed from the 1st, 6th, 8th and (e) none of these
9th letter of a word ‘EMANICIP ATE’ by using each letter 11. Which of the following points are inline?
once in the word? (a) P, R, S (b) Q, M, L
(a) two (b) one (c) B, S, T (d) M, S, L
(c) none (d) three (e) Q, S, L
(e) More than three
5. If all the letters in the word FIGURES are arranged in DIRECTIONS (Qs. 12-16): Read the following information
alphabetical order from left to right in such a way that vowels carefully and answer the given questions.
are arranged first followed by consonants, then how many Twelve persons sitting in two rows. D, A, F, K, L and M sitting
letters are there in between U and R after the arrangement? in row-1 and facing north. S, T, R, X, Y and Z sitting in row-
(a) two (b) one 2 and facing south direction. A sits third from one of the extreme
(c) none (d) three ends. S sits second to the left of the one who faces A. Only three
(e) four persons sit between S and T. K sits somewhere right of M. More
6. If in the number 39682147, 1 is added to each of the digit than three persons sit between X and T. F faces one of the
which is less than five and 1 is subtracted from each of immediate neighbours of T. Z sits second to the right of Y. The
the digit which is greater than five then how many digits one who faces L sits third to the left of R. D faces S.
are repeating in the number thus formed? 12. Who among the following faces K?
(a) two (b) one (a) T (b) S
(c) none (d) three
(c) X (d) Y
(e) four
(e) None of these
DIRECTIONS (Qs. 7-8): Read the information given below and
13. Who among the following faces the immediate neighbor
answer the questions.
of M?
SBI Clerk (Junior Associates) Prelim Solved Paper-2018 2018-25
(a) Z (b) K 20. What is the code of ‘right’ in a certain code language?
(c) D (d) L I. The code of ‘every right to reject’ is ‘%47 *32 $53
(e) None of these *95’,
14. Four of the following five from a group, which among the II. The code of ‘never reject right turn’ is’ %62 %47 $51
following does not belong to this group? *32’.
(a) T, A (b) R, D 21. Find the number of boys and number of girls in the row?
(c) Y, L (d) Z, A I. A sits 18th from left end of the row and Y sits 11th
(e) Z, K from the right end of the row. A and Y interchange
15. Who among the following faces the one who sit to the their positions, after interchanging the position A’s
immediate left of Y? position is 20th from left end.
(a) R (b) D II. Total 43 students are in the row and all are facing
is same direction.
(c) X (d) Z
(e) None of these DIRECTIONS (Qs. 22-26): Study the following arrangement
16. How many persons sit between M and D? carefully and answer the questions given below:
(a) one (b) two B 5 R 1 @ E K 4 F 7 © LAM 2 P 3 % 9 H I W 8 * 6 U J
(c) three (d) five $VQ#
(e) four 22. Which of the following is the fifth to the left of the
17. Five people C, K, X, Y, and Z live on five different floors seventeenth from the left end of the above arrangement?
of a building (such as ground floor numbered as 1 and (a) L (b) W
top is numbered as 5). There are three floors between C (c) * (d) 4
and K. X lives immediate above the floors where c lives. (e) None of these
Who among the following lives on third floor? 23. Which of the following is exactly in the middle between
(a) K (b) C L and U in the above arrangement?
(c) X (d) Z (a) % (b) H
(e) Cannot be determined (c) 9 (d) 3
18. Which of the following elements should come in a place ‘?’? (e) None of these
AB4, CE6, FI9, JN13?
(a) OT 20
(c) OT 21
Tg:- @NextGenBankers
(b) TO 21
(d) TS 21
24. Four of the following five are alike in a certain way based
on their position in the above arrangement and so form
a group. Which is the one that does not belong to that
(e) None of these group?
(a) R1E (b) F7L
DIRECTIONS (Qs.19-21): Each of the questions below consists (c) M23 (d) 9HW
of a question and two statements numbered I and II given below it.
(e) UJ6
You have to decide whether the data provided in the statement are
sufficient to answer the question. Read both the statements and 25. How many such symbols are there in the above
arrangement each of which is immediately preceded by a
Given answer: number but not immediately followed by a consonant?
(a) If the data in statement I alone are sufficient to (a) None (b) One
answer the question, while the data in statement II (c) Two (d) Three
alone are not sufficient to answer the question. (e) More than three
(b) If the data in statement II alone are sufficient to
26. Which of the following is the tenth to the left end of the
answer the question, while the data in statement I
alone are not sufficient to answer the question. thirteenth from the right end?
(c) If the data either in statement I alone or in statement (a) F (b) M
II alone are sufficient to answer the question. (c) @ (d) %
(d) If the data even in both statements I and II together (e) 3
are not sufficient to answer the question.
(e) If the data in both statement I and II together are DIRECTIONS (Qs. 27-28): Read the information carefully and
necessary to answer the question. answer the questions:
19. Who sits immediate to the left of Ram who is sitting in Eight persons Q, B, C, D, E, F, G, H, are sitting around a circular
a row. All the persons who are sitting in a row are facing table facing centre. H faces B. Two persons sit between F and
north direction? B. E sits 2nd right to D. F sits 2nd right to C, who is one of the
I. There are only two persons sit between Laxman and immediate neighbors of G. C is not an immediate neighbor of B.
Hanuman. More than three persons sit to the left of 27. Who among the following sits 3rd left to F?
Hanuman. (a) D (b) C
II. Not more than 8 persons can sit in a row. Ram sits (c) B (d) Q
second to the left Laxman. Diya sits 6 places away
from Hanuman. (e) None of these
GP_4458
2018-26 SBI Clerk (Junior Associates) Prelim Solved Paper-2018
DIRECTIONS (Qs. 29-30): Study the following information 36. 28, 13, 11, 19, 72, ?
(a) 571 (b) 251
carefully and answer the given questions.
(c) 254 (d) 896
In a certain code language, (e) 260
‘good key friends’ is coded as ‘xo pe ky’ 37. 72, 36, 12, 6, 2, ?
‘key law found’ is coded as ‘xo og bt’ (a) 0.75 (b) 0.25
‘data key good’ is coded as ‘tu xo pe’ (c) 0.5 (d) 45
(e) 1
29. Which of the following is the code for ‘good’?
38. 19, 30, 43, 54, ?, 78
(a) xo (b) pe (a) 62 (b) 64
(c) tu (d) ky (c) 65 (d) 66
(e) None of these (e) 67
30. Which of the following word is coded as ‘og’? 39. 8, 251, 170, 197, 188, ?
(a) law (b) good (a) 640 (b) 191
(c) found (d) Either (a) or (c) (c) 187 (d) 190
(e) 751
(e) key
40. 178, ?, 112, 89, 70, 53
DIRECTIONS (Qs. 31-35): The following questions are based (a) 215 (b) 217
on the six three digits numbers given below: (c) 223 (d) 221
(e) 141
563, 426, 556, 491, 929
31. If 2 is subtracted from the second digit of all odd DIRECTIONS (Qs. 41-45): Pie-chart given below shows total
numbers and 2 is added in the first digit of all even number of students who opted different subjects for exam in
Tg:- @NextGenBankers
numbers, then which number is lowest number after the
arrangement?
January 2018. Study the data carefully and answer the
following questions:
(a) 426 (b) 556 Total students = 300
(c) 491 (d) 929
(e) None of these
32. If third digit of highest number is divided by the first Maths Sociology
digit of lowest number, then what will be the resultant? 13% 18%
(a) 4 (b) 6 Urdu
(c) 2.25 (d) 5 17%
(e) None of these 12%
33. If all the digits in each number are arranged in increasing
order, then which number will be the highest number after History
Sanskrit 16%
the rearrangement? 24%
(a) 426 (b) 556
(c) 491 (d) 563
(e) None of these 41. Total number of students who opted for Sociology and
34. How many numbers will be there in the given series in Philosophy together is how much less than total number
which addition of first and third digit is greater than of students who opted for Sanskrit and History together?
second digit? (a) 24 (b) 27
(a) One (b) Two (c) 30 (d) 33
(c) Three (d) Four
(e) 36
(e) None of these
35. How many numbers will be there in the given series in 42. Find the total number of students who gave exams in
February 2018. If total number of students is increased
which difference of first and third digit is greater than
by 20% in February 2018 as compared to January 2018.
second digit?
(a) 450 (b) 420
(a) One (b) Two
(c) 390 (d) 330
(c) Three (d) Four
(e) 360
(e) None of these
SBI Clerk (Junior Associates) Prelim Solved Paper-2018 2018-27
43. Find the central angIe of total number of students who (a) 446 (b) 340
opted for History? (c) 405 (d) 468
(a) 57.6° (b) 54°
(e) 348
(c) 50.4° (d) 43.2°
(e) 64.8° 53. Interest earned on an amount after 2years at 20% p.a
44. Find the average number of students who opted for compounded yearly is ` 3432. Find the interest earned on
Maths, Sociology and Sanskrit together? same amount after 3 years at 15% p.a at Simple interest.
(a) 54 (b) 55 (a) ` 1620 (b) ` 3510
(c) 56 (d) 57 (c) ` 1665 (d) ` 1710
(e) 58 (e) ` 1750
45. Find the ratio between total number of students who 54. In place of 18% profit an article is sold at 32% profit and
opted for Urdu, History and Sociology together to total seller gets ` 112 more. Find the selling price of article if
number of students who opted for Maths, Urdu and
it were sold at 25% profit?
Sanskrit together?
(a) 8 : 9 (b) 17 : 19 (a) ` 440 (b) ` 460
(c) 15 : 16 (d) 17 : 18 (c) ` 1000 (d) ` 550
(e) 5 : 6 (e) ` 525
55. Raja and Baja working alone can do a work in 20 days
DIRECTIONS (Qs. 46-49): What should come in place of
and 15 days respectively. They started the work together
question mark (?) in the following questions?
but Baja left after sometime and Raja finished remaining
46. ?= 18 ´ 32 + 11 ´ 5 + 9 ´ 5 work in 6 days. Find after how many days from start Baja
left the work?
(a) 26 (b) 24
(a) 5 days (b) 4 days
(c) 28 (d) 27
(e) 35 (c) 6 days (d) 3 days
(e) 7 days
47.
(c) 328.8
Tg:- @NextGenBankers
32% of 420 + 36% of 540 = ?
(a) 312 (b)
(d)
288
318
DIRECTIONS (Qs. 56-60): In each of these questions, two
equations are given. You have to solve both the equations and
(e) 324
give answer
48. 80% of 450 + 20% of 850 = ?
(a) 540 (b) 580 (a) if x > y (b) if x > y
(c) 530 (d) 560 (c) if x < y (d) if x < y
(e) 555 (e) if x = y or no relation can be established between x
49. 7569 + ? = 104 and y.
(a) 256 (b) 400 56. (i) x2 = 256
(c) 361 (d) 321 (ii) y2 + 2y – 48 = 0
(e) 289 57. (i) x2 – 11x + 28 = 0
50. Present average age of Ram, Shyam and Rohit is 22 (ii) y2 – 14y + 45 = 0
years. Three years ago, Average age of Shyam and Rohit
58. (i) 2x2 – 5x + 3 = 0
is 18 years, then find Ram’s age 9 years hence?
(a) 24 years (b) 27 years (ii) 2y2 – y – 1 = 0
(c) 30 years (d) 33 years 59. (i) x2 – 17x + 72 = 0
(e) 36 years (ii) y = 64
51. Ratio between speed of yatch in still water to speed of 2
stream is 8 : 1. If 67.5km is travelled downstream in 2.5 60. (i) x – x – 132 = 0
hours then find the difference between speed of yatch in (ii) y2 – 6y + 8 = 0
still water to speed of stream (in kmph)?
(a) 15 (b) 3 DIRECTIONS (Qs. 61-64): What should come in place of
question mark (?) in the following questions?
(c) 24 (d) 21
(e) 17.5 61. 441 - 169 = ?
52. The perimeter of a rectangle whose length is 12m more (a) 64 (b) 9
than its breadth is 84 m. What will be the area of the (c) 100 (d) 10
rectangle? (in m2) (e) 121
GP_4458
2018-28 SBI Clerk (Junior Associates) Prelim Solved Paper-2018
There are a few factors driving the demand for increased flexibility. (I) a full time worker
Globalization is one. The development of a 24/7 marketplace, and (II) an employee who is fully committed to work
the rapid expansion of the services economy are also having a (III) the employee has no personal or family commitments
transformational effect on the workplace, requiring organizations that impact his availability
to think creatively about how they can best organize jobs and
(a) Only (II)
work to respond to an increasingly diverse and demanding
(b) Only (III)
customer base. Similarly technology is driving - and enabling -
greater flexibility. It is dramatically reshaping our workplaces, (c) Both (II) and (III)
blurring the boundaries between work and home and diversifying (d) Both (I) and (III)
where, when and how employees work. Advances in mobile, (e) All of these.
internet and cloud technologies, the rapid development of 73. According to the passage, how is technology driving and
computing power, and the digital connection between multiple enabling greater flexibility?
objects have all driven workplace innovations such as remote
(a) It is identifying the stereotype of an ideal worker.
working, telecommuting, co-working spaces, video/
teleconferencing, and virtual teams and collaboration. (b) It is dramatically reshaping our workplaces, blurring
the boundaries between work and home and
So the future of work demands new approaches to work design - diversifying where, when and how employees work.
but have workplaces risen to the challenge? The evidence
(c) It is training the workers on how to transform the ways
suggests we have yet to grasp this opportunity to be more
in which flexibility can be introduced
innovative. While some employers are making flexible work more
available, there is still a high prevalence of bolted-on temporary (d) It is one of the factors that is contributing in declining
arrangements. These arrangements are seen as the exception to the demand for flexibility at the workplace worldwide.
the rule, with the full-time, "face-time", long hours "ideal worker" (e) None of these
still the model to which everyone is expected to adhere. Many 74. Which of the following were the steps taken by the UK
people make assumptions about flexible workers, including that Bakery with regards to the working time of their bakers?
they're not interested in training and development, aren't (I) The UK Bakery came up with a flexible system of two
Tg:- @NextGenBankers
committed to the organization, or don't have any career
aspirations. We need to explore and challenge these biases.
to three baking shifts a day to maintain a steady supply
of fresh bread.
There are good international examples of successful work redesign (II) The Bakery team agreed to rotate their hours each
that have involved the input of a team of employees. For example, week so no team member permanently worked a shift
a UK bakery sat down with their bakers and came up with a that did not suit.
flexible system of two to three baking shifts a day to maintain a
steady supply of fresh bread. The team agreed to rotate their (III) The bakery sales increased by more than 65% in the
hours each week so no team member permanently worked a shift first year and employee satisfaction in the bakery has
that did not suit. After the change was made, bakery sales risen 10% since the change to 93%.
increased by more than 65% in the first year and employee (a) Only (III) (b) Both (I) and (III)
satisfaction in the bakery has risen 10% since the change to 93%.
(c) Both (II) and (III) (d) Both (I) and (II)
So work redesign is not only doable, it can deliver business
benefits, although it does require a completely new approach. By (e) All of these.
changing our thinking and focusing on the team and the 75. The most appropriate title of the passage is:
organization as a whole, rather than the individual, we have the
(a) Complex lives of the workforce
opportunity to create more adaptable and sustainable workplaces.
71. According to the author, how is flexible working still (b) Role of technology in increasing flexibility of workers
regarded as? (c) The brightest workforce requires more flexible work
(a) the way of designing work and jobs.
(d) How a UK Bakery increased their sales
(b) the way to identify the stereotype of an ideal worker.
(c) an add-on, something we do for mothers for a few (e) Importance of Globalization in increasing workforce
months when they are back from parental leave. Directions
(d) advances in mobile, internet and cloud technologies 76. Choose the word which is most nearly the SAME in meaning
and the rapid development of computing power. to the word 'PREDICT' printed in bold as used in the
(e) None of these. passage.
72. According to the passage, in today's workforce, which (a) ignore (b) forecast
among the following is/are the stereotype(s) of an ideal (c) misunderstand (d) recount
worker? (e) narrate
GP_4458
2018-30 SBI Clerk (Junior Associates) Prelim Solved Paper-2018
77. Choose the word which is most nearly the OPPOSITE in (a) A (b) B
meaning to the word 'TEMPORARY' printed in bold as used (c) C (d) D
in the passage. (e) E
(a) substitute (b) alternate 85. The blunder mistake (A)/was the apparent failure of
(c) expedient (d) provisional detectives (B)/to inform the Parole Board that the murderer
(e) permanent (C)/had threatened to return to kill her. (D)/No Error (E)
DIRECTIONS (Qs. 78-85): Read each sentence to find out (a) A (b) B
whether there is any grammatical or idiomatic error in it. The (c) C (d) D
error, if any, will be in one part of the sentence. The alphabet (e) E
corresponding to that part is your answer. If there is 'No error',
the answer is (e). (Ignore errors of punctuation, if any.) DIRECTIONS (Qs. 86-90): In each question below some
sentences are given which are divided into five parts. The first
78. The priest together with (A)/his followers were fatally
part of the sentence (1) is correct and is given in bold followed
injured (B)/in the accident which occurred last night (C)/
by four parts named A, B, C and D. Rearrange the four parts of
near the unmanned railway crossing. (D)/No Error (E)
(a) A (b) B the sentence to make a coherent paragraph. The rearranged
(c) C (d) D sequence of the parts will be your answer. If the given sentence
(e) E is correct as it is then choose option (e).
79. The young woman who is watching television in that room 86. Crashes in the early days (1)/ to be caused by technical
(A)/lived here for more than (B)/a year but she has never faults, (A)/ of commercial jets tended (B)/ such as metal
created (C)/any problem for us. (D)/No Error (E)
fatigue (C) /in the airframe or engines (D)
(a) A (b) B
(a) DCAB (b) ABCD
(c) C (d) D
(c) BACD (d) CABD
(e) E
80. Each of the employees, (A)/whom the company has chosen (e) No arrangement required
to take part (B)/in the international seminar to be conducted 87. There have been a lot of (1)/ drivers not obeying (A)/
(a) A
Tg:- @NextGenBankers
(C)/in the City Hall, are up to the mark. (D)/ No Error (E)
(b) B
complaints recently about (B)/ in downtown Boston (C)/
the speed limits (D)
(c) C (d) D (a) BADC (b) ABCD
(e) E (c) BCAD (d) CABD
81. Needless to say, (A)/no sooner were all these large and
(e) No arrangement required
rather expensive operations finished (B)/when the main
electricity was brought in (C)/and the turbine became 88. Chinese officials say (1)/ dropped to a three-year (A)/ low because
obsolete. (D)/No Error (E) of (B)/ economic growth has (C)/ the world economy (D)
(a) A (b) B (a) BADC (b) CABD
(c) C (d) D (c) BCAD (d) CABD
(e) E (e) No arrangement required
82. Plastic bags less than 50 microns thick (A)/are banned, (B)/ 89. I think it's a shame that (1)/ some foreign language
but neither the states nor the city corporations (C)/cares to
teachers(A)/ studied with a native speaker (B)/ college
enforce this rule. (D)/No Error (E)
without ever having (C)/ were able to graduate from (D)
(a) A (b) B
(c) C (d) D (a) BADC (b) CABD
(e) E (c) BCAD (d) ADCB
83. Tribal angst over economic issues (A)/leading to the (e) No arrangement required
scapegoating of nontribal longtime residents (B)/reflects 90. The Gita is a spiritual philosophy (1)/ addressed to all and
the continued failure (C)/to forge a more inclusive politics we know that (A)/ there are all kinds of people, (B)/ each
in Meghalaya. (D)/No Error (E) kind differing (C)/ quite significantly from the other (D)
(a) A (b) B
(a) BADC
(c) C (d) D
(b) CABD
(e) E
84. The Prime Minister has great power of (A)/implementing (c) BCAD
some useful (B)/schemes but the ministers (C)/have even (d) ADCB
greatest ability to foil them. (D)/No Error (E) (e) No arrangement required
SBI Clerk (Junior Associates) Prelim Solved Paper-2018 2018-31
DIRECTIONS (Qs. 91-95): Given below are sentences with a 96. Despite of being most efficient method ever, it is still highly
blank in each. Identify the most suitable alternative among the inefficient, and this inefficiency inspires hope.
five given that fits into the blank to make the sentence logical (a) Despite being the mostly efficient
and meaningful. (b) Despite of being a most efficient
91. In the same amount of time it would take me to correct all (c) Despite of being the most efficient
the ________________ in your report, I could write a (d) Despite being the most efficient
better report myself. (e) No replacement required
(a) mistakes (b) problems 97. A satisfactorily number of contestant must register for the
(c) accuracies (d) obstacles contest in order for it to take place.
(e) disputes
92. I have recently used the services of his _____________ (a) satisfactory number of contestants
agency to book a cruise in the Mediterranean. (b) satisfaction of number of contestants
(a) progress (b) deportation (c) satisfaction in the number of contestants
(c) travel (d) transfer (d) satisfactory number of contestant
(e) mover (e) No replacement required
93. They would like local authorities to be given greater _____ 98. The next class of wave or oscillation detector is the magnetic
as to how the money is spent.
detector depending in the powers of electric oscillations to
(a) affairs (b) function
(c) omission (d) discretion affect the magnetic state of iron.
(e) statement (a) depend on the power in
94. In a 10-billion-year-old galaxy there should have been ample (b) depending upon the power of
_________ for at least one species to escape its own mess, (c) depends upon the power in
and to spread across the stars, filling every niche. (d) deepening upon the power of
(a) negligence (b) opportunity
(e) No replacement required
(c) surveillance (d) supply
(e) advocacy 99. James had teaching at the university for more than a year
95. A true ________________ of the resources involved in before he left for Asia.
sport would include the unpaid labour services. (a) was taught (b) had been taught
(a) growth
(c) guidance
(e) estimation
Tg:- @NextGenBankers
(b) consideration
(d) suggestion
(c) had been teaching
(e) No replacement required
(d) has been teaching
100. His tail was short and scraggly, and his harness had been
DIRECTIONS (Qs. 96-100): In the question given below, there broken in many places and fastened together again with
is a sentence in which one part is given in bold. The part given cords and bits of wire.
in bold may or may not be grammatically correct. Choose the (a) was broke from
best alternative among the four given which can replace the
part in bold to make the sentence grammatically correct. If the (b) has broke from
part given in bold is already correct and does not require any (c) have been broken in
replacement, choose option (e), i.e. "No replacement required" (d) have been breaking on
as your answer. (e) No replacement required
ANSWER KEY
1 (a) 11 (d) 21 (d) 31 (c) 41 (c) 51 (d) 61 (a) 71 (c) 81 (c) 91 (a)
2 (d) 12 (d) 22 (a) 32 (c) 42 (e) 52 (c) 62 (c) 72 (e) 82 (d) 92 (c)
3 (a) 13 (a) 23 (c) 33 (b) 43 (a) 53 (b) 63 (d) 73 (b) 83 (e) 93 (d)
4 (a) 14 (d) 24 (e) 34 (d) 44 (b) 54 (c) 64 (b) 74 (d) 84 (d) 94 (b)
5 (a) 15 (b) 25 (d) 35 (e) 45 (d) 55 (c) 65 (e) 75 (c) 85 (a) 95 (e)
6 (b) 16 (c) 26 (a) 36 (a) 46 (a) 56 (e) 66 (a) 76 (b) 86 (c) 96 (d)
7 (e) 17 (e) 27 (e) 37 (e) 47 (c) 57 (e) 67 (b) 77 (e) 87 (a) 97 (a)
8 (c) 18 (e) 28 (d) 38 (e) 48 (c) 58 (b) 68 (a) 78 (b) 88 (b) 98 (b)
9 (b) 19 (e) 29 (b) 39 (b) 49 (e) 59 (b) 69 (a) 79 (b) 89 (d) 99 (c)
10 (d) 20 (d) 30 (d) 40 (e) 50 (d) 60 (e) 70 (e) 80 (d) 90 (e) 100 (e)
GP_4458
2018-32 SBI Clerk (Junior Associates) Prelim Solved Paper-2018
F D
Q 15 m
Q
27. (e) 28. (d)
15 m
M L Solutions (29-30):
S
30 m
20 m Word Code
friends ky
R key xo
P 25 m
good pe
9. (b) 10. (d) 11. (d)
Solution (12-16): law/found og/bt
T Z R Y S X data tu
Row-2:
29. (b) 30. (d)
SBI Clerk (Junior Associates) Prelim Solved Paper-2018 2018-33
31. (c) 543 626 756 471 909 50. (d) Sum of present ages of Ram, Shyam and Rohit
9 = 66 years
32. (c) = 2.25 Sum of present age of Shyam and Rohit
4 = 18 × 2 + 6 = 42
33. (b) 356 246 556 149 299 Present age of Ram = 66 – 42 = 24
34. (d) 563 426 556 929 Ram’s age nine years hence = 24 + 9 = 33 years
35. (e) 51. (d) Let speed of yatch in still water and speed of stream
36. (a) 28 13 11 19 72 571 be 8x and x respectively.
ATQ
×0.5–1 ×1–2 ×2–3 ×4 – 4 ×8–5 67.5 27
= 8x + x ; x =
6 1 2.5 9
37. (e) 72 36 12 2
x=3
Required difference = 8x – x = 7x = 7 × 3 = 21
52. (c) Breadth of rectangle = x metre
38. (e) 19 30 43 54 67 78 Length = (x + 12) metre
\ 2(x + 12 + x) = 84
Þ 4x + 24 = 84
Þ 4x = 60
39. (b) 8 251 170 197 188 191 Þ x = 15
\ Length = 15 + 12 = 27 metre
\ Area of rectangle = Length × Breadth
178 141 112 89 70 53 = 27 × 15 = 405 sq. metre
40. (e) 53. (b) Overall rate for 2 years at 20% p.a compounded
7 20 ´ 20
41. (c) Required difference yearly is equivalent to 20 + 20 + = 44%
100
(24 + 16) - (18 + 12) ATQ
´ 300
42.
=
100
Tg:- @NextGenBankers
(e) Total number of students who gave exam in Feb. 2018
44% of sum = 3432
100% of sum = 7800
120 7800 ´ 15 ´ 3
= 300 × = 360 Simple interest earned = = ` 3510
100 100
43. (a) Required central angle = 16 × 3.6 = 57.6° 54. (c) Let cost price of article = 100x
44. (b) Required average ATQ
32x – 18x = 112
1 æ 13 + 18 + 24 ö 14x = 112
= ç ÷ø ´ 300 = 55
3è 100 x=8
45. (d) Required Ratio Cost price of article = 8 × 100 = 800
17 + 16 + 18 51 17 125
= = = Selling price to earn 25% profit = 800 ´ = 1000
13 + 17 + 24 54 18 100
46. (a) ? = 18 ´ 32 + 11 ´ 5 + 9 ´ 5 55. (c) Efficiency Total work
?= 576 + 100 3 Raja 20
? = 676
? = 26 60
32 36
47. (c) ? = ´ 420 + ´ 540 Baja 15
100 100
? = 134.4 + 194.4 A+ B
? = 328.8
48. (c) 80% × 450 + 20% × 850 = ?
? = 360 + 170 Work done by Raja in last 6 days = 6 × 3 = 18 work.
? = 530 Remaining work done by Raja + Baja
= 60 – 18 = 42 work
49. (e) ? = 104 – 7569
42
? = 104 – 87 Baja left the work after =
7
= 6 days.
? = 172 = 289
GP_4458
2018-34 SBI Clerk (Junior Associates) Prelim Solved Paper-2018
56. (e)
2 1 2
(i) x2 = 256 62. (c) 81 - 7 = ? + 17
x = ± 16 3 4 3
(ii) y2 + 2y – 48 = 0 2 1 2
81 - 7 + - = ? + 17 +
y2 + 8y – 6y – 48 = 0 3 4 3
y(y + 8) –6 (y + 8) = 0 2 1 2
57 + - - = ?
(y – 6) (y + 8) = 0 3 4 3
y = 6, – 8 3
No relation can be established between x and y ? = 56
4
57. (e)
(i) x2 – 11x + 28 = 0 63. (d) 729 × 169 = ? + 50% of 312
x2 – 7x – 4x + 28 = 0 50
x (x – 7) – 4 (x – 7) = 0 27 × 13 = ? + ´ 312
100
(x – 7) (x – 4) = 0
x = 7, 4 351 = ? + 156
(ii) y2 – 14y + 45 = 0 ? = 195
y2 – 9y – 5y + 45 = 0 64. (b) 152 + 722 = ? × 3 2197
y (y – 9) – 5 (y – 9) = 0 225 + 5184 = ? × 13
(y – 5) (y – 9) = 0
y = 5, 9 5409
=?
No relation can be established between x and y 13
58. (b) ? » 416
(i) 2x2 – 5x + 3 = 0 65. (e) Let cost price of article = 100x
2x2 – 3x – 2x + 3 = 0 Selling price of one article = 125x
x (2x – 3) – 1 (2x – 3) = 0 ATQ
(x – 1) (2x – 3) = 0 3 × 25x – 2 × 25x = 80
x = 1, 3/2 25x = 80
(ii) 2y2 – y – 1 = 0 16
Tg:- @NextGenBankers
2y2 – 2y + y – 1 = 0
2y (y – 1) + l (y – 1) = 0
(2y + 1) (y – 1) = 0
x=
5
Cost price of article = `
16
× 100 = ` 320
5
66. (a) Quantity I:
1 Length of train ‘A’ = x
y=– ,1 ; x > y
2 Length of train ‘B’ = 0.5x
59. (b) ATQ
(i) x2 – 17x + 72 = 0 x + 0.5x = 12 × (25 + 15)
x2 – 9x – 8x + 72 = 0 1.5x = 480
x (x – 9) – 8 (x – 9) = 0 x = 320 meters
(x – 8) (x – 9) = 0 Quantity II: 160 meters
x = 8, 9 Quantity I > Quantity II
(ii) y = 64 67. (b) Let average of a, k and c be x
y=8 a + k + c = 3x
x ³ y And k + c + d = 3x + 3
60. (e) Þ d–a=3
(i) x2 – x – 132 = 0 And, d + a = 39
x2 – 12x + 11x – 132 = 0 d = 21 and a = 18
x (x – 12) + 11 (x – 12) = 0 Quantity I:
(x – 12) (x + 11) = 0 a = 18
x = 12, – 11 Quantity II: 21
(ii) y2 – 6y + 8 = 0 Quantity II > Quantity I
y2 – 2y – 4y + 8 = 0 68. (a) Quantity I: Due to leakage only 80% of the cistern
y(y – 2) – 4 (y – 2) = 0 is filled this means 20% of tank is leaked out by
(y – 2) (y – 4) = 0 leakage which is equal to 60 liters
y = 2, 4 20% = 60
No relation can be established between x and y 100% = 300 liters
61. (a) Capacity of tank = 300 liters
441 – 169 = ?
Quantity II: 250 liters
21 – 13 = ? Quantity I > Quantity II
8= ? ? = 64
SBI Clerk (Junior Associates) Prelim Solved Paper-2018 2018-35
69. (a) Quantity I: because it was the result of the steps taken by the
Let speed of boat in still water and speed of stream bakery, not the step itself. Hence, option (d) will be
be 3x and 2x respectively the correct answer.
ATQ 75. (c) Option (c), i.e. "The brightest workforce requires
more flexible work" is the most appropriate title of
72 72
Þ 32 = + the passage.
3x 2 x 76. (b) Predict means say or estimate that (a specified
60 thing) will happen in the future or will be a
Þ x= consequence of something. Forecast has the same
32
meaning as predict. Hence, option (b) will be the
Downstream speed = 3x + 2x = 5x
correct answer. Recount means tell someone about
60 something; give an account of an event or experience.
=5× = 9.37 kmph Narrate means give a spoken or written account of.
32
Quantity II: 9 kmph 77. (e) Temporary means lasting for only a limited period of
Quantity I > Quantity II time; not permanent. Permanent has the most
70. (e) Quantity I: opposite meaning to temporary. Hence, option (e)
will be the correct answer. Expedient means (of an
Side of square = 361 = 19 cm action) convenient and practical although possibly
Let length of rectangle be x and breadth of rectangle improper or immoral Provisional means arranged or
be (x – 4) cm existing for the present, possibly to be changed
ATQ later.
4 ´19 78. (b) In the second part of the sentence, the word "were"
(x + x – 4) = = 38 will be replaced by the word "was" because when
2
x = 21 two subjects are combined with 'together with', the
Area of rectangle = 21 × 17 = 357cm2 main subject is the one which is written before
Quantity II: 357cm2 together with, and the verb follows the main subject.
Quantity I = Quantity II All other parts of the sentence are correct. Hence,
option (b) will be the correct answer.
71. (c)
Tg:- @NextGenBankers
Refer to the first paragraph of the passage where it
is clearly mentioned, "Yet despite this overwhelming
evidence, access to flexible work and careers is not
79. (b) In the second part of the sentence, the word "lived"
will be replaced by "has lived" or "has been living"
widespread. Flexible work is still regarded as an add- because "is watching" used in first part of the
on, something we do for mothers for a few months sentence and "has never created" used in the third
when they are back from parental leave." Hence, part of the sentence signify that the sentence is in
option (c) will be the correct answer. present tense. Hence, option (b) will be the correct
72. (e) Refer to the second paragraph of the passage where answer.
it is mentioned, "In today's workforce, fewer people 80. (d) In the fourth part of the sentence, "are" should be
identify with the stereotype of the ideal worker - a replaced by "is" because the subject of the statement
full-time, fully committed employee without personal starts with "Each of" and such sentences always
or family commitments that impact on availability". follow singular Verb. Hence, option (d) will be the
Hence, option (e) will be the correct answer. correct answer.
73. (b) Refer to the third paragraph of the passage where it 81. (c) In the third part of the sentence, "when" will be
is mentioned, "Similarly, technology is driving - and replaced by "than" because 'No sooner' is always
enabling - greater flexibility. It is dramatically followed by 'than' and 'Hardly/Scarcely' is followed
reshaping our workplaces, blurring the boundaries by 'when/before'. Hence, option (c) will be the
between work and home and diversifying where, correct answer.
when and how employees work". Hence, option (b) 82. (d) In the fourth part of the sentence, "cares" should be
will be the correct answer. replaced by "care" because if two subjects are
74. (d) Refer to the last paragraph of the passage where it combined using 'Either…or' or 'Neither…nor' then
is mentioned, "For example, a UK bakery sat down the verb always follows the subject near to it, which
with their bakers and came up with a flexible system in this case is 'the city corporations'. So the verb will
of two to three baking shifts a day to maintain a be plural. Hence, option (d) will be the correct
steady supply of fresh bread. The team agreed to answer.
rotate their hours each week so no team member 83. (e) All the parts of the given sentence are grammatically
permanently worked a shift that did not suit. After correct. Hence, option (e) will be the correct answer.
the change was made, bakery sales increased by 84. (d) In the fourth part of the sentence, "greatest" should
more than 65% in the first year and employee be replaced by "greater" because here we are
satisfaction in the bakery has risen 10% since the comparing between two types of people, 'Prime
change to 93%." Option (III) will not be the answer Minister' and 'Ministers', and when we compare two
GP_4458
2018-36 SBI Clerk (Junior Associates) Prelim Solved Paper-2018
types of people or things in Comparative Degree All the other words do not fill the blank appropriately,
then the adjective used should also be of hence option (b) is the most suitable answer choice.
Comparative Degree. Hence, option (d) will be the Negligence means failure to take proper care over
correct answer. something. Surveillance means close observation,
85. (a) The error lies in the first part of the sentence where especially of a suspected spy or criminal.
the usage of the words 'blunder' and 'mistake' 95. (e) The most appropriate word that would fill the blank
together is superfluous as 'blunder' itself means 'a is 'estimation' which means a judgement of the worth
stupid or careless mistake'. Hence, option (a) will be or character of someone or something. All the other
the correct answer. words do not fill the blank appropriately, hence
86. (c) The correct sequence of the other parts to form a option (e) is the most suitable answer choice.
grammatically correct and contextually meaningful Consideration means careful thought, typically over
sentence is BACD. Hence, option (c) is the most a period of time. Suggestion means an idea or plan
suitable answer choice. put forward for consideration. Guidance means
87. (a) The correct sequence of the other parts to form a advice or information aimed at resolving a problem
grammatically correct and contextually meaningful or difficulty, especially as given by someone in
sentence is BADC. Hence, option (a) is the most authority.
suitable answer choice. 96. (d) The most appropriate phrase to replace the phrase
88. (b) The correct sequence of the other parts to form a given in bold is "despite being the most efficient".
grammatically correct and contextually meaningful It is to be noted that "despite" does not take any
sentence is CABD. Hence, option (b) is the most preposition with it. It is always followed with a
suitable answer choice. noun, pronoun or a gerund. Moreover, when we use
89. (d) The correct sequence of the other parts to form a a superlative adjective (most) before the noun, we
grammatically correct and contextually meaningful generally use it with 'the'. This is because there's
sentence is ADCB. Hence, option (d) is the most only one (or one group) of the things we are talking
suitable answer choice. about. Since option (d) is in the precise grammatical
90. (e) The given parts of the sentence are in their precise syntax, it becomes the most suitable answer choice.
position forming a coherent sentence. Since, they 97. (a) The most appropriate phrase to replace the incorrect
do not require any rearrangement option (e) becomes bold phrase in the sentence is "satisfactory number
91. (a)
Tg:- @NextGenBankers
the most suitable answer choice.
The most appropriate word that would fill the blank
of contestants". It is to be noted that 'satisfactory'
is an adjective which means 'fulfilling expectations
is 'mistakes' which means an act or judgement that or needs; acceptable, though not outstanding or
is misguided or wrong. All the other words do not perfect' while 'satisfactorily' is an adverb. Moreover,
fill the blank appropriately, hence option (a) is the 'number of' reflects the plural nature of the noun,
most suitable answer choice. Accuracies means the hence "contestant" should be replaced by
quality or state of being correct or precise. Obstacles "contestants". Therefore, option (a) becomes the
means a thing that blocks one's way or prevents or most suitable answer choice.
hinders progress. Disputes means a disagreement or 98. (b) The most suitable phrase that should replace the
argument. phrase given in bold is "depending upon the power
92. (c) The most appropriate word that would fill the blank of". Except for option (b) none other options are in
is 'travel' which means journeys, especially abroad. absolute grammatical syntax. Hence, it becomes the
All the other words do not fill the blank appropriately, most viable answer choice.
hence option (c) is the most suitable answer choice. 99. (c) The most appropriate phrase to replace the given
Deportation means the action of deporting a foreigner phrase in bold to make the sentence grammatically
from a country. Transfer means move from one place correct is "had been teaching". It is to be noted past
to another. perfect continuous tense is used for ongoing
93. (d) The most appropriate word that would fill the blank continuous events over a period of time before
is 'discretion' which means the freedom to decide another action or event in the past. These must be
what should be done in a particular situation. All the continuous verbs, and cannot be actions that are
other words do not fill the blank appropriately, still happening in the present. Hence, considering
hence option (d) is the most suitable answer choice. the given rationale option (c) becomes the most
Omission means someone or something that has suitable answer choice.
been left out or excluded. Affairs means an event or 100. (e) The given phrase in bold is accurate and does not
sequence of events of a specified kind or that has require any corrections or replacement. Hence,
previously been referred to. option (e) becomes the most suitable answer choice.
94. (b) The most appropriate word that would fill the blank
is 'opportunity' which means a time or set of
circumstances that makes it possible to do something.
SBI Clerk Mains Solved Paper-2016
Held on 25 June 2016
(Based on Memory)
Time : 160 minutes Max. Marks : 200
Reasoning Ability • There are two persons between those two persons who like
Dhoni and Yuvraj and neither of them sits opposite A. C
DIRECTIONS (QS. 1 - 5) : Answer the questions on the basis of and G are immediate neighbours and neither of them likes
the information given below. Micael or Yuvraj.A, who likes Ashwin, sits second to the
right of the girl who likes Micael. B and E neither like Micael
A number arrangement machine when given an input of words,
nor sit adjacent to A. E, who likes Yuvraj, sits second to the
rearranges them following a particular rule in each step. The
following is an illustration of input and steps of rearrangement. left of G, who likes Dravid. C does not like Dhoni.
Input : west 45 28 83 bunch find 39 hut 12 8 guy cool • The immediate neighbour of E faces C, who sits third to the
Step I: 83 45 28 bunch find 39 hut 12 8 guy cool west left of the one who likes Ashwin. F does not like Micael.
Step II: 8 83 45 28 bunch find 39 12 guy cool west hut Both D and H do not like Dhoni and Sachin. B likes Suhaib
Step III: 45 8 83 28 bunch find 39 12 cool west hut guy while D likes Irfan. B is not an immediate neighbour of A.
Step IV: 12 45 8 83 28 bunch 39 cool west hut guy find 6. Who of the following is the favourite Cricketer of H?
Step V: 39 12 45 8 83 28 bunch west hut guy find cool (a) Sachin (b) Micael
Step VI: 28 39 12 45 8 83 west hut guy find cool bunch (c) Dhoni (d) Dravid
This is the final arrangement and step VI is the last step for this input. (e) None of these
1. Which of the following steps will be the last but one for 7. Sachin is the favourite Cricketer of which of the following
following input? persons?
Input : 76 from 48 super itself 56 18 went 22 crazy (a) A (b) B
(a) V (b) III (c) C (d) D
2.
(c) IV
(e) None of these Tg:- @NextGenBankers
(d) VII
12. Statements: - A%B, B*C, C#D, D*E 20. Which of the following may represent 'better generate
Conclusions: - (1) A%C (2) E$B wealth' in the given code language?
(a) Only conclusion one follows (a) fs gh dj (b) dj mk gh
(b) Only conclusion second follows (c) gh pn st (d) gh fs mk
(c) Either conclusion one or two follows (e) xs dj ak
(d) Neither conclusion one nor second follow
DIRECTIONS (Qs.21-25): Study the following carefully to
(e) Both conclusion one and two follows
answer the given questions
13. Statements: - A*B, B$C, C#D, D@E
Conclusions: - (1) E*A (2) C%A Eight friends H, I, J, K, L, M, N and O are sitting in a row facing
(a) Only conclusion one follows north. All of them like different colours - Red, Pink, Orange, Green,
(b) Only conclusion second follows Yellow, Black, Violet and Blue.
(c) Either conclusion one or two follows • There is only 1 person between J and one who likes Violet. N is
(d) Neither conclusion one nor second follow neither an immediate neighbour of J nor he likes Green.H sits
(e) Both conclusion one and two follows fourth to the left of the one who likes Violet but she does not
14. Statements: - A$B, B$C, C*D, D@E like pink. The person who likes Black is sits third to the right of
Conclusions: - (1) A$E (2) E%C the onw who likes Green and sits on the immediate right of H.
(a) Only conclusion one follows • The one who likes Green sits at one of the extreme ends of
(b) Only conclusion second follows the row. I does not like Green. M is an immediate neighbour
(c) Either conclusion one or two follows of both N and J. O sits at one of the extreme end of the row
(d) Neither conclusion one nor second follow but he does not like green.The one who likes Blue sits
(e) Both conclusion one and two follows second to the right of the one who likes Orange. The one
15 Statements: - A%E, E@C, C%B, B*D who likes Black and Pink are immediate neighbours. L sits
Conclusions: - (1) C%D (2) B@A third to the left of J and likes Yellow. There is only one
(a) Only conclusion one follows person sitting between the one who likes yellow and black.
(b) Only conclusion second follows 21. How many persons are there between I and N ?
(c) Either conclusion one or two follows (a) One (b) Two
(d) Neither conclusion one nor second follow (c) Three (d) Four
Tg:- @NextGenBankers
(e) Both conclusion one and two follows
DIRECTIONS (Qs.16 -20): Study the information below and
(e) None of these
22. Who among the following sits third to the right of the person
answer the following question: one who likes Pink ?
(a) One who like Blue (b) One who likes Black
In a certain code language (c) One who likes Red (d) One who likes Green
'economy and wealth balance' is written as 'gh mk ru st' (e) None of these
'wealth of nations depleting' is written as 'tl zm ak gh' 23. Who among the following likes Orange ?
'taxes balance nations better' is written as 'dj ru zm pn' (a) O (b) N
'better to revive economy' is written as 'br ht dj st' (c) M (d) L
(All codes are two-letter codes only) (e) None of these
16. What does the code 'tl' stand for in the given code language? 24. N likes which of the following colour ?
(a) and (a) Red (b) Black
(b) Either 'of' or 'depleting' (c) Red (d) Violet
(c) taxes (e) None of these
(d) to 25. Who likes Red sitting between
(e) either 'nations' or 'taxes' (a) Immedaite right of the one who likes Green
17. Which of the following may possibly represent 'revive to' (b) Immedaite left of the one who likes Black
in the given code? (c) Immedaite right of the one who likes Yellow
(a) zm ht (b) bt zm (d) Immedaite right of the one who likes Pink
(c) ht co (d) br ht (e) Both B and C
(e) br dj
18. What is the code for 'economy' in the given code language? DIRECTIONS (Qs. 26-30): Study the following arrangement to
(a) st (b) Either 'mk' or 'ru' answer the given questions
(c) gh (d) dj H3 R% MA$ K 2 P5 E © N 4 W@ F & Q 1U/ 9 J ID 7 8
(e) mk 26. Which of the following is the fifth to the right of the 12th
19. What is the code for 'balance' in the given code language? from the left end of the above arrangement ?
(a) st (b) gh (a) $ (b) @
(c) ru (d) ak (c) / (d) W
(e) zm (e) None
SBI Clerk Mains Solved Paper-2016 2016-3
27. If all the numbers are dropped from the above arrangement II. Point F is3m to south of point D. Point C is 4m to the east
then what will be the 10th from the right end ? of point A. Point A is 3m to the north of point B. Point A
(a) W (b) @ lies on the line formed by joining points C and D.
(c) / (d) N (a) Only I (b) Only II
(e) None (c) Both I and II (d) Either I or II
28. How many such symbol in the above arrangement
immediately preceded by letter and immediately followed (e) Neither I or II
by a number ? 37. How many people are standing in a straight line. All are
(a) None (b) One facing North
(c) Two (d) Three I. M stands third from the left end of the line. Only one
(e) More than three person stands between M and O. P stands second to
29. What should come in the following series based on the the right of O. P stands at one of the extreme lines
above arrangement ? II. K stands exactly at the centre of the line. Only two
HR% AK2 5©N ? people stand between K and L. Only three people
(a) @Q1 (b) F&Q stands between L and O. Only one persons stands
(c) W@F (d) WF& between O and P
(e) None (a) Only I (b) Only II
30. How many such numbers are there in the above arrangement, (c) Both I and II (d) Either I or II
each of which is immediately preceded by letter and
(e) Neither I or II
immediately followed by letter ?
38. What does ' Zee' represent in a code language ?
(a) None (b) One
(c) Two (d) Three I. In that code language ' ah koj zee pig' mean ' can you
(e) More than three take that '
II. In that code language 'et zee lin ter' means ' you may
DRECTIONS(Qs. 31-35) : Study the following information to come now'
answer the given questions: (a) Only I (b) Only II
A, B, C ,D , E, F and H live on eight different floors , Ground floor is no (c) Both I and II (d) Either I or II
1, 1stfloor is no 2 and so on. There are 2 floors between the floor on (e) Neither I or II
which B and H live. D lives on an odd - numbered floor and he does not 39. Among K, L, M, N, O and P each has different age, who is
live immediate below or above F's floor. F lives below H's floor.
Tg:- @NextGenBankers
D does not live immediately below or above B's floor. A does not live
on an even numbered floor. C does not live on the topmost floor. G
the youngest among them ?
I. L is younger than only K, and P. N is neither the oldest
nor the youngest
lives on 6th floor. E does not live below H and B lives on floor no E) II. M is older than N but not the oldest
31. How many floors are there between the floors on which D
(a) Only I (b) Only II
and H live ?
(a) One (b) Two (c) Both I and II (d) Either I or II
(c) Three (d) Four (e) Neither I or II
(e) None of the above 40. How is H related to C?
32. Who among the following lives on the floor no 3? I. M has two daughters. One of them is Z, who is married
(a) A (b) B to H.
(c) C (d) F II. C is the mother of Y, the younger sister of Z.
(e) None of the above III. M is C's husband.
33. Four of the following five are alike in a certain way and so (a) Only I and II
form a group. Which does not belong to that group ? (b) Only I and III
(a) ED (b) BC (c) Only I and either II or III
(c) HF (d) BA (d) Any two of the three
(e) None of the above (e) All are necessary
34. E lives on which of the following floor ?
(a) 4th floor (b) 8th floor DIRECTIONS (Qs. 41-45) : Study the following information
(c) 3rd floor (d) 8th floor carefully to answer the given questions.
(e) None of the above Seven actors A, B, C, D, E, F and G act in different types of drama
35. Who lives between A and F ? like Action, Thriller, Romantic, Suspense, Comedy, Horror and
(a) C (b) G Inspirational, but not necessarily in the same order. All the seven
(c) B (d) H
(e) None of the above actors are engaged on three different days of the week i.e. Sunday,
Monday and Tuesday. A is engaged on Sunday only with E and
DIRECTIONS (Qs. 36-40): Study the following information his drama is Thriller. D is acting in Horror drama and does not act
carefully to answer the given questions on Tuesday. The one who is acting in Comedy acts on Monday.
36. How far is point A from point D ? B and C do not act on the same day. Those who act on Sunday do
I. Point A is 4m to the north of point B .Point C is 3m to not perform Romantic drama. F acts in Suspense drama but does
the east of point A. Point D is to the west of point A not act on Monday. G acts on the same day as F. C acts in Action
such that points C, A and S form a straight line of 7m drama.
GP_4458
2016-4 SBI Clerk Mains Solved Paper-2016
DIRECTIONS (Qs. 61-65): Each of the questions below consists (a) 370 (b) 340
of a question and two statements numbered I and II given (c) 320 (d) 360
below it. You have to decide whether the data provided in the (e) None of these
statements are sufficient to answer the question. 68. Fresh grapes contain 80% water, while dry grapes contain
Read both the statements and give answer 10% water. If the weight of dry grapes is 500 kg, then what
(a) if the data in statement I alone are sufficient to is its totalweight when it is fresh?
answer the question, while the data in statement II alone (a) 2350 kg (b) 2085 kg
are not sufficient to answer the question. (c) 2255 kg (d) 2250 kg
(b) if the data in statement II alone are sufficient to (e) None of these
answer the question, while the data in statement I alone 69. What will be the ratio of simple interest earned by certain
are not sufficient to answer the question. amount at the same rate of interest for 12 yr and for 18 yr?
(c) if the data either in statement I aloneor in statement II (a) 2 : 5 (b) 1 : 3
alone are sufficient to answer the question. (c) 2 : 3 (d) 3 : 1
(d) if the data even in both statements I and II together
(e) None of these
are not sufficient to answer the question.
(e) if the data in both statements I and II together are 70. Number of employees in a factory decreases in the
necessary to answer the question. ratio of 8 : 7 and salary of employees increase in the ratio of
61. Who amongst A, B, C, D and E is the tallest? 5 : 6. Find whetherthe total salary given to the employees
I. A is taller than B but shorter than C. D is not the tallest. is increased or decreased and in what ratio?
II. Two people are taller than C. (a) 19 : 20, increased (b) 20 : 19, decreased
62. Which direction is Ali facing? (c) 19 : 21, increased (d) 21 : 20, decreased
I. If Pawn, who is currently facing East,turns 90º (e) None of the above
towards his right, he would face a direction exactly 71. A and B started a business by investing `20000 and
opposite to the direction which Ali is facing.
`25000 respectively. After 4 months B left and C joined by
II. If Preeti who, is currently facing South,turns left,
invested `15000. At the end of the year, there was a profit of
walks 1 m and then takes aleft turn again, she would
Tg:- @NextGenBankers
face the same direction as Ali.
63. Did 300 candidates appear for the written examination for
`23000. What is C's share?
(a) `8000
(c) `6000
(b) `9000
(d) `12000
admission into College ABC ?
I. The principal of the college correctly mentions (e) None of these
that the number of candidates who had appeared for 72. What would be the cost of building a fence around a square
the examination was more than 200. plot with area equal to 361 sq ft, if the price per foot of
II. According to a statistical report, only 175 building the fence is `62?
candidates could qualify the examination. (a) `4026 (b) `4712
64. How far is Point P from Point Q ?
(c) `3948 (d) Can't be determined
(All the points lie on a straight line.)
(e) None of these
I. Point T is exactly midway between Points P and
Q. Point T is 5 km towards west of Point R. 73. Mr. X sold a good to Mr.Y at 10% discounted value of printed
II. Point Q is 2 km towards west of Point R. rate. The discounted value is `1,242. If the printed rate is
65. How many brothers does A have? 15% profit of the purchase rate, what is the purchase rate?
I. A, who is B’s brother, has two siblings. (a) `1,242 (b) `1,380
II. D is brother of A and is youngest in the family. (c) `1,280 (d) `1,200
66. A has double the money of B and B has 50% more money (e) None of these
than C. If average money of all the three persons is 12000, 74. The average age of five officers in a department is 32
how much money A have? years. If the age of their supervisor is added the average
211000 315000 increased by 1. What is the supervisor's age?
(a) (b) (a) 32 years (b) 48 years
11 11
(c) 38 years (d) 42 years
216000 316000
(c) (d) (e) None of these
11 11
75. A 175 metres long train crosses a 35 metres platform in 12
(e) None of the above
67. In an examination out of 480 students, 85% of the girls and seconds. What is the speed of the train in km/hr?
70% of the boys have passed. (a) 42 (b) 64
How many boys appeared in the examination,if total pass (c) 63 (d) 59
percentage was 75%? (e) None of these
GP_4458
2016-6 SBI Clerk Mains Solved Paper-2016
DIRECTIONS (Qs. 76-80): Study the following profile of DIRECTIONS(Qs. 81-85): Study the following pie-charts
parliament carefully and answer thequestions given below it. carefully to answer the given questions
PROFILE OF PARALIMENT IN YEAR XXXX
Total members in Parliament = 640 G
12% A
(492 from Lok Sabha and 150 from Rajya Sabha) 18%
F B
12%
25%
C
3%
D
E 20%
10%
% Profit
*
15 *
15 *
10 12 * 10
5
0
A B C D E F G
Companies
81. What was the ratio of the expenditure of company 'A' to
that of company 'C'?
87. The average score of Rahul, Manish and Suresh is 63. third number. The value of the third number is 2960. What is
Rahul's score is 15 less than Ajay and 10 more than 30% of the first number?
Manish. If Ajayscored 30 marks more the average score of (a) 88.8 (b) 99.9
Rahul, Manish and Suresh, what is the sumof Manish's and (c) 66.6 (d) Can't be determined
Suresh's scores? (e) None of these
(a) 120 (b) 111 95. In a school there are 250 students, out of whom 12% are
(c) 117 (d) Can't be determined girls. Each girl's monthly fee is `450 and each boy's monthly
(e) None of these fee is 24% more than of a girl. What is the total monthly
88. Mr. X invested a certain amount in Debtand Equity fee of girls and boys together?
Funds in the ratio of 4 : 5. At the end of one year, he earned (a) `1,36,620 (b) `1,36,260
a total dividend of 30% on his investment. After one year, (c) `1,32,660 (d) `1,32,460
he reinvested the amount including the dividend in the (e) None of these
ratio of 6 : 7 in the Debt and Equity Funds. If the amount DIRECTIONS (Qs. 96-100): In the following question two
reinvested in Equity Funds was ` 94,500, what was the
equations numbered I and II are given, You have to solve both
original amount invested in Equity Funds?
euation and – give answer
(a) ` 75,000 (b) ` 81,000
(c) ` 60,000 (d) ` 65,000 (a) if x > y (b) if x > y
(e) None of these (c) if x < y (d) if x < y
89. If the length of a rectangular field is increased by 20% (e) if x = y or the relationship cannot be established.
and the breadth is reduced by 20%, the area of the rectangle 96. I. 289 x + 25 = 0
will be 192 m2 . What is the area of the original II. 676 y + 10 = 0
rectangle?
97. I. 8x2 – 78x + 169 = 0
(a) 184 m2 (b) 196 m2
2 II. 20y2 – 117y + 169 = 0
(c) 204 m (d) 225 m2
(e) None of these 15 9
98. I. + = 11 x
90. In how many different ways can the letters of the word x x
'THERAPY' be arranged so that the vowels never come together?
(a) 720
(c) 5040
Tg:- @NextGenBankers
(b) 1440
(d) 3600
II. 4
y 5 y
+
12
=
1
y
(e) 4800
91. Car 'A' travels at the speed of 65 km/hr andreaches its 8 6
99. I. + = x
destination in 8 hours. Car 'B'travels at the speed of 70 x x
km/hr andreaches its destination in 4 hours. What isthe
(14)2
ratio of the distance covered by car Aand car B respectively? II. y3 – =0
(a) 11 : 7 (b) 7 : 13 y
(c) 13 : 7 (d) 7 : 11 100. I. x2 – 208 = 233
(e) None of these II. y2 – 47 + 371 = 0
92. Arun ordered 15 chapattis, 4 plates of rice,6 plates of mixed
vegetable and 5 ice-cream cups. The cost of one chapatti is
English Language
`5, oneplate of rice is `50, one plate of mixed vegetable
DIRECTIONS (Qs. 101-110): Read the following passage
is `75 and one ice-cream cup is`20. How much amount did
carefully and answer the following questions given below it.
Arun pay to the cashier?
Certain words/ phrases have been printed in bold to help you
(a) `850 (b) `795
locate them while answering some of the questions:
(c) `825 (d) `750
(e) None of these Our current approach to solving global warming will not work. It
93. Four examiners can examine a certainnumber of answer is flawed economically, because carbon taxes will cost a fortune
papers in 10 days by working for 5 hours a day. For how and do little, and it is flawed politically because negotiations to
manyhours in a day would 2 examiners have to work in order reduce carbon-dioxide emissions will become even more fraught
and divisive. And even if you disagree on both counts, the current
to examine twice the numberof answer papers in 20 days?
approach is also flawed technologically.
1 Many countries are now setting ambitious carbon cutting goals
(a) 8 hours (b) 7 hours
2 ahead of global negotiations. Let us imagine that the world
1 ultimately agrees on an ambitious target. Say, we decide to reduce
(c) 10 hours (d) 8 hours carbon-dioxide emissions by three-quarters by the year 2100 while
2
(e) None of these maintaining reasonable growth. Herein lies that the technological
94. Five-ninths of a number is equal to 25% of the second problem to meet this goal, non-carbon based sources of energy
number. The second number is equal to one-fourth of the would have to be astounding 2.5 times greater in 2100 than that
was in the year 2000.
GP_4458
2016-8 SBI Clerk Mains Solved Paper-2016
These figures were calculated by economists of a foreign university. 103. Why, according to the author, are the international negotiations
Their research shows that confronting global warming effectively barely a solution to the problem of global warming?
requires nothing short of a technological revolution. We are not (a) Many countries fail to confine to the carbon-cut norms
taking this challenge seriously. If we continue on our current path, as set in these negotiations.
technological development will be nowhere near significant enough (b) These negotiations emphasize on the amount of carbon
to make non-carbon based energy sources competitive with fossil to be cut and not on the ways in which it can be done.
fuels on price and effectiveness.
(c) Recent research on the carbon-cut methods is
Sadly, during the international negotiations, the focus is on how overlooked by the politicians.
much carbon to cut, rather than on how to do so. Little or no
(d) Such negotiations produce dominance of powerful
consideration will be given to whether the means of cutting emissions
countries over the others, thus hampering their
are sufficient to achieve the goals.
industries development.
Politicians wilt base their decisions on global warming models that
(e) None of these
simply assume that technological breakthroughs will happen by
themselves. This faith is sadly and dangerously misplaced. 104. Which of the following is intended in the given passage?
Economists examine the state of non-carbon based energy today- (a) To suggest the ways in which alternate forms of energy
nuclear, wind, solar and geothermal and find that, taken together, can reduce climate change.
alternative energy sources would get us less than halfway toward a (b) To explain that the current technological developments
path of stable carbon emissions by 2050, and only a tiny fraction of are flawed and thus, fail to control the climate change.
the way toward stabilization by 2100. We need many times more (c) To explain that the stable carbon emissions are
non-carbon based energy than is currently produced. Yet the needed impossible to achieve in this century.
technology will not be ready in terms of scalability of stability. In (d) To suggest to the policy makers to invest in research
many cases, there is still a need for the most basic research and rather than futile negotiations.
development. We are not even close to getting this revolution started.
(e) None of these
Current technology is so inefficient that to take just one example, if
we were serious about wind power, we would have to blanket most 105. Which challenge according to the author is not being taken
countries with wind turbines to generate enough energy for seriously in the phrase "we are not taking this challenge
everybody, and we would still have the massive problem of storage: seriously" as used in the passage?
Tg:- @NextGenBankers
We don't know what to do when the wind does not blow.
Policy makers should abandon fraught carbon reduction
(A) Cutting the carbon emissions to a scale of 2.5 times in
the next decade.
negotiations and instead make agreements to invest in research (B) The challenge of locating new fossil fuel reserves since
and development to get this technology to the level where it the existing ones are depleting at a fast rate.
needs to be. (C) To carry out technological innovations for developing
101. Which of the following is suggested in the passage by non-carbon based energy resources.
which global warming can be reduced? (a) Only (C) (b) Both (A) & (C)
(a) To make the approach towards global warming (c) Only (B) (d) Both (A) & (C)
completely free from political intervention.
(e) None of these
(b) By making non-carbon based energy methods as
efficient and cost effective as the fossil fuels. 106. Which of the following is true in the context of the passage?
(c) To force every country to confine to stricter terms of (a) Many countries are refraining from taking part in global
carbon emissions. negotiations to check global warming.
(d) To avoid international negotiations on carbon cuts (b) There has been no improvement in carbon emissions
until the technological research comes out with a world over.
scalable and stable solution. (c) Technological advancement through research is the
(e) All of the above need of the hour in order to meet the directed goal of
102. Which of the following is not true in the context of the combating global warming.
given passage? (d) Most countries in the world are well equipped to
(A) Non-carbon fuels are too expensive so they should produce sufficient non-carbon based energy sources.
not be used. (e) All the above statements are true.
(B) Political ignorance is one of the main reasons behind DIRECTIONS (Qs. 107-108): Choose the word which is most
the inappropriate approach to combat global warming. similar in meaning to the word printed in bold as used in the
(C) The generation of energy from non-carbon sources has passage.
to be increased for significant reduction in global warming.
107. COUNTS
(a) Only (A) (b) Only (C)
(a) numbers (b) matters
(c) Only (A) & (C) (d) Only (B) & (C)
(c) calculations (d) values
(e) All (A), (B) & (C)
(e) attributes
SBI Clerk Mains Solved Paper-2016 2016-9
108. CUTTING inspired them culturally and politically. Never before had a poet
(a) slicing (b) breaking left such an imprint and wielded so deep an influence on the
(c) reducing (d) tearing psyche of the vast majority of the people. While India chose his
(e) interrupting Jana gana manas the national anthem in 1947, Bangladesh has
had one of his songs as the national anthem since its birth.
DIRECTIONS (Qs. 109-110): Choose the word which is most Sri Lanka's national anthem was also penned by Tagore: Apa Sri
opposite in meaning to the word printed in bold as used in the Lanka, Nama Nama Nama Nama Mata, Sundar Sir Borni was
passage. originally Nama Nama Sri Lanka Matain Bangla, written and set to
109. REASONABLE its tune by Tagore. He did it at the request of his favourite Sri
Lankan student at Santiniketan, Ananda Samarkun, in 1938. In
(a) fair (b) logical
1940, Ananda returned to his native land and translated the song
(c) irrational (d) inadequate into Sinhalese and recorded it in Tagore's tune. Indeed,
(e) considerable Rabindranath is not only the preeminent literary genius of Bengal
110. MASSIVE but all of South Asia, perhaps the whole of Asia.
(a) light (b) large 111. What did Tagore make the Bengali middle class feel?
(c) significant (d) short (a) that he (Tagore) was alien to them because of his
(e) unreasonable religion.
(b) that they (the Bengalis) were alien to them because of
DIRECTIONS (Qs. 111-115): Read the following passage their religion.
carefully and answer the questions given below it. Certain words/ (c) that he (Tagore) was an essential part of their national
phrases are given in bold to help you to locate while answering ethos.
some of the questions. (d) that they (the Bengalis) were an essential part of their
national ethos.
The simultaneous celebration of Rabindranath Tagore's 150th birth (e) None of these
anniversary in India and Bangladesh marked an exceptional move 112. Out of the following options, Tagore was not a(n) -
to honor the poet-philosopher. It also symbolized the deep (a) Short-story writer (b) Painter
admiration that exists in both countries for the man who enriched
(c) Novelist (d) Lawyer
Tg:- @NextGenBankers
literature as much as he did humanity as a whole.
The versatile genius, who was much ahead of his time, wrote in
his mother tongue of Bangla. But he did not limit his message to
(e) Singer
113. Tagore, at critical moments, has been an inspiration for the
people of ______.
the people who lived around him. His creative works introduced (a) Pakistan and India
a powerful dose of live and internationalism. This Indian rose to (b) India and Bangladesh
international heights: he was the first non-European to receive (c) Bangladesh and Pakistan
the Nobel Prize for Literature, in 1913. (d) Only Bangladesh
Tagore was poet, novelist, short story writer, essayist, playwright, (e) None of these
educationist, spiritualist, painter, lyricist, composer and singer - a 114. What happened after Tagore's songs were banned by
rare set of distinctions, an unbelievable conjunction of talents. Pakistan's first military ruler, Ayub Khan?
His creative works, which still influence billions of people globally, (a) Tagore became more relevant than before.
are a matter of pride for the people of India and Bangladesh. He (b) A strong sense of linguistic nationalism grew around him
was born, grew up, worked and died here. (c) He wielded a deep influence and enriched literature
At critical moments he has been an inspiration for the people of (d) All 1, 2 and 3
what is now Bangladesh. Protagonists of the two-nation theory (e) Only 1 and 2
wanted to wipe out his influence. Pakistan's first military ruler. 115. Which of the following statements about Tagore, according
Ayub Khan, banned his songs. to the given passage, is not true?
But the poet only became more relevant then before. A strong (a) He was an unbelievable conjunction of talents.
sense of linguistic nationalism grew ground him. Finally, the (b) He was much ahead of his time and limited his message
people launched a strong cultural and political movement that to the people who lived around him.
culminated in the formation of Bangladesh. (c) He penned the national anthems of India, Bangladesh
Tagore made the Bengali middle class feel that he an essential and Sri Lanka.
part of their national ethos. The emerging middle class, including (d) He was the first non-European to receive the Nobel
students and intellectuals, regarded him as one of them. In no Prize for Literature, in 1913.
way could they think that Tagore was alien to them because of (e) None of these
his religion. DIRECTIONS (Qs. 116-125): In the following passage there
Strangely, as in Pakistan's case, the successive military regimes are blanks, each of which has been numbered. These numbers
in Bangladesh showed little interest in upholding his legacy. are printed below the passage and against each, five words are
Tagore's songs and poems inspired Bengalis in their fight against suggested, one of which fits the blanks appropriately. Find out
Pakistan in the 1971 war of liberation. His songs and poetry the appropriate word in each case.
GP_4458
2016-10 SBI Clerk Mains Solved Paper-2016
Information technology, and the hardware and software (21) with (D) Any signal can be classified into one of the two types:
the IT industry, are a/an (22) part of nearly (23) major global analog and digital.
industry. IT industry has become one of the most robust (E) In contrast, a digital signal takes the form of pulses, where
industries in the world. IT, more than any other industry or we have something or nothing.
economic (24), has an increased productivity, particularly in the 126. Which of the following should be the FIRST sentence after
developed world, and therefore is a key driver of global economic rearrangement?
growth. Economies of scale and (25) demand from consumers and (a) A (b) B
enterprises (26) this rapidly growing sector. The Information (c) C (d) D
Technology Association of America (ITAA) explains 'information (e) E
technology' as (27) all possible aspects of information systems 127. Which of the following should be the THIRD sentence
based on computers. Both software development and the hardware after rearrangement?
involved in the IT industry include everything from computer (a) A (b) B
systems, to the design, implementation, study and development of (c) C (d) D
IT and management systems. (28) to its easy accessibility and the (e) E
wide range of IT products available, the demand for IT services has 128. Which of the following should be the SECOND sentence
increased (29) over the years. The IT sector has emerged as a major after rearrangement?
global (30) of both growth and employment. (a) A (b) B
116. (a) use (b) amalgamation (c) C (d) D
(c) associated (d) vision (e) E
(e) regulated 129. Which of the following should be the FOURTH sentence
117. (a) integral (b) fundamental after rearrangement?
(c) increased (d) vital (a) A (b) B
(e) eager (c) C (d) D
118. (a) most (b) all (e) E
(c) every (d) few 130. Which of the following should be the FIFTH sentence their
(e) some rearrangement?
119. (a) world (b) opinion (a) A (b) B
(c) stature (d) profit (c) C (d) D
(e) facet
120. (a) multiply
Tg:- @NextGenBankers
(b) insatiable
(e) E
DIRECTIONS (Qs. 131-135): Read each of the following
(c) decreasing (d) unquenchable
(e) unreasonable sentences to find out whether there is any error in it. The error,
121. (a) forage (b) thwart if any, will be in one part of the sentence. The number of that part
(c) motivate (d) fuelling is the answer. If there is no error, the answer is (e). (Ignore
(e) characterize errors of punctuation, if any).
122. (a) making (b) qualifying 131. The scheme failed because (a)/some states could not, (b)/
(c) inclusive (d) encompassing manage not to raise (c)/the necessary funds. (d)/No error (e).
(e) trusting 132. By so early as next year (a)/that leading investment bank
123. (a) owing (b) since (b)/has plans to open (c)/ an office in New Delhi. (d)/No
(c) catering (d) in order error (e).
(e) complementing 133. Experts have recommended that (a)/the government
124. (a) regularly (b) substantially reconsidered (b)/restrictions imposed on foreign (c)/
(c) minimally (d) exponential investment in real estate. (d)/No error (e).
(e) savagely 134. He wants to (a)/set up a laboratory (b)/to undertake research
125. (a) fortune (b) meltdown (c)/into a vaccine for cancer. (d)/No error (e).
(c) spring (d) source 135. Because of the pace at (a)/which the company is growing
(e) economy (b)/I believe it will easily (c)/acheive their target. (a)/No
error (e).
DIRECTIONS (Qs. 126-130): Rearrange the following five sentences
(A), (B), (C), (D), and (E) in the proper sequence to form a meaningful DIRECTIONS (Qs. 136-140): Each question given below has
paragraph; then answer the questions given below them. two blanks, each blank indicating that something has been
(A) For instance, if we measure the room temperature omitted. Choose the set of words for each blank that best fits the
continuously and plot its graph with time on X-axis and meaning of the sentence as a whole.
temperature on the Y-axis, we get a continuous waveform, 136. After having been friends for more than a decade, they had
which is an analog signal. Analog is always continuous. a ______ last year and have not ______ each other ever
(B) The absence or presence of something can be used to plot since.
a digital signal. (a) fight, talked (b) argument, met
(C) An analog signal is a continuously varying signal, similar (c) dispute, seen (d) quarrel, admired
to a sinusoidal waveform. (e) difference, introduced
SBI Clerk Mains Solved Paper-2016 2016-11
137. The workers, several of ____ had complained about their 145. Which bank has become the top merchant acquiring bank
low wages earlier have now ______ to move to the court in the country?
for the labour rights. (a) ICICI (b) SBI
(a) who, indicated (b) whom, decided (c) PNB (d) BoB
(c) which, threatened (d) them, resolved (e) None of the above
(e) number, warmed 146. In which year was the 1st Malti Gyan Peeth Puraskar
138. The hutment dwellers were jubilant when the government ceremony held?
______ an apartment to each of them at ______ rates. (a) 1997 (b) 2013
(a) demolished, fast (b) announced, less (c) 2016 (d) 2015
(c) provided, high (d) acquired, low (e) 1986
(e) promised, subsidised 147. Who is the Chairman of Indian Space Research Organisation
139. The organization was deeply ______ by difficulties a (ISRO)?
decade ago, but the new CEO brought many ______ (a) Thekkethil Kochandy Alex
changes in it and took it to a new high. (b) K. Radhakrishnan
(a) indebted, necessary (b) plagued, vital (c) Tapan Misra
(c) coping, more (d) hurt, critical (d) Satya N. Atluri
(e) shaken, inevitable (e) A. S. Kiran Kumar
140. The Prime Minister who is ______ in his holiday home at 148. In which year did the Foreign Account Tax Compliance Act
the moment said that he was very ______ by the news of (FATCA) come into effect?
India winning the World Cup. (a) 2007 (b) 2008
(a) visiting, happy (b) residing, obliged (c) 2010 (d) 2013
(c) intruding, dejected (d) staying, pleased (e) 2015
(e) resting, cheerful 149. What is India’s rank in creating jobs in renewable energy
General Awareness
sector in 2015?
(a) 6 (b) 7
141. What was India’s per capita income for 2015-16? (c) 5 (d) 9
(a) 65000 rupees (b) 72000 rupees (e) 10
(c) 85000 rupees (d) 110000 rupees 150. The Reserve Bank of India (RBI) directed that all banks and
(d) 77000 rupees Tg:- @NextGenBankers
142. What is Special Drawing Right(SDR)?
white-label ATM operators should have................ chips and
pin based card infrastructure on the cards.
(a) an urban free market economic process of (a) EMP (b) MS-CHAP
entrepreneurs replacing the imports of the city with (c) Dual EC DRBG (d) EMV
production from within the city. (e) SHA-1
(b) is an interest-bearing international reserve asset created 151. Which country is going to fund two road projects in North-
by the IMF in 1969 to supplement other reserve assets East India?
of member countries. (a) Germany (b) Israel
(c) a series of pre-defined commercial terms published by (c) Ukraine (d) UK
the International Chamber of Commerce (ICC). They (e) Japan
are widely used in International commercial 152. Which of the following businessmen is listed in Forbes’ list
transactions or procurement processes. of 40 most powerful people in the financial world?
(d) a policy followed by some international markets in (a) Jamshyd Godrej (b) Mukesh Ambani
which countries’ governments do not restrict imports (c) Uday Kotak (d) Gautam Thapar
from, or exports to, other countries. (e) Savji Dholakia
(e) a government policy to encourage export of goods and 153. Name the cricket club which has released the first ever app
discourage sale of goods on the domestic market through on Laws of Cricket.
direct payments, low-cost loans, tax relief for exporters, (a) Lancashire (b) Sydney Thunder
or government-financed international advertising. (c) Sydenham Cricket Club (d) Cornwall Cricket Club
143. Who was the first Chief of Naval Staff of the Indian Navy? (e) Marylebone Cricket Club
(a) Madhvendra Singh 154. A.R. Rahman who is India’s famous composer, singer-
(b) Ronald Lynsdale Pereira songwriter, music producer, musician and philanthropist is
(c) Charles Thomas Mark Pizey from which state?
(d) Sir Stephen Hope Carlill (a) Kerala (b) Tamil Nadu
(e) Sureesh Mehta (c) Maharashtra (d) Delhi
144. What is the tagline of State Bank of India (SBI)? (e) Andhra Pradesh
(a) A tradition of Trust 155. Which country has unveiled a new intellectual property
(b) Your Faithful and Friendly Financial Partner rights policy (IPR)?
(c) We understand your world (a) China (b) USA
(d) Pure Banking, Nothing Else (c) India (d) Japan
(e) Much more to do with you in Focus (e) UK
GP_4458
2016-12 SBI Clerk Mains Solved Paper-2016
156. Name the web portal launched by the Government which is (a) Hemchandra Vikramaditya
an e-Bidding & e-Reverse Auction portal for acquisition of (b) Vashwantrao Holkar
short-term by the Power Distribution Companies (DISCOMs)? (c) Prithviraj Chauhan
(a) DEEP (b) DEEPA (d) Maharana Pratap
(c) DEEPIKA (d) DEEPAM (e) Nana Fadnavis
(e) DEEPAK 168. Which bank has launched India’s G-Secs facility to retail
157. Which bank has launched India’s first contactless credit customers via ATMs?
card for small and mid-sized enterprises (SMEs)? (a) IDBI (b) ICICI
(a) Bank of Baroda (b) ICICl (c) SBI (d) HDFC
(c) HDFC (d) SBI (e) Federal Bank
(e) Punjab National Bank 169. Which state has banned diesel taxis from May 1, 2016?
158. Government has launched 2nd phase of Pradhanmantri (a) Delhi (b) Maharashtra
Ujjwala Yojana in (c) Haryana (d) Tamil Nadu
(a) Uttar Pradesh (b) Gujarat (e) Bihar
(c) Maharashtra (d) Bihar 170. Name the city which has been declared as a declared ‘smoke-
(e) Tamil Nadu free city.
159. In which year will the government set up the Monetary (a) Delhi (b) Imphal
Policy Panel Committee? (c) Agartala (d) Gangtok
(a) 2016 (b) 2018 (e) Kohima
(c) 2017 (d) 2019 171. Who is the Chairman of National Bank for Agriculture and
(e) 2020 Rural Development (NABARD)?
160. How many banks will merge with the State Bank of India(SBI) (a) Harsh Kumar Bhanwala
in the year 2016? (b) Rajiv Takru
(a) 12 (b) 6 (c) Prakash Bakshi
(c) 8 (d) 9 (d) N. Kamakodi
(e) 5 (e) Sri Umesh Chandra
161. Name the Act under which taxes are collected in India? 172. Name the law firm which is news due to the Panama Papers.
(a) The Income Tax decree of 1968 (a) Conyers Dill & Pearman
Tg:- @NextGenBankers
(b) The Income Tax Act, 1953
(c) The Income Tax Act, 1949
(b) Harney Westwood & Riegels
(c) Mourant Ozannes
(d) Income Tax Act, 1961 (d) Maples and Calder
(e) The Income Tax Act, 1950 (e) Mossack Fonseca
162. Which bank has launched SmartUp, a dedicated solution 173. India has recently signed four Memorandum of
for startups, to fulfil all their banking needs? Understandings (MoUs) in various sectors to boost bilateral
(a) SBI (b) ICICI cooperation with which country?
(c) HDFC (d) Bank of Baroda (a) Singapore (b) Malaysia
(e) Punjab National Bank (c) Indonesia (d) Samoa
163. In which country is the Internet Corporation for Assigned (e) Papua New Guinea
Names and Numbers (ICANN) headquartered? 174. Government has raised the employees provident fund (EPF)
(a) Sweden (b) UK rate to .................. % for 2015-16.
(c) France (d) Switzerland (a) 9 (b) 10
(e) USA (c) 8.9 (d) 8.8
164. In response to which disease did the World Bank launch a (e) 9.2
$500 million insurance fund? 175. Name the committee formed by the Reserve Bank of India
(a) Malaria (b) Dengue (RBI) to ease compliance reporting norms for banks.
(c) Encephalitis (d) Tropical Nulcer (a) Ghosh (b) Sharma
(e) Ebola (c) Jilani (d) Patwardhan
165. Aadhar card has a ........digit unique identity number. (e) Patil
(a) 12 (b) 13 176. Name the scheme launched by the Railways Ministry to
(c) 15 (d) 20 prevent accidents.
(e) 18 (a) Rashtriya Rail Sanraksha Kosh
166. The government appoints ............. Deputy Governors to the (b) Rail Suraksha Kawach
Reserve Bank of India (RBI). (c) Rashtriya Rail Kushal
(a) 5 (b) 3 (d) Rail Kushalta Yojana
(c) 2 (d) 6 (e) Rashtriya Rail Salamati Kosh
(e) 4 177. In which year was the Goldman Sachs bank established?
167. To mark the 475th birth anniversary of ........................., (a) 1869 (b) 1876
Government has released a commemorative coin of `100 (c) 1901 (d) 1799
and a circulation coin of `10. (e) 2002
SBI Clerk Mains Solved Paper-2016 2016-13
178. Government has hiked the minimum wage for contract 185. The Reserve Bank of India(RBI) has permitted business
workers to Rs. .............. per month houses to invest in the banks to less than .......... %.
(a) 9500 (b) 9000 (a) 15 (b) 9.5
(c) 10000 (d) 11000 (c) 10 (d) 20
(e) 12000 (e) 8
179. Name the country which launched the X-2 fighter craft. 186. The government has given its approval for a currency swap
(a) Israel (b) USA agreement between the Reserve Bank of India (RBI) and
(c) China (d) Spain ................. .
(e) Japan (a) National Bank of Austria
180. Name the businessman who has been named as World’s (b) Central Bank of the United Arab Emirates
Most Admired Man 2016. (c) Czech National Bank
(a) Carlos Slim (b) Mark Zuckerberg (d) Deutsche Bundesbank
(c) Warren Buffett (d) Bill Gates (e) Reserve Bank of Australia
(e) Larriy Ellison 187. According to ................ FDI Confidence Index India is ranked
181. Which bank has decided to close 24 branches in India? 9th in the 2016 Foreign Direct Investment (FDI) Confidence
(a) ABN AMRO (b) HSBC Index.
(c) Credit Suisse (d) Standard Chartered (a) The Global Dow (b) MSCI World
(e) Deutsche Bank (c) BBC Global 30 (d) FTSE All-World index series
182. Which bank has reported a massive loss of ` 5,367 crore (e) AT Kearney
for the quarter ending March 2016? 188. Which country has been ranked as a top destination for
(a) Punjab National Bank Foreign Direct Investment (FDI)?
(b) State Bank of Bikaner & Jaipur (a) China (b) India
(c) UCO Bank (c) Malaysia (d) Singapore
(d) Union Bank of India (e) Japan
(e) Allahabad Bank 189. The 8th Banking & Finance Conference was organised by
183. In which city is the Multi Commodity Exchange of India Ltd Indian Merchants’ Chamber (IMC) and the ...............
(MCX) headquartered? (a) ICTACEM (b) IOV
(a) Bengaluru (b) Mumbai (c) ISTE (d) ICSI
(c) Delhi
(e) Hyderabad
Tg:- @NextGenBankers
(d) Chennai (e) ICAI ‘
190. Which company has teamed up with Indian Railways to
184. Which company has launched a youtube like video service? provide free wi-fi service at railway stations?
(a) Netflix (b) Amazon (a) Google (b) Facebook
(c) Redbox (d) King and McGaw (c) Twitter (d) Microsoft
(e) Wuaki.tv (e) HCL
ANSW ER KEY
1 (c) 21 (b) 41 (b) 61 (e) 81 (c) 101 (d) 121 (e) 141 (d) 161 (d) 181 (b)
2 (a) 22 (a) 42 (d) 62 (c) 82 (b) 102 (a) 122 (d) 142 (b) 162 (c) 182 (a)
3 (a) 23 (c) 43 (b) 63 (d) 83 (e) 103 (b) 123 (a) 143 (c) 163 (e) 183 (b)
4 (d) 24 (d) 44 (e) 64 (e) 84 (c) 104 (d) 124 (b) 144 (d) 164 (e) 184 (b)
5 (c) 25 (e) 45 (b) 65 (d) 85 (a) 105 (a) 125 (d) 145 (b) 165 (a) 185 (c)
6 (b) 26 (b) 46 (e) 66 (c) 86 (e) 106 (c) 126 (d) 146 (b) 166 (e) 186 (d)
7 (c) 27 (a) 47 (c) 67 (c) 87 (b) 107 (b) 127 (a) 147 (e) 167 (d) 187 (e)
8 (a) 28 (b) 48 (e) 68 (d) 88 (a) 108 (c) 128 (c) 148 (c) 168 (a) 188 (b)
9 (d) 29 (d) 49 (c) 69 (c) 89 (e) 109 (d) 129 (e) 149 (c) 169 (a) 189 (e)
10 (d) 30 (e) 50 (d) 70 (e) 90 (d) 110 (a) 130 (b) 150 (d) 170 (e) 190 (a)
11 (a) 31 (d) 51 (d) 71 (c) 91 (c) 111 (c) 131 (c) 151 (e) 171 (a)
12 (e) 32 (a) 52 (b) 72 (b) 92 (c) 112 (d) 132 (a) 152 (c) 172 (e)
13 (d) 33 (d) 53 (b) 73 (d) 93 (c) 113 (d) 133 (b) 153 (e) 173 (e)
14 (b) 34 (b) 54 (e) 74 (c) 94 (b) 114 (e) 134 (e) 154 (b) 174 (d)
15 (a) 35 (d) 55 (c) 75 (c) 95 (b) 115 (b) 135 (d) 155 (c) 175 (c)
16 (b) 36 (d) 56 (a) 76 (a) 96 (a) 116 (b) 136 (b) 156 (a) 176 (a)
17 (d) 37 (a) 57 (d) 77 (b) 97 (a) 117 (a) 137 (c) 157 (b) 177 (a)
18 (a) 38 (c) 58 (b) 78 (e) 98 (a) 118 (c) 138 (e) 158 (b) 178 (c)
19 (c) 39 (c) 59 (e) 79 (c) 99 (a) 119 (a) 139 (a) 159 (c) 179 (e)
20 (a) 40 (c) 60 (c) 80 (d) 100 (a) 120 (b) 140 (d) 160 (b) 180 (d)
GP_4458
2016-14 SBI Clerk Mains Solved Paper-2016
so both × and +
Tg:- @NextGenBankers
Q to be mother of K - P must be son or daughter of Q. 30 16 10 8 8 9 10.5
61. (e) From I: C > A > B and – > D Now, using the condition from the question,
From II: – > – > C Let the number of boys be x.
Combining we get Then, 70% of x + 85% of (480 – x) = 360
E > D > C>A> B
75 ´ x 85 ´ (480 - x)
62. (c) Using statement I: Þ + = 360
If Pawan turns 90° towards his right he will face, South. 100 100
So Ali is facing North. Þ 70x – 85x + 40800 = 36000
Using statement II: Þ 40800 – 36000 = 85x – 70x
4800
Þ 48000 = 15x Þ x = = 320
15
\ There are 320 boys who appeared for the examination.
1m 68. (d) Let the weight of fresh grapes be x.
In the end Preeti faces North. so Ali aloso faces North. 80 4x
63. (d) Even by using both the stetements together we can Quantity of water in it = ×x=
100 5
only determine that number of candidates appreared
more than 200. æ 4x ö x
64. (e) Using I: Quantity of pulp in it = ç x - ÷ =
è 5 ø 5
5
T R and T is midway between P and Q. Quantity of water in 500 kg dry
Using II:
10
2 grapes = × 500 = 50 kg
R Q 100
Using Both: \ Quantity of pulp in it = (500 – 50) = 450 kg
7 5 2 x
P T Q = 450
5
65. (d) Even by using both the statements together we
\ x = 2250 kg
66. (c)
or two brothers.Tg:- @NextGenBankers
cannotdetermine the gender of B. So A can have one
80.
Tg:- @NextGenBankers
(d) Reuired percentage =
45
225
´ 100 = 20%
Hence, the number of ways, all vowels will come
together
81. (c) Ratio of income (A's : C's) = 18 : 3 = 6 : 1 = 6! × 2! = 1 × 2 × 3 × 4 × 5 × 6 × 2 = 1440
Total number of ways in which vowels will never come
100
6x ´ together
\ Ratio of their exp. = 120 = 25 : 4
100 = 5040 – 1440 = 3600
x´ 92. (c) 15 × 5 = 75
225
4 × 50 = 200
83. (e) Profit fo these Cos. in ` cr 6 × 75 = 450
A C D E F 5 × 20 = 100
90 18 64.3 50 145 Total = 75 + 200 + 450 + 100 = `825
84. (c) ` 280 cr 92. (c) Here, there are four quantities, examiner, no. of
answer papers, day and hour. We have to calculate
124 hours. Hence the quantity 'hour' should be in the last
85. (a) Income of Company 'F' = 750 ×
100 column. Following relationship exists:
\ Total income of all the Cos. (i) Less examiners, more hours (inverse)
(ii) More answer papers, more hours (direct)
124 100
= 750 × ´ = ` 3720 cr (iii) More days, less hours (inverse)
100 25 Hence,
86. (e) Let the present ages of Raj and Radha be 4x and 9x
Then, accoding to the question, Examiner Answer Papers Days Hours
4 1¯ 10 5 ¯
9x = 2(9x – 12) – 32 2 2 20 x
Þ 9x = 27 Þ x = 3 yrs Again,
Raj's present age = 4x = 12 yrs 2 : 4 üï
After 5 yrs, 1 : 2 ý =: : 5 : x
Raj's age will be 12 + 5 = 17 yrs 20 : 10 ïþ
87. (b) Ajay's score = 63 + 30 + 93 or , 2 ´ 1´ 20 ´ x = 4 ´ 2 ´ 10 ´ 5
Rahul's score = 93 – 15 = 78 4 ´ 2 ´10 ´ 5
\x = = 10hour per day
Manish's + Suresh's score 2 ´1´ 20
= 63 × 3 – 78 = 189 – 78 = 111
GP_4458
2016-18 SBI Clerk Mains Solved Paper-2016
1 13 13
94. (b) Second number = × 2960 = 740 \ y= or = 2.6 or 3.25
4 5 4
Let the first number be x.
\ x> y
5 25
x= ´ 740
9 100 15 9
98. (a) I. + = 11 x
9 1
x x
x= ´ × 740 = 333
5 4 15 + 9
= 11 x 24 = x
30 x
30% of 1st number = ´ 333 = 99.9
100
24
\ x= = 2.18
12 11
95. (b) Total girls = ´ 250 = 30
100
y 5 y 1
II. + =
Total bots = 250 – 30 = 220 4 12 y
Each boy's mountly fee
3 y +5 y 1
= 1.24 × 450 = 558 =
12 y
Total monthly fee of boys and girls together
8y = 12
= (220 ×558) + (30 × 450) = 122760 + 13500 = ` 136260
y = 1.5
96. (a) I. 289 x + 25 = 0 8 6
99. (a) I. + = x
or, x x
289 x = - 25
Tg:- @NextGenBankers
squaring both sides, we get 289x = 25 14
= x
x
25
x= x = 14
289
x> y
4.
(e) None of these
144. ¸ 8 ¸ ? = 9
(a) 3
Tg:- @NextGenBankers
(b) 2
13. Bina's monthly income is 90% of Anita's monthly income.
The total of both their monthly incomes is Mr. Sen's monthly
income. Mr. Sen's annual income is 7,75,200. What is Bina's
(c) 4 (d) 6 monthly income?
(e) None of these (a) ` 34,000 (b) ` 36,000
5. ?% of 590 – 11.8 = 236 (c) ` 30,600 (d) ` 30,000
(a) 48 (b) 45 (e) None of these
(c) 42 (d) 41 14. What is the value of three fourth of sixty per cent of 480?
(e) None of these (a) 216 (b) 218
6. (8)2 % of ? = 723 + 45 (c) 212 (d) 214
(a) 1200 (b) 1400 (e) None of these
(c) 1100 (d) 1020 15. The height of 5 boys is recorded as, 146 cm, 154 cm, 164 cm,
(e) None of these 148 cm and 158 cm. What is the average height of all these
7. 5554 – 333 + 45 = ? + 2525 boys?
(a) 2174 (b) 2417 (a) 152 cm (b) 158 cm
(c) 2714 (d) 2741 (c) 156 cm (d) 154 cm
(e) None of these (e) None of these
1 1 1 16. Pravin purchased 25 kg of rice at the rate ` 45 per kg and 12
8. 3 +2 +6 = ?
4 2 6 kg of pulses at the rate of ` 28 per kg. What is the total
amount that he paid to the shopkeeper?
11 11 (a) ` 1, 466 (b) ` 1,416
(a) 13 (b) 11
12 12 (c) ` 1,461 (d) ` 1, 471
(e) None of these
11 11
(c) 12 (d) 15 17. If one man or three women or five boys can do a piece of
12 12 work in 46 days then how many days will one man, one
(e) None of these woman and one boy together take to complete the same
9. (450 ¸ 30)2 – (12)2 = (?)2 piece of work?
(a) 92 (b) – 92 (a) 30 days (b) 32 days
(c) 9 (d) 81 (c) 35 days (d) 40 days
(e) None of these (e) None of these
GP_4458
2016-20 SBI Clerk Prelim Solved Paper-2016
18. The perimeter of a rectangle is 60 cm and its breadth is 12 29. 3 100 297 594 991 (?)
cm. What is the area of the rectangle? (a) 1489 (b) 1479
(a) 261 cms2 (b) 263 cms2 (c) 1478 (d) 1498
(c) 213 cms 2
(d) 216 cms2 (e) None of these
(e) None of these 30. 112 119 140 175 224 (?)
19. A train crossed a platform in 43 seconds. The length of the (a) 277 (b) 276
train is 170 metres. What is the speed of the train? (c) 287 (d) 266
(a) 233 km./hr (b) 243 km./hr (e) None of these
(c) 265 km./hr (d) Cannot be determined
(e) None of these DIRECTIONS (Qs. 31-35): Study the following table carefully
20. If a number is multiplied by three- fourth of itself, the value to answer the questions that follow: Number (N) of Six types of
thus obtained is 10800. What is that number? Electronic Product Sod by Six different stores in a month and
(a) 210 (b) 180 the price per product (P) (price in Rs. 000) charged by each
(c) 120 (d) 160 store.
(e) 140
21. If three fourth of a number is subtracted from the number; Store A B C D E F
the value so obtained is 163. What is that number? Product N P N P N P N P N P N P
(a) 625 (b) 562 L 54 135 48 112 60 104 61 124 40 136 48 126
(c) 632 (d) 652 M 71 4.5 53 3.8 57 5.6 49 49 57 5.5 45 4.7
(e) None of these
N 48 12 47 18 52 15 54 11.5 62 10.5 56 11
22. What will be the compound interest accrued on a sum of `
35000 at the rate of 8 p.c.p.a in 2 years? O 52 53 55 48 48 50 54 49 59 47 58 51
(a) ` 5,884 (b) ` 5,284 P 60 75 61 68 56 92 44 84 46 76 59 78
(c) ` 5,524 (d) ` 5,428 Q 43 16 44 15 45 14.5 48 15.6 55 18.2 55 14.9
(e) None of these
23. The area of a square is twice the area of a circle. The area of 31. The number of L type products sold by Store F is what per
the circle is 392 cm2. What is the length of the side of the cent of the number of the same type of products dold by
square?
(a) 28 cm Tg:- @NextGenBankers
(b) 26 cm
Store E ?
(a) 76.33 (b) 124
(c) 24 cm (d) 22 cm (c) 83.33 (d) 115
(e) None of these (e) None of these
24. It is required to get 40% marks to pass an exam. A candidate
32. What is the ratio of the total number of N and L type products
scored 200 marks and failed by 8 marks. What were the
together sold by Store D and that of the sme products sold
maximum marks of that exam?
(a) 530 (b) 540 by Store A?
(c) 502 (d) Cannot be determined (a) 119 : 104 (b) 102 : 115
(e) None of these (c) 104 : 115 (d) 117 : 103
25. The sum of five consecutive numbers is 190. What is the (e) None of these
sum of the largest and the smallest number? 33. What is the average price per product charged by all the
(a) 75 (b) 77 Stores together for Product Q?
(c) 76 (d) 73
(e) None of these (a) Rs 14,700 (b) Rs 15,700
(c) Rs 15,200 (d) Rs 14,800
DIRECTIONS (Qs. 26-30): What will come in place of the (e) None of these
question mark (?) in the following number series?
34. What is the difference in the amout earned by Store A
26. 1 7 49 343 (?) through the sale of P type products and that earned by
(a) 16807 (b) 1227 Store B through the sale of Q type products?
(c) 2058 (d) 2401 (a) Rs 38.4 lakhs (b) Rs 0.384 lakh
(e) None of these
(c) Rs 3.84 lakhs (d) Rs 384 lakhs
27. 13 20 39 78 145 (?)
(a) 234 (b) 244 (e) None of these
(c) 236 (d) 248 35. What is the total amount earned by Store C through the
(e) None of these sale of M and O type products together?
28. 12 35 81 173 357 (?) (a) Rs 2719.2 lakhs (b) Rs 271.92 lakhs
(a) 725 (b) 715 (c) Rs 2.7192 lakhs (d) Rs 27.192 lakhs
(c) 726 (d) 736 (e) None of these
(e) None of these
SBI Clerk Prelim Solved Paper-2016 2016-21
Reasoning Ability 44. What will come in place of question mark in the following
series based upon the given arrangement?
DIRECTIONS (Qs. 36-40): Study the following information 7#G, EF4, 1W%, 251, ?
carefully and answer the questions given below? (a) YH@ (b) Q@6
(c) 52N (d) YBQ
Eight friends, A, B, C, D, E, F, G and H, are sitting around a (e) None of these
circular table facing the centre. Further, it is known that 45. Four of the following five are alike in a certain way based on
(A) D sits opposite H and E sits on the immediate left of H. their positions in the above arrangement and so form a
(B) B cannot be an immediate neighbour of either D or H and group. Which is the one that does not belong to that group?
sits third to the left of F. (a) EGR (b) K$F
(C) G sits second to the right of C. (c) b @ M (d) NB2
(D) F is not an immediate neighbour of H. (e) 5% KU
36. Who is sitting exactly between E and F?
(a) B (b) A DIRECTIONS (Qs. 46-50): In these questions, the following
(c) H (d) C symbols are used with different meaning as follows:
(e) None of these 'A#B' means ' A is neither greater than nor equal to B'.
37. How many persons are sitting between A and C, starting 'A©B' means 'A is neither equal to nor smaller than B'.
from C in clockwise direction? 'A%B' means 'A is neither smaller than nor greater than B'.
(a) None (b) One 'A$B' means 'A is not smaller than B'.
(c) Two (d) Three 'A@B' means 'A is not greater than B'.
(e) More than three Now, in each of the following questions, assuming the given
38. What will come in place of question mark in the following statements to be true, find which of the two conclusions I and II
series according to the above seating arrangement? given below them is/are true. Give answer.
D, A, C, F, ? (a) if only conclusion I is true.
(a) H (b) E (b) if only conclusion II is true.
(c) G (d) B (c) if either conclusion I or II is true.
(e) None of these (d) if neither conclusion I nor II is true.
39. Four of the following five are alike in a certain way based on (e) if both conclusions i and II are true.
Tg:- @NextGenBankers
their seating positions, and so form a group. Which is the
one that does not belong to that group?
46. Statements : Z # F,
Conclusions : I. Z # R
R @ F,
II. D © Z
D© R
52. Which of the following combinations of exam - Day - Time 60. Which of the following information is not required for
Duration is correct ? the above arrangements?
(a) English - Wednesday - 75 mins (a) Only (i) (b) Only (ii)
(b) Maths - Thursday - 50 mins (c) All are required (d) Only (iii)
(c) History - Thursday - 60 mins (e) None of these
(d) Hindi - Tuesday - 100 mins 61. Ravi is Raju's father, Sudha is Raju's grandmother, Radha is
(e) None is correct Sudha's sister, Rani is Ravi's wife. How is Ravi related to
53. What is the time duration of science exam? Radha?
(a) 90 mins (b) 75 mins (a) Sister (b) Niece
(c) 50 mins (d) 40 mins (c) Sister-in-law (d) Maternal uncle
(e) None of these (e) None of these
54. On which day is Economics exam scheduled?
(a) Monday (b) Saturday DIRECTIONS (Qs. 62-63): Study the following information to
(c) Tuesday (d) Friday answer the given questions.
(e) Cannot be determined In a certain code, 'strong financial economy' is written as 'mo tic
55. Which day is sunday? su', 'financial inclusion needed' is written as 'da ra su' and 'ecnomy
(a) 3rd march (b) 2nd march crisis inclusion' is written as 'ye da mo'.
(c) 5th march (d) 6th march 62. What is the code for 'financial'?
(e) Cannot be determined (a) da (b) su
(c) mo (d) ra
DIRECTIONS (Qs. 56-60): Read the followingstatements
(e) Can't be determined
carefully and answer the questionsgiven below:
63. What does 'tic' stand for?
The Director of the institute XYZ hasannounced that six guest (a) economy (b) financial
lecturers on different areas like Leadership, Decision Making, (c) strong (d) needed
Syllogism, Motivation, Verbal Reasoning and Group Discussions (e) Either economy or strong
are to be organised only oneon each day from Monday to Sunday. 64. Which of the following expressions is correct if the
(i) Motivation should be organisedimmediately after Verbal expression 'Z > Y > W < V' is definitely correct?
Reasoning.
Tg:- @NextGenBankers
(ii) Syllogism should be organised onWednesday and should
not be followed by Group Discussion.
(a) V > Y
(c) Z > V
(e) None of these
(b) W < Z
(d) Y > V
(iii) Decision Making should be organisedon Friday and there 65. Town D is 13 km towards the East of town A. A bus starts
should be a gap oftwo days between Leadership andGroup from town A, travels 8 km towards West and takes a right
Discussion. turn. After taking the right turn, it travels 5 km and reaches
(iv) One day there will be no lecture(Saturday is not that town B. From town B the bus takes a right turn again, travels
day), just beforethat day Group Discussion will 21 km and stops.
beorganised. How far and towards which direction must the bus travel to
56. Which of the Pairs of lectures were organised on first reach town D?
and last days? (a) 13 km towards South (b) 5 km towards West
(a) Syllogism and Motivation (c) 21 km towards South (d) 5 km towards South
(b) Group Discussion and Sylloyism (e) None of these
(c) Group Discussion and Decision Making DIRECTIONS (Qs. 66-70): Study the following information
(d) Cannot be determined and answer the given questions:
(e) None of these In a certain code language -
57. How many lecturers were organised between Motivation
'hospital is far from house' is written as 'to ga di ba ni'. 'house' is
and Syllogism?
the hospital bus' is written as 'ru to ni di zi'. 'come from hospital'
(a) One (b) Two
is written as 'ga ni mo'. 'is the bus late' is written as 'ru zi fa to'.
(c) Three (d) Four
66. What does the code 'ga' stand for in the given code
(e) None of these
language?
58. Which day will the lecture on Leadership be organised?
(a) from (b) house
(a) Tuesday (b) Wednesday
(c) far (d) is
(c) Friday (d) Can't be determined
(e) None of these
(e) None of these
67. What is the code for 'late' in the given code language?
59. On which day there is no lecture?
(a) Sunday (b) Monday (a) to (b) fa
(c) Tuesday (d) Wednesday (c) zi (d) ru
(e) None of these (e) None of these
SBI Clerk Prelim Solved Paper-2016 2016-23
68. What will the code 'fa mo ba' stand for in the given code as declared in scriptures : "For protecting the righteous, for
language? destroying the wicked and for firmly upholding the law, I am
(a) come far late (b) bus far late born on Earth age after age." Dhrishtadyumna also consoled his
(c) come far hospital (d) come late from sister and told her how nemesis would overtake the Kauravas.
(e) None of these He said : "I will kill Drona. Shikhandi will cause Bhishma's fall.
69. What may the code 'ru mo di' stand for in the given code Bhima will take the lives of the wicked Duryodhana and his
language? brothers. Arjuna will slay Karna, the charioteer's son."
(a) the bus house (b) come from house Sri Krishna said : "When this calamity befell you, I was not
(c) come the late (d) come the house in Dwaraka. Has I been there, I would never have allowed this
(e) late is come fraudulent game of dice to take place. Uninvited, I would have
70. What is the code for 'hospital' in the given code langauge? gone there and stirred up Drona, Kripa and the other elders to a
(a) ba (b) di sense of duty. I would, at all costs, have prevented this
(c) ni (d) ga destructive play of dice. When Sakuni was cheating vou, I was
(e) None of these fighting King Salva, who had besieged my city. It was only after
I had defeated him that I came to know of the game of dice and
English Language the subsequent sordid story. It grieves me that I am not able to
remove your sorrows immediately but you know, some water
DIRECTIONS (Qs. 71-80) : Read the passage carefully and must be lost before a broken dam is restored."
answer the questions given below it. Certain words/phrases are 71. Why was Krishna unable to prevent the destructive game
given in bold to help you locate them while answering some of of dice?
the questions. (a) Because he himself was fond of dicing.
As soon as Krishna learnt of the events at Hastinapura, the (b) Because he wanted the Pandavas to be robbed of their
game of dice and the exile of the Pandavans, he set out for the property.
forest where the Pandavas, were living. Along with Krishna went (c) Because he hated Bhima, Draupadi's husband.
many, including men of the Bhoja and Vrishni tribes. Dhrishtaketu, (d) Because he was fighting king Salva, who had besieged
the king of Chedi country, and the Kekayas, who were all devoted his city, when she was being cheated by Sakuni
(e) None of these
Tg:- @NextGenBankers
to the Pandavas. They were filled with righteous indignation
when they heard of Duryodhana's perfidy and cried out that
surely the earth would drink the blood of such wicked people.
72. Why did the sons of Dhritarashtra insult Draupadi?
(a) Because Draupadi had become their slave like her five
husbands.
Draupadi approached Sri Krishna and, in a voice drowned in
(b) Because Draupadi was of low birth and breeding and
tears and broken with sobs, told the story of her wrongs.
was merely a kept for them.
She said : "I was dragged to the assembly when I had but a
(c) Because she was an ally of Krishna, whom the sons of
single garment on my body. The sons of Dhritarashtra insulted
Dhristrashtra hated.
me most outrageously and gloated over my agony. They thought
(d) Because Draupadi had abused Dhritarashtra.
that I had become their slave and accosted me and treated me as
(e) None of these
one. Even Bhishma and Dhritarashtra forgot my birth and
73. Why were Draupadi's husbands helpless to protect her from
breeding and my relationship to them. O Janardhana, even my the insults of wicked sons of Dhritarashtra?
husbands did not protect me from the jeers and the ribald insults (a) Because they were enjoying Draupadi being dragged
of those foul ruffians. Bhima's bodily strength and Arjuna's Gandiva into and insulted in the court.
were alike of no avail. Undner such supreme provocation even (b) Because they were turned into slaves and had no right
weaklings would have found strength and courage to strike the vile before their masters to interfere with them.
insulter dead. The Pandavas are renowned heroes and yet (c) Because Draupadi's husbands were disrobed of their
Duryodhana lives! I, the daughter-in-law of the emperor Pandu, clothes and were more concerned about hiding their
was dragged by my hair. I, the wife of five heroes, was dishonoured. nudity.
O Madhusudana, even you had deserted me." She stood trembling, (d) Because the Pandavas had no weapons with them to
utterly unable to continue, for the grief convulsed her. protect Draupadi.
Krishna was deeply moved and he consoled the weeping (e) None of these
Draupadi. He said : "Those who tormented you will be stricken 74. What was Krishna's vow to Draupadi?
to death in the bloody quagmire of a lost battle. Wipe your eyes. (a) That he will punish the Pandavas for not protecting
I solemnly promise that your grievous wrongs shall be amply Draupadi.
avenged. I shall help the Pandavas in every way. You will become (b) That he will kill Dhritarashtra for just sitting and
an empress. The heavens may fall, the Himalayas may split in watching the sorrows inflicted on Draupadi like a mute
twain, the earth may crumble or the boundless sea may dry up- spectator.
but, I tell you verily, my words shall stand, swear this," and (c) That he will help Pandavas to avenge the Kauravas.
Krishna took a solemn vow before Draupadi. This vow, it will be (d) That he will destroy the whole Bharata race.
seen, was in perfect accord with the purpose of the Lord's avatars, (e) All the above
GP_4458
2016-24 SBI Clerk Prelim Solved Paper-2016
75. Which of the following is not the purpose of avatars of 83. (a) exercise (b) expense
Lord as declared in the scriptures? (c) experience (d) examination
(a) To protect the righteous (e) job
(b) To uphold the law 84. (a) experienced (b) devoted
(c) To curse the inflictors (c) motivated (d) targeted
(d) To destroy the wicked (e) upset
(e) None of these 85. (a) pleasant (b) enthusiastic
(c) motivated (d) vigorous
DIRECTIONS (Qs. 76-78) : Choose the word/group of words
(e) jolly
which is MOST SIMILAR in meaning to the word/group of words
printed in bold as used in the passage. DIRECTIONS (Qs. 86-90) : In each question below, four words
76. Gloated printed in bold type are given. These are numbered (a), (b), (c)
(a) incited (b) choked and (d). One of these words printed in bold may either be
(c) rejoiced (d) muffled wronglky spelt or in appropriate in the context of the sentence.
(e) sympathised Find out the word that is inappropriate or wrongly spelt, if any.
77. Accosted The number of that word is your answer. If all the words printed
(a) addressed (b) attributed in bold are correctly spelt and appropriate in the context of the
(c) conceded (d) ignored sentence then mark (e) i.e. 'All Correct', as your answer.
(e) scorned 86. Anushka works with a (a) / garment export house (b) / and
78. Tormented is thirty year old lady (c) / with very attractive and gorgeuos
(a) agitated (b) exhausted personality. (d) / No error (e).
(c) pleased (d) relieved 87. The management might not be (a) / successful in
(e) tortured implementing changes (b) / if the informed (c) / organisation
DIRECTIONS (Qs. 79-80) : Choose the word/group of words opposes them. (d) / No error (e).
which is MOST OPPOSITE in meaning of the word/group of 88. The practise of telecasting (a) / of vacant posts (b) / over
words printed in bold as used in the passage. television is gaining (c) / importance these days. (d) / No
error (e).
79. Convulsed
(a) disturbed
Tg:- @NextGenBankers
(b) bothered
89. By using external sources (a) / or recruitment, the
management (b) / can be attracted (c) / qualified and trained
(c) liked (d) pacified people. (d) / No error (e).
(e) praised 90. The department of pharmaceuticals has told (a) / a group of
80. Besieged ministers (b) / that competition does not (c) / necessarily
(a) captured (b) freed lead to reduction in prices. (d) / No error (e).
(c) attacked (d) acknowledged
(e) plundered DIRECTIONS (Qs. 91-95) : Rearrange the following six sentences
(A), (B), (C), (D), (E) and (F) in the proper sequence to form a
DIRECTIONS (Qs. 81-85) : In the following passage, some of meaningful paragraph and then answer the questions given
the words have been left out, each of which is indicated by a below.
number. Find the suitable word from the options given against
each number and fill up the blanks with appropriate words to (A) It so happened that in 1977 it was able to collaborate with
make the paragraph meaningful. other political parties.
(B) Power was never its objective.
Bangalorean Clawin D'Souza, 22, is a (81) online poker fan. So, (C) Then, in 1998, they for power at the centre.
much so that he gave up his job to take up a career in the game. (D) The job of the Jan Sangh was to defend the RSS.
His parents, of course, did not like the idea, but D'Souza went (E) They (Hindutva forces) started getting into power.
ahead with this decision. Though he started a bit slow and made (F) When this happened their old leaders Vajpayee and Advani
very little money (82), gradually he became an expert at the game. started to see RSS chiefs and leaders as their juniors.
"The (83) has been great because I am my own boss. I set my 91. Which of the following sentences will come 'FIRST' after
goals and I am very (84) to achieve them. I love what I,m doing rearrangement?
and it doesn't feel like a (85) job at all," said D'Souza, who now (a) A (b) B
makes around `90,000 a month playing poker online. (c) C (d) D
81. (a) inflexible (b) conservative (e) E
(c) bigot (d) diehard 92. Which of the following sentences will come 'THIRD' after
(e) dogmatic rearrangement?
82. (a) finally (b) initially (a) A (b) B
(c) making (d) market (c) C (d) E
(e) lending (e) F
SBI Clerk Prelim Solved Paper-2016 2016-25
93. Which of the following sentences will come 'SECOND' after 96. Cleansing, toning (a) / and moisturiser (b) / are three steps
rearrangement?
(a) B (b) A to (c) / accentuate beauty. (d) / No error (e).
(c) C (d) D 97. The principle of unity of (a) / command states that each (b)
(e) E / participant should be received (c) / orders for only one
94. Which of the following sentences will come 'LAST' after
rearrangement? superior. (d) / No error (e).
(a) A (b) B 98. An important development (a) / in the second halves (b) / of
(c) F (d) C the 19th century was (c) / the large-scale industries in India.
(e) D
95. Which of the following sentences will come 'FOURTH' after (d) / No error (e).
rearrangement? 99. Sylvia Plath gained (a) / a reputation of one (b) / of the most
(a) A (b) C
(c) D (d) B important American (c) / female poets (d) / No error (e).
(e) E 100. A more serious citicism of English (a) / by whom attempting
DIRECTIONS (Qs. 96-100) : Read each sentence to find out to master it (b) / is the chaotic character of our spelling and
whether there is any grammatical error in it. The error, if any, the frequent lack of (c) / correlation between spelling and
will be in one part of the sentence. The number of that part is the
pronunciation. (d) / No error (e).
answer. if there is no error, the answer is (e). (Ignore errors of
punctuation, if any.)
ANSW ER KEY
1 (d ) 11 (b ) 21 (d ) 31 (e) 41 (b ) 51 (e) 61 (e) 71 (d) 81 (d) 91 (d)
2 (e) 12 (b ) 22 (e) 32 (e) 42 (c) 52 (b ) 62 (b) 72 (a) 82 (b) 92 (a)
3
4
(a)
(b )
13
14
(c)
(a) Tg:- @NextGenBankers
23
24
(a)
(e)
33
34
(b )
(a)
43
44
(c)
(b )
53
54
(d )
(a)
63
64
(c)
(b)
73
74
(e)
(c)
83
84
(c)
(c)
93
94
(a)
(c)
5 (c) 15 (d ) 25 (c) 35 (d ) 45 (d ) 55 (d ) 65 (d) 75 (c) 85 (d) 95 (e)
6 (a) 16 (c) 26 (d ) 36 (b ) 46 (d ) 56 (e) 66 (a) 76 (c) 86 (d) 96 (b)
7 (d ) 17 (a) 27 (d ) 37 (c) 47 (b ) 57 (c) 67 (b) 77 (a) 87 (e) 97 (c)
8 (b ) 18 (d ) 28 (a) 38 (d ) 48 (b ) 58 (e) 68 (a) 78 (e) 88 (a) 98 (b)
9 (e) 19 (d ) 29 (e) 39 (d ) 49 (e) 59 (c) 69 (d) 79 (d) 89 (b) 99 (b)
10 (b ) 20 (c) 30 (c) 40 (b ) 50 (d ) 60 (c) 70 (c) 80 (b) 90 (e) 10 0 (e)
GP_4458
2016-26 SBI Clerk Prelim Solved Paper-2016
6. (a) Q ? ×
( 8 )2
100
Tg:- @NextGenBankers
= 723 + 45
æ 5 ´ 46 ´ 3 ö
\ Required number of days = ç
è 23 ø
÷ = 30 days
768 æ1 ö
Þ 0.64 ´ ? = 768 \ ? = = 1200 18. (d) Q Length of the rectangle = ç ´ 60 - 12 ÷ = 18cm
0.64 è2 ø
7. (d) ? + 2525 = 5554 – 333 + 45 \ Area of the rectangle = (18 × 12 ) = 216 sq. cm
\ ? = 5266 – 2525 = 2741 19. (d) Length of the platform is not given.
1 1 1 20. (c) Let the number be x.
8. (b) ? = 3 + 2 + 6 Now, according to the question,
4 2 6
3
æ1 1 1ö æ 3+ 6 + 2 ö 11 x´ x = 10800
= ( 3 + 2 + 6 ) + ç + + ÷ = 11 + ç ÷ = 11 4
è 4 2 6 ø è 12 ø 12
4
(e) Q (?)2 = (450 ¸ 30)2 – (12)2=(15)2–(12)2=225–144= 81
2
9. Þ x = 10800 ´ = 14400
3
= (9)2
\?= 9 \ x = 14400 = 120
10. (b) ? = 56.73 + 32.88 + 45.23 = 134.84 21. (d) Let the number be x.
Now, according to the question,
æ 1230 ö
11. (b) Q Speed of the train = ç ÷ = 246km / hr 3x
è 5 ø x- = 163
4
æ1 ö
\ Speed of the truck = ç ´ 246 ÷ = 82km / hr \ x = 163 ´ 4 = 652
è3 ø
22. (e) Required Compound Interest = ` 35000
12. (b) ? = 343 = 18.52 » 18 éæ 2 ù
8 ö é108 ´ 108 - 10000 ù
êç 1 + ÷ - 1ú = ` 35000 ê úû
æ 775200 ö ë è 100 ø û ë 10000
13. (c) Sen's monthly income = Rs ç ÷ = 64600
è 12 ø
Let the monthly income of Anita be Rs. x. 35000 ´ 1664
=` = `5824
10000
SBI Clerk Prelim Solved Paper-2016 2016-27
23. (a) Q Area of the square = 2 × 392 = 784 sq. cm 35. (d) Required total amount earned by selling both products
M and O by store C = Rs (57 × 5.6 + 48 × 50) × 103
\ Length of the side of the square = 784 = 24cm = ` (319.2 + 2400) × 103 = ` 27.192 lakhs
100 ´ 208 (36-40) : H
24. (e) Maximum marks = = Rs520
40 C E
25. (c) Let the lowest number be x.
Now according to the question, B A
x + (x+1) + (x+2) + (x+3) + (x+4) = 190
Þ 5x + 10 = 190 G F
D
190 - 10 180
Þx= = = 36 37. (c) Only two persons H and E are siting.
5 5
D A C F B
\ Required sum = 36 + 40 = 76 38. (d)
F EH BGD AEHC
26. (d) 1×7=7
39. (d) All other are sitting at consecutive positions.
7 × 7 = 49 41. (b) If all the symbols and numbers are dropped, the new
49 × 7 = 343 arrangement is P G R E F K U W H N I B Q Y M V D
343 × 7 = 2401 b 8 V
42. (c) Only two $ 4 F
27. (d) 13 + 22 + 3 = 20
20 + 42 + 3 = 39 43. (c) Required position is = (7 + 12)th
39 + 62 + 3 = 78 = 19th from the right end in the given arrangement.
Element at 19th from the right end = U
78 + 82 + 3 = 145
145 + 102 + 3 = 248 +2 +2 +2 +2
+3 +3 +3 +3 +3
28. (a) 12 × 2 + 11 = 35 Q @6
44. (b) 7 # G E F 4 1 W % 2 5 1
35 × 2 + 11 = 81
Tg:- @NextGenBankers
81 × 2 + 11 = 173 -1
–1
-1
–1
-1 -1
–1
-1
173 × 2 + 11 = 357
357 × 2 + 11 = 725 45. (d) E G R K $ F b @M
29. (e) 3 + 97 = 100 –3 –3 –3
100 + 197 = 297 –1 –1
297 + 297 = 594 N B 2 %KU
597 + 397 = 991 +3 –3
991 + 497 = 1488 (46–50) :
30. (c) 112 + 7 × 1 =119 A# B ®A < B
119 + 7 × 3 = 140 A© B ®A> B
140 + 7 × 5 = 175 A% B®A= B
175 + 7 × 7 = 224 A$ B®A>B
224 + 7 × 9 = 287
A@ B ®A < B
31. (e) Number of L type products sold by
46. (d) Z # F, R @ F, D© R
Store F = 48
Z < F, R < F, D>R
Store E = 40
I. Z < R
48 II. D > Z
Required percentage = ´ 100 = 120
40 Combining the given expressions,
32. (e) Required ratio = (61 + 54) : (54 + 48) = 115 : 102 D>R> F> Z
3
33. (b) Average price= ` (16 + 15 + 14.5 + 15.6 + 18.2 + 14.9 ) ´10 comparision is not possible
6 So, neither I nor II is true.
94.2 47. (b) R @ D, D © W, B@ W
=` ´ 103 = Rs15700
6 R £ D, D > W, B £W
34. (a) Required difference = Rs (60 × 75 – 44 × 15) × 103 I. W > R,
= ` (45000 – 660) × 103 = ` 38.4 lakhs II. B < D
GP_4458
2016-28 SBI Clerk Prelim Solved Paper-2016
M>R £ (56–60) :
Days Lectures
Thus only II follows. Monday Group Discussion
49. (e) H $ V, V % M, K©M Tuesday —
H ³ V, V = M, K>M Wednesday Quality Circles
I. K > V Thursday Leadership
II. M £ H
Combining all the given expressions, Friday Decision Making
Saturday Assessment Centre
H³ V =M < K
E555F
H³V<K
combining Tg:- @NextGenBankers Sunday Motivation
21. Which of the following is not emitted by radio active substance? 33. Which scientist discovered radioactive element radium?
(a) Electrons (a) Isaac Newton (b) Albert Einsten
(b) Electromagnetic radiations (c) Benyamen Franklen (d) Marie Curie
(c) Alpha particles (e) None of these
(d) Neutrons 34. ICICI is the name of a
(e) None of these (a) Chemical industry (b) Bureau
22. Ordinary table salt is Sodium Chloride. What is baking soda? (c) Corporation (d) Financial institution
(a) Potassium Chloride (b) Sodium Hydrocide (e) None of these
(c) Sodium Bicarbonate (d) Potassium Hydrocide 35. ‘World Chronic obstructive pulmonary Disease day for the
(e) None of these year 2015 was observed on which date?
23. Ozone hole refers to (a) 16 November (b) 17 November
(a) hole in ozone layer (c) 18 November (d) 19 November
(b) decrease in ozone layer in troposphere (e) None of these
(c) decrease in thickness of ozone layer in stratosphere 36. The famous Dilwara Temples are situated in
(d) increase in thickness of ozone layers in troposphere (a) Uttar Pradesh (b) Rajasthan
(e) None of these (c) Maharashtra (d) Madhya Pradesh
24. Pulses are a good source of (e) Karnatka
(a) carbohydrates (b) fats 37. The government has decided to increase the foreign direct
(c) proteins (d) vitamins investment (FDI) limit in private banks from the existing
(e) None of these 74% to
25. Oxygen in our blood in transported by a protein called (a) 90% (b) 100%
(a) haemoglobin (b) Keratin (c) 95% (d) 80%
(c) collagen (d) Myoglobin (e) None of these
(e) None of these 38. Who is called as the father of geometry?
26. The ratio of width to length of our National Flag is (a) Aristotle (b) Euclid
(a) 3 : 5 (b) 2 : 3 (c) Pythagoras (d) Kepler
(c) 2 : 4 (d) 3 : 4
(e) Newton
(e) 3 : 2
Tg:- @NextGenBankers
27. For which of the following disciplines in Nobel prize awarded?
(a) Physics and Chemistry
39. Who was known as from man of India?
(a) Govind Ballabh pant (b) Jawarlal Neharu
(c) Subhash Chandra Bose (d) Sardar Vallabhbai Patel
(b) Physiology or Medicine
(e) Mahatma Gandhi
(c) Literature, and Economics
40. Dr. M. S. Swaminathan is associated with
(d) Peace
(a) Agriculture (b) Medicine
(e) All of the above
28. Exposure to sunlight helps a person improve his health because (c) Astrophysics (d) Physics
(a) the infrared light kills bacteria in the body (e) None of these
(b) resistance power increases General English
(c) the pigment cells in the skin get stimulated and
produce a healthy tan DIRECTIONS (Qs. 41-50) : Read the following passage carefully
(d) the ultraviolet rays convert skin oil into vitamin D. and answer the questions given below. Certain words are printed in
(e) None of these bold to help you to locate them while answering some of the questions.
29. Fileria is caused by
(a) Bacteria (b) Mosquito Comfort is now one of the causes of its own spread. It has now
(c) Protozoa (d) Virus become a physical habit, a fashion, an ideal to be pursued for its
(e) None of these own sake. The more comfort is brought into the world, the more
30. Ecology deals with it is likely to be valued. To those who have known comfort,
(a) Birds discomfort is a real torture. The fashion which now decrees the
(b) Cell formation worship of comfort is quite as imperious as any other fashion.
(c) Relation between organism and their environment Moreover, enormous material interests are bound up with the
(d) Tissue supply of the means of comfort. The manufacturers of furniture,
(e) None of these of heating apparatus, of plumbing fixtures cannot afford to let
31. Headquarters of UNO are situated at the love of comfort die. In modern advertisements they have
(a) New York, USA (b) Hague (Netherlands) found a means for compelling it to live and grow. A man of
(c) Geneva (d) Paris means today, who builds a house, is in general concerned
(e) China primarily with the comfort of his future residence. He will spend
32. Fathometer is used to measure a great deal of money on bathrooms, heating apparatus , padded
(a) Earthquakes (b) Rainfall furnishings, and having spent he will regard his house as perfect.
(c) Ocean depth (d) Sound intensity His counter part in an earlier age would have been primarily
(e) Intensity of light
SBI Clerk Solved Paper 2015-3
concerned with the impressiveness and magnificence of his (c) It facilitates mental life.
dwelling with beauty, in a word, rather than comfort. The money (d) It encourages a blend of materialistic and spiritual
our contemporary would spend on baths and central heating thinking.
would have been spent on marble staircases, frescoes, pictures (e) None of these
and statues. I am inclined to think that our present passion for 48. Why would manufacturers of various devices not permit
comfort is a little exaggerated. Though I personally enjoy comfort, comfort to die?
I have lived most happily in houses devoid of everything that (a) They want to manufacture more and more comfort
Anglo-Saxons deem indispensable. Orientals and even South goods.
Europeans who know not comfort and live very much as our (b) Manufacturers are mainly interested in creating new
ancestors did centuries ago seem to go on very well without our things.
elaborate apparatus and padded luxuries. However, comfort for (c) Manufacturers' emphasis is on producing beautiful
me has a justification; it facilitates mental life. Discomfort things.
handicaps thought; it is difficult to use the mind when they (d) Their prosperity is closely linked with the people's
body is cold and aching. desire for comfort.
41. Choose the word that is SIMILAR in meaning to the phrase (e) None of these
devoid of as used in the passage. 49. Choose the word which is MOST OPPOSITE in meaning
(a) available (b) lacking of the word ‘indispensable’ as used in the passage.
(c) empty (d) false (a) unattractive (b) avoidable
(e) deficient (c) favourable (d) unelegant
42 How do people manage to keep the love of comfort a live? (e) comfortable
(a) By pumping in more comfort goods in the market 50. Which of the following statements is NOT TRUE in the
(b) By sacrificing high profit on comfort goods context of the passage?
(c) By targeting youths in the sales campaign (a) Discomfort is not liked by those who live in comfort.
(d) By appealing to the emotionality of people (b) The affluent man of an earlier age was interested more
(e) None of these in beauty than in comfort.
43. What is the author’s prediction about comfort? (c) Discomfort handicaps thought.
(a) The value of comfort will increase (d) Orientals and South Europeans love comfort immensely.
Tg:- @NextGenBankers
(b) People will value more spirituality thus reducing the
value of comfort.
(c) People will desire simple lifestyle
(e) The author of the passage enjoys comfort.
55. (a) forced (b) reluctant 69. The head ______ was annoyed to see a ______ in the
(c) bound (d) prepared soup.
(e) curious (a) chief, house fly (b) chef, housefly
56. (a) farming (b) traditional (c) chief, house-fly (d) chef, house fly
(c) improved (d) powerful (e) None of these
(e) old 70. When you want to digitalise a city _________ with millions,
57. (a) routine (b) monotonous you don't bet _________ the odds.
(c) excessive (d) wasteful (a) proceeding, into (b) teeming, against
(e) effective (c) undergoing, adhere (d) dangling, for
58. (a) accruing (b) helping (e) None of these
(c) enabling (d) belonging
DIRECTIONS (Qs. 71-75): Rearrange the following five
(e) referring
59. (a) eager (b) capable sentences into a meaningful paragraph and answer the questions
(c) indifferent (d) antagonistic given below:
(e) unwilling (A) However, with innovation coming into play unit-linked/
60. (a) theories (b) techniques market-linked products have also found a place in insurance
(c) desires (d) hours business after privatisation.
(e) policies (B) It is also worth mentioning here that world over unitlinked
products constitute quite a substantial chunk of the total
DIRECTIONS (Qs. 61 - 65) : Read each sentence to find out
portfolio of insurance companies.
whether there is any error in it. The error, if any, will be in one part (C) There was a time when only traditional insurance products
of the sentence. The number of that part is the answer, if there in no used to dominate the arena.
error, the answer is (e). (Ignore errors of punctuation, if any). (D) The emergence of these products of various insurance
61. You may not know it (a)/ but this engine is (b)/ claimed to have companies combines the characteristics of both endowment
twice (c)/ as powerful as the previous one. (d)/ No error (e) insurance policies and mutual funds.
62. Nothing ever becomes real (a)/ till it is experienced. (b)/ (E) The insurance industry in India is evolving and assuming
Even proverb is no proverb to you (c)/ till your life has different proportion since it was privatised.
76. Meena loved to shop and goes out with her friends
87. 17956 + 24025 = ?
whenever she got time.
(a) went outside along (b) went out with (a) 256 (b) 289
(c) goes for outing to (d) go outing and (c) 155 (d) 19
(e) None of these
(e) No corrections required
77. Manoj was many better at sports than Anurag. 1 2 1
88. 5 +6 +7 =?
(a) much good than (b) many good to 4 3 6
(c) much better at (d) much better than 11
(e) No corrections required (a) 19.5 (b) 19
12
78. It was very dark and Trisha was too scary to go home alone.
1
(a) too scaring (b) to scary top (c) 19 (d) 19
(c) to scare too (d) too scared to 12
(e) None of these
(e) No corrections required
89. 40% of 250 = 50% of ?
79. Many people not like being interrupted when they are busy (a) 200 (b) 100
working. (c) 150 (d) 400
(a) do not like being (b) do not like (e) None of these
(c) not liking when (d) no like being 90. ?% of 658 + 40% of 845 = 568.3
(e) No corrections required (a) 46 (b) 42
80. The crowd loved her performance and gave her a stand (c) 38 (d) 35
ovation as she left the stage. (e) None of these
(a) stand ovate (b) stood ovation
DIRECTIONS (Qs. 91-95) : What should come in place of the
(c) stand the ovation (d) standing ovation
question mark (?) in the following number series?
(e) No correction required
91. 9 19 40 83 ? 345 696
Quantitative Aptitude (a) 162 (b) 170
Tg:- @NextGenBankers
DIRECTIONS (Qs. 81-90): What should come in place of question
mark (?) in the following questions. 92.
(c) 175
(e) None of these
980 484 236 112 50 ?
(d) 166
(a) 25 (b) 17
81. 3251 + 587 + 369 – ? = 3007
(c) 21 (d) 29
(a) 1250 (b) 1300 (e) None of these
(c) 1376 (d) 1200 93. 8 9 20 63 256 1285 ?
(e) None of these (a) 6430 (b) 7450
82. (800 ÷ 64) × (1296 ÷ 36) = ? (c) 7716 (d) 7746
(a) 420 (b) 460 (e) None of these
(c) 500 (d) 540 94. 1015 508 255 129 66.5 ? 20.875
(a) 34.50 (b) 35
(e) None of these
(c) 35.30 (d) 35.75
83. 107 × 107 + 93 × 93 = ? (e) None of these
(a) 19578 (b) 19418 95. 12 12 18 36 90 270 ?
(c) 20098 (d) 21908 (a) 945 (b) 810
(e) None of these (c) 1080 (d) 1215
84. 34.667 – 15.597 – 8.491 – 0.548 = ? (e) None of these
(a) 14.403 (b) 10.031 DIRECTIONS (Qs. 96-100) : Study the following table carefully
(c) 18.301 (d) 21.043 and answer the questions given below it.
(e) None of these
85. [(140)2 ÷ 70 × 16] ÷ 8 = 14 × ? Shows(Number in hundreds)
(a) 38 (b) 22 Stand-up
Cities Dance Music Drama Mimicry
(c) 55 (d) 40 Comedy
(e) None of these M 15 21 24 0.8 0.9
86. 456 + 24 × 0.75 – 12 = ? N 12.4 13 26 2 0.5
(a) 462 (b) 672 O 5.7 8 12 0.3 0.2
(c) 348 (d) 624 P 11.3 6 18 1 1.5
(e) None of these Q 17 12.4 11 3 0.4
R 14 10.5 9.8 0.7 0.1
GP_4458
2015-6 SBI Clerk Solved Paper
96. The mimicry shows held in city M are what percent of the 106. A person sold an article from ` 3600 and got a profit of
drama shows held in city O ? 20%. Had he sold the article for ` 3150, how much profit
(a) 7 (b) 8.5 would he have got?
(c) 6.5 (d) 8 (a) 4% (b) 5%
(e) None of these (c) 6% (d) 10%
97. What is the average number of entertainment shows held (e) None of these
in city P ? 107. A water pipe is cut into two pieces. The longer piece is
(a) 756 (b) 678 70% of the length of the pipe. By how much percentage is
(c) 786 (d) 698 the longer piece longer than the shorter peice?
(e) None of these 400
98. If the number of music shows in city N and Q is increased (a) 140% (b) %
by 5%, what will be the total number of music shows in 3
both the cities together ? (c) 40% (d) None of these
(a) 2,602 (b) 2,667 (e) None of these
(c) 2,540 (d) 2,667 108. Age of X is six times that of Y. After 4 years, X is four times
(e) None of these. elder to Y. What is the present age of Y ?
99. What is the ratio of the number of dance shows held in (a) 4 years (b) 5 years
city N to the number of drama shows held in city R? (c) 6 years (d) 7 years
(a) 49: 62 (b) 49 : 51 (e) None of these
(c) 62 : 45 (d) 62 : 49 109. In a certain school, the ratio of boys to girls is 7 : 5. If there
(e) None of these are 2400 students in the school, then how many girls are
100. What is the total number of stand–up comedy shows held there ?
in all the cites together ? (a) 500 (b) 700
(a) 820 (b) 740 (c) 800 (d) 1000
(c) 780 (d) 810 (e) None of these
(e) None of these. 110. A person invested part of ` 45000 at 4% and the rest at 6%.
101. Which one of the following is a prime number ? If his annual income from both are equal, then what is the
average rate of interest?
(a) 161
(c) 173
(e) None of these
Tg:- @NextGenBankers
(b) 171
(d) 221 (a) 4.6%
(c) 5.0%
(b) 4.8%
(d) 5.2%
(e) None of these
2 111. In an examination, 40% of the candidates wrote their answers
102. When a ball bounces, it rises to of the height from
3 in Hindi and the others in English. The average marks of the
which it fell. If the ball is dropped from a height of 36 m, candidates written in Hindi is 74 and the average marks of the
how high will it rise at the third bounce? candidates written in English is 77. What is the average marks
1 2 of all the candidates ?
(a) 10 m (b) 10 m (a) 75.5 (b) 75.8
3 3 (c) 76.0 (d) 76.8
1 2 (e) None of these
(c) 12 m (d) 12 m
3 3 112. If the rate of interest is 10% per annum and is compound
(e) None of these half-yearly, then the principle of ` 400 in 3/2 years will
103. LCM of two numbers is 16 times their HCF. The sum of amount to
LCM and HCF is 850. If one number is 50, then what is the (a) ` 463.00 (b) ` 463.05
other number ? (c) ` 463.15 (d) ` 463.20
(a) 800 (b) 1200 (e) None of these
(c) 1600 (d) 2400 113. The simple interest on a certain sum of money for 3 years
(e) None of these at 8% per annum is half the compound interest on ` 4000
104. The sum of two numbers is 232 and their HCF is 29. What for 2 years at 10% per annum. What is the sum placed on
is the number of such pairs of numbers satisfying the above simple interest?
condition? (a) ` 1550 (b) ` 1650
(a) One (b) Two
(c) ` 1750 (d) ` 2000
(c) Four (d) Five
(e) None of these
(e) None of these
105. A man losses 20% of his money. After spending 25% of 114. A man buys 4 tables and 5 chairs for ` 1000. If he sells the
the remaining, he has `480 left. What is the amount of tables at 10% profit and chairs 20% profit, he earns a profit
money he originally had? of ` 120. What is the cost of one table?
(a) ` 600 (b) ` 720 (a) ` 200 (b) ` 220
(c) ` 800 (d) ` 840 (c) ` 240 (d) ` 260
(e) None of these (e) None of these
SBI Clerk Solved Paper 2015-7
115. A refrigerator and a camera were sold for ` 12000 each. The (a) One (b) Two
refrigerator was sold at a loss of 20% of the cost and the (c) Three (d) More than three
camera at a gain of 20% of the cost. The entire transaction (e) None of these
results in which one of the following? 124. Arrange the words given below in a meaning sequence.
(a) No loss or gain (b) Loss of ` 1000 1. Key 2. Door
(c) Gain of ` 1000 (d) Loss of ` 2000 3. Lock 4. Room
(e) None of these 5. Switch on
116. Two cars A and B start simultaneously from a certain place (a) 5, 1, 2, 4, 3 (b) 4, 2, 1, 5, 3
at the speed of 30 km/h and 45 km/hr, respectively. The car (c) 1, 3, 2, 4, 5 (d) 1, 2, 3, 5, 4
B reaches the distination 2 h earlier than A. What is the (e) 5, 4, 3, 2, 1
distance between the starting point and destination? 125. If it is possible to form a word with the first, fourth, seventh
(a) 90 km (b) 180 km and eleventh letters of the word ‘SUPERFLUOUS’, write
(c) 270 km (d) 360 km the first letter of that word. Otherwise, X is the answer.
(e) None of these (a) S (b) L
117. A train running at the speed of 72 km/h goes past a pole in (c) O (d) X
15 s. What is the length of the train ? (e) None of these
(a) 150 m (b) 200 m DIRECTIONS (Qs. 126-130) : Study the following arrangement
(c) 300 m (d) 350 m
carefully and answer the questions given below.
(e) None of these
118. A person walks a distance in 114 days, when he rests 9h a RD15E%K3J19P8#A2B$KM6W@N4©T*7FH
day. How long will he take to walk twice the distance, if he 126. Which of the following is the third to the right of the twelfth
walks twice as fast and rests twice as long each day as from the left end?
before? (a) J (b) A
(a) 57 days (b) 228 days (c) B (d) @
(c) 285 days (d) 324 days (e) None of these
(e) None of these 127. Which of the following is the sixth to the right of the
119. The sides of a triangular field are 41 m, 40 m and 9 m. The eighteenth from the right end?
number of rose beds that can be prepared in the field if
is
Tg:- @NextGenBankers
each rose bed, on an average, needs 900 square cm space,
(a) P
(c) W
(e) None of these
(b) 3
(d) M
(a) 2000 (b) 1800 128. How many such numbers are there in the above
(c) 900 (d) 800 arrangement, each of which is immediately preceded by a
(e) None of these consonant and immediately followed by a symbol?
120. The ratio of the outer and inner perimeters of a circular (a) One (b) Two
path is 23 : 22 If the path is 5 m wide, the diameter of the (c) Three (d) Four
inner circle is (e) None of these
(a) 55 m (b) 110m 129. How many such consonants are there in the above
(c) 220 m (d) 230 m arrangement, each of which is immediately preceded by a
(e) None of these symbol and immediately followed by a number?
Reasoning Ability (a) One
(c) Three
(b) Two
(d) Four
121. If each of the digits in the number 92581473 are arranged in (e) None of these
ascending order, what will be the difference between the 130. Which of the following is exactly in the middle between the
digits which are fourth from the right and third from the left tenth from the left end and the eight from the right end?
in the new arrangement? (a) $ (b) #
(a) One (b) Two (c) B (d) 7
(c) Three (d) Four (e) None of these
(e) None
122. If ‘R’ denotes ‘¸’ ‘P’ denotes ‘X’, ‘W’ denotes ‘+’ and ‘V’ DIRECTIONS (Qs. 131 – 135) : In each of the questions below
denotes ‘–’ then 14W16R4V3P5=? are given four statements followed by four conclusions numbered
(a) 15 (b) 4 I, II, III and IV. You have to take the given statements to be true
(c) 6 (d) 3 even if they seem to be at variance from commonly known facts.
(e) None of these Read all the conclusions and then decide which of the given
123. How many such pair of letters are there in the word conclusions logically follows from the given statements
WONDERS, each of which has as many letters between disregarding commonly known facts.
them in the word (in both forward and backward direction) 131. Statements :
as they have between them in the English alphabetical All silver are metals.
order? All metals are steel.
GP_4458
2015-8 SBI Clerk Solved Paper
142. Who sits exactly between Q and R? (d) Finance, Computers, Marketing
(a) T (b) P (e) None of these
(c) K (d) M 150. If Finance lecturer is related to English lecture and
(e) S and K Mathematics lecturer is related to Marketing lecturer in a
143. Which of the following pairs represents the persons seated certain way based upon the given arrangement, then
in middle of the sides who face each other? Accounts lecturer will be related to which of the following,
(a) S, Q (b) K, L following the same pattern?
(c) M, P (d) R, T (a) English (b) Physics
(e) T, Q (c) Computers (d) Marketing
144. Who among the following sit between R and K when (e) None of these
counted in anti-clockwise direction from K?
DIRECTIONS (Qs. 151-155): Study the following information
(a) No one sits between R and K as R and K are immediate
neighbours of each other to answer the given questions:
(b) S, P and L In a certain code, 'she sat on chair' is written as 'ik ma ja de', 'he
(c) P and Q sat on table' is written as 'de da ik phi', 'chair is on cart' is written
(d) L and R as 'ik pa ma ki', and 'he is looking smart' is written as 'jo va ki phi'.
(e) M, S and T 151. What does 'de' mean in the given code language?
145. If K is made to face the opposite direction, who would sit to (a) on (b) cart
his immediate right? (c) sat (d) chair
(a) R (b) Q (e) Can't be determined
(c) P (d) T 152. What does 'da ik ja' stand for?
(e) S (a) cart chair table (b) she on chair
DIRECTIONS (Qs. 146 - 150) : Study the information given (c) is he table (d) she on table
below and answer the given questions: (e) None of these
153. Which is the code for 'chair'?
7 people - A, B, C, D, E, F and G stay on a 7 floors building (No (a) ma (b) ik
body stays on ground floor which is named 0 floor). They are (c) pa (d) ja
professors of different subjects - Finance, English, Computer,
Tg:- @NextGenBankers
Physics, Marketing, Accounts and Mathematics (but not
necessarily in the same order).
(e) ki
154. 'she cart smart' is written as what in the code language?
(a) va pa ja (b) pa je ik
The professor of Finance stays on either 2nd or 5th floor. (c) ja jo pa (d) Either A or C
There are three floors between the persons who teach English (e) Cannot be determined
and Finance. And also there are 2 floors between the persons 155. Which of the following may represent 'table was on cart'?
who teach English and Computers. Physics Lecturer stays (a) ik ki jo va (b) de phi da pa
immediately below the Marketing Lecturer. Accounts Lecturer (c) da pa je ik (d) ik pa su da
stays below Mathematics Lecturer (not necessarily immediately (e) ik da ma va
below Mathematics Lecturer). Accounts lecturer does not stay
on 1st or 2nd floor. DIRECTION (Qs. 156-160): Relationship between different
146. Which of the following subject's professor stays on 1st elements is shown in the statements. Find if the conclusions also
floor? follow or not.
(a) Finance (b) Marketing 156. Statements : P = Q ³ R = S ³ T; V £ W = T; R > X
(c) Physics (d) English Conclusions:
(e) Data inadequate I. P > X
147. How many floors are between Finance Lecturer and English II. Q ³ W
Lecturer?
(a) only I follows (b) only II follows
(a) None (b) One
(c) either I or II follows (d) neither I nor II follow
(c) Two (d) Three
(e) More than three (e) both I and II follow
148 . Which of the following combination of floor-lecturer is not 157. Statements: P = Q ³ R = S > T; V £ X £ T
correct? Conclusions:
(a) English-1 (b) Marketing-3 I. P > X
(c) Mathematics-7 (d) Computers-4 II. P ³ P
(e) Physics-6 (a) only I follows (b) only II follows
149. Which of the following subject's lecturers stay between the (c) either I or II follows (d) neither I nor II follow
lecturers of Accounts and Physics? (e) both I and II follow
(a) Finance, Marketing 158. Statements: P = T ³ A < D > W; W £ X > L; P < K = N
(b) Mathematics, Computers Conclusions: I. D > L
(c) Computers II. A £ N
GP_4458
2015-10 SBI Clerk Solved Paper
(a) only I follows (b) only II follows (a) Internet commerce (b) Web commerce
(c) either I or II follows (d) neither I nor II follow (c) Computer commerce (d) Electronic commerce
(e) both I and II follow (e) None of these
159. Statements: A > L ³ J = I; K < L £ H; G = H £ B Conclusions: 168. In terms of special product life cycles, a _______ is a basic
I. J £ B II. J > B and distinctive mode of expression.
(a) only I follows (a) Genre (b) Style
(b) only II follows (c) Fashion (d) Fad
(c) either I or II follows (e) None of these
(d) Error! Not a valid embedded object. neither I nor II 169. The process of evaluating each market segment’s
follow attractiveness and selecting one or more segments to enter
(e) both I and II follow is called_______.
160. Statements: T > P £ B = H; J £ P > M; L < G = J Conclusions: (a) Mass Marketing (b) Market segmentation
I. T < J II. G £ H (c) Market targeting (d) Market positioning
(a) only I follows (b) only II follows (e) None of these
(c) either I or II follows (d) neither I nor II follow 170. Which of the following factors are the most popular bases
(e) both I and II follow for segmenting customer groups.
(a) Geographic (b) Demographic
Marketing Aptitude/ (c) Psychographic (d) Behavcoral
Computer Knowledge 171.
(e) None of these
Which strategy calls for using the sales force and trade
161. Which of the following is a part of Marketing Management? promotion to move the product through channels.
(a) Identification of Business opportunities (a) Push strategy (b) Pull strategy
(b) Understanding the customer needs (c) Blocking strategy (d) Integrated strategy
(c) Producing according to customer needs (e) None of these
(d) Delivering as per customer convience 172. What is ‘Globalization’?
(e) All of the above (a) Opening the economy for word trade
162. Consumer buys_______. (b) Export and import trade
(a) Goods
(c) Both (a) and (b)
Tg:- @NextGenBankers
(b) Services
(d) Neither (a) nor (b)
(c) Foreign tour
(d) Marketing foreign products
(e) None of these (e) None of these
163. Which of the following is not correct with regard to 173. Web marketing involves_______
marketing? (a) Selling web cameras
(a) Marketing is management function (b) Web advertisements
(b) Marketing is a philosphy (c) E-mail chatting
(c) Marketing is not related to business activity alone (d) Browsing the web
(d) Marketing means selling. (e) Door-to-door canvasing
(e) Objective of marketing is to achieve customer 174. Marketing share means_______
satisfaction. (a) Paid up capital
164. Which of the following describes changes in individuals (b) Sharesheld by employees
behaviour arising from experience?
(c) Share of business volume as compared to other companies
(a) Modeling (b) Motivation
(d) Share price of the company quoted in the market
(c) Perception (d) Learning
(e) Sensex
(e) None of these
175. “USP” in marketing means
165. If a company wants to reach masses of buyers that were
(a) Unique selling practices
geographically dispersed at a low cost per exposure, which
promotional form company would choose? (b) Uniform selling practices
(a) Advertising (b) Personal selling (c) United sales persons
(c) Public relations (d) Sales promotion (d) Unique selling proposition
(e) None of these (e) Useful sales person
166. Anything can be offered to a market for attention, 176. Online marketing is useful for marketing of
acquisition, use or consumption that might satisfy a want (a) Saving accounts (b) Credit card
or need is called a(n). (c) Home loans (d) NRI deposits
(a) Idea (b) Demand (e) Business accounts
(c) Product (d) Service 177. Digital marketing is similar to ________
(e) None of these (a) Online marketing (b) Cold calling
167. _________ is the general term for buying and selling (c) Web designing (d) Market for cost
process that is supported by electronic means. (e) Outdoor marketing
SBI Clerk Solved Paper 2015-11
178. Target group means 189. What is the process of copying software programs from
(a) All purchasers (b) All sales persons secondary storage media to the hard disk is called
(c) Targeted purchasers (d) All consumers (a) Configuration (b) download
(e) Delivery persons (c) storage (d) upload
179. Referral means_______- (e) installation
(a) Sales person 190. When the pointer is positioned on a ________, it is shaped
(b) All customers like a hand.
(b) Lead provided by operation staff (a) grammar error (b) formatting error
(c) Calling the existing purchasers (c) screen tip (d) hyperlink
(e) All purchasers (e) spelling error
180. Market segmentation means 191. When you save your data to_______, it will remain intact
(a) Segmentation of sales team even when the computer is turned off.
(b) Allocation of teritory (a) RAM (b) Mother board
(c) Sales arrangement
(c) Secondary storage (d) Primary storage device
(d) Segmentation of target group according to their needs
(e) None of these
(e) All hospitals
192. The_________ allows you to access object and start
181. A person who uses his expertise to gain access to other
programme.
computers to get information illegally or do damages is
(a) Hacker (b) Analyst (a) Default menu (b) XP menu
(c) Instant messenger (d) Programmer (c) Start menu (d) Stop menu
(e) Spammer (e) None of these
182. A device that connects to a network without the use of 193. Which of the following is an operating system?
cables is said to be_______ (a) MS Windows (b) Mac
(a) distributed (b) free (c) MS DOS (d) Windows NT
(c) centralized (d) open source (e) All of the above
(e) None of these 194. What is correcting errors in a program called?
183. The most common type of storage devices are (a) Interpreting (b) Translating
(a) Persistence
(c) Magnetic
(e) None of these
Tg:- @NextGenBankers
(b) Optical
(d) flash
(c) Debugging
(e) None of these
(d) Compiling
ANSW ER KEY
1 (a) 21 (d) 41 (b) 61 (a) 81 (d) 101 (c) 121 (b) 141 (d) 161 (e) 181 (a)
2 (c) 22 (c) 42 (a) 62 (e) 82 (e) 102 (b) 122 (d) 142 (b) 162 (e) 182 (b)
3 (b) 23 (c) 43 (a) 63 (d) 83 (c) 103 (a) 123 (d) 143 (e) 163 (d) 183 (c)
4 (d) 24 (c) 44 (e) 64 (c) 84 (b) 104 (b) 124 (c) 144 (c) 164 (d) 184 (a)
5 (a) 25 (a) 45 (b) 65 (d) 85 (d) 105 (c) 125 (b) 145 (b) 165 (a) 185 (b)
6 (a) 26 (b) 46 (a) 66 (d) 86 (a) 106 (b) 126 (b) 146 (d) 166 (c) 186 (a)
7 (d) 27 (e) 47 (c) 67 (d) 87 (e) 107 (b) 127 (d) 147 (e) 167 (d) 187 (a)
8 (c) 28 (d) 48 (d) 68 (c) 88 (c) 108 (c) 128 (b) 148 (d) 168 (b) 188 (c)
9 (b) 29 (b) 49 (b) 69 (b) 89 (a) 109 (d) 129 (b) 149 (b) 169 (c) 189 (e)
10 (d) 30 (c) 50 (d) 70 (a) 90 (d) 110 (b) 130 (c) 150 (c) 170 (b) 190 (d)
11 (d) 31 (a) 51 (a) 71 (d) 91 (b) 111 (b) 131 (d) 151 (b) 171 (a) 191 (c)
12 (a) 32 (c) 52 (c) 72 (c) 92 (e) 112 (b) 132 (c) 152 (a) 172 (a) 192 (a)
13 (b) 33 (d) 53 (e) 73 (b) 93 (c) 113 (c) 133 (b) 153 (a) 173 (b) 193 (e)
14 (b) 34 (d) 54 (d) 74 (e) 94 (d) 114 (a) 134 (c) 154 (d) 174 (c) 194 (c)
15 (b) 35 (c) 55 (b) 75 (a) 95 (a) 115 (b) 135 (d) 155 (e) 175 (d) 195 (d)
16 (d) 36 (b) 56 (c) 76 (b) 96 (e) 116 (b) 136 (a) 156 (c) 176 (d) 196 (a)
17 (a) 37 (b) 57 (e) 77 (c) 97 (a) 117 (c) 137 (b) 157 (d) 177 (a) 197 (c)
18
19
(a)
(b)
38
39
(b)
(d)
Tg:- @NextGenBankers
58
59
(d)
(a)
78
79
(d)
(a)
98
99
(d)
(d)
118
119
(c)
(a)
138
139
(d)
(d)
158
159
(a)
(b)
178
179
(c)
(c)
198
199
(a)
(d)
20 (a) 40 (a) 60 (b) 80 (d) 100 (c) 120 (c) 140 (b) 160 (a) 180 (d) 200 (b)
89. (a) 40% of 250 = 50% of ? 104. (b) Let two numbers by 29x and 29y.
1 \ 29x + 29y = 232 Þ x + y = 8
Þ 100 = ´ ? Þ 100 × 2 = 200 Þ (x, y) = (1, 7), (3, 5)
2
Since, one such pair is 87 and 145.
568.3 - 40% ´ 845 568.3 - 338 Hence, the other pairs is 203 and 29.
90. (d) ? = =
6.58 6.58 105. (c) Let man has originally ` x
230.3 x ´ 80 8 x
= = 35 After 20% loss = =
6.58 100 10
91. (b) The given series is
8 x 75 8 x 3
× 2 + 1, × 2 + 3, × 2 + 4, .... After spending 25% = ´ = ´
10 100 10 4
92. (e) The given series is ÷ 2 – 6
According to the question,
Reqd no. = 50 ÷ 2 – 6 = 25 – 6 = 19
8x 3
93. (c) The given series is ´ = 480
× 1 + 1, × 2 + 2, × 3 + 3, × 4 + 4, × 5 + 5, × 6 + 6, 10 4
94. (d) (1015 + 1) ÷ 2 = 508 Þ 8x ´ 3 = 480 ´ 4 ´ 10
(508 + 2) ÷ 2 = 255 (255 + 3) ÷ 2 = 129 480 ´ 4 ´ 10
\ x= = 800
(129 + 4) ÷ 2 = 66.5 (66.5 + 5) ÷ 2 = 35 .75 8´3
(35.75 + 6) ÷ 2 = 20.875 106. (b) Let the cost price of the article be ` x
95. (a) The given series is 120 x
× 1, × 1.5, × 2, × 2.5, .... After 20% profit Þ = 3600
100
96. (e) Required percentage
x = 3000
0.9
= ´100 = 7.5% Now, profit percentage, when the article is sold for `3150
12
37.8 ´ 100 3150 - 3000 150
97. (a) Required average = = 756 = x100 - x100 = 5%
5
Tg:- @NextGenBankers
98. (d) Required number of music shows after the increase
= 25.4 ×
105
= 26.67 × 100 = 2667
3000 3000
\ Length of train = Speed of train × Time taken to 127. (d) Sixth to the right of the eighteenth from the right end
cross the stationary object mens twelfth from right end which is M.
128. (b) 2 Such numbers which is preceded by a consonant
72 ´ 1000 ´ 15
= = 300 m and immediately followed by a symbol
3600 P8$ and N4©.
118. (c) Distance Speed Hour/Day Speed 129. (b) 2 Such consonants each of which is immediately
1 1 15 114 preceded by a symbol and immediately followed by a
2¯ 2 6 x ¯
number are : %K3, @N4.
1: 2 ü 130. (c) Tenth from left is I and 8th from right is N. So B between
ï I and N.
2 :1 ý114 : x
6 :15ïþ
Steels
131. (a) Stands
\1 ´ 2 ´ 6 ´ x = 2 ´ 1 ´ 15 ´ 114 Metal
2 ´ 15 ´114 Stones
Þ x= Silver
2´6
= 285days
119. (a) Area of triangular field
= s ( s – a)( s – b)( s – c )
Chairs
Rhythms
Pillows
r2 23
120. (c) =
r1 22
r2 – r1 23 – 22
Þ =
r1 22
From this Venn Diagram, it is clear
r2 – r1 1 I. Some tables are chairs
Þ = IV. All chairs are songs
r1 22
5 1 133. (e) Papers
Þ =
r1 22
Mobiles Pens Plates
r1 = 110 m
Diameter of inner circle = 110 × 2 m = 220 m covers
121. (b) Number after writing digits in ascending order is 1 2 3
4 5 7 8 9. So difference between the digits which are
fourth from right and third from left = 5 – 3 = 2 From this Venn Diagram, it is clear none of the conclusion
122. (d) 14 W 16R4V3P5 follows.
= 14 + 16 ¸ 4 – 3 × 5 = 14 + 4 – 15 = 3
123. (d) DE, NO, RW, RS (4 pair of letters) have as many letters
between them as in English alphabets. 134. (b)
Lanes Rows
124. (c) The correct order is. Tables
Key Lock Door Room Switch on
1 3 2 4 5 Shoes
125. (b) The word from I, IV, VII, XI letters (S, E, L, S) is LESS. Caps
126. (b) Third to right of twelfth means 15th from left and which
is A.
GP_4458
2015-16 SBI Clerk Solved Paper
M S K
(out) (in) (out)
141. (d) 142. (b) 143. (e) 144. (c) 145. (b)
SBI Clerk Solved Paper-2014
Held on August 2014
(Based on Memory)
Time : 135 minutes Max. Marks : 200
General Awareness 11. Which bank has tied with New India Assurance to launch health
insurance products for women account holders of the bank.
1. Which is a tactical surface-to-surface short-range ballistic (a) State bank of Inadia
missile (SRBM) developed by DRDO of India under the (b) ICICI bank
Integrated Guided Missile Development Program ? (c) Punjab National Bank
(a) Prithvi (b) Agni
(d) Bhartiya Mahila Bank
(c) Prahar (d) Dhanush
(e) None of these (e) None of these
2. The Commonwealth Heads of Government Meeting 2015, 12. Recently which car plant has been shut down by Hindustan
also known as CHOGM 2015t will be held in ______: Motors Ltd, the maker of the Ambassador car ?
(a) Mauritius (b) Malta (a) Uttarpara plant near (Kolkata)
(c) Malaysia (d) Vanuatu (b) The Tiruvallur plant near Chennai
(e) None of these (c) Port Okha plant near Gujarat
3. Which place hosted the two-day 24th ASEAN summit to (d) Pithampur (Madhya Pardesh)
discuss the future development of the group ? (e) None of these
(a) New Delhi (b) Indonesia 13. Recently in which state Solar Thermal Power Plant has been
(c) Brunei (d) Bangkok installed ?
(e) Nay Pyi Taw
(a) Madhya Pardesh (b) Gujarat
4. Azlan Shah who died recently is a -
(a) Hockey Player
(c) Cricket player
Tg:- @NextGenBankers
(b) Football Player
(d) Polo player
(c) Rajsthan
(e) Maharashtra
(d) Odisha
(e) None of these 14. Why the government has banned on imports of milk and its
5. Which city is best known for its oil refinery ? products from China for one more year till June 2015?
(a) Baiji (b) Mumbai (a) Due to presence of melamine, used for making plastics
(c) Perth (d) Chad and fertiliser
(e) None of these (b) Due to increase in procurement costs.
6. Who is the Chairman of Bharatiya Mahila Bank ? (c) Due to wrong policies
(a) Aisha De Sequeira (b) Arundhati Bhattacharya (d) Due to lack of products
(c) Chanda Kochhar (d) Archana Hingorani (e) None of these
(e) Usha Ananantha Subramanium 15. Recently Navaratna status has been given to which
7. Which Article of the Constitution of India, is the annual organization?
budget of the Republic of India ? (a) National Buildings Construction Corporation Limited
(a) Article 12 (b) Article112 (NBCC) and Engineers India Limited (EIL)
(c) Article 21 (d) Article 11 (b) Airport Authority of India
(e) None of these (c) Balmer Lawrie & Co. Limited & Bharat Coking Coal Limited
8. Which state has the highest rural population according to (d) Airport Authority of India & Bharat Coal limited
census 2011? (e) None of these
(a) Bihar (b) Gujarat 16. Who has been chosen for the prestigious 49thJnanpith
(c) Uttar Pradesh (d) Andhra Pradesh award for 2013?
(e) Himachal Pradesh (a) Kedarnath Singh
9. Which state has the highest urban population according to (b) Chandrashekhara Kambara
census 2011? (c) Pratibha Ray
(a) Bihar (b) Gujarat (d) Ravuri Bharadvaja
(c) Uttar Pradesh (d) Maharashtra (e) None of these
(e) Himachal Pradesh 17. Who is the Minister of Consumer Affairs, Food and Public
10. Indian Oil Corporation is going to set up an oil refinery at Distribution ?
(a) Paradip (b) Jatni (a) Narendra Modi (b) Sushma Swaraj
(c) Ranchi (d) Raipur (c) Rajnath Singh (d) Arun Jailtey
(e) Korbi (e) Ram Vilas Paswan
GP_4458
2014-2 SBI Clerk Solved Paper-2014
18. Who is the Minister for Road Transport and Highways ? 28. Which of the following Prizes/Awards is given for excellence
(a) Nitin Gadkari (b) Narendra Modi in the field of Sports ?
(c) Sushma Swaraj (d) Rajnath Singh (a) Pulitzer prize
(e) Arun Jailtey (b) Shanti Swarup Bhatnagar Award
19. Who is the current SBI chairman ? (c) Arjuna Award
(a) Aisha De Sequeira (d) Shram Vir Purushkar
(b) Arundhati Bhattacharya (e) None of these
(c) Chanda Kochhar 29. Manavjit Singh Sandhu is a well known________.
(d) Archana Hingorani (a) Film Actor (b) Journalist
(e) Usha Ananantha Subramanium (c) Social Worker (d) Sportsman
20. The Head of the Reserve Bank of India is officially known as (e) Author
(a) President of RBI 30. Which of the following is not a public sector company ?
(b) Chief Executive of RBI (a) HPCL (b) BPCL
(c) Managing Director (MD) of RBI (c) BHEL (d) Gillette
(d) Executive Director of RBI (e) All of these
(e) Governor of RBI 31. A customer drawing a cheque on a bank has a right to
21. An engineer working in a big city in India wishes to send (a) Take back the cheque from the bank after it is paid
some money to his/her parents in a small village. How can a (b) Take back the cheque from the bank after it is paid with
bank help him/her ? (Both of them have accounts in the the permission of RBI
branches of the same bank.) (c) Stop payment of the cheque before it is paid
A. By issuing a Demand Draft. (d) Stop payment of the cheque after it is paid
B. By opening a letter of credit in the name of his/her (e) Stop payment of the cheque before it is issued
parents.
32. Which of the following States is among the North East States
C. Through E-transfer of money
of India ?
(a) Only A (b) Only B
(a) Jammu & Kashmir (b) Punjab
(c) Only C (d) Both A and B
(c) Himachal Pradesh (d) Meghalaya
(e) Both A and C
(e) Rajasthan .
22.
Tg:- @NextGenBankers
Coins of which of the following denominations are easily
available in India and are used by all of us in our day-to-day
shopping ?
33. Which of the following is a mineral?
(a) Isabgol (b) Camphor
A. ` 5 B. ` 2 C. ` 50 (c) Tobacco (d) Nickel
(a) Only A (b) Only B (e) Jute
(c) Only C (d) Both A and B 34. The Banking Ombudsman
(e) All A, B and C (a) is in charge of bank loans for buses
23. The two main seasons of cropping in India are known as (b) fixes the rates of interest for loans
______. (c) resolves complaints of customers
(a) Hot-Cold (b) Winter-Spring (d) issues licences for new bank branches
(c) Summer-Winter (d) Rainy-Cold (e) is the head of all nationalised banks
(e) Kharif-Rabi 35. What does the letter ‘L’ stands for in the term LAF commonly
24. International Day of Non-violence is observed on the birth used in financial/economic news ?
day of ________. (a) Liquidity (b) Least
(a) Jawahar Lal Nehru (b) Indira Gandhi (c) liabilities (d) Long
(c) Rajiv Gandhi (d) Sonia Gandhi (e) Liquid
(e) Mahatma Gandhi
36. What is the maximum period for which domestic term deposits
25. The National Stock Exchange is located in ______.
are normally accepted by banks in our country ?
(a) New Delhi (b) Mumbai
(a) 3 years (b) 5 years
(c) Kolkata (d) Chennai
(c) 7 years (d) 10 years
(e) Bangalore
26. Which of the following countries does not play international (e) 12 years
cricket ? 37. Which of the following is the name of the currency of a
(a) Russia (b) England country ?
(c) South Africa (d) Pakistan (a) Mexico (b) Peru
(e) India (c) Syria (d) Zambia
27. Which of the following is the autobiography of the film (e) Lira
actor Dev Anand ? 38. Which of the following is the name of a Scientific Instrument ?
(a) My Story (b) Sunny Days (a) Plasma (b) Pascal
(c) Romancing With Life (d) Wings of Fire (c) Radiation (d) Lactometer
(e) None of these (e) Adrenal Cortex
SBI Clerk Solved Paper-2014 2014-3
39. Many times we read a term DoT in various newspapers. In “(52) me will he come, or will he not ?” – asked the queen.
connection of 2G Spectrum. What is its full form ? “I do not know whether he put his right foot on the stirrup, or he
(a) Department of Transport put his left foot on the ground after I left”. Everybody (53) for the
(b) Department of Training king. He came the next day and said to the queen. “The wise
(c) Directorate of Technology Mamad, who never lies, (54) to you yesterday.” But the queen
(d) Director of Tele services told him Mamad’s exact words. And the king (55), that the wise
(e) Department of Telecom man never lies, and says only that, which he see’s with his own eyes.
40. Which of the following terms is used in the game of Chess ? 46. (a) name (b) sound
(a) Checkmate (b) Hoops (c) call (d) identity
(c) Stroke (d) Heave (e) label
47. (a) demanded (b) send
(e) Deuce
(c) ordered (d) request
General English (e) sanctioned
48. (a) Several (b) Most
DIRECTIONS (Qs. 41-45) : In each question below a sentence (c) Lots (d) Glorious
(e) Long
with four words printed in bold type is given. These are numbered
49. (a) just (b) tired
as (a), (b), (c) and (d). One of these four words printed in bold
(c) schedule (d) planned
may be either wrongly spelt or inappropriate in the context of (e) about
the sentence. Find out the word, which is wrongly spelt or 50. (a) wished (b) order
inappropriate, if any. The number of that word is your answer. If (c) said (d) featured
all the words printed in bold are correctly spelt and also (e) send
appropriate in the context of the sentence, mark (e) i.e. ‘All 51. (a) leave (b) prepare
Correct’ as your answer. (c) figure (d) show
(e) prove
41. Bye (a) / the summer of 1939, Hitler was ready (b) / to unleash 52. (a) Say (b) Rescue
(c) / his army on Europe. (d) / All correct (e). (c) Reveal (d) Tell
42. The two national (a) / emblems (b) / of India are of (c) / (e) Understand
Lumbini is a beautiful place in the southern part of Nepal. About 64. What did the King wish for his son, Siddharth ?
2,500 years ago, a baby boy was born to the king and queen. The (a) He wanted Siddharth to become a great ruler.
baby was named Siddharth. His mother, died when he was five (b) He wished that Siddharth would not marry Yashodhara.
days old. The baby boy grew into a handsome prince. His father (c) He wished that Siddharth’s son would take over his
tried to keep him happy. The little prince had everything he needed kingdom.
— fine clothes, the best food and good toys. But he was not (d) He wished that Siddharth would behave like the other
interested in them. He wanted to be alone and was always found princes.
in deep thought. Later, he was married to a beautiful princess. She (e) He wished that Siddharth would find the answer to all
was called Yashodhara. They had a son and named him Rahul. the suffering in the world.
The king hoped that Siddharth would become a great ruler. 65. What incident changed Siddharth’s life forever ?
One day Siddharth was driving through the street in his (a) His mother’s death.
chariot. He saw an old man and then a sick man. The oldman could (b) The incident where he saw a monk free from the misery
hardly walk. The sick man groaned in pain. Then he saw some of life.
people carrying a dead body, others were wailing and weeping at (c) The birth of his son
the loss of a dear one. Siddharth was very upset to see so much (d) His marriage to Yashodhara.
suffering and unhappiness. He was shocked. Then he saw an (e) The time he met Lord Buddha.
entirely different sight. A man in yellow robes was walking along 66. Why did Siddharth leave his home ?
the street. There was no trace of sadness on his radiant face, (a) He wanted to run away from his wife and son
instead it shone with peace. He was a monk who had given up the (b) He did not want to become heir to his father’s throne
world to escape the misery of life. (c) He was in search of a bigger kingdom
Siddharth wanted to find out why there was so much (d) He wanted to see the world
suffering in the world. He wanted to find out how men could be (e) He wanted to search for a way out of suffering and
free of misery. He could find neither peace nor happiness in the spend time in meditation.
life he was leading at the palace. One night Siddharth left his 67. According to Siddharth what was the reason for suffering
home, his wife and his little son. He went into the forest. He and unhappiness in the world ?
wanted to search for a way out of suffering and sorrow for all (a) The increased number of deaths
mankind. He meditated and got enlightenment. He became Lord (b) The advocacy of the Middle Path
Tg:- @NextGenBankers
Buddha, the enlightened one. Truth was revealed to him and he
learnt all the secrets of life and the world.
He found out that the world was full of sorrow and
(c) Human greed and selfishness
(d) People were unaware of the benefits of meditation.
(e) None of these
unhappiness. The reason for it was greed and selfishness. To be 68. What according to passage is the root cause for all human
free from suffering, we must be free from greed and desire. Desire suffering ?
is the root cause of all human suffering. He advocated the Middle (A) Desire (B) Happiness
Path and asked his followers to avoid the two extremes. (C) Meditation
61. Which of the following sentences is true according to the (a) Only (A) (b) Only (B) and (C)
passage ? (c) Only (B) (d) Only (C)
(a) Lord Budha was born in Lumbini. (e) All (A). (B) and (C)
(b) The little boy was very happy with fine clothes, good 69. Why was Siddharth called the enlightened one ?
food and toys. (a) He was smarter than all the princes of his age.
(c) Siddharth divorced his first wife. (b) He was the King’s son.
(d) Siddharth and Yashodhara had no children and so they (c) Truth was revealed to him through meditation.
were very unhappy. (d) He had a great ability to resist temptation
(e) None of these (e) He always wanted to be left alone
62. What was Siddharth interested in as a child ? 70. Why did Lord Buddha advocate the Middle Path ?
(a) He was interested in making new friends as he did not (a) He believed that it was the only way to eliminate poverty.
have any siblings. (b) He believed that it was the only way of obtaining true
(b) His interests were largely in studying and reading books. happiness.
(c) He was interested in the best of clothes and good toys. (c) He was a staunch believer of living an extreme life.
(d) In spending time alone in deep thought. (d) He did not have a happy life being a prince.
(e) In spending time with nature. (e) He was not a risk-taker.
63. Which of the following can be inferred about Siddharth ?
(A) Siddharth was different from other princes his age. DIRECTIONS (Qs. 71-75) : In the following questions, sentences
(B) Siddharth was a spoilt child. are given with blanks to be filled with an appropriate word.
Choose the best alternative among them.
(C) Siddharth was lonely because he did not have
a mother. 71. Mr. Murugan has been in this college ________ 2010.
(a) Only (A) (b) Only (B) (a) for (b) since
(c) Only (C) (d) Only (B) and (C) (c) after (d) before
(e) All (A), (B) and (C) (e) None of these
SBI Clerk Solved Paper-2014 2014-5
72. We attended a ________ discourse. 80. Which of the following should be the THIRD sentence after
(a) spiritual (b) spirituous rearrangement ?
(c) spirituality (d) spiritually (a) A (b) B
(e) None of these (c) C (d) D
73. The valley is known for its ________ growth of vegetation. (e) E
(a) luxurious
(c) luxuriant
(b) luxury
(d) luxuriously
Quantitative Aptitude
(e) None of these DIRECTIONS (Qs. 81-95) : What should come in place of the
74. Satyajitray’s films ________ all barriers of caste, creed and question mark (?) in the following questions ?
religion. They are universal.
(a) transcend (b) transcends 81. 3
13824 ´ ? = 864
(c) trancend (d) transend (a) 1296 (b) 1156
(e) None of these
(c) 1600 (d) 1024
75. I could hardly recognize him ________ I saw him.
(e) None of these
(a) after (b) but
(c) and (d) when 82. (91)2 + (41) 2 - ? = 9858
(e) None of these (a) 11236 (b) 10816
DIRECTIONS (Qs. 76-80) : Rearrange the following six sentences (c) 10404 (d) 9604
(A), (B), (C), (D), (E) and (F) in the proper sequence to form a (e) None of these
meaningful paragraph; then answer the questions that follow : 83. 4900 ÷ 28 × 444 ÷ 12 = ?
(a) 6575 (b) 6475
(A) The woodcutter thankfully broke off from work and sat down
(c) 6455 (d) 6745
to eat the delicious meal that his wife had sent for him.
(e) None of these
(B) He was in a good mood that particular morning and soon
started singing as he swung his axe at the log of wood in 84. 125% of 260 + ? % of 700 = 500
front of him. (a) 32 (b) 56
Tg:- @NextGenBankers
(C) After he had eaten his meal and taken rest for a while the
woodcutter got back to work.
(c) 23
(e) None of these
(d) 46
(D) The hours passed and the sun became hotter than ever and 7 3 1
very soon perspiraton started breaking out on the 85. 3 + 7 ´1 = ?
11 11 2
woodcuter's hands and face.
10 6
(E) One hot summer's morning a woodcutter was hard at work, (a) 13 (b) 14
chopping wood into small pieces, so that he could sell them 11 11
in the market. 9 17
(F) As it neared afternoon, his wife sent their little son to him (c) 14 (d) 10
11 22
with food for the afternoon. (e) None of these
76. Which of the following should be the FOURTH sentence
after rearrangement ? .23 - .023
86. =?
(a) F (b) E .0023 ¸ 23
(c) D (d) C (a) 0.207 (b) 207
(e) B (c) 2070 (d) 0.0207
77. Which of the following should be the FIRSTsentence after (e) None of these
rearrangement ? 87. 1.05% of 2500 + 2.5% of 440 = ?
(a) A (b) B
(a) 37.50 (b) 37.25
(c) C (d) D
(c) 370.25 (d) 372.50
(e) E
(e) None of these
78. Which of the following should be the SECOND sentence
after rearrangement ? 17 ´ 4 + 42 ´ 2
(a) A (b) B 88. =?
90 ¸ 5 ´ 12
(c) C (d) D
(e) F 25 22
79. Which of the following should be the LAST (SIXTH) (a) (b)
54 57
sentence after rearrangement ?
(a) A (b) B 11 13
(c) (d)
(c) C (d) D 27 27
(e) E (e) None of these
GP_4458
2014-6 SBI Clerk Solved Paper-2014
107. The total number of people who prefer either Sarod or Guitar, B, the respective ratio becomes 5 : 6. What is the number of
is greater than the total number of people who prefer either students in college B ?
Violin or Sitar by : (a) 450 (b) 500
(a) 1200 (b) 1600 (c) 400 (d) 600
(c) 1100 (d) 1400 (e) None of these
(e) None of these 116. The average age of 60 boys in a class was calculated as 12
108. The number of people who prefer the musical instrument years. It was later realised that the actual age of one of the
Sarod is : boys in the class was 12.5 years but it was calculated as 14
(a) 7400 (b) 8400 years. What is the actual average age of the boys in the class?
(c) 6400 (d) 8600 (a) 11 years (b) 11.275 years
(e) None of these (c) 11.50 years (d) 11.975 years
(e) None of these
2
109. If 16 % of the people who prefer Piano, would go with the 117. An aeroplane takes off 30 minutes later than the scheduled
3 time and in order to reach its destination 1500 km away in
people who prefers Flute, the percentage of people who time, it has to increase its speed by 250 km/h from its usual
prefer Flute would have been : speed. Find its usual speed.
(a) 13.5% (b) 14.5% (a) 1000 km/h (b) 750 km/h
(c) 15.5% (d) 12.5% (c) 850 km/h (d) 650 km/h
(e) None of these (e) None of these
110. The number of people who prefer Guitar is greater than the 118. Pipes A and B can fill a tank in 5 and 6 hours, respectively.
total number of people who prefer either Flute or Piano by : Pipe C can empty it in 12 hours. The tank is half full. All the
(a) 1200 (b) 1100 three pipes are in operation simultaneously. After how much
(c) 1300 (d) 1400 time, the tank will be full ?
(e) None of these
9
111. The sum of the two digits of a two-digit number is 15 and (a) 3 h (b) 11 h
the difference between the two digits of the two digit number 17
is 3. What is the product of the two digits of the two digit 8
number ?
(a) 72
Tg:- @NextGenBankers
(b) 56
(c) 2
11
h
(e) None of these
(d) 1 h
13
17
(c) 54 (d) Cannot be determined 119. A sum of money becomes eight times in 3 years if the rate is
(e) None of these compounded annually. In how much time, the same amount
112. A shopkeeper purchased 200 bulbs for ` 10 each. However, at the same compound interest rate will become sixteen times ?
5 bulbs were fused and had to be thrown away. The (a) 6 years (b) 4 years
remaining were sold at ` 12 each. What will be the (c) 8 years (d) 5 years
percentage profit ? (e) None of these
(a) 25 (b) 15 120. If the compound interest on a certain sum of money for
(c) 13 (d) 17 3 years at 10% p.a. be ` 993, what would be the simple
(e) None of these interest ?
113. The profit earned after selling a pair of shoes for ` 2,033 is (a) ` 800 (b) ` 950
the same as loss incurred after selling the same pair of shoes (c) ` 900 (d) ` 1000
for ` 1,063. What is the cost of the shoes ? (e) None of these
(a) ` 1,650
(b) ` 1,548 Reasoning Ability
(c) ` 1,532 121. In a certain code, a number 13479 is written as AQFJL and
(d) Cannot be determined 2568 is written as DMPN. How is 396824 written in that code ?
(e) None of these (a) QLPNMJ (b) QLPNMF
114. Ajay spends 25 per cent of his salary on house rent, 5 per (c) QLPMNF (d) QLPNDF
cent on food, 15 per cent on travel, 10 per cent on clothes (e) None of these
and the remaining amount of ` 27,000 is saved. What is 122. In the following sequence or instructions, 1 stands for Run,
Ajay’s income ? 2 stands for Stop, 3 stands for Go, 4 stands for Sit and 5
(a) ` 60,000 (b) ` 80,500 stands for Wait. If the sequence is continued, which
(c) ` 60,700 (d) ` 70,500 instruction will come next ?
(e) None of these 44 5453 4531 453 1245 4534 53
115. Number of students studying in colleges A and B are in the (a) Wait (b) Sit
ratio of 3 : 4 respectively. If 50 more students join college A (c) Stop (d) Run
and there is no change in the number of students in college (e) None of these
GP_4458
2014-8 SBI Clerk Solved Paper-2014
123. If the first and second letters in the word DEPRESSION 130. Statements:
were interchanged, also the third and the fourth letters, the All terrorists are human.
fifth and the sixth letters and so on, which of the following All humans are bad.
would be the seventh letter from the right ? Conclusions:
(a) R (b) O I. All terrorists are bad.
(c) S (d) P II. No human can be a terrorist.
(e) None of these 131. Statements:
124. In a certain code ‘na pa ka so’ means ‘birds fly very high’, Some teachers are followers.
‘ri so la pa’ means ‘birds are very beautiful’ and ‘ti me ka Some followers are famous.
bo’ means ‘the parrots could fly’. Which of the following is Conclusions:
the code for ‘high’in that language ? I. Some teachers are famous.
(a) na (b) k a II. Some followers are teachers.
(c) bo (d) so 132. Statements:
(e) None of these Some books are pens.
125. If 'P' denotes '–'; 'Q' denotes ' ¸ ', 'R' de notes '×' and 'W' No pen is pencil.
denotes '+' then- Conclusions:
48 Q 12 R 10 P 8 W 4=? I. Some books are pencils.
(a) 56 (b) 40 II. No book is pencil.
(c) 52 (d) 44 133. Statements:
(e) None of these Some dedicated sourls are angles
126. Laxman went 15 km to the west from my house, then turned All social workers are angles.
left and walked 20 km. He then turned East and walked 25 km Conclusions:
and finally turning left covered 20 km. How far was he from I. Some dedicated souls are social workers
my house ? II. Some social workers are dedicated souls
(a) 5 km (b) 10 km
DIRECTIONS (Qs. 134-135) : Study the information given below
(c) 40 km (d) 80 km
and answer the questions following it:
(e) None of these
Mohan is son of Arun’s father’s sister. Prakash is son of Reva,
Tg:- @NextGenBankers
127. If ‘yellow’ means ‘green’, ‘green’ means ‘white’, white means
‘red’, ‘red’ means ‘black’, ‘black’ means ‘blue’ and ‘blue’
means ‘violet’, which of the following represents the colour
who is mother of Vikash and grandmother of Arun. Pranab is
father of Neela and grandfather of Mohan. Reva is wife of Pranab.
of human blood ? 134. How is Mohan related to Reva ?
(a) black (b) violet (a) Grandson (b) Son
(c) red (d) blue (c) Nephew (d) Data inadaequate
(e) None of these (e) None of these
128. A trader in order to code the prices of article used the letters 135. How is Vikash’s wife related to Neela ?
of PSICHOLAZY in the form of ‘0 to 9’ respectively. Which (a) Sister (b) Niece
of the following code stands for ` 875.50 ? (c) Sister-in-law (d) Data inadaequate
(a) AIL.HP (b) AIL.HS (e) None of these
(c) ZYA.HO (d) ZCA.OP DIRECTIONS (Qs. 136-140) : In the following questions the
(e) None of these
symbols @, —
@ , =, © and —
Ó are used with the following meaning:
DIRECTIONS (Qs. 129-133) : In each of the following questions P © Q means P is less than Q.
there are three items. These three items may or may not be related P @ Q means P is greater than Q.
with one another. Each group of items may fit into one of the
diagrams (a), (b), (c), (d) and (e). You have to decide in which of P —@ Q means P is greater than or equal to Q.
the following diagrams and groups of items may fit. The number P = Q means P is equal to Q.
of that diagram is the answer. P —Ó Q means P is either smaller than or equal to Q.
Give answer (a) if only conclusion I follows. Now in each of the following questions, assuming the given
Give answer (b) if only conclusion II follows. statements to be true, find which of the two conclusions I and II
Give answer (c) if either I or II follows. given below them is/are definitely true ? Give answer.
Give answer (d) if neither I nor II follows. (a) if only conclusion I is true.
Give answer (e) if both I and II follow. (b) if only conclusion II is true.
129. Statements: (c) if either I or II is true.
All leaders are good team workers. (d) if neither I nor II is true, and
All good team workers are good orators. (e) if both I and II are true.
Conclusions: 136. Statements: B @ V, K © C, C — Ó B
I. Some good team workers are leaders. Conclusions : I. V @ C
II. All good orators are leaders. II. B @ K
SBI Clerk Solved Paper-2014 2014-9
Ó R
137. Statements : K @ T, S = K, T — 146. Who is to the immediate right of D ?
(a) M (b) Q
Conclusions : I. S @ R
(c) B (d) Data inadequate
II. T = R
(e) None of these
138. Statements : U = M, P —
@ U, M —
@B 147. Who is second to the right of M ?
Conclusions : I. P = B (a) B (b) R
II. P @ B (c) T (d) Q
(e) None of these
139. Statements: L —
@ N, J —Ó P, P —
@L
148. Who is second to the left of D ?
Conclusions : I. J = L (a) A (b) Q
II. P = N (c) B (d) P
140. Statements: H —
@ G, D @ E, H = E (e) Data inadequate
149. Which of the following pairs represents the immediate
Conclusions : I. D @ H neighbours of A ?
II. G ©D (a) PT (b) PB
DIRECTIONS (Qs. 141- 145) : Read the following information (c) TQ (d) PD
carefully to answer the questions that follow. (e) None of these
150. In which of the following pairs the first person is sitting to
There are six teachers A, B, C, D, E and F in a school. Each of the the immediate right of the second person ?
teachers teaches two subjects, one compulsory subject and the (a) DM (b) BT
other optional subject. D’s optional subject is History while three (c) RA (d) PQ
others have it as compulsory subject. E and F have Physics as (e) PA
one of their subjects. F’s compulsory subject is Mathematics which
is an optional subject of both C and E. History and English are A’s DIRECTIONS (Qs. 151-155) : Each of the following questions
subjects but in terms of compulsory and optional subjects, they consists of unmarked figures followed by five figures marked a,
are reverse of those of D’s. Chemistry is an optional subject of b, c, d and e. Select a figure from the marked figures which will
any one of them. There is only one female teacher in the school continue the series established by the unmarked figures.
who has English as her compulsory subject.
(a) History
Tg:- @NextGenBankers
141. What is C’s compulsory subject ?
(b) Physics
(c) Chemistry (d) English 151.
(e) None of these
142. Who is a female member in the group ?
(a) A (b) B
(c) C (d) D
(e) None of these
143. Who among the following has same optional subjects as
that of the compulsory subject of F ? (a) (b) (c) (d) (e)
(a) D (b) B
(c) A (d) C
(e) None of these 152.
144. Disregarding which is compulsory and which is the optional
subject, who has the same two subjects combination as F ?
(a) A (b) B
(c) E (d) D
(e) None of these
145. Which of the following groups of teachers has History as
the compulsory subject ?
(a) A, C and D (b) B, C and D
(c) C and D (d) A, B and C
(e) None of these
153.
DIRECTIONS (Qs. 146-150) : Study the following information
carefully and answer the questions given below
170. Networking helps in marking Marketing Function 180. Marketing channels mean
(a) a difficult task (a) Delivery objects (b) Sales targets
(b) a laborious task (c) Deliavery outlets (d) Delivery boys
(c) an easier task (e) Sales teams
(d) Networking is worthless in Marketing 181. A repair for a known software bug, usually available at no
(e) Networking has a partial role change on the Internet, is called a (n)
171. A DSA helps in _____ . (a) version (b) patch
(a) Boosting direct sales (c) tutorial (d) FAQ
(b) Boosting sales through internet (e) None of these
(c) Strong Indirect Marketing 182. A Website address is a unique name that identifies a specific
(d) Effective Telemaketing _______ on the web.
(e) All of these (a) Web browser (b) Web site
172. Mark the incorrect statement (c) PDA (d) link
(a) marketing has no relevance for public sector banks (e) None of these
(b) marketing has no relevance in Private Sector Banks 183. What is the process of copying software programs from
(c) marketing has no relevance in Foreign Banks secondary storage media to the hard disk called?
(d) All of these (a) configuration (b) download
(e) None of these (c) storage (d) upload
173. Marketing is required in banks due to (e) installation
(a) globalisation (b) computerisation 184. What characteristic of read-only memory (ROM) makes it
(c) larger population (d) government policies useful ?
(e) None of these (a) ROM information can be easily updated.
174. Online Marketing is the function of which of the following ? (b) ROM provides very large amounts of inexpensive data
(a) Purchase Section (b) Production Department storage.
(c) IT Department (d) Designs Section (c) Data in ROM is nonvolatile, that is , it remains there
(e) A collective function of all staff even without electrical power.
Tg:- @NextGenBankers
175. Customisation is useful for
(a) Designing customer specific products
(b) Call centres
(d) ROM chips are easily swapped between different
brands of computer.
(e) None of these
(c) Publicity 185. Which of the following is true ?
(d) Motivating the staff (a) Byte is a single digit in a binary number.
(e) Cold calls (b) Bit represents a grouping of digital numbers
176. The key challenge to market-driven strategy is (c) An eight-digit binary number is called a byte.
(a) Selling maximum products (d) An eight-digit binary number is called a bit.
(b) Employing maximum DSAs (e) None of these
(c) Delivering superior value to customers 186. A device that connects to a network without the use of
(d) Being rigid to changes cables is said to be
(e) Adopting short-term vision (a) distributed (b) wireless
177. Effective selling skills depends on (c) centralized (d) open source
(a) Size of the sales team (e) None of these
(b) Age of the sales team 187. The _________, also called the Web, contains billions of
(c) Peer strength documents.
(d) Knowledge level of the sales team (a) World Wide Web (b) HTTP
(e) Educational level of the sales team (c) Web Portal (d) Domain
178. Generation of sales leads can be improved by (e) None of these
(a) Being very talkative 188. In database, a field is a
(b) Increasing personal and professional contacts (a) label
(c) Being passive (b) table of information
(d) Engaging Recovery Agents (c) group of related records
(e) Product designs (d) category of information
179. A Market Plan is (e) None of these
(a) Performance Appraisal of marketing staff 189. Computers send and receive data in the form of ______
(b) Company Prospectus signals
(c) Documented marketing strategies (a) analog (b) digital
(d) Business targets (c) modulated (d) demodulated
(e) Call centre (e) All of these
GP_4458
2014-12 SBI Clerk Solved Paper-2014
190. Where are programs and data kept while the processor is 195. In a spreadsheet, a cell is defined as the
usingthem? (a) intersection of a table and a tuplet
(a) Main memory (b) Secondary memory (b) intersection of a file and a database
(c) Disk memory (d) Program memory (c) intersection of a row and column
(e) None of these (d) intersection of a field and a record
191. In order to Save an .existing docu-ment with a different name (e) None of these
you need to 196. The particular field of a record that uniquely identifies each
record is called the
(a) Retype the document and give it a different name
(a) key field (b) primary field
(b) Use the Save as .. command
(c) master field (d) order field
(c) Copy and paste the original document to a new
(e) None of these
document and then-save
197. _______ represents raw facts, whereas _______ is data
(d) Use Windows Explorer to copy the document to a
made meaningful.
different location and-then rename it
(a) Information, reporting (b) Data, information
(e) None of these (c) Information, bits (d) Records, bytes
192. are lists of commands that appear on the screen.
(e) Bits, bytes
(a) GUIs (b) Icons
198. Information on a computer is stored as
(c) Menus (d) Windows
(a) analog data (b) digital data
(e) None of these
(c) modern data (d) watts data
193. The feature in Word auto matically corrects certain spelling.
(e) None of these
typing, capitalization, or grammar errors.
199. A megabyte is actually equal to ______kilobytes.
(a) Auto Fix (b) Auto Spell
(a) 100 (b) 1000
(c) Auto Mark (d) Auto Correct
(c) 1024 (d) 1024 × 1024
(e) None of these
(e) None of these
194. What is the main folder on a storage device called ?
200. Underlined text, such as text and folder names is referred to
(a) platform
as a
(b) interface
(c) root directory
(d) device driver
Tg:- @NextGenBankers (a) icon
(c) menu
(b) hyperlink
(d) source drive
(e) None of these (e) None of these
ANSW ER KEY
1 (a) 21 (e) 41 (a) 61 (a) 81 (a) 10 1 (c) 12 1 (d) 14 1 (a) 16 1 (d) 18 1 (b)
2 (b) 22 (d) 42 (e) 62 (d ) 82 (b ) 10 2 (d) 12 2 (d) 14 2 (d ) 16 2 (c) 18 2 (d)
3 (e) 23 (e) 43 (b ) 63 (a) 83 (b ) 10 3 (e) 12 3 (d) 14 3 (d ) 16 3 (e) 18 3 (c)
4 (a) 24 (e) 44 (a) 64 (a) 84 (e) 10 4 (e) 12 4 (a) 14 4 (c) 16 4 (e) 18 4 (e)
5 (a) 25 (b) 45 (d ) 65 (b ) 85 (b ) 10 5 (b) 12 5 (e) 14 5 (d ) 16 5 (e) 18 5 (c)
6 (e) 26 (a) 46 (a) 66 (e) 86 (c) 10 6 (c) 12 6 (b) 14 6 (a) 16 6 (c) 18 6 (b)
7 (b) 27 (c) 47 (c) 67 (c) 87 (b ) 10 7 (a) 12 7 (e) 14 7 (c) 16 7 (e) 18 7 (a)
8 (c) 28 (c) 48 (a) 68 (a) 88 (a) 10 8 (b) 12 8 (e) 14 8 (d) 16 8 (b) 18 8 (b)
9 (d) 29 (d) 49 (e) 69 (c) 89 (b ) 10 9 (d) 12 9 (a) 14 9 (e) 16 9 (c) 18 9 (b)
10 (a) 30 (d) 50 (c) 70 (b ) 90 (e) 11 0 (a) 13 0 (a) 15 0 (e) 17 0 (c) 19 0 (a)
11 (d) 31 (c) 51 (b ) 71 (b ) 91 (a) 11 1 (c) 13 1 (b) 15 1 (c) 17 1 (c) 19 1 (b)
12 (a) 32 (d) 52 (d ) 72 (c) 92 (b ) 11 2 (d) 13 2 (c) 15 2 (d) 17 2 (d) 19 2 (a)
13 (c) 33 (d) 53 (e) 73 (a) 93 (c) 11 3 (b) 13 3 (d) 15 3 (a) 17 3 (a) 19 3 (b)
14 (a) 34 (c) 54 (c) 74 (a) 94 (c) 11 4 (a) 13 4 (a) 15 4 (e) 17 4 (c) 19 4 (c)
15 (a) 35 (a) 55 (c) 75 (d ) 95 (d ) 11 5 (d) 13 5 (c) 15 5 (c) 17 5 (a) 19 5 (c)
16 (a) 36 (d) 56 (b ) 76 (a) 96 (b ) 11 6 (d) 13 6 (b) 15 6 (d) 17 6 (c) 19 6 (a)
17 (e) 37 (e) 57 (b ) 77 (e) 97 (a) 11 7 (b) 13 7 (d) 15 7 (e) 17 7 (d) 19 7 (b)
18 (a) 38 (d) 58 (a) 78 (b ) 98 (d ) 11 8 (d) 13 8 (c) 15 8 (b) 17 8 (b) 19 8 (b)
19 (b) 39 (e) 59 (c) 79 (e) 99 (c) 11 9 (b) 13 9 (d) 15 9 (d) 17 9 (c) 19 9 (c)
20 (e) 40 (a) 60 (d ) 80 (d ) 100 (d ) 12 0 (c) 14 0 (e) 16 0 (a) 18 0 (c) 20 0 (b)
SBI Clerk Solved Paper-2014 2014-13
82.
Tg:- @NextGenBankers
\ ? = 36 × 36 = 1296
(b) (91)2 + (41)2 – ? = 9858
90. (e)
250 ´ 136 550 ´ ?
100
+
100
= 670
101. (c)
3
+20
23
Tg:- @NextGenBankers
+20
43
+20
63
+20
83
+20
103 Required difference = 13200 – 12000 = 1200.
111. (c) Let the nubmer be 10x + y where x > y.
According to the question,
102. (d) 1 9 25 49 81 121 169 x + y = 15
and x – y = 3
Solving both the equations,
12 32 52 72 92 112 132 x = 9, y = 6
103. (e) Pattern of the series would be as follows \ x × y = 9 × 6 = 54
112. (d) Total cost price = 200 × 10 = ` 2000
5× 1+1=6
Total selling price = 12 × 195 = ` 2340
6 × 2 + 2 = 14
14 × 3 + 3 = 45 2340 – 2000
\ Profit per cent = ´ 100 = 17%
\ 45 × 4 + 4 = 184 2000
104. (e) The pattern of the number series is: 113. (b) Le the CP of the shoes be ` x.
7× 1+1=8 \ 2033 – x = x – 1063
8 × 2 + 2 = 18 Þ 2x = 2033 + 1063 = 3096
18 × 3 + 3 = 57
3096
57 × 4 + 4 = 232 Þx= = ` 1548
2
105. (b) Can you see that the pattern is
12, 23, 32, 43, 52, 63, 72 114. (a) Saving percentage = (100 – 55) % = 45%
106. (c) No. of people who prefer flute = 11% of 60,000 If the income of Ajay be ` x, then,
11 45 ´ x
= ´ 60000 = 6600 = 27000
100 100
2100 people be less from the people who prefer flute. 27000 ´ 100
Therefore, 6600 – 2100 = 4500 Þ x= = ` 60000
45
4500 115. (d) Let total number of students in college A = 3x
Required percentage = ´ 100 = 7.5%
60000 and total number of students in college B = 4x
After 50 more students join college A
SBI Clerk Solved Paper-2014 2014-15
3x + 50 5 æ 3000 ´ 3 ´ 10 ö
New Ratio = = \ Simple interest = ` çè ÷ø = ` 900
4x 6 100
Þ 18 x + 300 = 20 x Þ 2x = 300
121. (d) 1 3 4 7 9 2 5 6 8
300
Þ x= = 150 A Q F J L D M P N
2
Total number of students in college
B = 4x = 4 × 150 = 600 3 9 6 8 2 4
Thus, Q L P N D F
116. (d) Let S be the sum of ages of 60 boys
S 122. (d) 4, 45, 453, 4531, 45312, 45, 453, 4531
Then, 12 = ... (1)
60 The next coded digit will be 1. Hence, the instruction
Run will come next.
S - 14 + 12.5 S 105
New average A = = - 123. (d) The new letter sequence is EDRPSEISNO.
60 60 60 The seventh letter from the right is P.
= 12 – 0.025 = 11.975
117. (b) Let the usual speed of the aeroplane be x km/h.
D E P R E S S I O N
Then, 1500 - 1 = 1500
x 2 (x + 250) 1 2 3 4 5 6 7 8 9 10
1500 1500 1 1500x + 750000 - 1500x 1
- = Þ =
x x + 250 2 x(x + 250) 2 124. (a) na pa ka so ® birds fly very high
Þ 750000 = x2 + 250x or x2 + 250x – 750000 = 0 ri so la pa ® birds are very beautiful
Þ x2 + 1000 x – 750 x – 75000 = 0 ti me ka bo ® the parrots could fly
Þ (x + 1000) (x – 750) = 0 Þ x = 750, – 1000 Thus high is coded as na.
Speed cannot be negative 125. (e) 48 Q 12 R 10 P 8 W 4 = ?
We get x = 750 km/h Þ ? = 48 ÷ 12 × 10 – 8 + 4
Tg:- @NextGenBankers
118. (d) Part of the tank filled by the three pipes working
131. (b) I does not follow. But II follows because it is conversion 146. (a) From the above arrangements, M is to the immediate
of the first statement. right of D.
147. (c) From the above arrangements, T is second to the right
(134-135) : Pranab Û Reva of M.
(+) (-) 148. (d) From the above arrangements, P is second to the left
¯ ¯ ¯ of D.
149. (e) From the above arrangements, P and R are the immediate
Neela Prakash Vikash neighbours of A.
(–) ( +)
150. (e) From the above arrangements, in the following pairs, P
¯ is immediate to the right of A.
Mohan Arun 151. (c) The elements move one side CW and two of the
(+) adjacent elements interchange places alternately. The
half-shaded circle rotates 90° CW, 180°, 180°, 90°... and
136. (b) B > V ....(i) K < C ...(ii); C £ B ...(iii) so on. The quarter shaded circle rotates by 180° and
No relationship can be find out between V and C. 90° ACW.
Hence I does not follow. 152. (d) The whole figure rotates by 45° ACW in each step.
From (ii) and (iii), B > K. Hence II follows. One and two arcs forming petals are added in alternate
137. (d) K > T ...(i) ; S = K ...(ii); T £ R ...(iii) steps. One arc is added on the ACW side.
Neither relationship can be established . 153. (a) The half-shaded squares move one step ACW and
rotate by 90° ACW. A new square on the ACW side
138. (c) U = M ...(i) P ³ U ...(iii); M ³ B ...(iii)
gets shaded and its shading is 90° ACW to its
Combining, we get P ³ U = M ³ B Þ P ³ B counterpart on the CW side.
Þ P = B or P > B 154. (e) In each step a new arrow is added while the elements
139. (d) L ³ N ...(i) ; J £ P ...(ii); P ³ L ...(iii) rotate by 90° CW. The pre-existing elements shift in a
Neither relationship can be established. cyclic order.
155. (c) In each step one element remains static while other
140. (e) H ³ G ....(i); D > E ...(ii); H = E ...(iii)
three shift closer or away from the axis.
Combining, we get D > E = H ³ G
Þ D > H and G < D
For (Qs. 141 to 145)
Tg:- @NextGenBankers 156. (d) From I: ja na pit sod = beautiful bunch of flowers
From II: na sod pa tok = huge bunch of twigs
Even using I & II together, we can’t determine the code
The given information is summarised in a table as follows : of pit. It may be beautiful or flowers.
Subjects 157. (e) From I: C > B > D From II: A > E > C
Teachers Compulsory Optional Combining both, we get, A > E > C > B > D
A History Englis h Hence, both statements together are necessary.
B History Chemis try 158. (b) From I: (A + C) > B > (E + D) We can’t answer the
C History M athematics question on the basis of statement I. We need some
more information.
D (Female) Englis h His tory
From II: C > A > (E + D) And ‘C’ has the second position
E Phys ics M athematics
in descending order of their salaries. Hence B is the
F M athematics Phys ics
highest salary earner.
159. (d) From I: Ramakant _ _ _ Chandrakant Suresh
141. (a) History is the compulsory subject of C.
Hence, cannot be determined.
142. (d) D is a female member in the group.
From II: Ramakant _ _ Suresh _ Naresh
143. (d) The compulsory subject of F (Mathematics) is the
Hence, cannot be determined.
optional subject of C.
The combination of both statements is not possible.
144. (c) E has physics and Mathematics as his two subjects.
160. (a) From I: (17–1 2) = 5 girl students are taller than Samir
145. (d) A, B and C all have History as the compulsory subjects.
From II: (29 – 18) = 11 girl students are shorter than
For Q. 146-150. Samir. But, from II alone it is not known how many girls
Sitting arrangements of 8 persons would be as follows : are there in the class.
D
M Q
B P
T A
R
SBI Clerk Solved Paper-2012
Held on June-2012
(Based on Memory)
Time : 135 minutes Max. Marks : 200
General Awareness
(a) Energy Efficiency (b) Indira Awas Yojana
(c) Mid Day Meal Scheme (d) Bharat Nirman
1. Which of the following agencies/organisations has put (e) National Old Age Pension Scheme
economic sanctions on Syria? 9. Which of the following organizations/agencies has decided
(a) OPEC (b) NATO to establish a National Innovation Foundation (NIF) to
(c) World Bank (d) European Union encourage early stage innovations in India?
(e) G - 8 (a) University Grants Commission
2. Who among the following was elected unanimously as the (b) Confederation of Indian Industry (CII)
Secretary General of the UNO for the second term ? (c) Department of Science & Technology, Government of
(a) Ellen Johnson Sirleaf (b) Nelson Mandela India
(c) Hillary R Clinton (d) Jacob Mathew (d) Atomic Energy Commission (AEC)
(e) Ban-Ki moon (e) Council for Industrial and Scientific Research (CSIR)
3. International Atomic Energy Agency (IAEA) has decided 10. Which of the following countries is trying to get membership
to begin New Round of Negotiations with which of the of European Union ?
following nations as it is not satisfied with the previous (a) Myanmar (b) Brazil
rounds of discussions? (c) Croatia (d) Italy
(a) Iraq (b) India (e) South Africa
(c) Japan (d) Iran 11. The General Elections took place in which of the following
4.
(e) Germany
Tg:- @NextGenBankers
As decided by UNESCO the ‘International Literacy Day’ is
countries in January 2012 in which Muslim-Brotherhood
party won with crushing majority?
observed on which of the following days every year ? (a) Lebanon (b) Libya
(a) 8th September (b) 18th March (c) Syria (d) Egypt
(c) 28th March (d) 18th September (e) Iran
(e) None of these 12. Web based complaints redressal system ‘SCORES’ is
5. As per decision taken by the European Union and IMF, launched by which of the following organizations/agencies?
which of the following countries will receive a fresh (a) Ministry of Corporate Affairs
assistance of 100 billion Euro by the year 2014 ? (b) SEBI
(a) Italy (b) Greece (c) RBI
(c) Portugal (d) Spain (d) State Bank of India
(e) Belgium (e) IBA
6. ‘Kovvada’ a town in Andhra Pradesh was in news recently 13. Which of the following countries has decided to free political
as ___________ . prisoners and pardon them as a measure to adopt democratic
(a) a nuclear plant is being developed there reforms?
(b) Maruti has decided to build a small car plant there (a) Myanmar (b) Bangladesh
(c) RBI is going to establish a New Note Printing Press (c) Mali (d) Afghanistan
there (e) Sri Lanka
(d) an earthquake hit the town in which lot of people died 14. Which of the following books is written by Satish Gujral?
(e) it is the place selected for developing a new Space (a) A General and His Army
Research Centre (b) A Bend in the River
7. Which of the following is India’s new rating index developed (c) A Secular Agenda
by Ministry of Finance ? (d) A Brush with Life
(a) CECA (b) CRIS (e) Best and the Brightest
(c) SENSEX (d) TIEA 15. Sri Jatin Das who was awarded Padma Bhushan in January
(e) RTGS 2012, is a famous ___________ .
8. Very often we read terms ‘Perform, Achieve and Trade’ (PAT) (a) Painter (b) Cine actor
in newspapers/magazines these days. PAT is related to (c) Film Director (d) Sportsman
which of the following fields ? (e) Social Worker
GP_4458
2012-2 SBI Clerk Solved Paper-2012
16. The economy in which there is a mixture of public sector 26. Thomas Cup is associated with the game of ________ .
and private sector both, is called ___________ . (a) Golf (b) Cricket
(a) Closed economy (b) Mixed economy (c) Badminton (d) Football
(c) Open economy (d) Free trade economy (e) Lawn Tennis
(e) Public economy 27. MCX is the index of which of the following stock exchanges
17. Asian Development Bank (ADB) has agreed to give a loan of of India ?
US $ 500 million to India to construct which of the following? (a) National Stock Exchange
(a) Roads in Naxal hit areas (b) Bombay Stock Exchange
(b) Railway track in North Eastern states (c) Delhi Stock Exchange
(c) Railway phataks at unmanned crossings (d) Kolkata Stock Exchange
(d) Six bridges over river Kali which is known as ‘Sorrow (e) Multi Commodity Stock Exchange
of Bihar’ 28. Ma Ying-jeou is the present President of ________ .
(e) School buildings in villages of Odisha (a) Thailand (b) Hong Kong
18. Robert Rock of England recently won an International sports (c) Taiwan (d) Myanmar
event held in Abu Dhabi. He is a well known __________. (e) Fiji
(a) Golf player (b) Badminton player 29. Which of the following is the currency used in Brazil ?
(c) Lawn Tennis player (d) Table Tennis player (a) Dollar (b) Euro
(e) Chess player (c) Real (d) Pesso
19. Which of the following teams won the Ranji Trophy Final (e) Franc
played in January 2012 ? 30. Which of the following will be the venue of the World
(a) Tamil Nadu (b) Punjab Athletic Championship to be organized in 2017 ?
(c) Maharashtra (d) Rajasthan (a) Tokyo (b) Paris
(e) Haryana (c) New York (d) London
20. As per new estimate what is per capita income in India ? (e) Mumbai
(per annum) 31. Which of the following terms is used in Botany ?
(a) ` 30,000/- (a) Recession (b) Dumping
(b) ` 40,000/-
Tg:- @NextGenBankers
(c) More than ` 50,000/- but less than ` 58,000/-
(d) ` 60,000/- 32.
(c) Barter System
(e) Biennial
(d) Cartel
37. Which of the following awards is given for exemplary she was dragging a log of wood with a rope. Again, the bullies
contribution in the area of science & technology? shouted, “Magic in the air, Grandma.” After a few minutes, the boys
(a) Arjun Award untied the log of wood too and ran away with it. When Sheela finally
(b) Kalidas Samman reached the market, she found that she had nothing but a rope in her
(c) Dronacharya Award hand. She came back home dejected as she had lost the cow. She told
(d) Dada Saheb Phalke Award Jairam the whole story. He immediately understood what had
(e) Shanti Swarup Bhatnagar Award happened. “Make chapatti, vegetable and kheer for lunch tomorrow,”
38. Which of the following terms is NOT used in banking/ he said. “Cook for atleast four people. I will come home with some
finance? guests. As soon as they come you must say, ‘I cooked what the
(a) Public Debt rabbit told me. Come, eat your lunch.’ Leave everything else to me,”
(b) Plasma Jairam reassured her. The next morning Jairam went and borrowed
(c) Joint Venture two identical rabbits from a friend. He left one at home, tied the other
(d) Net Demand & Time Liability one with a string and started walking towards the market with it. On
the way he too met the four bullies. ‘Hey Grandfather !’ they yelled,
(e) Treasury Bill
“your wife’s cow vanished yesterday. Where are you taking this
39. Who among the following was awarded Sahitya Akademi
rabbit now ?” Jairam sighed sadly and said, “This rabbit is like my
Award for the year 2011 ?
son. It obeys everything I say. But now I am sick and we need
(a) Anand Kumar (b) Srilal Shukla money, so I am going to sell it in the market.” The four bullies were
(c) Mrinal Pandey (d) Su. Venkatesan surprised when they heard this. “Does it really understand what you
(e) Vasudevan Nair say, Grandfather ?” they asked. Jairam replied, “Of course it does.
40. Victoria Azarenka won Australian Open Women’s Single Here, watch me.” Jairam turned towards the rabbit and said, “Hop
Tennis Tournament 2012 by defeating ............ . home and tell Sheela to make chapattis, vegetable and kheer for four
(a) Maria Sharapova (b) Vera Zvonareva people.” Then he untied the string and let the rabbit hop away. He
(c) Svetlana Kuznetsova (d) Bethanie Mattek Sands turned towards the four boys and said, “Come home and have lunch
(e) Sania Mirza with me.” When they reached his house his wife welcomed them
General English
and said, “I cooked what the rabbit asked me to cook. Come, eat your
Tg:- @NextGenBankers
DIRECTIONS (Qs. 41-55) : Read the following passage carefully
lunch.” She served the chapattis, vegetable and kheer to all of them.
The four bullies were stunned when they saw the rabbit sitting in the
corner. They told Jairam, “We will buy your rabbit.” Jairam pretended
and answer the questions given below it. Certain words/phrases
to think and said, “It is very precious to me.” The four bullies
have been printed in bold to help you locate them while
immediately offered him a higher price. Jairam showed some
answering some of the questions.
reluctance. The moment he agreed to sell the rabbit they paid him the
Sheela and Jairam were a poor old couple. Their only possession money and left with the rabbit immediately. The four bullies decided
was one cow. Once, Jairam fell very sick. Soon all their money was to test the rabbit’s abilities. They had been blackmailing a landlord
used up in buying medicines and they realised that they would for money. So they told the rabbit, “Go and tell the landlord to bring
have to sell their cow in order to bear the rest of the expenses. us the money within ten minutes.” The rabbit hopped off. They
Sheela decided to go to the market and sell the cow. She set off, waited for an hour but the landlord did not come with the money.
leading the cow by a rope. One the way, she met four young men. They marched to his house and yelled, “Give us the money and our
They were the local bullies who enjoyed teasing and tormenting rabbit.” The landlord had been waiting for a chance to teach these
old people. When they saw old Sheela with her cow, they decided bullies a lesson. He ordered his strongest bodyguard to give them a
to play a trick. On of them sneaked up behind her, untied the cow good thrashing. Bleeding and bruised they went back to Jairam’s
and tied a goat in its place. Sheela had been walking immersed in house and said, “You fooled us. Return our money at once.” Jairam
thought, worried about Jairam. Suddenly she heard a goat bleat
simply smiled and said, “The money has disappeared! There is magic
behind her. She turned around and was surprised to see that her
in the air.”
cow had vanished. She was leading a goat to the market. The four
41. Why did Sheela and Jairam decide to sell the cow ?
bullies came up to her and said, “There is magic in the air these
days. See, it turned your cow into a goat. “Poor Sheela walked on (a) They had spent all their money on Jairam’s sickness
with the goat. After a little while, the boys untied the goat and and needed more money
tied a rooster in its place. The rooster crowed and Sheela was (b) Sheela wanted to buy a rooster and they needed money
surprised again. The goat had turned into a rooster in its place. for that
The rooster crowed and Sheela was surprised again. The goat (c) They were fed up of the four bullies and wanted to
had turned into a rooster! The four bullies shouted, “Magic in the teach them a lesson
air, Grandma.” She resumed walking with the rooster in tow. After (d) They wanted to invite the four bullies for lunch and
a few minutes the boys crept up again, untied the rooster and tied needed money for buying the ingredients
a log of wood in its place. A few moments later Sheela realised that (e) They owned money to the landlord
GP_4458
2012-4 SBI Clerk Solved Paper-2012
42. Why was Jairam reluctant to sell the rabbit? 48. Arrange the following incidents in a chronological order as
(a) He only pretended to be reluctant in order to fool the they occurred in the passage.
bullies A. Jairam and Sheela needed money
(b) He was fond of the little rabbit and did not want to sell B. The four bullies sent the rabbit to the landlord
it to the bullies C. Sheela cooked chapattis, vegetable and kheer
(c) He wanted to gift the rabbits to the landlord D. The four bullies tied a log of wood to the rope
(a) ADBC (b) ACDB
(d) He knew that the bullies would not take good care of
(c) ADCB (d) DABC
the rabbit
(e) DACB
(e) He wanted to sell the rabbit in the market and get a 49. Which of the following is true according to the passage ?
better price for it (a) The four boys liked to help old people
43. Which of the following may be an appropriate title to the (b) Jairam asked Sheela to cook lunch for atleast four people
passage? (c) The rabbit could understand Jairam
(a) Magic in the air (d) Jairam could not trick the four boys
(b) Never steal a cow (e) None is true
(c) The strongest bodyguard 50. What did the landlord do when the four bullies went to his
(d) The smart bullies house and yelled ?
(e) The obedient rabbit (a) He got scared and immediately gave them the money
44. What did Sheela do after reaching the market empty handed? (b) He ran away and hid in the market
(a) Determined to earn money, she managed to sell the (c) He complained to Jairam about this
rope that she was left with (d) He ordered his strongest bodyguard to thrash them
(b) She returned home dejected and narrated the whole (e) He offered them lunch in order to calm them down
incident to her husband
(c) She bought two rabbits and devised a plan to get back DIRECTIONS (Qs. 51-53) : Choose the word/group of words
at the bullies which is most similar in the meaning to the word/group of words
(d) She went to the landlord and complained about the printed in bold as used in the passage.
bullies
herself
Tg:- @NextGenBankers
(e) She went home and cooked lunch for her husband and
51. Thrashing :
(a) Garbage
(c) Shouting
(b) Beating
(d) Warning
45. What did the four bullies tie to Sheela’s rope immediately (e) Rejection
after untrying the cow? 52. Possession :
(a) A log of wood (a) Control (b) Power
(b) A rooster (c) Custody (d) Keeping
(c) A rabbit (e) Belonging
(d) A goat 53. Vanished :
(e) Not mentioned in the passage (a) Gone missing (b) Was found
46. Why were the four bullies surprised when they were talking (c) Was killed (d) Was left behind
to Jairam? (e) Had exchanged
(a) They knew that Jairam was sick and had not expected
to see him DIRECTIONS (Qs. 54-55) : Choose the word which is most
(b) They had not expected Jairam to invite them for lunch opposite in meaning to the word printed in bold as used in the
(c) They could not believe that the rabbit could understand passage.
and obey Jairam 54. AGREED:
(d) They knew that Jairam did not own a rabbit and were
(a) Decided (b) Arranged
surprised to see him with one
(e) They could not believe that Jairam knew the rabbit’s (c) Accepted (d) Declined
language (e) Fixed
47. What did the four bullies see when they reached Jairam’s 55. DEJECTED :
house? (a) Happy (b) Hurried
(a) They saw that two rabbits were sitting in the corner (c) Crestfallen (d) Slowly
(b) They saw that Sheela had not prepared any lunch (e) Angrily
(c) They saw the cow they had stolen from Sheela on the
previous day DIRECTIONS (Qs. 56-60) : Each sentence below has two blanks,
(d) They saw the log of wood they had tied to Sheela’s each blank indicating that something has been omitted. Choose
rope the set of words for each blank which best fits the meaning of the
(e) They saw that the same rabbit was sitting in the corner sentence as a whole.
SBI Clerk Solved Paper-2012 2012-5
56. The students _________ not reach on time _________ of 68. She was just looking outside the window when a beautiful
the transport strike. bird caught the eye.
(a) can - as (b) could - because (a) catch the eye (b) eye catching
(c) may - account (d) will - despite (c) caught her eye (d) catch her eyes
(e) should - for (e) No correction required
57. They were _________ to vacate that house as _________ 69. They sent out the invitations last evening.
as possible. (a) send out (b) sending out
(a) asked - soon (b) ordered - easy (c) sent at (d) sending in
(c) shown - early (d) told - later (e) No correction required
(e) found - fast 70. He had to bear the brunt of his father’s mistakes.
58. The teacher _________ the concept by _________ (a) bear the burnt (b) bear the burns
practical examples. (c) bear a brunt (d) bear and burn
(a) showed - telling (b) gave - speaking (e) No correction required
(c) liked - citing (d) found - looking
DIRECTIONS (Qs. 71-80) : In the following passage there are
(e) explained - quoting
blanks, each of which has been numbered. These numbers are
59. The manager told us _________ Ramesh was very anxious
printed below the passage, against each, five words are
_________ the meeting.
suggested, one of which fits the blank appropriately. Find out
(a) about - in (b) that - before
the appropriate word in each case.
(c) like - during (d) the - for
(e) said - after Emperor Akbar was fond of (71) tricky questions to Birbal. One
60. She did not tell _________ that she _________ attended day he asked Birbal what he would (72) if he were given a choice
the party. between justice and a gold coin. “The gold coin,” said Birbal. Akbar
(a) someone - have (b) no one - has was (73) aback. He had known Birbal for many years and he knew
(c) him - not (d) her - can that Birbal was a just person. Then how could he choose the gold
(e) anyone - had coin. “You would prefer a gold coin to justice ?” He asked,
DIRECTIONS (Qs. 61-65) : Read each sentence to find out incredulously. “Yes,” said Birbal. The other courtiers were amazed
by Birbal’s (74) of idiocy. For years they had been trying to discredit
Tg:- @NextGenBankers
whether there is any grammatical error or idiomatic error in it.
The error, if any, will be in one part of the sentence. The letter of
that part is the answer. If there is “No Error” the answer is (e).
Birbal in the emperor’s eyes but without success and now the man
had gone and (75) it himself ! They could not believe their good
(Ignore errors of punctuation if any.) fortune. “I would have been dismayed if even the lowliest of my
servants had said this,” continued the emperor. “But coming from
61. The shepherd counted (a) / his sheep and found (b) / that you it’s shocking - and sad. I did not (76) you were so debased !
one of (c) / them is missing. (d) / No Error (e) I never expected this from you. How could you be so shallow ?”
62. The teacher were (a) / impressed by her performance (b) /
and asked her to (c) / participate in the competition. (d) No One (77) for what one does not have, Your Majesty !” said
Error (e) Birbal, quietly. “You have (78) to it that in our country justice is
63. She asked her (a) / son for help her (b) / find a place to bury available to everybody. So as justice is already available to me
(c) / the gold ornaments (d) No Error (e) and as I’m always (79) of money I said I would choose the gold
64. The painter was (a) / ask to paint a (b) / picture of the king, coin.” The emperor laughed. He thought to himself, ‘I should
(c) / sitting on his throne (d) No Error (e) have known that Birbal would come up with a witty reply as
65. The story was (a) / about how an (b) / intelligent man had always.’ He was so pleased with Birbal’s reply that he gave him
saving (c) / himself from being robbed (d) No Error (e) (80) one but a thousand gold coins.
71. (a) showing (b) asking
DIRECTIONS (Qs. 66-70) : Which of the phrases (a), (b), (c)
and (d) given below should replace the phrase given in bold in (c) naming (d) finding
the following sentence to make the sentence grammatically (e) telling
meaningful and correct ? If the sentence is correct as it is and 72. (a) look (b) said
‘No correction is required’, mark (e) as the answer. (c) think (d) choose
66. They are yet to decided about buying the new furniture. (e) find
(a) still decide (b) yet to decision 73. (a) pushed (b) fallen
(c) yet to decide (d) still decided (c) pulled (d) sent
(e) No correction required
(e) taken
67. Rohan’s mother was feed up of his laziness.
(a) fed up in (b) fed off 74. (a) idea (b) display
(c) feeds up of (d) fed up of (c) reply (d) place
(e) No correction required (e) showing
GP_4458
2012-6 SBI Clerk Solved Paper-2012
99. Cost of 36 pens and 42 pencils is ` 460/-. What is the cost of DIRECTIONS (Qs. 104-106) : Study the information given below
18 pens and 21 pencils ? and answer the questions that follow :
(a) ` 230/- (b) ` 203/-
(c) ` 302/- (d) ` 320/- An article was bought for ` 5600. Its price was marked up by 12%.
(e) None of these Thereafter it was sold at a discount of 5% on the market price.
100. The ratio of the ages of A and B seven years ago was 3 : 4 104. What was the market price of the article ?
respectively. The ratio of their ages nine years from now (a) ` 6207/- (b) ` 6242/-
will be 7 : 8 respectively. What is B’s age at present ? (c) ` 6292/- (d) ` 6192/-
(a) 16 years (b) 19 years (e) ` 6272/--
(c) 28 years (d) 23 years 105. What was the percent profit on the transaction ?
(e) None of these (a) 6.8% (b) 6.3%
101. In how many years will ` 4600 amount to ` 5428 at 3 p.c.p.a. (c) 6.4% (d) 6.6%
simple interest ? (e) 6.2%
(a) 3 (b) 5
106. What was the amount of discount given ?
(c) 6 (d) 4
(a) ` 319.6 (b) ` 303.6
(e) None of these
102. What will be the average of the followings set of scores ? (c) ` 306.3 (d) ` 313.6
59, 84, 44, 98, 30, 40, 58 (e) ` 316.9
(a) 62 (b) 66 107. The area of a rectangle is 1209 square meters. Its length
(c) 75 (d) 52 measures 39 meters. How much is its perimeter ?
(e) 59 (a) 122 meters
103. The sum of three consecutive odd numbers is 1383. What is (b) 134 meters
the largest number ? (c) 148 meters
(a) 463 (b) 49 (d) 144 meters
(c) 457 (d) 461 (e) None of these
(e) None of these
Tg:- @NextGenBankers
DIRECTIONS (Qs. 108-112) : Study the following graph carefully and answer the questions that follow :
The graph given below represents the number of users of two broadband services A and B across 5 cities P, Q, R, S and T.
800
700
A
600 B
500
400
P Q R S T
CITY
108. What is the total number of users of brand B across all five 110. What is the average number of users of brand A across all
cities together ? five cities together ?
(a) 2700 (b) 3000 (a) 560 (b) 570
(c) 3100 (d) 2900 (c) 580 (d) 590
(e) 3200 (e) 550
111. What is the difference between the total number of users of
109. The number of users of brand A in city T is what percent of
Brand A and B together in city R and the total number of
the number of users of brand B in City Q ?
users of brand A and B together in city P ?
(a) 150 (b) 110 (a) 170 (b) 140
(c) 140 (d) 160 (c) 130 (d) 150
(e) 120 (e) 160
GP_4458
2012-8 SBI Clerk Solved Paper-2012
112. What is the respective ratio of the number of users of brand (a) None (b) One
A in city P to the number of users of brand B in city S ? (c) Two (d) Three
(a) 5 : 7 (b) 4 : 7 (e) More than three
(c) 2 : 5 (d) 3 : 4 122. How many letters are there in the English alphabetical series
(e) 5 : 6 between second letter of the word which is second from the
113. 21 articles were bought for ` 6531 and sold for ` 9954. How right and the third letter of the word which is third from the
much was the approximate profit percentage per article ? left of the given words?
(a) 56% (b) 43% (a) One (b) Two
(c) 52% (d) 49% (c) Three (d) Four
(e) 61% (e) Five
114. A and B together can complete a particular task in 8 days. If 123. If in each of the given words, each of the consonants is
B alone can complete the same task in 10 days, how many changed to previous letter and each vowel is changed to
days will A take to complete the task if he works alone ? next letter in the English alphabetical series, in how many
(a) 28 (b) 36 words thus formed will no vowels appear ?
(c) 40 (d) 32 (a) None (b) One
(e) None of these
(c) Two (d) Three
115. The cost price of an article is ` 1700. If it was sold at a
(e) More than three
price of ` 2006, what was the percentage profit on the
transaction ? 124. If the last alphabet in each of the words is changed to the
(a) 18 (b) 12 next alphabet in the English alphabetical order, how many
(c) 10 (d) 15 words having two vowels (same or different vowels) will be
(e) 20 formed ?
(a) None (b) One
DIRECTIONS (Qs. 116-120) : What should come in place of the (c) Two (d) Three
question mark (?) in the following number series ? (e) Four
116. 1 1 2 6 ? 120 125. If the given words are arranged in the order as they would
(a) 24 (b) 60 appear in a dictionary from left to right, which of the
(c) 100
(e) None of these
Tg:- @NextGenBankers
(d) 30 following will be fourth from the left ?
(a) WIT (b) BAR
117. 7 8 16 43 ? 232 (c) URN (d) ELF
(a) 204 (b) 107 (e) TOP
(c) 119 (d) 89 126. Veena walked 5m towards north, took a left turn and walked
(e) None of these 7 m. She took a left turn again and walked 8m before taking
118. 4 13 17 ? 30 39 a left turn and walking 7 m. She then took a final left turn and
(a) 29 (b) 21 walked 1 m before stopping. How far is Veena from the
(c) 26 (d) 19 starting point ?
(e) None of these (a) 3 m (b) 6 m
119. 982 977 952 827 822 ? (c) 4 m (d) 2 m
(a) 779 (b) 817
(e) 7 m
(c) 797 (d) 697
127. In a certain code IDEAS is written as HEDBR and WOULD
(e) None of these
is written as VPTMC. How will RIGHT be written in the
120. 41472 5184 576 72 8 ?
same code ?
(a) 0 (b) 9
(a) QJHIS (b) QJFGS
(c) 1 (d) 8
(c) SHHGU (d) QJFIU
(e) None of these
(e) QJFIS
Reasoning Ability DIRECTIONS (Qs. 128-130) : Study the following information
to answer the given question.
DIRECTIONS (Qs. 121-125) : Following questions are based
on five words given below : Amongst five friends, A, B, C, D and E, each bought a mobile
phone for a different price. A paid more than both C and E. Only B
WIT BAR URN ELF TOP paid more than D. E did not pay the minimum amount. E paid `
(The new words formed after performing the mentioned operations 8,000 for the phone.
may or may not necessarily be meaningful English words) 128. Which of the following is true with regard to the given
121. If in each of the words, all the alphabets are arranged in information?
English alphabetical order within the word, how many words (a) Only two people paid a price less than the price paid
will NOT begin with a vowel ? by B
SBI Clerk Solved Paper-2012 2012-9
143. Which of the following is true regarding S ? 149. Which of the following represents such engineers who are
(a) S sits exactly between R and P MBA degree holders but not bank employees ?
(b) S sits second to left of Q (a) G and B (b) Only F
(c) P is an immediate neighbour of S (c) D (d) G
(d) D is an immediate neighbour of the person who faces S (e) None of these
(e) None is true 150. Which of the following correctly represents such engineers
who are neither bank employees nor MBA degree holders?
144. Who amongst the following faces Q ?
(a) Only G (b) C and B
(a) A (b) B
(c) A and D (d) C and G
(c) C (d) D
(e) Only C
(e) Cannot be determined
145. Who amongst the following faces the person who sits DIRECTIONS (Qs. 151-160) : In each of the questions given
exactly between B and C ? below which one of the five answer figures on the right should
(a) P (b) Q come after the problem figures on the left, if the sequence were
(c) R (d) S continued ?
(e) Cannot be determined 151. Problem Figures
DIRECTIONS (Qs. 146-150) : Each of the questions given below
is based on the given diagram. The diagram shows three figures
each representing Engineers, MBA degree holders and Bank
employees.
Answer Figures
A Bank Employees
G
E
B C Engineers
D F Tg:- @NextGenBankers
MBA degree holders
(a) (b)
152. Problem Figures
(c) (d) (e)
(a) (b) (c) (d) (e) (a) (b) (c) (d) (e)
155. Problem Figures 160. Problem Figures
Answer Figures
Answer Figures
Answer Figures
Tg:- @NextGenBankers
161. Marketing is the function of _______
(a) Only Sales persons (b) Only counter staff
(c) Only qualified persons (d) Top Bosses
(e) A collective function of all staff
162. The performance of a sales person depends on _______
(a) Ability and willingness of the sales person
(b) Incentives paid
(a) (b) (c) (d) (e)
157. Problem Figures (c) Size of the sales team
(d) Team leader’s attitude
(e) His aggressive nature
163. Good marketing strategy envisages good and proper
_______
(a) Product distribution
Answer Figures (b) Networking of branches
(c) High Pricing
(d) Placement of counter staff
(e) Relationship management
164. Service Marketing is resorted to in _______
(a) (b) (c) (d) (e) (a) All MNCs
158. Problem Figures (b) All production houses
(c) Industrial units
(d) Insurance companies and banks
(e) Fish markets
165. Customisation results in _______
Answer Figures (a) Customer exit
(b) Customer retention
(c) Customer complaints
(d) Better Balance Sheet figures
(a) (b) (c) (d) (e) (e) Better technology
GP_4458
2012-12 SBI Clerk Solved Paper-2012
166. Current Accounts are basically meant for _______ (c) Selling arrangements
(a) Investment purpose (b) Savings purpose (d) The counter staff
(c) Identity purpose (d) To earn Foreign Exchange (e) The back-office staff
(e) Day-to-day needs of one’s business 176. Target group means _______
167. The sole aim of marketing is to _______ (a) All employers (b) All sales persons
(a) Improve the Balance Sheet figures (c) Intended buyers (d) All industries
(b) Increase recruitment (e) Call Centre persons
(c) Increase profits 177. SME means _______
(d) Increase production (a) Selling and Marketing Employees
(e) Increase branch network (b) Sales and Mergers of Entities
168. Aggressive Marketing is necessitated due to _______ (c) Small and Micro Entities
(a) Globalisation (d) Small and Medium Enterprises
(b) Increased competition (e) Sales Performance Measurement Program
(c) Increased production 178. A short term loan is repayable within _______
(d) Increased job and opportunities (a) 20 years
(e) Increased staff (b) 3 years
169. Value added services means _______ (c) As per the borrowers’ wish
(a) Substituted products (d) As per the guarantor’s wish
(b) Highly valuable products (e) There is no need to repay short term loans
(c) Old products 179. “USP” in marketing language means _______
(d) Extra services in addition to existing ones (a) Uniform Selling Practices
(e) At par services (b) Unique Sales Person
170. Savings Accounts can be opened by _______ (c) Unique Selling Proposition
(a) All individuals fulfilling KYC norms (d) Unique Savings Plans
(b) All tax payers only (e) Useful Sales Persons
Tg:- @NextGenBankers
(c) All individuals above the age of 18
(d) All businessmen only
(e) All students below the age of 18
180. An IPO Loan is meant for
(a) Meeting personal needs
(b) For higher education
171. The best promotional tool in any marketing is _______ (c) Medical treatment
(a) Pamphlets (d) For starting a new industry
(b) Newsletters (e) For purchase of shares
(c) Word of mouth publicity 181. Devices that enter information and let you communicate
(d) Regional Advertisement with the computer are called _______.
(e) Viral marketing (a) Software (b) Output devices
172. Market information means _______ (c) Hardware (d) Input devices
(a) Knowledge level of DSAs (e) Input/Output devices
(b) Information about marketing staff 182. In electronic device, operating under the control of
(c) Information regarding Share market information, that can accept data, process the data, produce
(d) Knowledge of related markets output and store the results for future use _______
(e) Latest knowledge about technology progress (a) Input (b) Computer
173. Efficient marketing style requires _______ (c) Software (d) Hardware
(a) Proper planning (e) None of these
(b) Good debating skills 183. What is the function of the Central Processing Unit of a
(c) Arrogant staff Computer ?
(d) Knowledge of many languages (a) Creates invoices
(e) Ignorant customers (b) Performs calculations and processing
174. The target group for SME loans is _______ (c) Deletes Data
(a) All SSIs (b) All College Professors (d) Corrupts the data
(c) All students (d) All Nurses (e) None of these
(e) All salaried persons 184. All the characters that a device can use is called its ?
175. Market segmentation means grouping _______ (a) Skill Set (b) Character Alphabet
(a) The sales teams (c) Character Codes (d) Keyboard Characters
(b) The customers as per their needs and tastes (e) Character Set
SBI Clerk Solved Paper-2012 2012-13
185. If your computer keeps rebooting itself, then it is likely that (a) virtual private network(b) LAN
_______ (c) intranet (d) extranet
(a) It has a virus (e) internet
(b) It does not have enough memory 194. Tangible, physical computer equipment that can be seen
(c) There is no printer and touched is called _______
(d) There has been a power surge (a) hardware (b) software
(e) It needs a CD-ROM (c) storage (d) input/output
186. What does RAM stand for ? (e) None of these
(a) Read Access Memory 195. Which of the following is the second largest measurement
(b) Read Anywhere Memory of RAM ?
(c) Random Anything Memory (a) Terabyte (b) Megabyte
(d) Random Access Module (c) Byte (d) Gigabyte
(e) Random Access Memory (e) Mega Hertz
187. What type of device is a 3½ inch floppy drive ? 196. What resides on the motherboard and connects the CPU to
(a) Input (b) Output other components on the motherboard ?
(c) Software (d) Storage (a) Input Unit (b) System Bus
(e) None of these (c) ALU (d) Primary Memory
188. What utility to you use to transfer files and exchange (e) None of these
messages ? 197. By firmware we understand _______
(a) Web browsers (b) WWW (a) physical equipment used in a computer system
(c) Email (d) Hpertext (b) a set of instructions that causes a computer to perform
(e) search engines one or more tasks.
189. Which unit controls the movement of signals between CPU (c) the people involved in the computing process.
and I/O ? (d) a set of programs that is pre-installed into the read
(a) ALU (b) Control Unit only memory of a computer during the time of
(c) Memory Unit
(e) None of these Tg:- @NextGenBankers
(d) Secondary Storage
190. The three main parts of the processor are _______ 198.
manufacturing
(e) None of these
Supercomputers _______
(a) ALU, Control Unit and Registers (a) are smaller in size and processing capability than
(b) ALU, Control Unit and RAM mainframe computers
(c) Cache, Control Unit and Registers (b) are common in majority of households
(d) Control Unit, Registers and RAM (c) contain thousands of microprocessors
(e) RAM, ROM and CD-ROM (d) are rarely used by researchers due to their lack of
191. Which of the following memory chip is faster ? computing capacity
(a) There is no certainty (e) are of the same size as laptops
(b) DRAM 199. The cost of a given amount of computing power has
(c) SRAM _______ dramatically with the progress of computer
(d) DRAM is faster for larger chips technology.
(e) None of these (a) stayed the same
192. Which of the following does not relate to Input Unit ? (b) changed proportionally with the economy
(a) If accepts data from the outside world. (c) increased
(b) It converts data into binary code that is understandable (d) fluctuated
by the computer (e) decreased
(c) It converts binary data into the human readable form 200. The basic computer processing cycle consists of _______
that is understandable to the users. (a) input, processing and output
(d) It sends data in binary form to the computer for further (b) systems and application
processing (c) data, information and applications
(e) None of these (d) hardware, software and storage
193. Which of the following terms is just the connection of (e) None of these
networks that can be joined together ?
GP_4458
2012-14 SBI Clerk Solved Paper-2012
ANSW ER KEY
1 (d) 21 (d) 41 (a) 61 (d) 81 (d) 101 (c) 121 (b) 141 (c) 161 (e) 181 (e)
2 (e) 22 (c) 42 (a) 62 (a) 82 (a) 102 (e) 122 (e) 142 (a) 162 (a) 182 (d)
3 (d) 23 (b) 43 (a) 63 (b) 83 (e) 103 (e) 123 (c) 143 (e) 163 (e) 183 (b)
4 (a) 24 (e) 44 (b) 64 (b) 84 (e) 104 (e) 124 (a) 144 (d) 164 (d) 184 (d)
5 (b) 25 (c) 45 (d) 65 (c) 85 (b) 105 (c) 125 (c) 145 (b) 165 (d) 185 (a)
6 (a) 26 (c) 46 (c) 66 (c) 86 (b) 106 (d) 126 (d) 146 (e) 166 (e) 186 (e)
7 (b) 27 (e) 47 (e) 67 (d) 87 (b) 107 (e) 127 (e) 147 (c) 167 (c) 187 (d)
8 (a) 28 (c) 48 (c) 68 (c) 88 (d) 108 (b) 128 (e) 148 (b) 168 (b) 188 (c)
9 (c) 29 (c) 49 (b) 69 (e) 89 (d) 109 (c) 129 (a) 149 (b) 169 (d) 189 (b)
10 (c) 30 (d) 50 (d) 70 (e) 90 (d) 110 (c) 130 (a) 150 (e) 170 (c) 190 (a)
11 (d) 31 (d) 51 (b) 71 (b) 91 (e) 111 (d) 131 (b) 151 (d) 171 (c) 191 (c)
12 (b) 32 (b) 52 (e) 72 (d) 92 (a) 112 (a) 132 (a) 152 (b) 172 (c) 192 (c)
13 (a) 33 (e) 53 (a) 73 (e) 93 (b) 113 (c) 133 (a) 153 (a) 173 (b) 193 (e)
14 (d) 34 (e) 54 (d) 74 (b) 94 (e) 114 (c) 134 (c) 154 (e) 174 (a) 194 (a)
15 (a) 35 (e) 55 (c) 75 (c) 95 (d) 115 (a) 135 (b) 155 (b) 175 (b) 195 (d)
16 (b) 36 (c) 56 (b) 76 (c) 96 (b) 116 (a) 136 (e) 156 (a) 176 (c) 196 (b)
17 (b) 37 (e) 57 (a) 77 (a) 97 (e) 117 (b) 137 (d) 157 (d) 177 (d) 197 (d)
18 (a) 38 (b) 58 (e) 78 (d) 98 (e) 118 (b) 138 (b) 158 (a) 178 (b) 198 (c)
19 (d) 39 (b) 59 (b) 79 (c) 99 (a) 119 (e) 139 (d) 159 (b) 179 (c) 199 (b)
20 (c) 40 (a)
Tg:- @NextGenBankers
60 (e) 80 (b) 100 (d) 120 (d) 140 (a) 160 (d) 180 (d) 200 (a)
current prices as the barometer. According to the 48. (c) The first event is of the couple being needy as in (A).
revised GDP data for the last financial year, per capita Among the various things they tie to the rop is a log of
income is estimated to have risen 16.9% to ` 53,331 wood as in (D). The next given event in terms of the
compared to ` 46,117 in the previous year. four sentences can be found in (C) where Sheela cooks
22. (c) Permanent Account Number (PAN) for the four bullies. Once they buy the rabbit they
23. (b) Nicaragua President Daniel Ortega send it to the landlord to extract money as in (B) and
24. (a) “Zindagi Na Milegi Dobara” grab 7 most popular when they reach his house they get good thrashing.
awards-Best Film, Best Director, Best Dialogue, Best 49. (b) Fouram decides to take revenge on the bullies and
Supporting Actor (Male), Best Movie (Critics), Best trick them in the same manner as they had tricked his
Cinematography and Best Choreography. wife. So he asks her to cook for them.
26. (c) The Thomas Cup, sometimes called the World Men’s 50. (d) The landlord was so angry that he had the bullies
Team Championships, is an international badminton thrashed by his strongest bodyguard.
competition among teams representing member 51. (b) Thrashing means to give a sound beating.
nations of the Badminton World Federation (BWF), 52. (e) In this case possession means belonging.
the sport’s global governing body. The championships 53. (a) Vanished means had got lost.
have been conducted every two years since the 1982 54. (d) In this case agreed means to say yes, so the opposite
tournament, amended from being conducted every is declined which means to say no.
three years since the first tournament held in 1948- 55. (c) Here dejected means sad or upset which is the same as
1949. crest fallen.
27. (e) Multi Commodity Exchange of India Ltd. (MCX) (BSE: 58. (e) Obviously the concept has to be explained and
534091) is an independent commodity exchange based practical examples can only be quoted.
in India. It was established in 2003 and is based in 60. (e) She would not have told any one that she lad attended
Mumbai. the party.
30. (d) 16th World Championships in Athletics will be 61. (d) Since the sentence begins in past tense. It should end
organized in London in 2017 in past tense also since it the subject is singular the
32. (b) Total Sanitation Campaign (TSC) is a comprehensive verb will also be singular. Hence “them is missing”
programme to ensure sanitation facilities in rural areas should be “them was missing”.
Tg:- @NextGenBankers
with the broader goal to eradicate the practice of open
defecation.
62. (a) The teacher that is the subject is singular so “were”
will be replaced with “was”.
33. (e) Comptroller and Auditor General (CAG) of India is 63. (b) “Son for help her” should be “son to help her”.
not a financial institute. 64. (b) The verb “ask” will be in the past tense it will become
36. (c) Sania Mirza related to tennis. “asked”.
37. (e) The Shanti Swarup Bhatnagar Prize for Science and 65. (c) “Saving” will be replaced with past tense of the verb
Technology is awarded annually by the Council of “Save” that is saved because it is preceded by had.
Scientific and Industrial Research (CSIR) for notable 66. (c) Since the whole sentence is in the present tense “yet
and outstanding research, applied or fundamental, in to decided” should be replaced with “yet to decide”.
biology, chemistry, environmental science, 67. (d) Rohan's mother cannot be feed up. It has no meaning.
engineering, mathematics, medicine and Physics This should be replaced with fed up of.
38. (b) Plasma is a state of matter similar to gas in which a 68. (c) Since the subject is a female “caught the eye” should
certain portion of the particles are ionized. be replaced with “caught her eye”.
41. (a) Sheela and Jairam had to sell their cow because they 81. (d) 3.6 + 36.6 + 3.66 + 0.36 + 3.0 = 47.22
needed money for Jairam’s sickness. They were a very 82. (a) 23 × 45 ¸ 15 = 69
poor old couple. 5 1 5 29 15 63
42. (a) Jairam was pretending to be reluctant to sell the rabbit 83. (e) 4 + 7 -8 = + -
6 2 11 6 2 11
so that the bullies would realise how precious the rabbit
was to him. 319 + 495 - 558 256 128 29
= = = =3
43. (a) ‘Magic in the Air ’ has been used repeatedly 66 66 33 33
throughout the passage. It is the most appropriate title.
44. (b) She was very upset. So she came back home and told 210 17
84. (e) ´ ´ ? = 4046
her husband the whole story. 14 15
45. (d) The four bullies first tied a goat to Sheela’s rope.
4046 ´ 15 ´ 14
46. (c) The four bullies were surprised because it was different ?= = 238
to believe that the rabbit could understand obey Jairam. 210 ´ 17
47. (e) The four bullies thought it was the same rabbit sitting 85. (b) 83% of 2350 = ?
in a corner.
83 ´ 2350
?= = 1950.50
100
GP_4458
2012-16 SBI Clerk Solved Paper-2012
99. (a) Let one pen cost be ` x and one pencil cost be ` y
86. (b) 1089 + 3 = (?)2
36 x + 42 y = 460 ... (i)
33 + 3 = (6)2 18 x + 21 y = ?
87. (b) 96 + 32 × 5 – 31 = 225 Divided eq. (i) by 2
88. (d) ? ¸ 36 = (7)2 – 8 18 x + 21 y = 230
?
= 49 – 8 100. (d) Let the present age of A = x and B = y years
36
According to first condition
\ ? = 36 × 41
= 1476 x -7 3
= Þ 4 x - 28 = 3 y - 21Þ 4 x - 3 y = 7 ... (i)
89. (d) 8281 = ? y -7 4
= 91 According to second condition
90. (d) (63)2 – (12)2 = 3825 x+9 7
= Þ 8 x + 72 = 7 y + 63
9 18 43 y+9 8
91. (e) + =? -
4 5 10 Þ 7y – 8x = 9 ... (ii)
9 18 43 8 x - 6 y =14
?= + +
4 5 10 7 y - 8x =9
45 + 72 + 86 203 3 y = 23 years.
= = = 10
20 20 20
p´r´t
92. (a) 17 × 19 × 4 ¸ ? = 161.5 101. (c) P + = 5428
100
1 1615
= 4600 ´ 3 ´ t
? 10 ´17 ´19 ´ 4 = 5428 - 4600 = 828
100
?=8
93.
348 ´ 31
Tg:- @NextGenBankers
(b) 1798 ¸ 31 × ? = 348
Þt=
828 ´ 100
4600 ´ 3
= 6 years
?= =6
1798 59 + 84 + 44 + 98 + 30 + 40 + 58
94. (d) (9.8 × 2.3 + 4.46) ¸ 3 = 3? 102. (e) Required average = = 59
7
27 ¸ 3 = 3? 32 = 3? 103. (e) x + (x + 1) + (x + 2) = 1383
\ ?=2 Þ 3x + 3 = 1383
95. (d) 43% of 600 + ?% of 300 = 399 Þ 3x = 1380
43 × 6 + 3x = 399
3x = 141 1380
Þ x= = 460
x = 47 3
2 Largest number = x + 2 = 462
æ 4 ö (104-106) :
96. (b) Total amount = 7500 ç1 +
è 100 ÷ø 104. (e) Cost price of article = ` 5600
æ 26 26 ö 12
= 7500 ç ´ ÷ = 8112 Marked price = 5600 + 5600 ´
è 25 25 ø 100
C I = Total amount – sum 5
= 8112 – 7500 = ` 612 SP = 6272 + 6272 ´ = 5958.4
100
CR EAM = ` 6272
97. (e) 1 2 3 4 5
5958.4 - 5600
Required number of ways = 5! 105. (c) Profit% = ´ 100
= 5 × 4 × 3 × 2 × 1 = 120 5600
98. (e) Circumference = 792 358.4
2p r = 792 = = 6.4%
56
792 792 ´ 7 5
r = = = 126 m
2p 22 ´ 2 106. (d) Amount of discount = 6272 ´
100
= 313.6
SBI Clerk Solved Paper-2012 2012-17
1 1 1
+ =
x 10 8
Tg:- @NextGenBankers 126. (d) N
1 1 1 10 - 8 2 1 W E
= - = = =
x 8 10 80 80 40
x = 40 S
2006 - 1700 306 127. (e) Coding for: I D E A S
115. (a) %profit = ´ 100 = ´ 100 = 18% –1¯ +1¯ –1¯ +1¯ –1¯
1700 1700
116. (a) 1 × 1 = 1 H E D B R
1×2=2 Coding for: W O U L D
2×3=6 –1¯ +1¯ –1¯ +1¯ –1¯
V P T M C
6 × 4 = 24 Similarly, R I G H T
24 × 5 = 120 –1¯ +1¯ –1¯ +1¯ –1¯
117. (b) 8 – 7 = 1 = 13 Q J F I S
16 – 8 = 8 = 23 (128-130):
43 – 16 = 27 = 33 A > B and C
B > D only
? – 43 = 64 = 43
E did not pay the minimum amount and paid ` 8.000
232 – 107 = 125 = 53 128. (e) A > C & E
118. (b) 4 × 3 + 1 = 13 only B > D
4 × 4 + 1 = 17 B> D >A> E > C
4 × 5 + 1 = 21 ¯
8,000 `
119. (c) 982 – 977 = 5
129. (b) E paid = ` 8, 000
977 – 952 = 25
D paid = 17, 000 + 8,000 = 25,000
952 – 827 = 125
Possibly amount paid by A = 16,000
827 – 822 = 5
Because D > A > E
822 – ? = 25
GP_4458
2012-18 SBI Clerk Solved Paper-2012
Fires (141-145) : S R Q P
Tyres
133. (a) clip A C D B
pin (146-150):
badges A
G Bank employees
B E
MBA degree D F C Engineers
holders.
134. (c) colour paint
146. (e) B is common to all diagram
147. (c) Letter A and E are common to such rectangle
148. (b) Letter D represents only MBA degree holders.
149. (b) Letter F represents such engineers who are MBA
degree holders but not bank employees.
brush
150. (e) Letter C represents only engineers (neither MBA nor
bank employees).
Tg:- @NextGenBankers
brush 151. (d) Next figure of each problem figure changes like that
the horizontal line shifted to downward and the letters
changes their position on either side of vertical lines
and middle letter of horizontal line changes by new
135. (b) stars planets letter.
Galaxies 153. (a) Next second figure of each problem figure is rotated
clockwise by 90° and in next figure of each problem
figure one more circle is darken.
154. (e) First right of each problem figure change like : The
137. (d) The given arrangement is:
figures on hexagonal side are rotated clockwise one
side and two side respectively and one more same
8th to the left of 20th
figure added on particular side.
155. (b) Next right figure of each problem figure is rotated
B U B D C E D B D E U B A D anticlockwise clockwise respectively by so and the
figure at appex is changes to new Figure.
C B E A C D A E B A U A C D 156. (a) Next figure of each question figure is related by 45°
and 90°, 135°, 180° anticlockwise respectively and the
B C A C figure at end of line shifted to opposite corner.
157. (d) Corner figure is shifted to vertices of triangle and one
20th from the left
corner figure is changed by new figure.
158. (a) In next figure of each question figure lower part of
138. (b) B UCE combination figure is doubled crossed and upper left
C U BE corner figure is attached at power part.
SBI Clerk Solved Paper-2011
Held on : 16-01-2011 (2nd Sitting)
Based on Memory
Time : 135 minutes Max. Marks : 200
General Awareness 8. Hashim Amla who was awarded the player of the year in his
category is a famous______
1. The Govt. of India and RBI are making all the efforts to provide (a) Tennis player (b) Badminton player
Banking facilities to all the villages/habitations in the country. (c) Hockey player (d) Chess player
This is the part of their policy of: (e) Cricket player
(a) PURA 9. What does the letter 'F' denote in the abbreviated name 'IIFCL'?
(b) Financial Inclusion (a) Functional (b) Foreign
(c) Swawalamban (c) Forwarded (d) Finance
(d) Pradhan Mantri Adarsh Gram Yojana (e) None of these
(e) None of these 10. What is the purpose of Mahatma Gandhi National Rural
2. Which of the following is NOT a Private Bank in India? Employment Guarantee Act?
(a) Yes Bank (b) HDFC Bank (a) To provide employment to all those who are unemployed
(c) Axis Bank (d) Kotak Mahindra Bank (b) To provide 100 days employment to those who are
(e) IDBI Bank willing to take a job
3. Which of the following is/are NOT the functions of a bank? (c) To ensure that nobody in a village remains unemployed
(A) Accepting deposits from the public (d) To ensure that people get enough income during off
(B) Grant of loans and advances season
to survive
Tg:- @NextGenBankers
(C) Providing finance to all those who cannot earn enough
11.
(e) None of these
'Guangzhou' was recently in news as ______
(a) Only (A) (b) Only (B) (a) G-20 Summit took place there
(c) Only (C) (d) All (A), (B) & (C) (b) Commonwealth Games were organized there
(e) Only (B) & (C) (c) ASEAN Summit took place there
4. David Cameron who was in news recently is the______ (d) Asian Games 2010 were organized there
(a) President of Australia (e) None of these
(b) Prime Minister of United Kingdom 12. Which of the following is a treaty associated with the use of
(c) President of Brazil Nuclear Power?
(d) Prime Minister of Brazil (a) NPT (b) SAFTA
(e) None of these (c) GATT (d) Look East Policy
5. Farmers in India are covered by some welfare schemes. Which (e) None of these
of the following is the name of the same? 13. Nicole Faria who was crowned 'Miss Earth - 2010' is from
(a) Swasthya Bima Yojana which of the following countries?
(b) Varsha Bima Yojana (a) Brazil (b) India
(c) National Pension Scheme for unorganized sector (c) Britain (d) Italy
(d) Food for work scheme (e) France
(e) None of these 14. Which of the following awards/honours is given to Mr.
6. Which of the following Awards is given to a sports coach? Pranab Mukherjee?
(a) Arjuna Award (a) Parliamentarian of the year
(b) Khel Ratna Puraskar (b) Best Union Minister
(c) Sportsman of the year Award (c) Finance Minister of the year for Asia
(d) Dronacharya Award (d) SAARC Parliamentarian of the year
(e) None of these (e) None of these
7. Who amongst the following has taken over as the Chief 15. Which of the following names is associated with a Health
Minister of Bihar after recent assembly elections there? scheme launched by the Govt. of India?
(a) Lalu Prasad Yadav (b) Ramvilas Paswan (a) VAT (b) MAT
(c) Nitish Kumar (d) Syeed Shahnawaj Hussain (c) CAR (d) AAFI
(e) None of these (e) ASHA
GP_4458
2011-2 SBI Clerk Solved Paper-2011
16. Gagan Narang who won 4 gold medals in Delhi Common - 27. Which of the following trophies/cups is NOT associated
wealth Games is basically a______ with the game of Football?
(a) Weight lifter (b) Badminton player (a) Challenge Cup
(c) Table Tennis player (d) Swimmer (b) Nehru Gold Cup
(e) Air Rifle shooter (c) Carling Cup
17. An individual going to Kuwait will have to make all his/her (d) FIFA World Cup
payments in which of the following currencies? (e) Merdeka Cup
(a) Pula (b) Ruble 28. Shri S M Krishna recently signed an agreement with Dr. Guido
(c) Curo (d) Lari Westerwelle when he came to India in October 2010. This
(e) Dinar means this was an agreement between India and _______
18. World's largest bourse was inaugurated in Mumbai recently. (a) USA (b) Russia
The bourse will deal in which of the following commodities? (c) Germany (d) France
(a) Textiles (b) Gold (e) Brazil
(c) Silver (d) Diamond
29. The first general election was held in India in_______
(e) None of these
(I) 1951-52 (b) 1962
19. HINDALCO is a company operating in the area of_______
(c) 1957 (d) 1947-48
(a) Car & Automobile
(e) 1949
(b) Textiles
(c) Cement manufacturing 30. Which one of the following taxes/cess is levied by States in
(d) Software development India?
(e) Aluminium & Copper rolling (a) Tax on motor vehicles (b) Educational cess
20. Who has written the book "The White Tiger" ? (c) Tax on hotels (d) Tax on wealth
(a) Saul Bellow (b) Amitav Ghosh (e) All of these
(c) Salman Rushdie (d) V. S. Naipaul 31. Who among the following has been declared as Businessman
(e) Aravind Adiga of the decade?
21. Which of the following is a form of irrigation used in India? (a) Mukesh Ambani (b) Anil Ambani
(a) Dip
(c) Amalga Tg:- @NextGenBankers
(b) Booting
(d) Extension Service
(c) Ratan Tata
(e) Aditya Birla
(d) Azim Premji
38. Money laundering refers to _______ hence are arguing. Can you please help us and tell us which of us
(a) Conversion of assets into cash is right"?
(b) Conversion of money which is illegally obtained The zoo keeper laughed before answering, "My dear men,
(c) Conversion of cash into gold each of you has touched just one portion of the animal. The
(d) Conversion of gold into cash animal you see is neither a snake, nor any of the other things you
(e) Money power have mentioned. The animal in front of you is an elephant!" As
39. Who won the Commonwealth Writer' Prize on April 12, 2010? the six men bowed their head, ashamed of the scene they had
(a) Shobha De (b) Tara Joshi created, the zoo keeper said, "My dear men, this is a huge animal,
(c) Arvind Virmani (d) Rana Dasgupta and luckily, it is tame. It stood by calmly as each of you touched it.
(e) Khushwant Singh
You are extremely lucky that it stayed calm even during your
40. Brahmos is a/ an_______
argument, for if it had got angry, it would have trampled all of you
(a) Fighter plane (b) Pattan tank
to death!" He continued further, "It is not enough to gather
(c) Submarine (d) Supersonic missile
knowledge, but it is also important to learn to share and pool your
(e) Helicopter
knowledge. Instead of fighting amongst yourselves, if you had
General English tried to put all your observations together, you might have had an
idea of the animal as a whole! Also, when you cannot see the
DIRECTIONS (41-47) : Read the following passage carefully entire truth, it is better to go to someone who does know the
and answer the questions given below it. Certain words are complete truth, rather than guess about small parts of it. Such
printed in bold to help you locate them while answering some of half- knowledge is not only useless, but also dangerous. If you
the Questions. had come directly to me, I would have helped you identify all the
Once upon a time in a village, there lived six blind men. In spite of animals without putting you in danger!" The six men apologized
their blindness, they had managed to educate themselves. Seeking to the zoo keeper, and assured him that they had learnt their lesson.
to expand their knowledge, they decided to visit a zoo and try out From now on, they would seek true knowledge from qualified
their skills in recognising animals by their touch. The first animal people, and would also try to work together as a team so that they
they came across, as soon as they entered the zoo, was an elephant. could learn more.
As the first man approached the elephant, the elephant waved 41. Which part of the elephant resembled a big fan?
Tg:- @NextGenBankers
its trunk, and the man felt something brush past him. Managing to
hold on to it, he felt it, and found something long and moving. He
(a) The wide ears
(b) The mouth
jumped back in alarm, shouting "Move away! This is a snake!"
(c) The long slender trunk
Meanwhile, the second man had moved closer, and walked right
near its legs. As the man touched the thick, cylindrical–shaped (d) The big wrinkled body
legs, he called out "Do not worry. These are just four trees here. (e) The end of the tail
There is certainly no snake!" The third man was curious hearing 42. Why did the six blind men visit the zoo?
the other two, and moved forward. As he walked towards the (a) They wanted to touch an elephant
elephant, he felt his hand touch one of the tusks. Feeling the (b) They had heard a lot about animals
smooth, sharp ivory tusk, the man cried out "Be careful! There is (c) They wanted to visit the animals in the zoo
a sharp spear here". The fourth man cautiously walked up behind (d) They wished to recognise animals by their touch and
the elephant, and felt its swinging tail. "It's just a rope! There is feel
nothing to be afraid of ! "he said. The fifth man had meanwhile (e) They had never been to a zoo before
reached out and was touching the huge ears of the animal. "I 43. What was the first thing the blind men came across as they
think all of you have lost your sense of touch!" he said. "This is entered the zoo?
nothing but a huge fan!" The sixth man did not want to be left out. (a) A large mud wall (b) The zoo keeper
As he walked towards the elephant, he bumped into its massive
(c) The trees (d) The elephant
body, and he exclaimed, "Hey! This is just a huge mud wall! There
(e) A snake
is no animal at all!" All six of them were convinced that they were
right, and began arguing amongst themselves. 44. Why is it that each of the six blind men had different
Wondering what the commotion was all about, the zoo keeper impressions of the elephant?
arrived at the scene, and was surprised to see six blind men (A) Each of them touched only a portion of the elephant
surrounding an elephant, each of them shouting at the top of (B) Each of the six blind men approached different animals
their voice! "Quiet" he shouted out, and when they had calmed (C) The blind men were touching the surroundings instead
down, he asked, "Why are all of you shouting and arguing in this of the elephant
manner ?"They replied, "Sir, as you can see, we are all blind. We (D) They had never touched an elephant before
came here to expand our knowledge. We sensed an animal here, (a) Only (A) (b) Only (A) and (C)
and tried to get an idea of its appearance by feeling it. However, (c) Only (C) (d) Only (B) and (D)
we are not able to arrive at a consensus over its appearance, and (e) Only (B)
GP_4458
2011-4 SBI Clerk Solved Paper-2011
45. Why were the six men arguing and shouting amongst (b) ashamed of
themselves? (c) pretending to be unaware
(a) Each of them wanted his voice to be heard over and (d) unhappy about
above the others (e) None of these
(b) Each of them thought he was right about the animal 53. I am in touch with the police, and they will be here in ten
(c) There was a lot of noise in the zoo and they couldn't minutes.
hear each other (a) in communication with
(d) They were having an interesting debate (b) in close proximity with
(e) None of these (c) in good terms with
46. What advice did the zoo keeper give to the six blind men? (d) familiar with
(a) That the elephant was tame and obedient (e) None of these
(b) That they were very lucky to have had the opportunity 54. I stumbled upon some interesting old letters in my
to visit the zoo Grandfather's desk.
(c) That it was important to share knowledge and work (a) deliberately went through
together as a team (b) surveyed
(d) That they were not qualified to be knowledgeable (c) tripped over
individuals (d) discovered by chance
(e) That the elephant is made up of different parts (e) None of these
47. Which statement best describes the zoo keeper's behaviour 55. The secretary made an entry of the arrangement.
towards the six blind men? (a) initiated discussion (b) made a record
(a) The zoo keeper insulted the six men (c) brought notice (d) showed approval
(b) The zoo keeper presented himself to be as ignorant as (e) None of these
they were DIRECTIONS (56-60) : Read each sentence to find out whether
(c) The zoo keeper helped them and assisted them further there is any grammatical error in it. The error, if any, will be in
(d) The zoo keeper was indifferent towards the six blind one part of the sentence. The number of that part is the answer. If
men there is no error, the answer is (e) i.e. 'No Error'. (Ignore the
63. The lawyer's ______ led to the resolution of the problem. (a) A (b) D
(a) behaviour (b) fees (c) F (d) B
(c) advice (d) impact (e) E
(e) approval 73. Which of the following should be the FIRST sentence after
64. The government claims that ____ in the telecommunications the rearrangement?
industry will mean lower prices for customers. (a) E (b) B
(a) budget (b) finance (c) D (d) C
(c) installments (d) decrease (e) F
(e) competition 74. Which of the following should be the SIXTH (LAST)
65. Sarah was walking along the street __ she tripped over. sentence after the rearrangement?
(a) when (b) as (a) A (b) D
(c) while (d) then (c) F (d) E
(e) however (e) C
DIRECTIONS (66-70) : In each question below, four words 75. Which of the following should be the FIFTH sentence after
printed in bold type are given. These are numbered (a), (b), (c) the rearrangement?
and (d). One of these words printed in bold may either be wrongly (a) E (b) D
spelt or inappropriate in the context of the sentence. Find out (c) B (d) C
the word that is inappropriate or wrongly spelt, if any. The (e) F
number of that word is your answer. If all the words printed in DIRECTIONS (76-80) : In the following passage, there are
bold are correctly spelt and appropriate in the context of the blanks, each of which has been numbered. These numbers are
sentence then mark (e) i.e. 'All Correct' as your answer. printed below the passage and against each, five words are
66. All the competitors (a)/ completed (b)/ the race (c), with just suggested, one of which fits the blank appropriately. Find out
one acception (d)/. All Correct (e) the appropriate word in each case.
67. Poor posture (a)/ can lead (b)/ to muscular (c)/ problems Day dreaming is often overlooked as a proper dream and (76)
(d)/ in later life. All Correct (e) instead as wandering thoughts. However, the meanings to your
68. The pump (a)/ shut off (b)/ as a result (c)/ of a mechenical
Tg:- @NextGenBankers
(d) failure. All Correct (e)
69. The Principal (a)/ gave a very pompous (b)/ spech (c)/ about
nightly dream symbols are also (77) to your day dreams. The
content in your day dreams are helpful in understanding your
true feelings and will help you in (78) your goals. Day dreaming is
'The portals (d) / of learning'. All Correct (e) the spontaneous imagining or recalling of various images or
70. Copeing (a)/ with her mother's long illness (b) / was a heavy experiences in the past or the future. When you daydream, you
load (c)/ to bear (d)/. All Correct (e) are accessing your right brain, which is the creative and feminine
DIRECTIONS (71-75) : Rearrange the following six sentences/ side of your personality. Worrying about something creates visual
group of sentences (A), (B). (C). (D). (E) and (F) in the proper images in your brain of the worst outcome that you are imagining
sequence to form a meaningful paragraph; then answer the and is a form of daydreaming. By repeating these negative images
questions given below. in your mind, you are more likely to make them happen. So the
next time you start worrying, try to think of a positive outcome.
(A) To his surprise, a little honeybee came before his throne and Positive daydreaming is very healthy and acts as a temporary
said. "Of all the gifts you could give me, only one will do. I'd (79) from the demands of reality. It is also a good way to (80) built
like the power to inflict great pain whenever I choose to." up frustrations without physically acting them out.
(B) I hereby give you a sharp sting. But, I am sure you will use 76. (a) composed (b) determined
this weapon carefully only in times of anger and strife. (c) thought (d) felt
(C) “What an awful wish!” said great Zeus, “But I will grant it”. (e) regarded
(D) And to this day, the little honeybee dies after it stings. 77. (a) duplicated (b) present
(E) One day, Zeus, the King of Mount Olympus, was giving out (c) established (d) applicable
gifts to beasts, birds and insects. (e) depictive
(F) “You will get to use it only once, for using it will cost you 78. (a) thinking (b) holding
your life.” (c) achieving (d) realise
71. Which of the following should be the FOURTH sentence (e) capturing
after the rearrangement? 79. (a) solitude (b) healing
(a) D (b) F (c) gateway (d) passage
(c) B (d) E (e) escape
(e) C 80. (a) adjust (b) confirm
72. Which of the following should be the SECOND sentence (c) capture (d) release
after the rearrangement? (e) demonstrate
GP_4458
2011-6 SBI Clerk Solved Paper-2011
(a) 2556.7 (b) 2456.7 98. If the numerator of a certain fractions increased by 100% and
(c) 2546.7 (d) 2645.7 the denominator is increased by 200%; the new fraction thus
(e) None of these
4
5 1 7 formed is . What is the original fraction ?
89. + + =? 21
8 4 12
2 3
11 13 (a) (b)
(a) 1 (b) 1 7 7
24 24
9 7 2 4
(c) 1 (d) 1 (c) (d)
26 24 5 7
(e) None of these (e) None of these
SBI Clerk Solved Paper-2011 2011-7
99. In how many different ways can the letters of the word 100. 52% students from a college participated in a survey. What
'SIMPLE' be arranged ? is the respective ratio between the number of students who
(a) 520 did not participate in the survey to the number of students
(b) 120 who participated ?
(c) 5040 (a) 11 : 13 (b) 12 : 13
(d) 270 (c) 12: 17 (d) Cannot be determined
(e) None of these (e) None of these
DIRECTIONS (101-105) : Study the following table carefully and answer the questions that follow:
Number of students specialising in different fields from six different colleges. M - Males, F - Females
S pecialisation
Name of
College Political
Economics Marketing HRM S ociology Psychology
S cience
M F M F M F M F M F M F
K 53 45 34 47 18 36 25 45 21 34 24 56
L 62 65 34 14 15 58 18 56 56 65 58 86
M 18 28 67 25 26 85 25 35 64 85 68 46
N 33 24 23 19 45 54 24 34 54 75 57 86
O 28 10 45 25 14 47 48 54 21 36 46 56
P 53 34 34 30 27 20 32 78 21 96 79 35
101. What is the average number of females specialising in HRM (a) ` 4,324.8 (b) ` 4,432.8
from all the colleges together ? (c) ` 4,342.8 (d) ` 4,234.8
(a) 30 (b) 45 (e) None of these
(c) 50
(e) None of these
Tg:- @NextGenBankers
(d) 55 107. The area of a rectangle is twice the area of a triangle. The
perimeter of the rectangle is 58 cm. What is the area of the
102. The total number of males in college L are approximately triangle ?
what percent of total females from the same college ? (a) 106 cm2 (b) 108 cm2
(c) 104 cm 2 (d) Cannot be determined
(a) 10 (b) 30
(c) 50 (d) 110 (e) None of these
(e) 70 108. Yesterday Shweta completed 300 units of work at the rate of
103. What is the respective ratio between the total number of 15 units per minute. Today she completed the same units of
students specialising in Economics from college P and the work but her speed was 40% faster than yesterday. What is
total number of students specialising in Psychology from the approximate difference in the time she took to complete
the same college ? the work yesterday and the time she took today ?
(a) 29 : 31 (b) 25: 34 (a) 16 minutes (b) 26 minutes
(c) 28: 39 (d) 25: 39 (c) 46 minutes (d) 36 minutes
(e) None of these (e) 6 minutes
104. The total number of females specializing in Political Science 109. The average speed of a bus is 8 times the average speed of a
from colleges K, N and P together are approximately what bike. The bike covers a distance of 186 km in 3 hours. How
percent of the males specialising in the same field from the much distance will the bus cover in 10 hours ?
same colleges ? (a) 4069 km (b) 4096 km
(a) 210 (b) 90 (c) 4960 km (d) 4690 km
(c) 190 (d) 150 (e) None of these
(e) 110 110. What is the value of (x) in the following equation ?
105. How many students are there in a college M from all the
specialisations together ? ( x )1.2 28
= 1.8
(a) 574 (b) 576 98 ( x)
(c) 572 (d) 568 (a) 18 (b) 12
(e) None of these
(c) 16 (d) 14
106. How much will be the compound interest to be paid on a
principal amount of ` 53,000 after 2 years at the rate of 4 (e) None of these
p.c.p.a. ?
GP_4458
2011-8 SBI Clerk Solved Paper-2011
Kerala
Maharashtra
28%
Reasoning Ability
15%
121. In a certain language 'to be polite' is coded as 'fa so la', 'she is
Odisha polite' is coded as 'so me pa' and 'to have manners' is coded
12% as 'na la ma'. Which of the following is the code for 'be' in that
language?
Percentage of female students from different
(a) s o (b) fa
states attending a national seminar
(c) la (d) me
Total Students: 3500
(e) na
122. 'Lead' is related to 'Pencil' in the same way as 'Ink' is related to
(a) Bottle (b) Pen
(c) Stamp (d) Pot
(e) Colour
SBI Clerk Solved Paper-2011 2011-9
123. How many meaningful English words can be formed with the (a) 9$7# (b) 59#$
letters ONFT using each letter only once in each word? (c) 9b7$ (d) 7$59
(a) None (b) One (e) $27#
(c) Two (d) Three
DIRECTIONS (131-135) : In each of the questions below are
(e) More than three given three statements followed by two conclusions numbered I
124. If the digits in the number 25673948 are arranged in ascending and II. You have to take the given statements to be true even if
order from left to right, what will be the sum of the digits they seem to be at variance from commonly known facts and then
which are fourth from the right and third from the left in the decide which of the given conclusions logically follows from the
new arrangement? statements disregarding commonly known facts.
(a) 10 (b) 9
Give answer (a) if only conclusion I follows.
(c) 4 (d) 6
Give answer (b) if only conclusion II follows.
(e) 8
Give answer (c) if either conclusion I or conclusion II follows.
125. In a certain code GIVEN is written as MDVJH. How is SHARK
Give answer (d) if neither conclusion I nor conclusion II follows.
written in that code?
Give answer (e) if both conclusions I and II follow.
(a) JSAIT (b) JQAIT
131. Statements:
(c) TIAQJ (d) JQBTI
(e) JQIAT All petals are flowers.
126. If each of the vowels in the word HONESTLY is changed to Some flowers are not petals.
the next letter in the English alphabetical series and each Some petals are colours.
consonant is changed to the previous letter in the English Conclusions:
alphabetical series, and then the alphabets so formed are I. Some flowers are colours.
arranged in alphabetical order from left to right, which of the II. Some flowers are not colours.
following will be fifth from the left of the new arrangement 132. Statements:
thus formed? All desks are tables.
(a) S
(c) M
(e) P
Tg:- @NextGenBankers
(b) R
(d) F
Some tables are drawers.
Some drawers are big.
Conclusions:
127. If it is possible to make only one meaningful word with the I. Some tables are big.
first, second, fifth and sixth letters of the word PYGMALION, II. No desk is a drawer.
which of the following would be the second letter of that 133. Statements:
word from the right end? If no such word can be made, give All colleges are buildings.
'X' as your answer and if more than one such word can be
All buildings are concrete.
formed, give your answer as' Z '.
Some concrete are strong.
(a) X (b) P
Conclusions :
(c) Y (d) A
I. Some colleges are strong.
(e) Z
II. At least some strong are concrete.
128. Four of the following five are alike in a certain way and so
134. Statements:
form a group, which is the one that does not belong to that
group? Some trees are tall.
(a) Feathers (b) Leaves All tall are healthy.
(c) Twigs (d) Nest All healthy are not tall.
(e) Hay Conclusions:
129. How many such pairs of letters are there in the word I. Some healthy are tall.
DUPLICATE each of which has as many letters between II. Some trees are not tall.
them in the word (in both forward and backward directions) 135. Statements:
as they have between them in the English alphabetical order? All books are interesting.
(a) None (b) One All magazines are books.
(c) Two (d) Three Some interesting that are not books are journals.
(e) More than three Conclusions:
130. In a certain language ‘GUST’ is coded as ‘@7$2’ and ‘SNIP’ I. All books are journals.
is coded as ‘957#’ and ‘GAPE’ is coded as ‘b$35’. How will II. All magazines are interesting.
‘SING’ be coded in the same code?
GP_4458
2011-10 SBI Clerk Solved Paper-2011
DIRECTIONS (136-140) : Read the following information 144. If all the digits in each of the numbers are arranged in
carefully and answer the questions which follow: descending order within the number, which of the following
will form the highest number in the new arrangement of
If 'A * Z' means 'A is the wife of Z·. numbers?
If 'A ´ Z' means 'A is the husband of Z·. (a) 612 (b) 589
If 'A + Z' means 'A is the sister of Z' . (c) 743 (d) 468
If 'A – Z' means 'A is the brother of Z'.
(e) 297
If 'A > Z' means 'A is the son of Z'.
If 'A < Z' means 'A is the daughter of Z'. 145. If all the numbers are arranged in ascending order from left to
136. Which of the following relations will not be true if the right, which of the following will be the sum of all the three
expression 'A < P ´ T + F > L ´ M' is definitely true? digits of the number which is second from the right of the
(a) A is the daughter of T new arrangement thus formed?
(b) F is the son of M (a) 14 (b) 9
(c) P is the son-in-law of L (c) 18 (d) 16
(d) A is the cousin of F (e) 12
(e) M is the grandmother of A
137. Which of the following means N is the daughter-in-law of A? DIRECTIONS (146-150): Study the following information
(a) M + N * P > A (b) N < M ´ P + A carefully and answer the given questions.
(c) M – N ´ P < A (d) A< P + N ´ M Six friends L, M, N, O, P and Q work in three different cities namely
(e) A< N < P * M Chennai. Pune and Nasik (not more than two work in a single
138. How is P related to F if 'Q ´ P < B + F'? city). Each of them has a different profession, viz." banker, software
(a) Daughter (b) Niece engineer, lawyer, lecturer, doctor and psychologist, but not
(c) Daughter-in-law (d) Grand daughter necessarily in the same order.
(e) Aunt O works in Chennai and is not a lecturer. M is a banker by
139. Which of the following means P is the father of R ? profession and works in Pune with only Q, who is a software
(a) R > S < P * J engineer by profession. N works in Nasik and is not a lawyer by
(b) J + R – S < P profession. P is a doctor and does not work in Chennai. The only
(c) R > S * P– J
(d) S + J ´ R < P Tg:- @NextGenBankers other person who works in Chennai is a lecturer by profession.
146. Which of the following is true for L?
(e) None of these
(a) L is a lecturer by profession.
140. How is M related to B if 'A * B > Z ´ S + M'?
(a) Aunt (b) Grandfather (b) L works in the same place as N.
(c) Uncle (d) Cousin (c) L is neither a lecturer nor a lawyer by profession.
(e) Cannot be determined (d) L works in Pune.
(e) None of these
DIRECTIONS (141-145) : The following questions are based on
147. Which two people work in Nasik?
the five three digit numbers given below:
(a) L and N (b) N and P
612 589 743 468 297 (c) L and Q (d) N and O
141. If two is added to the first digit of each of the numbers, how (e) Cannot be determined
many numbers thus formed will be completely divisible by 148. Which of the following combinations of person, place and
three? profession is correct?
(a) None (b) One
(a) Q – Nasik – Psychologist
(c) Two (d) Three
(b) P – Pune – Doctor
(e) Four
142. If the position of the second and the third digits of each of (c) L – Nasik – Lecturer
the numbers are interchanged, in how many numbers thus (d) N – Chennai – Software engineer
formed will the last digit be a perfect square? ('1' is also a (e) O – Chennai – Lawyer
perfect square) 149. Which of the following is the occupation of N?
(a) One (b) Two (a) Software engineer (b) Psychologist
(c) Three (d) Four (c) Lecturer (d) Lawyer
(e) Five (e) None of these
143. What will be the resultant if the third digit of the second 150. Who among the following five is a lawyer by profession
lowest number is divided by the second digit of the highest among the group of friends?
number?
(a) O (b) L
(a) 4 (b) 1
(c) N (d) Q
(c) 6 (d) 5
(e) 2 (e) None of these
SBI Clerk Solved Paper-2011 2011-11
DIRECTIONS (151-160) : In each of the questions given below which one of the five answer figures on the right should come after
the problem figures on the left, if the sequence were continued?
Problem Figures Answer Figures
151. T P U U P C U C P T U T U T U
C Z U C C Z Z C T Z C T T Z Z C Z Z C
P U T Z T P T P U Z P U P P C P
(a) (b) (c) (d) (e)
152. U Z U Z U U Z U Z U
T P Z P
C R R P P P R
C CR P R P R R R R
(a) (b) (c) (d) (e)
153.
C C C C
154.
C C C C C C
(a) (b) (c) (d) (e)
Tg:- @NextGenBankers
D
D
155.
C U U U U
156. C C C C C
C
(a) (b) (c) (d) (e)
157. T C
T C
U
(a) (b) (c) (d) (e)
158. T O TU C P
U O C U PC Z P Z Z Z
Z U O
C Z O OZ Z T P P O P T O
ZC P T Z U T
O
U U O
U
C
U
C C
O P P T P C C T T T U
(a) (b) (c) (d) (e)
159. O U C O O O U O O
R U R CC U R C U R C U C C R R
C U O R O R U O R O R C U C U
(a) (b) (c) (d) (e)
160.
CUTE CUT TOC T OA AT O TAZ O T A O TZ OT Z O TZ
Section - V : Marketing Aptitude/ 171. Editing a document consists of reading through the document
you've created, then
Computer Knowledge (a) correcting your errors
(b) printing it
161. Which of the following groups consist of only input devices? (c) saving it
(a) Mouse. Keyboard. Monitor (d) deleting it
(b) Mouse. Keyboard. Printer (e) None of these
(c) Mouse. Keyboard. Plotter 172. The Internet allows you to
(a) send electronic mail
(d) Mouse. Keyboard. Scanner (b) view web pages
(e) None of these (c) connect to servers all around the world
162. A menu contains a list of (d) All of these
(a) commands (b) data (e) None of these
173. Most mail programs automatically complete the following
(c) objects (d) reports
two parts in an e-mail
(e) None of these (a) From : and Body : (b) From : and Date :
163. What is output? (c) From : and To : (d) From : and Subject :
(a) What the processor takes from the user (e) None of these
(b) What the user gives to the processor 174. Where is the disk put in a computer?
(c) What the processor gets from the user (a) in the modem (b) in the hard drive
(d) What the processor gives to the user (c) into the CPU (d) in the disk drive
(e) None of these (e) None of these
164. An error is also known as 175. The name a user assigns to a document is called a(n)
(a) bug (b) debug (a) filename (b) program
(c) cursor (d) icon (c) record (d) data
(e) None of these (e) None of these
165. Computer_________ is whatever is typed, submitted, or 176. An e-mail address typically consists of a user ID followed
transmitted to a computer system. by the_________ sign and the name of the e-mail server
(a) input
(c) data
Tg:- @NextGenBankers
(b) output
(d) circuitry
that manages the user's electronic post office box.
(a) @ (b) #
(e) None of these (c) & (d) «
166. Which process checks to ensure the components of the (e) None of these
computer are operating and connected properly? 177. A personal computer is designed to meet the computing
needs of a(n)
(a) Booting (b) Processing
(a) individual (b) department
(c) Saving (d) Editing (c) company (d) city
(e) None of these (e) None of these
167. Unsolicited commercial email is commonly known as 178. Ctrl, shift and alt are called _________ keys.
(a) spam (b) junk (a) adjustment (b) function
(c) hoaxes (d) hypertext (c) modifier (d) alphanumeric
(e) None of these
(e) None of these
179. File _________ shrinks the size of a file so it requires less
168 _________ is processed by the computer into information. storage space.
(a) numbers (b) processor (a) scanning (b) synthesizing
(c) input (d) data (c) defragmenting (d) compression
(e) None of these (e) None of these
169. A web site address is a unique name that identifies a specific 180. In addition to the basic typing keys, desktop and notebook
computer keyboards include a(n) _________ keypad to
_________ on the web.
efficiently move the screen-based insertion point.
(a) web browser (b) web site (a) editing (b) number
(c) PDA (d) link (c) locked (d) docked
(e) None of these (e) None of these
170. A device that connects to a network without the use of cables 181. Safe Deposit Locker can be canvassed among
is said to be (a) all existing account holders
(a) distributed (b) centralised (b) persons below poverty line
(c) students
(c) open source (d) wireless (d) jewelers
(e) None of these (e) ATM cardholders
SBI Clerk Solved Paper-2011 2011-13
ANSW ER KEY
1 (b) 21 (a) 41 (a) 61 (e) 81 (a) 101 (c) 121 (b) 141 (b) 161 (d) 181 (a)
2 (e) 22 (a) 42 (d) 62 (b) 82 (d) 102 (e) 122 (b) 142 (c) 162 (a) 182 (d)
3 (c) 23 (e) 43 (d) 63 (c) 83 (c) 103 (e) 123 (b) 143 (e) 163 (d) 183 (a)
4 (b) 24 (d) 44 (a) 64 (e) 84 (e) 104 (e) 124 (a) 144 (b) 164 (a) 184 (c)
5 (b) 25 (b) 45 (b) 65 (a) 85 (b) 105 (c) 125 (b) 145 (b) 165 (a) 185 (e)
6 (d) 26 (c) 46 (c) 66 (d) 86 (d) 106 (a) 126 (e) 146 (a) 166 (a) 186 (c)
7 (c) 27 (a) 47 (c) 67 (e) 87 (c) 107 (d) 127 (d) 147 (b) 167 (a) 187 (d)
8 (c) 28 (c) 48 (a) 68 (d) 88 (c) 108 (e) 128 (a) 148 (e) 168 (d) 188 (d)
9 (d) 29 (a) 49 (b) 69 (c) 89 (a) 109 (c) 129 (b) 149 (b) 169 (b) 189 (e)
10 (b) 30 (b) 50 (b) 70 (a) 90 (d) 110 (d) 130 (a) 150 (a) 170 (d) 190 (a)
11 (d) 31 (c) 51 (d) 71 (c) 91 (c) 111 (a) 131 (a) 151 (e) 171 (a) 191 (d)
12 (a) 32 (a) 52 (a) 72 (a) 92 (a) 112 (e) 132 (d) 152 (c) 172 (d) 192 (c)
13 (b) 33 (b) 53 (a) 73 (a) 93 (d) 113 (a) 133 (b) 153 (b) 173 (b) 193 (a)
14 (c) 34 (c) 54 (d) 74 (b) 94 (b) 114 (d) 134 (a) 154 (a) 174 (d) 194 (b)
15 (e) 35 (a) 55 (b) 75 (e) 95 (a) 115 (b) 135 (b) 155 (a) 175 (a) 195 (e)
16 (e) 36 (b) 56 (c) 76 (e) 96 (c) 116 (b) 136 (d) 156 (b) 176 (a) 196 (c)
17 (e) 37 (c) 57 (d) 77 (d) 97 (c) 117 (d) 137 (a) 157 (d) 177 (a) 197 (d)
18
19
(d)
(e)
38
39
(b)
(d)
Tg:- @NextGenBankers
58
59
(a)
(a)
78
79
(c)
(e)
98
99
(a)
(e)
118
119
(c)
(a)
138
139
(b)
(c)
158
159
(d)
(e)
178
179
(c)
(d)
198
199
(b)
(a)
20 (e) 40 (d) 60 (b) 80 (d) 100 (b) 120 (e) 140 (e) 160 (c) 180 (b) 200 (a)
57. (d) Here, Past Simple i.e. the educated class did not support 85. (b) ? = 3
21952 + 33
him .... should be used as the sentence shows past
time. = 3
28 ´ 28 ´ 28 + 33
58. (a) Here, Depletion (Noun) of the Ozone layer .... should
= 28 + 33 = 61
be used.
59. (a) Here, Most of the people who should be used. Who is 5600 ´ ? 3500 ´ 28
86. (d) - = 1988
used to show which person or people you mean. 100 100
Look at the sentence: Þ 56 × ? – 980 = 1988
The people who called yesterday want to buy the
Þ 56 × ? = 1988 + 980 = 2968
house.
60. (b) Here, in reducing human suffering .... should be used. 2968
\ ?= = 53
61. (e) used to 56
62. (b) I know
63. (c) advice 3 1
87. (c) 32000 ´ ´ ? ´ = 4800
64. (e) competition 4 2
65. (a) when Þ 12000 × ? = 4800
66. (d) The correct spelling is : exception.
4800 2
67. (e) All correct Þ ?= =
68. (d) The correct spelling is : mechanical. 12000 5
69. (c) The correct spelling is : speech. 88. (c) ? = ( 8536 - 2209 ) ´ 0.3
70. (a) The correct spelling is : coping.
= (8536 – 47) × 0.3
71. (c) B
= 8489 × 0.3 = 2546.7
72. (a) A
5 1 7
73. (a) E + +
74. (b) D
Tg:- @NextGenBankers 89. (a) ? =
8 4 12
15 + 6 + 14 35 11
75. (e) F = = =1
24 24 24
76. (e) regarded
3584
77. (d) applicable 90. (d) ?= - 11
32
78. (c) achieving = 112 – 11 = 101
79. (e) escape \ ? = 101 × 101 = 10201
80. (d) release 91. (c) The pattern of the number series is :
4 + 1 × 15 = 19
81. (a) ? = 6389 – 1212 – 2828 = 2349
19 + 2 × 15 = 49
31 86 41 82 49 + 3 × 15 = 94
82. (d) ? = ´ ´ = 94 + 4 × 15 = 154
43 95 93 285
83. (e) 526 × 12 + 188 = 50 × ? 154 + 5 × 15 = 229
Þ 6312 + 188 = 50 × ? 92. (a) The pattern of the number series is :
Þ 6500 = 50 × ? 1 1
+ =1
6500 2 2
\ ?= = 130
50 1 1
1+ =1
84. (e) ( 62 + 32) (62 – 32) + ? 2 = 3144 2 2
Þ 94 × 30 + ?2 = 3144
1 1
Þ 2820 + ?2 = 3144 1 + =2
2 2
Þ ?2 = 3144 – 2820 = 324 1 1
\ ?=3+ = 3
\ ?= 324 = 18 2 2
GP_4458
2011-16 SBI Clerk Solved Paper-2011
93. (d) The pattern of the number series is : 99. (e) The word SIMPLE consists of 6 distinct letters
101 + 2 = 103 \ Number of arrangements = 6!
103 – 4 = 99 = 6 × 5 × 4 × 3 × 2 × 1 = 720
99 + 6 = 105 100. (b) Required ratio = 48 : 52
105 – 8 = 97 = 12 : 13
101. (c) Average number of females specializing in HRM
97 + 10 = 107 36 + 58 + 85 + 54 + 47 + 20 300
= = = 50
94. (b) The pattern of the number series is : 6 6
102. (e) Total number of males in college L
219 – 3 = 216 = 63 = 62 + 34 + 15 + 18 + 56 + 58 = 243
344 – 219 = 125 = 53 Total number of females in college L
408 – 344 = 64 = 43 = 65 + 14 + 58 + 56 + 65 + 86 = 344
\ ? = 408 + 33 243
\ Required percentage = ´ 100 » 70
344
= 408 + 27 435
103. (e) Required ratio
95. (a) The pattern of the number series is :
= (53 + 34) : (21 + 96)
7 + 3 = 10
= 87 : 117 = 29 : 39
10 + 12 (= 2 × 3) = 16
104. (e) Total number of females specialising in Political Science
16 + 12 (= 2 × 6) = 28
from colleges K, N and P together
28 + 24 (= 2 × 12) = 52 = 56 + 86 + 35 = 177
52 + 48 (= 2 × 24 ) = 100 Number of males in these colleges
x é 676 ù
= 53000 ê -1
98. (a) Let the original fraction be = . ë 625 úû
y
53000 ´ 51
x ´ 200 4 = = ` 4324.8
\ = 625
y ´ 300 21
x 4 3 2
Þ = ´ =
y 21 2 7
SBI Clerk Solved Paper-2011 2011-17
Tg:- @NextGenBankers
\ Speed of bus = 8 × 62
= 496 kmph
she is polite
to have manners
so me pa
na la ma
Distance covered by bus in 10 hours
122. (b) Lead is used in pencil for writing. Similarly, ink is used
= 496 × 10 = 4960 km
in pen to write something.
110. (d) (x)1.2 × (x)1.8 = 28 × 98 123. (b) Meaningful Word Þ FONT
Þ (x)1.2 + 1.8 = 28 × 98 124. (a) 2 5 6 7 3 9 4 8
Þ (x)3 = 2 × 2 × 7 × 7 × 7 × 2 = 23 × 73
2 3 4 5 6 7 8 9
\ x = 3 23 ´ 73 = 2× 7 = 14 = 6 + 4 = 10
111. (a) ? » 11 because 11 × 11 × 11
125. (b) G I V E N M D V J H
= 1331 and 10 × 10 × 10 = 1000
–1
112. (e) ? » 508 + 253 + 200 » 961
–1
\ Required answer = 960
±0
7231
113. (a) ? = × 1.7 » 585 +1
21
+1
114. (d) ? = 9355 » 95
115. (b) ? = 170 × 14 ÷ 181 Similarly,
170 ´ 14 S H A R K J Q A I T
= » 13 –1
181
116. (b) Required ratio = 10 : 25 = 2 : 5 –1
117. (d) Total number of students from Maharashtra and ±0
Madhya Pradesh +1
8000 ´ (28 + 14) +1
=
100
Number of females from these states
GP_4458
2011-18 SBI Clerk Solved Paper-2011
G P M F R S K X
All tall are healthy.
FGKM P RSX I + A Þ I-type of Conclusion
“Some trees are healthy.”
127. (d) 1 2 3 4 5 6 7 8 9 Conclusion I is Converse of the second Premise.
P Y G M A L I O N 135. (b) All magazines are books.
Meaningful Word Þ PL A Y
128. (a) Feathers are found in birds. Leaves, twigs, nest and All books are interesting.
hay are related to vegetation. A + A Þ A-type of Conclusion
“All magazines are interesting.”
129. (b) 4 21 16 12 9 3 1 20 5 This is Conclusion II.
D U P L I C A T E 136. (d) A < P Þ A is the daughter of P.
P × T Þ P is the husband of T.
130. (a) G U S T @ 7 $ 2
# T + F Þ T is the sister of F.
S N I P 9 5 7
F > L Þ F is the son of L.
b $ 3 5
G A P E L × M Þ L is the husband of M.
Tg:- @NextGenBankers
Therefore, S I N G Þ 7 9 # $ A is the daughter of P and T.
F is the son of the L and M.
(131-135):
T is the daughter of L and M.
(i) All petals are flowers Þ Universal Affirmative
(A -type) Therefore, P is son-in law of L.
(ii) Some petals are colours Þ Particular Affirmative A is the niece of F.
(I-type) M is grandmother of A.
(iii) No desk is a drawer ÞUniversal Negative (E-type)
137. (a) M + N Þ M is the sister of Z.
(iv) Some desks are not drawers ÞParticular Negative
(O-type) N * P Þ N is the wife of P.
131. (a) Some colours are petals. P > A Þ P is the son of A.
N is the daughter in-law of A.
943 668
= 314.33; = 222.66;
3 3
497
= 165.66
3
142. (c) 612 Þ 621; 589 Þ 598;
152. (c) In each subsequent figure all the designs move in
743 Þ 734 ; 468 Þ 486;
297 Þ 279 Tg:- @NextGenBankers
6 2 1; 7 3 4 : 2 7 9
clockwise direction and in the first step the last design
is replaced with a new design and in the second step
the first design is replaced with a new design. Similarly,
the second last design and second design are replaced
143. (e) Second lowest number Þ 46 8 with new designs alternately.
153. (b) In each subsequent figure the cross sign and square
Highest number Þ 7 4 3 move in clockwise direction.
154. (a) In the subsequent figures the design (T) rotates
8
= =2 respectively 45°, 90°, 135°, 180°, 225° ...........,
4
anti-clockwise and the arrow reverses its direction in
144. (b) 612 Þ 621; 589 Þ 985;
each subsequent figure. The design (C) moves
743 Þ 743; 468 Þ 864; respectively one step clockwise, one step anti-
297 Þ 972 clockwise for three figures and again one step
Highest number Þ 985 Þ 589 clockwise.
145. (b) 297 < 468 < 589 < 612 < 743 155. (a) In each subsequent figure the dot moves in clockwise
direction. one new design appears at the lower left
6+ 1+2=9
(146-150): corner and in the next figure it rotates through 90°
anticlockwise. In the first step the second, the third
Friend City Profess ion and the fourth angles are inverted while in the second
L Chennai Lecturer step the first and the last angles are inverted.
M Pune Banker 156. (b) From Problem Figure (1) to (2) the design at upper left
N Nas hik Ps ychologis t corner is inverted and the design at upper right corner
O Chennai Lawyer is replaced with a new design while the middle and
P Nas hik Doctor lower left corner designs interchange positions. Similar
Q Pune Software
changes occur from Problem Figure (3) to (4) and from
Engineer
Problem Figure (5) to Answer Figure.
GP_4458
2011-20 SBI Clerk Solved Paper-2011
N
(3) to (4) N
N N
N
SBI Clerk Solved Paper-2009
Time : 135 minutes Max. Marks : 200
3.
(e) None of these Tg:- @NextGenBankers
Which of the following States introduced a high tech food 9.
(e) Jammu and Kashmir
Who amongst the following made the 41st test century of
grain rationing system to ensure timely supply of the his career recently?
foodgrains to people living below proverty line? (a) Saurav Ganguly (b) V.V.S. Laxman
(a) Maharashtra (b) Tamil Nadu (c) Rahul Dravid (d) Sachin Tendulkar
(c) Delhi (d) West Bengal (e) None of these
(e) Karnataka 10. Stephen Harper whose name was in news recently is the
4. Which of the following countries adopted its new (a) President of Canada
constitution recently? (b) Prime Minister of Canada
(c) President of Italy
(a) Bangladesh (b) Mexico
(d) Prime Minister of Switzerland
(c) South Africa (d) North Korea
(e) Foreign Minister of U.S.A.
(e) None of these
11. Barack Hussein Obama belongs to which of the following
5. Which of the following is/are NOT the feature(s) of India’s political parties?
Foreign Trade Policy (2004 to 2009) ? (a) Republican
A. To double India’s percentage share of global trade (b) Democratic
from present 0.7 per cent to 1.5 per cent by 2009 (c) Labour
B. Simplifying the procedures and bringing down the cost (d) American National Congress
C. Make SAARC countries India’s most preferred foreign (e) None of these
trade partners by 2009 12. Which of the following awards was given to Pt. Bhimsen
(a) Only (A) (b) Only (B) Joshi recently (in 2008)?
(c) Only (C) (d) All (A), (B) and (C) (a) Padma Bhushan
(b) Sangit Natak Academy Award
(e) Only (A) and (C) (c) Maharashtra Bhushan
6. Which of the following States will soon have its first Civil (d) Karnataka Ratna
Airports? (e) Bharat Ratna
GP_4458
2009-2 SBI Clerk Solved Paper-2009
13. Which of the following State Government has announced 22. Which of the following is/are the measure(s) taken by the
that it will provide a special package of incentives to Reserve Bank of India (RBI) to ease the liquidity crunch in
Employment Intensive Industries? the country?
(a) Punjab (b) Karnataka A. Cut in Cash Reserve Ratio and Statutory Liquidity
(c) Tamil Nadu (d) Uttar Pradesh Ratio.
(e) Bihar B. Increase the flow of foreign direct investment
14. Who amongst the following is the author of the book “The C. Supply of additional currency notes in the market.
exile”? (a) Only (A) (b) Only (B)
(a) B. G. Verghese (b) Philip Roth (c) Only (C) (d) All (A), (B) and (C)
(c) Aravind Adiga (d) Navtej Sarna (e) None of these
(e) None of these 23. Who amongst the following is the winner of the Nobel Prize
15. Which of the following nations has decided to increase its for Literature in 2008?
cooperation in the field of energy with India? (a) Jean Marie Gustave Le Clezio
(a) Russia (b) China (b) Martti Ahtisaari
(c) Bangladesh (d) Nepal (c) Yoichiro Nambu
(e) Japan (d) Francoise Barre Sinoussi
16. Which of the following is NOT a fertiliser product? (e) None of these
(a) Urea
24. A national level commission of India has asked about twenty
(b) Murate of Potash States to setup which of the following commissions in their
(c) Di Amonium Phosphate States as early as possible?
(d) Calcium Carbonate (a) Farmers Commission
(e) All are fertilisers (b) Women Commission
17. Which of the following programme has been implemented (c) Child Welfare Commission
in all the districts of the country?
Tg:- @NextGenBankers
(a) Sarva Siksha Abhiyan
(b) Navodaya Vidhyalay
(d) Law Commission
(e) None of these
25. Which of the following States has decided to setup Arsenic
(c) Ultra Mega Power Project
Removal Plants in all the districts of the State to enable
(d) Rajiv Gandhi Grameen Vidyuthikaran Yojana itself to provide arsenic free drinking water to all the people
(e) National Rural Employment Guarantee Programme by 2010-11?
18. The third India, Brazil and South Africa (IBSA) summit took (a) Karnataka (b) Kerala
place in which of the following cities recently? (c) West Bengal (d) Orissa
(a) Brasilia (b) New Delhi
(e) Maharashtra
(c) Sun City (d) Salvador
26. Which of the following services is NOT provided by the
(e) Durban post offices of India?
19. A Rs. 35,000 crore JSW Steel Plant project was inaugurated (a) Saving Bank Scheme
in West Bengal recently. The project is setup/developed in
(b) Retailing of Mutual Funds
(a) Singur (b) Nandigram
(c) Sale of Stamp Papers (Judicial)
(c) Salboni (d) Malda
(d) Issuance of Demand Drafts
(e) Joynagar
(e) Life Insurance Cover
20. The Government of India decided to provide a bonus price
of Rs. 50 per quintal on which of the following products 27. Which of the following diseases is not covered under
over and above its minimum support price of Rs. 850 per Integrated Disease Surveillance project?
quintal? (a) Cholera (b) Tuberculosis
(a) Wheat (b) Paddy (c) AIDS (d) Polio
(c) Sugarcane (d) Cotton (e) Cancer
(e) None of these 28. Tzipi Livini whose name was in news recently is from which
21. Mohammed Anni Nasheed is the newly elected President of of the following countries?
(a) Fiji (b) Maldives (a) Singapore (b) South Korea
(c) Iraq (d) Iran (c) Austria (d) Israel
(e) None of these (e) None of these
SBI Clerk Solved Paper-2009 2009-3
29. Who amongst the following is the author of the book “The 38. World Ozone day is observed on
Name sake”? (a) 16th September (b) 16th October
(a) Vikram Seth (b) V.S. Naipaul (c) 16th November (d) 26th September
(c) Arun Bhagat (d) Anita Desai (e) 26th October
(e) Jhumpa Lahiri 39. Which of the following is NOT a Government sponsored
30. The National Games scheduled in 2011 will be organised in organisation?
(a) Bangaluru (b) Goa (a) Small Industries Development Bank of India
(c) Kolkata (d) Kochi (b) NABARD
(e) Jaipur (c) National Housing Bank
31. Alena D’mintiva of Russia won which of the following titles (d) ICICI Bank
of tennis after defeating Dinara Safina? (e) All are government sponsored
(a) China Open 40. Which of the following state governments, in January 2009,
(b) Japan Open created a new record in the country’s power sector by
(c) New Zealand Open signing three separate memorandums of understanding on
(d) Sydney International a single day?
(e) None of these (a) Himachal Pradesh (b) Assam
32. Who amongst the following is the Minister of Home Affairs (c) Karnataka (d) Andhra Pradesh
in Union Cabinet of India at present? (e) None of these
(a) Shivraj Patil (b) Lalu Prasad Yadav
(c) P. Chidambaram (d) Sharad Pawar General English
(e) None of these
33. Dronacharya Award is given for excellence in DIRECTIONS (Qs. 41-52): Read the following passage carefully
and answer the questions given below it. Certain words have
(a) Literacy work (b) Social service
been printed in bold to help you locate them while answering
She would take the banana crop to market and get a good price. (a) only (B) (b) only (A)
Over the years the plantation grew and finally after six years (c) Both (A) and (B) (d) All (A), (B) and (C)
Govind had five litres of dew. He went to the sage who smiled, (e) None of these
uttered a mantra and sprinkled a few drops of dew on a copper 45. Why did Govind decide to cultivate a banana crop?
vessel. To Govind’s dismay, nothing happened. “you have cheated
(a) The soil of his land was suitable only for cultivating
me!” he shouted at the sage.
bananas
The sage however smiled. Govind’s wife then came forward
(b) It was the most highly priced commodity in the region
with a box. The sage opened it and revealed stacks of gold coins
inside. Turning to Govind he said, “you worked hard on your (c) It could be grown at any time of the year including
land and created a plantation. Your wife sold ‘the produce in the winter
market. It was your hard work which created this wealth, not (d) His wife pressurised him to do so
magic. If I had told you this earlier, you would not have listened.” (e) The ingredient for the magic potion could only be
Govind understood the wisdom behind the sage’s words and obtained from a banana tree
worked even harder from that day on. 46. What made Govind angry with the sage?
41. Why did Govind’s father give him large plots of land? (a) The sage had conspired with Govind’s wife against
(a) It was his way of instilling a sense of responsibility in him
his son (b) He had forgotten the magic spell and all Govind’s hard
(b) Govind was his only son and sole heir work was in vain
(c) To provide Govind with sufficient funds to pursue his (c) He had lost a good deal of money in cultivating
interest of discovering a magic potion bananas
(d) He wanted Govind to continue to look after the tenants (d) The sage had made a fool of him in front of other
(e) None of these villagers
42. Which of the following can be said about the sage? (e) None of these
(a) He was cunning and plotted with Govind’s wife to 47. How did Govind acquire his dream of gold finally ?
cheat him.
Tg:- @NextGenBankers
(b) He had no magical powers as such and used to swindle
(a) The sage gave him gold as a reward for his hard work
was in vain
people (b) His wife diligently saved the gold he had received from
(c) He was a good judge of people his father
(c) By selling the banana plantation
(d) He did not deserve his good reputation (d) His wife sold bananas at a higher price than others did
(e) He was dishonest because he had cheated Govind out (e) None of these
of his gold 48. Which of the following is TRUE in the context of the
43. Why was Govind’s wife worried ? passage?
A. Govind was easily fooled by people
(a) Govind had no knowledge of farming and could not
B. Govind was preserving by nature
cultivate the land he had inherited from his father
C. The sage had never actually been to the Himalayas
(b) Govind had not friends because he was obsessed with
(a) Only (A) (b) Both (A) and (B)
finding a potion which would turn any thing into gold
(c) Govind was only interested in studying under different (c) Only (C) (d) Both (A) and (C)
sages and neglected his family duties (e) None of these
DIRECTIONS (Qs. 51-52) : Choose the word which is most opposite DIRECTIONS (Qs. 58-62) : In each question below a sentence
in meaning to the word printed in bold as used in the passage. with four words printed in bold type is given. These are numbered
as (a), (b), (c) and (d). One of these four words printed in bold
51. DISMAY
may be either wrongly spelt or inappropriate in the context of
(a) joy (b) interest
the sentence. Find out the word which is wrongly spelt or
(c) desire (d) humour
inappropriate if any. The number of that word is your answer. If
(e) luck all the words printed in bold are correctly spelt and also
52. TENDED appropriate in the context of the sentence, mark (e) i.e. ‘All
(a) negligible (b) watched correct’ as your answer.
(c) inclined (d) ignored
58. Under existing (a) / regulations we are not permitted (b) / to
(e) spoil
owe (c) / more than a forty percent share (d) / of the family
DIRECTIONS (Qs. 53-57) : Which of the following phrases (a), business. All correct (e).
(b), (c) and (d) given below each sentences should replace the 59. In case of any land dispute (a) / panchayat officials (b) / will
phrase printed in bold in the sentence to make it grammatically determine (c) / how the property is to be dividend (d) / All
correct? If the sentence is correct as it is given and ‘No correction correct (e).
is required’, mark (e) as the answer. 60. The World Bank has consented (a) / to sanction (b) / the
53. Starting out my own business at this time would affect the necessary (c) / finance (d) / for the project. All correct (e).
financial stability of my family. 61. To obtain (a) / a refund you will have to fill (b) / a claim (c)
(a) Starting up my (b) For starting with / with the appropriate (d) / authority. All correct (e).
(c) To start out mine (d) By starting my 62. Experts predict (a) / there will be shortage (b) / of investment
(e) No correction required (c) / in the infrastructure (d) / sector. All correct (c).
54. Use a tactic for mixing the inferior with good quality rice is DIRECTIONS (Qs. 63-67) : Rearrange the following six
dishonest and you will lose your license. sentences (A), (B), (C), (D), (E) and (F) in the proper sequence
(a) Using tacti as
(c) To use tactics
Tg:- @NextGenBankers
(b) Using a tactic like
(d) Used to tactics like
to form a meaningful paragraph; then answer the questions
given below them.
(e) No correction required A. The hall was filled with children, teachers, students, family
55. The company will invest more six hundred crores in the members and those who were close to him.
next five years to expand its operations in Britain. B. Normally such ceremonies are attended by important people
(a) will future invest like industrialists, politicians and VIP’s.
(b) has invested more than C. What I saw when I stepped into the hall amazed me.
(c) have invested over D. I went home with the feeling that it was a most unusual oath
(d) will be invested above taking ceremony with only those who were ‘important’ to
(e) No correction required him present.
56. Several of our projects have delayed because the equipment E. When he was elected President, he invited me to the swearing
we ordered was delivered late. in ceremony in the Central Hall of Parliament.
(a) have been delayed when F. However in this case everyone who attended the ceremony
(b) delayed because of seemed to know him personally.
(c) are delayed since 63. Which of the following should be the FIRST sentence after
(d) were delayed with rearrangement ?
(e) No correction required (a) A (b) B
57. The committee has ruled out the possible raising taxes for (c) C (d) D
this financial year. (e) E
(a) possibly raised 64. Which of the following should be the SECOND sentence
(b) possible rise of after rearrangement ?
(c) possibility to raise (a) B (b) C
(d) possibility of raising (c) D (d) E
(e) No correction required (e) F
GP_4458
2009-6 SBI Clerk Solved Paper-2009
65. Which of the following should be the THIRD sentence after 73. (a) firstly (b) freshly
rearrangement ? (c) foremost (d) initially
(a) A (b) B (e) recently
(c) C (d) D 74. (a) copied (b) observed
(e) E (c) learned (d) understood
66. Which of the following should be the FIFTH sentence after (e) improving
rearrangement ? 75. (a) asked (b) insisted
(a) B (b) C (c) demanded (d) settled
(c) D (d) E (e) lend
(e) F 76. (a) severe (b) no
67. Which of the following should be the LAST (SIXTH) (c) additionally (d) variety
sentence after rearrangement ? (e) plenty
(a) A (b) B 77. (a) time (b) process
(c) C (d) D (c) return (d) event
(e) E (e) action
78. (a) neglected (b) abandoned
DIRECTIONS (Qs. 68-72) : Read each sentence to find out
(c) defaulted (d) depended
whether there is any grammatical error or idiomatic error in it.
(e) disappointed
The error, if any, will be in one part of the sentence. The number
of that part is the answer. If there is no error, the answer is (e). 79. (a) benefit (b) easier
(Ignore errors of punctuation, if any.) (c) reckless (d) disorganised
(e) secure
68. His proposal had (a) / to be send to (b) / the President of the 80. (a) sense (b) confidence
company (c) / for her approval (d). No error (e). (c) challenge (d) doubt
69. Each tuesday evening we visited (a) / the farmers in the (e) believe
Tg:- @NextGenBankers
area (b) / and held a meeting (c) / to discuss the problems
they faced (c). No error (e). Quantitative Aptitude
70. Though our training facilities (a) / are limited only a (b) / few
DIRECTIONS (Qs. 81-90): What should come in place of the
employees have been (c) / selected for training (d). No error
question mark (?) in the following questions ?
(e).
71. During the interview (a) / the panel asked me (b) / several 81. (786 × 64) ÷ 48 = ?
technical questions (c) / and I answered all of it (d). No error (a) 1050 (b) 1024
(e). (c) 1048 (d) 1036
72. He decided to work for (a) / an NGO, but most of his (b) / (e) None of these
classmates opted for high paid (c) / jobs in multinational 82. 3
13824 ´ ? = 864
companies (d). No error (e). (a) 1296 (b) 1156
DIRECTIONS (Qs. 73-80) : In the following passage there are (c) 1600 (d) 1024
blanks each of which has been numbered. These numbers are (e) None of these
printed below the passage and against each, five words are 3
suggested, one of which fits the blank appropriately. Find out 83. 60% of 20% of th of ? = 450
5
the approptiate words in each case.
(a) 6200 (b) 6,240
When we 73 started thirty years ago in 1977, we did not (c) 6150 (d) 6275
know anything about how to run a bank for the poor. We therefore (e) None of these
looked at how others ran their operations and 74 from their
84. 196 × 948 ÷ 158 = ?
mistakes. In Bangladesh, conventional banks and credit co-
(a) 1156 (b) 1200
operatives always 75 lump sum repayments. This created 76
problems because repaying in a lump sum was a mental hurdle for (c) 1188 (d) 1176
borrowers. They tended to delay repayment and get further into (e) None of these
debt in the 77. In the end they usually 78 totally on the loan, 85. 3.5 + 11.25 × 4.5 – 32.5 = ?
which was a loss to the bank. In structuring our own loans, I (a) 18.275 (b) 21.625
decided to ask for a daily payment, Monitoring repayment was (c) 32.375 (d) 25.45
79 and it filled people with 80 that they could repay their loans. (e) None of these
SBI Clerk Solved Paper-2009 2009-7
DIRECTIONS (Qs. 101-105): Study the following table carefully to answer the questions given below it.
Total Residents and Percentage of Woman out of those Living in Various Societies Over the Years
Residents A B C D E
Years % of Total % of Total % of Total % of Total % of Total
Women residents Women residents Women residents Women residents Women residents
2002 44 250 35 280 35 200 40 180 40 220
2003 45 260 30 270 40 250 60 170 65 240
2004 35 240 45 300 45 260 70 200 50 250
2005 40 250 55 320 55 280 60 210 45 220
2006 50 220 50 300 55 240 65 220 50 240
2007 60 240 60 340 60 250 60 230 40 260
GP_4458
2009-8 SBI Clerk Solved Paper-2009
101. What is the difference between the total residents living in (a) 56 (b) 48
all the societies together in the year 2006 and the total (c) 45 (d) Cannot be determined
residents living in all the societies together in the year 2007? (e) None of these
(a) 100 (b) 85 110. A car runs at the speed of 50 kmph when not serviced and
(c) 70 (d) 50 runs at 60 kmph, when serviced. After servicing the car
(e) None of these covers a certain distance in 6 hours. How much time will the
102. What is the average number of men living in all the societies car take to cover the same distance when not serviced ?
together in the year 2005? (a) 8.2 hours (b) 6.5 hours
(a) 110 (b) 125 (c) 8 hours (d) 7.2 hours
(c) 115 (d) 120 (e) None of these
(e) None of these DIRECTIONS (Qs. 111-115) : Study the following graph and
answer the questions given below it :
103. What is the respective ratio of men living in Society C in the
year 2002 to the year 2003 ? Number of Students from Various Faculties studying in
Various Colleges (Number in Thousands)
(a) 15 : 13 (b) 11 : 15
(c) 15 : 11 (d) 13 : 15 Arts Commerce Science
(e) None of these
104. What is the average number of women living in all the 35
societies together in the year 2005 ? 30
(a) 130 (b) 131 25
(c) 125 (d) 140 20
(e) None of these 15
105. What is the respective ratio of the number of women to the 10
total residents in Society E over the years ?
(a) 69 : 143
(c) 173 : 358
Tg:- @NextGenBankers
(b) 344 : 715
(d) 346 : 717
5
0
A B C D
(e) None of these
106. What approximate amount of compound interest can be
111. What is the ratio of the number of students from the faculty
obtained on an amount of Rs. 9, 650 at the rate of 6% p.a. at
of Arts studying in college C to the number of students
the end of 3 years ?
from the faculty of Arts studying in college D ?
(a) Rs. 1,737 (b) Rs. 1,920
(a) 13 : 9 (b) 13 : 10
(c) Rs. 1,720 (d) Rs. 1, 860
(c) 9 : 13 (d) 10 : 13
(e) Rs. 1,843
(e) None of these
107. A milkman sells 120 litres of milk for Rs. 3,360 and he sells
112. What is the average number of students studying in college
240 litres of milk for Rs. 6,120. How much concession does
A from all the faculties together ?
the trader give per litre of milk, when he sells 240 litres of
milk ? (a) 26,000 (b) 28,500
(a) Rs. 2 (b) Rs. 3.5 (c) 26,500 (d) 27,000
(c) Rs. 2.5 (d) Rs. 1.5 (e) None of these
(e) None of these 113. What is the average number of students from the faculty of
Arts from all the colleges together ?
108. When 3,626 is divided by the square of a number and the
answer so obtained is multiplied by 32, the final answer (a) 26,250 (b) 27,250
obtained is 2,368. What is the number ? (c) 27,000 (d) 26,500
(a) 7 (b) 36 (e) None of these
(c) 49 (d) 6 114. The students from the faculty of Commerce studying in
(e) None of these college B are approximately what per cent of the total number
of students in the same college ?
109. The sum of the digits of a two digit number is 14. The
difference between the first digit and the second digit of the (a) 30 (b) 35
two digit number is 2. What is the product of the two digits (c) 43 (d) 48
of the two digit number ? (e) 53
SBI Clerk Solved Paper-2009 2009-9
115. What is the difference between the total number of students 123. In a certain code ‘na pa ka so’ means ‘birds fly very high’,
from the faculty of Commerce from all the colleges together ‘ri so la pa’ means ‘birds are very beautiful’ and ‘ti me ka
and the total number of students from the faculty of Science bo’ means ‘the parrots could fly’. Which of the following is
from all the colleges together ? the code for ‘high’in that language ?
(a) 20,000 (b) 10,000 (a) na (b) k a
(c) 25,000 (d) 15,000 (c) bo (d) so
(e) None of these (e) None of these
DIRECTIONS (Qs. 116-120) : What approximate value should 124. If the digits in the number 86435192 are arranged in
come in place of the question mark (?) in the following questions ? ascending order, what will be the difference between the
(You are not expected to calculate the exact value.) digits which are second from the right and fourth from the
left in the new arrangement ?
116. 964 ´ 348 = ? (a) One (b) Two
(a) 575 (b) 570 (c) Three (d) Four
(c) 586 (d) 550 (e) None
(e) 579 125. If it is possible to make only one meaningful word with the
117. 37.35 + 13.064 × 3.46 = ? Third, Seventh, Eighth and Tenth letters of the word
(a) 89 (b) 83 COMPATIBILITY, which of the following would be the last
(c) 76 (d) 79 letter of that word ? If no such word can be made, give ‘X’
(e) 85 as your answer and if more than one such word can be
formed, give your answer as ‘Y’.
118. (4863 +1174 + 2829) ÷ 756 = ?
(a) I (b) B
(a) 18 (b) 16
(c) L (d) X
(c) 12 (d) 9
(e) Y
(e) 22
119. 54 × 746 ÷ 32 = ?
Tg:- @NextGenBankers 126. In a certain code FINE is written HGPC. How is SLIT written
in that code ?
(a) 1259 (b) 1268
(a) UTGR (b) UTKR
(c) 1196 (d) 1248
(c) TUGR (d) RUGT
(e) 1236
(e) None of these
120. [(1.5)2 ´ (3.2) 2 ] ¸ 2.3 = ? 127. If in a certain language LATE is coded as 8 & 4 $ and HIRE
is coded as 7*3$ then how will HAIL be coded in the same
(a) 6 (b) 4
language ?
(c) 12 (d) 10
(a) 7 & 8* (b) &7*8
(e) 16
(c) 7*& 8 (d) 7&*8
Reasoning Ability (e) None of these
128. Four of the following five are alike in a certain way and so
121. How many meaningful three letter English words can be
form a group. Which is the one that does not belong to that
formed with the letters AER, using each letter only once in
group ?
each word ?
(a) Stem (b) Tree
(a) None (b) One
(c) Root (d) Branch
(c) Two (d) Three
(e) Leaf
(e) Four
129. If ‘Apple’ is called ‘Orange’, ‘Orange’ is called ‘Peach’,
122. Each vowel of the word ADJECTIVE is substituted with the
‘Peach’ is called ‘Patato’, ‘Potato’ is called ‘Banana’,
next letter of the English alphabetical series, and each
‘Banana’ is called ‘Papaya’ and ‘Papaya’ is called ‘Guava’,
consonant is substituted with the letter preceding it. How
which of the following grows underground ?
many vowels are present in the new arrangement ?
(a) Potato (b) Guava
(a) Four (b) One
(c) Apple (d) Banana
(c) Two (d) Three
(e) None of these
(e) None of these
GP_4458
2009-10 SBI Clerk Solved Paper-2009
130. How many such pairs of letters are there in word ENGLISH, 135. Statements :
each of which has as many letters between its two letters as Some walls are windows.
there are between them in the English alphabets ? Some windows are doors.
(a) None (b) One All doors are roofs.
(c) Two (d) Three Conclusions :
(e) More than three I. Some doors are walls.
DIRECTIONS (Qs. 131-135) : In each of the questions below II. No roof is a window.
are given three statements followed by two conclusions
DIRECTIONS (Qs. 136-140) : Study the sets of numbers given
numbered I and II. You have to take the given statements to be
below and answer the questions, which follow :
true even if they seem to be at variance from commonly known
facts. Read both of the conclusions and then decide which of the 489 541 654 953 783
given conclusions logically follows from the given statements 136. If in each number, all the three digits are arranged in
disregarding commonly known facts. ascending order, which of the following will be the lowest
Read the statements and the conclusions which follow it and number ?
give answer (a) 489 (b) 541
(a) If only conclusion I is true. (c) 654 (d) 953
(b) If only conclusion II is true. (e) 783
(c) If either conclusion I or conclusion II is true. 137. If five is subtracted from each of the numbers, which of the
(d) If neither conclusion I nor conclusion II is true. following numbers will be the difference between the second
(e) If both conclusions I and II are true. digit of second highest number and the second digit of the
131. Statements : highest number ?
All stars are suns. (a) Zero (b) 3
Some suns are planets. (c) 1 (d) 4
Conclusions :
Tg:- @NextGenBankers
All planets are satellites.
138.
(e) 2
If in each number the first and the second digits are
I. Some satellites are stars. interchanged, which will be the third highest number ?
II. No star is a satellite. (a) 489 (b) 541
132. Statements : (c) 654 (d) 953
All curtains are rods. (e) 783
Some rods are sheets. 139. Which of the following numbers will be obtained if the first
Some sheets are pillows. digit of lowest number is subtracted from the second digit
Conclusions: of highest number after adding one to each of the numbers?
I. Some pillows are rods. (a) 1 (b) 2
II. Some rods are curtains. (c) 3 (d) 4
133. Statements : (e) 5
All switches are plugs. 140. If in each number, the first and the last digits are
Some plugs are bulbs. interchanged, which of the following will be the second
All bulbs are sockets. highest number ?
Conclusions: (a) 489 (b) 541
I. Some sockets are plugs. (c) 654 (d) 953
II. Some plugs are switches. (e) 783
134. Statements : DIRECTIONS (Qs. 141-145) : Read the following information
All fishes are birds. carefully and answer the questions, which follow :
All birds are rats.
‘A - B’ means ‘A is father of B’.
All rats are cows.
‘A + B’ means ‘A is daughter of B’.
Conclusions :
‘A ÷ B’ means ‘A is son of B’.
I. All birds are cows.
‘A × B’ means ‘A is wife of B’.
II. All rats are fishes.
SBI Clerk Solved Paper-2009 2009-11
Marketing Aptitude/
(a) (b) (c) (d) (e) Computer Knowledge
Answer Figures 161. _______ is a set of keywords, symbols, and a system of
rules for constructing statements by which humans can
communicate the instructions to be executed by a computer.
(a) A computer program
(a) (b) (c) (d) (e)
(b) A programming language
157. Problem Figures (c) An assemble
(d) Syntax
(e) None of these
(a) (b) (c) (d) (e) 162. A printer is this kind of device
(a) input (b) word processing
Answer Figures
(c) processing (d) output
(e) None of these
163. This can be another word for program
(a) (b) (c) (d) (e) (a) software (b) disk
158. Problem Figures Tg:- @NextGenBankers (c) floppy
(e) None of these
(d) hardware
169. Where is the disk put in a computer ? (d) the transmission of messages and files via a computer
(a) In the modem (b) In the hard drive network
(c) Into the CPU (d) In the disk drive (e) None of these
(e) None of these 178. Hardware includes
170. A hard copy of a document is (a) all devices used to input data into a computer
(a) printed on the printer (b) sets of instructions that a computer runs or executes
(b) stored on a floppy (c) the computer and all the devices connected to it that
(c) stored on a CD are used to input and output data
(d) stored in the hard disk (d) all devices involved processing information including
(e) None of these the central processing unit, memory, and storage
171. The name that the user gives to a document is referred to as (e) None of these
(a) document-name (b) file-name 179. The most common method of entering text and numerical
(c) name-given (d) document-identity data into a computer system is through the use of a
(e) None of these (a) keyboard (b) scanner
172. Restarting a computer that is already on is referred to as (c) printer (d) plotter
(a) shut down (b) cold booting (e) None of these
(c) warm booting (d) logging off 180. Which of the following groups consist of only input
(e) None of these devices?
173. E-mail (electronic mail) is (a) Mouse, Keyboard, Monitor
(a) an internet standard that allows users to upload and (b) Mouse, Keyboard, Printer
download files (c) Mouse, Keyboard, Plotter
(b) a real-time typed conversation that takes place on a
(d) Mouse, Keyboard, Scanner
computer
Tg:- @NextGenBankers
(c) an online area in which users conduct written
181.
(e) None of these
Marketing is required in banks due to
discussions about a particular subject
(d) the transmission of messages and files via a computer (a) globalisation
network (b) computerisation
(e) None of these (c) increase in population
174. The person who writes and tests computer programs is (d) government dictates
called a (e) None of these
(a) programmer (b) computer scientist 182. Marketing is
(c) software engineer (d) project developer (a) a day-to-day function
(e) None of these (b) a one-off affair
175. The information you put into the computer is called (c) a one-man show
(a) facts (b) data
(d) a collective process
(c) files (d) directory
(e) None of these
(e) None of these
183. Qualities needed by a good marketing staff are
176. The output devices make it possible to
(a) Aggressiveness (b) Pushy
(a) view or print data (b) store data
(c) Perseverance (d) Politeness
(c) scan data (d) input data
(e) None of these
(e) None of these
184. Marketing should be resorted
177. A chat is
(a) an internet standard that allows users to upload and (a) only among rich persons
download files (b) only among the poor
(b) a typed conversation that takes place on a computer (c) only in crowded areas
(c) an online area in which users conduct written (d) depends on the product
discussions about a particular subject (e) depends on the banks
GP_4458
2009-14 SBI Clerk Solved Paper-2009
ANSW ER KEY
1 (e) 21 (b) 41 (b) 61 (b) 81 (c) 101 (a) 121 (c) 141 (a) 161 (a) 181 (a)
2 (a) 22 (a) 42 (c) 62 (c) 82 (a) 102 (b) 122 (c) 142 (d) 162 (d) 182 (d)
3 (e) 23 (a) 43 (d) 63 (e) 83 (e) 103 (d) 123 (a) 143 (e) 163 (a) 183 (e)
4 (e) 24 (c) 44 (e) 64 (a) 84 (d) 104 (b) 124 (d) 144 (c) 164 (c) 184 (d)
5 (d) 25 (c) 45 (e) 65 (c) 85 (b) 105 (e) 125 (b) 145 (b) 165 (b) 185 (c)
6 (e) 26 (e) 46 (b) 66 (e) 86 (c) 106 (e) 126 (e) 146 (a) 166 (d) 186 (d)
7 (d) 27 (a) 47 (c) 67 (d) 87 (d) 107 (c) 127 (d) 147 (c) 167 (d) 187 (d)
8 (c) 28 (d) 48 (a) 68 (b) 88 (e) 108 (a) 128 (b) 148 (c) 168 (c) 188 (c)
9 (d) 29 (e) 49 (c) 69 (e) 89 (b) 109 (b) 129 (d) 149 (a) 169 (d) 189 (e)
10 (b) 30 (b) 50 (d) 70 (a) 90 (a) 110 (d) 130 (e) 150 (b) 170 (a) 190 (e)
11 (a) 31 (e) 51 (a) 71 (d) 91 (a) 111 (b) 131 (c) 151 (d) 171 (b) 191 (a)
12 (e) 32 (c) 52 (d) 72 (c) 92 (b) 112 (e) 132 (d) 152 (a) 172 (d) 192 (d)
13 (e) 33 (c) 53 (a) 73 (d) 93 (c) 113 (a) 133 (e) 153 (c) 173 (d) 193 (e)
14 (d) 34 (e) 54 (b) 74 (c) 94 (e) 114 (c) 134 (a) 154 (e) 174 (a) 194 (c)
15 (c) 35 (c) 55 (a) 75 (a) 95 (d) 115 (d) 135 (d) 155 (b) 175 (b) 195 (e)
16 (d) 36 (b) 56 (b) 76 (a) 96 (c) 116 (e) 136 (b) 156 (a) 176 (a) 196 (b)
17
18
(e)
(a)
37
38
(a)
(a)
Tg:- @NextGenBankers
57
58
(d)
(e)
77
78
(b)
(c)
97
98
(d)
(e)
117
118
(b)
(c)
137
138
(b)
(d)
157
158
(d)
(c)
177
178
(b)
(c)
197
198
(a)
(c)
19 (c) 39 (d) 59 (d) 79 (b) 99 (b) 119 (a) 139 (a) 159 (d) 179 (a) 199 (d)
20 (b) 40 (c) 60 (a) 80 (b) 100 (a) 120 (d) 140 (c) 160 (e) 180 (d) 200 (c)
48. (a) Refer to the third last sentence ‘people Twenty pens were distributed and the remaining were
took.............cheated him’ of the para 1 of the passage. returned.
49. (c) The word spend (verb) means to pay out, to emplay Thus, lying and remaining are synonymous.
(labour, time etc.) on some objects. For instance,
GP_4458
2009-16 SBI Clerk Solved Paper-2009
51. (a) The word dismay (noun) means feeling of fear and 84. (d) ? = 196 × 948 ÷ 158
discouragement. For instance,
196 ´ 948
They were struck with dismay at the news. = = 1176
158
While, the word joy (noun) means feeling of happiness.
85. (b) ? = 3.5 + 11.25 × 4.5 – 32.5
Thus, both are antonymous.
= 3.5 + 50.625 – 32.5
52. (d) The word tend (verb) means look after. For instance,
= 54.125 – 32.5 = 21.625
Nurse tended the injured persons.
While, the word ignore (verb) means take no notice of. 4096 ´ 56
86. (c) ?=
For instance, 764 – 652
71.
in the sentence.
Tg:- @NextGenBankers
(d) Use ‘and I answered’ properly to express cause and
Þ ? = 9962 – 9858 = 104
72. (c) Apply the adverb ‘highly’ before the adjective ‘paid’. 90. (a) ? = (2640 ÷ 48) × (2240 ÷ 35)
= 55 × 64 = 3520
786 ´ 64
81. (c) ?= = 1048
48
121 117 108 92 67 31
91. (a)
82. (a) 3
13824 ´ ? = 864 –2
2
–3
2
–4
2
–5
2
–6
2
3
24 ´ 24 ´ 24 ´ ? = 864
50 26 14 8 5 3.5
Þ 24 ´ ? = 864 92. (b)
÷2+1 ÷2+1 ÷2+1 ÷2+1 ÷2+1
864
Þ ?=
24 3 23 43 63 83 103
93. (c)
\ ? = 36 × 36 = 1296
+20 +20 +20 +20 +20
60 20 3
83. (e) ´ ´ ´ ? = 450
100 100 5 748 737 715 682 638 583
94. (e)
9 –11 –22 –33 –44 –55
Þ ´ ? = 450
125
1 9 25 49 81 121 169
450 ´ 125
Þ?= = 6250 95. (d)
9
12 32 52 72 92 112 132
SBI Clerk Solved Paper-2009 2009-17
96. (c) Let the number of ducks and frogs in the pond be 37x 103. (d) Required respective ratio
and 39x respectively.
65 ´ 200 60 ´ 250
ATQ, = :
100 100
37 x + 39 x
= 152 = 130 : 150
2
= 13 : 15
Þ 38x = 152
104. (b) Total number of women in the societies together in
152 year 2005
Þ x= =4
38
40 ´ 250 55 ´ 320 55 ´ 280
\ Number of frogs = 39x = + +
= 39 × 4 = 156 100 100 100
97. (d) The word ARISE has 5 distinct letters.
60 ´ 210 45 ´ 220
\ Number of arrangements = 5! + +
100 100
= 5 × 4 × 3 × 2 × 1 = 120
98. (e) The number of employees in companies A, B and C be = 100 + 176 + 154 + 126 + 99 = 655
4x, 5x and 6x respectively 655
After increase in the number of employees, required \ Required average = = 131
5
ratio will be
105. (e) Number of women in society E over the years
125 130 150
= 4x ´ : 5x ´ : 6x ´ 40 ´ 220 65 ´ 240 50 ´ 250 45 ´ 220
100 100 100 = + + +
100 100 100 100
= 4 × 25 : 5 × 26 : 6 × 30
= 10 : 13 : 18 50 ´ 240 40 ´ 260
+ +
Tg:- @NextGenBankers
99 (b) According to the questions, third number will be
= 5 × 213 – 2 × 233.5 – 2 × 271
100 100
= 88 + 156 + 125 + 99 + 120 + 104 = 692
= 1065 – 467 – 542 = 56 Total number of residents in the society E over the
100. (a) Let Sonali’s monthly income = Rs. x. years = 220 + 240 + 250 + 220 + 240 + 260 = 1430
Sonali’s percentage monthly spendings \ Required ratio = 692 : 1430 = 346 : 715
= (55 + 15)% = 70% éæ t ù
r ö
Percentage savings = 100 – 70 = 30% 106. (e) C .I . = P êç1 + ÷ - 1ú
êëè 100 ø úû
ATQ,
\ 30% of x = 12750 éæ 6 ö
3 ù
= 9650 êç1 + ÷ - 1ú
12750 ´ 100 è 100 ø
Þ x= = Rs. 42500 ëê ûú
30 = 9650 (1.191016 – 1)
101. (a) Number of all residents in year = 9650 × 0.191016 » Rs. 1843
2006 : 220 + 300 + 240 + 220 + 240 = 1220 107. (c) The rate of milk when milkman sells 120 litres of milk for
2007 : 240 + 340 + 250 + 230 + 260 = 1320 Rs. 3360
Required difference = 1320 – 1220 = 100
æ 3360 ö
\ SP = ç = Rs. 28
è 120 ÷ø
102. (b) Total number of men living together in year 2005
60 ´ 250 45 ´ 320 45 ´ 280 The rate of milk when milkman sells 240 litres of milk for
= + +
100 100 100 Rs. 6120.
40 ´ 210 55 ´ 220
+ + æ 6120 ö
100 100 \ SP = çè ÷ = Rs. 25.5
240 ø
= 150 + 144 + 126 + 84 + 121 = 625
\ Required discount
625
\ Required average = = 125 = (28 – 25.5) = Rs. 2.5
5
GP_4458
2009-18 SBI Clerk Solved Paper-2009
108. (a) Let the number be x. 115. (d) Total number of Commerce students
= 27500 + 32500 + 25000 + 22500
3626
ATQ, ´ 32 = 2368 = 107500
x2
Total number of Science students
2 3626 ´ 32 = 30000 + 20000 + 12500 + 30000
Þ x = = 49
2368 = 92500
\ x = 49 = 7 \ Difference = 107500 – 92500 = 15000
109. (b) Let the two digits number be 10n + m and n > m. 116. (e) ? = 964 ´ 348
As given, » 31.05 ´ 18.6 » 579
n + m = 14 117. (b) ? = 37.35 + 13.064 × 3.46
n–m=2 » 37.35 + 13 ´ 3.5
On solving the equation, » 37.35 + 45.5 » 82.85 » 83
n = 8, m = 6 118. (c) ? = (4863 + 1174 + 2829) ÷ 756
\ Product of digits = 8 × 6 = 48 = 8866 ÷ 756 = 11.72 » 12
110. (d) After servicing, speed of car = 60 km/h 54 ´ 746
119. (a) ?= = 1258.87 » 1259
\ Distance covered in 6 hours 32
= (60 × 6)km = 360 km
120. (d) ? = [(1.5)2 ´ (3.2)2 ] ¸ 2.3
Before servicing, time taken to cover 360 km
2.25 ´ 10.24
= » 10
\ Time taken = 360 km = 7.2 hours
Tg:- @NextGenBankers
50 km/h
2.3
121. (c) Meaningful words : ARE, EAR
111. (b) Number of students from the faculty of Arts
122. (c) A D J E C T I V E
Studying in College C = 32500 and
+1 ¯ - 1 ¯ - 1 ¯ + 1 ¯ - 1 ¯ - 1 ¯ + 1 ¯ - ¯ + ¯
College D = 25000.
B C I F B S J U F
\ Required ratio = 32500 : 25000 = 13 : 10
112. (e) Required Average 123. (a) na pa ka so ® birds fly very high
ri so la pa ® birds are very beautiful
25000 + 27500 + 30000 82500
= = = 27500 ti me ka bo ® the parrots could fly
3 3
Thus high is coded as na.
113. (a) Required average number of Arts students 124. (d) 1 2 3 4 5 6 7 8 9
25000 + 22500 + 32500 + 25000 Difference = 8 – 4 = 4
= 125. (b) 1 2 3 4 5 6 7 8 9 10 11 12 13
4
C O M PA T I B I L I T Y
105000
= = 26250 Meaningful word Þ L I M B
4
+2 -2
114. (c) Total number of students in college B 126. (e) As F ¾¾® H I ¾¾® G
= 22500 + 32500 + 20000 = 75000 +2 -2
N ¾¾® P E ¾¾® C
Number of Commerce students in college B
Similarly,
= 32500
+2 -2
S ¾¾® U L ¾¾® J
32500
\ Required percentage = ´ 100 » 43 +2 -2
75000 I ¾¾® K T ¾¾® R
SBI Clerk Solved Paper-2009 2009-19
I ¾¾
®* L ¾¾
®8 This is Conclusion I.
128. (b) Others relate to ‘parts of tree’. 135. (d) Some windows are doors.
129. (d) Since ‘potato’ is called Banana. Thus, ‘Banana’ grows
underground.
130. (e) EI, EG, GI and NL.
All doors are roofs.
131. (c) Some suns are planets.
(I + A Þ I-type)
“Some windows are roots”.
136. (b) Arranged in ascending order, we get
All planets are satellites.
489, 145, 456, 359 and 378.
(I + A Þ I-type)
\ Lowest number = 145 or 541.
“Some suns are satellites”.
137. (b) We have,
Conclusions I and II form Complementary Pair.
Tg:- @NextGenBankers
Therefore, either I or II follows.
489 – 5 = 484
541 – 5 = 536
654 – 5 = 649
953 – 5 = 948
132. (d) All curtains are rods. 783 – 5 = 778
Highest number = 948
Second highest number = 778
Some rods are sheets. \ Difference = 7 – 4 = 3
(A + I Þ No Conclusion) 138. (d) Interchanging the first and the second digits of given
numbers, we get 849, 451, 564, 593 and 873.
133. (e) Some plugs are bulbs
\ Third highest number = 593 or 953.
139. (a) On adding 1, we get 490, 542, 655, 954 and 784.
\ Required number = 5 – 4 = 1
All bulbs are sockets. 140. (c) Interchanging the first and the last digits of given
(I + A Þ I-type) numbers, we get 984, 145, 456, 359 and 387.
“Some plugs are sockets”. \ Second highest number = 456 or 654.
Conclusion I is Converse of thi3s Conclusion. 141. (a) P + S ® P is daughter of S.
Conclusion II is Converse of the first Premise. S – T ® S is father of T.
134. (a) All fishes are birds. (conversion) Therefore, P is sister of T.
142. (d) P × Q ® P is wife of Q.
Q – T ® Q is father of T.
All birds are rats. T is child of P and Q.
(A + A Þ A-type) The sex of T is not known.
“All fishes are rats”. T is either son or daughter of P.
GP_4458
2009-20 SBI Clerk Solved Paper-2009
143. (e) P × S ® P is wife of S. For (Qs. 151-155): Given information can be tabulated as follows
S ÷ T ® S is son of T.
Person Sex Company Specialisation
T is either father-in-law or mother-in-law of P. A Male X Finance
P ÷ S ® P is son of S. B Male Z Marketing
C Male Y Engineer
S × T ® S is daughter of T
D Female X HR
Therefore, T is father of P. E Male Y Doctor
P – S ® P is father of T. F Male Y Marketing
G Female Z Finance
P + T ® P is daughter of T
H Male Z HR
T ÷ S ® T is son of S.
156. (a) The movement and other changes in designs can be
Therefore, T is father of P.
shown as :
144. (c) P + Q ® P is daughter of Q.
Q – S ® Q is father of S. 1 to 2 2 to 3
Therefore, P is sister of S.
P ÷ Q ® P is son of Q.
Q × S ® Q is wife of S. N N
Therefore, P is son of S.
These two steps are repeated alternately.
P ÷ Q ® P is son of Q. 157. (d) In the subsequent figures respectively one, two
Q + S ® Q is daughter of S. zero............ curve(s) is/are added and curves move
along the line segment and get reversed in each
Tg:- @NextGenBankers
Therefore, P is grandson of S.
145. (b) P + Q ® P is daughter of Q.
158. (c)
subsequent figure.
In the subsequent figures one design is left intact while
Q × T ® Q is wife of P. other three designs are inverted.
Therefore, T is father of P. 159. (d) In the subsequent figures the star moves three steps
146. (a) B ® 9; A ® 2; R ® *; N ® %; I ® #; S ® 4 in clockwise direction inside the hexagon after every
two figures. The equal sign moves respectively one
147. (b) D ® 2; M ® @; B ® 9; N ® %; I ® #; A ® 2
and two step(s) in clockwise direction along the sides
Condition (i) is applied.
of the hexagon. The design C moves in and out the
148. (c) I ® $; J ® 8; B ® 9; R ® *; L ® £; G ® # hexagon in the subsequent figures and moves
Condition (ii) is applied. respectively two and one step(s) in clockwise direction.
In other words, this problem is based on the rule (1)
149. (a) B ® 9; K ® ©; G ® $; Q ® 7; J ® ©; N ® %
= (5) and hence (2) = (6).
Condition (iii) is applied.
160. (e) In the subsequent figures respectively two and three
150. (b) E ® £; G ® $; A ® 2; K ® 1; R ® *; L ® 3 designs change size alternately in a set order.
Condition (ii) is applied.